You are on page 1of 192

Machine Translated by Google

Machine Translated by Google

NGHỆ THUẬT VÀ THỦ CÔNG

GIẢI QUYẾT VẤN ĐỀ

Phiên bản thứ hai

Paul Zeitz
Đại học San Francisco

Là: TOÀN BỘ �
II

tôi 1 807 �
z z
� �WILEY�
z z
Năm
- 2007!
II � r
HAI TRĂM NĂM

John Wiley & Sons, Inc.


Machine Translated by Google

TRÌNH BIÊN SOẠN ĐIỀU KIỆN Trận chiến Angela


CHỈNH SỬA DỰ ÁN Jennifer Battista
TRỢ LÝ BIÊN TẬP Daniel Grace
SENIOR PRODUCTION EDITOR Ken Santor
QUẢN LÝ TIẾP THỊ Amy Bán
THIẾT KẾ BÌA Michael St. Martine
ẢNH BÌA Steve Casimiro / The Image Bank / Getty
Images, Inc.

Cuốn sách này được tác giả đặt trong LaTeX và được in và đóng gói bởi Malloy, Inc.
Bìa được in bởi Phoenix Color.

Cuốn sách này được in trên giấy không axit. 00

Bản quyền © 2007 John Wiley & Sons, Inc. Mọi quyền được bảo lưu.

Không một phần nào của ấn phẩm này có thể được sao chép, lưu trữ trong hệ thống truy xuất
hoặc truyền tải dưới bất kỳ hình thức nào hoặc bằng bất kỳ phương tiện nào, điện tử, cơ
khí, photocopy, ghi âm, quét hoặc bằng cách khác, trừ khi được cho phép theo Mục 1 07 hoặc 1
08 của 1 976 Đạo luật Bản quyền Hoa Kỳ, mà không có sự cho phép trước bằng văn bản của Nhà
xuất bản hoặc ủy quyền thông qua việc thanh toán phí mỗi bản sao thích hợp cho Trung tâm Xóa
bản quyền, Inc. 222 Rosewood Drive, Danvers, MA 01923, (978) 750-8400, fax (978) 646-8600. Các
yêu cầu cấp phép đối với Nhà xuất bản phải được gửi tới Phòng Cấp phép, John Wiley & Sons,
Inc., III River Street, Hoboken, NJ 07030-5774, (201) 748-601 1, fax (20 1) 748-6008.
Để đặt sách hoặc yêu cầu dịch vụ khách hàng, vui lòng gọi 1 -800-CALL WILEY (225-5945).

ISBN- IO 0-47 1 -78901-1


ISBN- 13 978-0-47 1 -78901-7

Được in tại Hoa Kỳ

10 9 8 7 6 5 4 3 2 I
Machine Translated by Google

Đến gia đình tôi


Machine Translated by Google
Machine Translated by Google

Người thám hiểm là người bị lạc.

-Tim Cahill, Báo đốm xé xác tôi

Khi các thám tử nói về thời điểm mà một tội phạm trở thành của họ để điều tra,
họ nói về "bắt một vụ án", và một khi bị bắt, một vụ án giống như một cơn cảm
lạnh: nó bốc lên và tiêu thụ tâm trí của người bắt cho đến khi, như một cơn sốt,

nó tan vỡ; hoặc, nếu nó vẫn chưa được giải quyết, nó sẽ được truyền lại như một
sự lây lan, từ thám tử này sang thám tử khác, mà không bao giờ hoàn toàn giải
phóng được cho những ai bắt được nó trên đường đi.

-Philip Gourevitch, Vỏ lạnh


Machine Translated by Google
Machine Translated by Google

Lời nói đầu của Ấn bản thứ hai

Ấn bản mới này của Nghệ thuật và Thủ công Giải quyết Vấn đề là một phiên bản mở rộng, và
tôi hy vọng là phiên bản cải tiến của tác phẩm gốc. Có một số thay đổi, bao gồm:

• Một chương mới về hình học. Nó dài nhiều trang như các chương tổ hợp và lý thuyết
số cộng lại - nhưng nó chỉ đơn thuần là phần giới thiệu về chủ đề. Các chuyên
gia chắc chắn sẽ không hài lòng với tốc độ của chương (chậm, đặc biệt là ở phần
đầu) và thiếu các chủ đề (hình học đặc, độ dài và góc có hướng, định lý Desargues,
đường tròn 9 điểm). Nhưng chương này là dành cho những người mới bắt đầu; do đó
có tựa đề là "Hình học cho người Mỹ." Tôi hy vọng rằng nó mang lại cho người
giải bài tập mới sự tự tin để nghiên cứu các vấn đề hình học một cách nhanh chóng
như họ có thể giải quyết các câu hỏi toán học rời rạc.

• Mở rộng chương giải tích, với nhiều bài toán mới. • Nhiều vấn đề

hơn, đặc biệt là những vấn đề "dễ", trong một số chương khác.

Để phù hợp với tài liệu mới và kiểm soát độ dài, các bài toán ở định dạng hai cột
với phông chữ nhỏ hơn. Nhưng đừng để kích thước nhỏ hơn này đánh lừa bạn nghĩ rằng
các vấn đề ít quan trọng hơn phần còn lại của cuốn sách. Như với lần xuất bản đầu
tiên, các vấn đề là trọng tâm của cuốn sách. Người đọc nghiêm túc, ít nhất, nên đọc
từng báo cáo vấn đề và cố gắng càng nhiều càng tốt. Để tạo điều kiện thuận lợi cho
việc này, tôi đã mở rộng số lượng các vấn đề được thảo luận trong phụ lục Gợi ý,
hiện có thể tìm thấy trực tuyến tại www. wi ley. com / col lege / zei tz.
Tôi vẫn mang ơn những người mà tôi đã cảm ơn trong lời nói đầu của ấn bản đầu tiên.
Ngoài ra, tôi muốn cảm ơn những người sau đây.

• Jennifer Battista và Ken Santor tại Wiley đã tận tình hướng dẫn tôi quá trình
hồi sinh, không bao giờ mất kiên nhẫn với sự trì hoãn của tôi. • Brian Borchers,

Joyce Cutler, Julie Levandosky, Ken Monks, Deborah Moore Russo, James Stein, và
Draga Vidakovic đã xem xét cẩn thận bản thảo, tìm thấy nhiều sai sót và đưa ra
nhiều đề xuất quan trọng.

• Tại Đại học San Francisco, nơi tôi đã làm việc từ năm 1992, Trưởng khoa Jennifer
Turpin và Phó Hiệu trưởng Brandon Brown đã hào phóng hỗ trợ các hoạt động ngoại
khóa của tôi, bao gồm cả việc chấp thuận cho nghỉ phép trong năm học 2005--06 để
thực hiện dự án này . • Kể từ năm 1 997, sự hiểu biết của tôi về cách giải quyết

vấn đề đã được nâng cao nhờ công việc của tôi với một số vòng thi và cuộc thi toán
học ở địa phương. Viện Nghiên cứu Khoa học Toán học (MSRI) đã tài trợ rất nhiều

cho hoạt động này, và tôi đặc biệt biết ơn các quan chức MSRI Hugo Rossi, David
Eisenbud, Jim Sotiros và Joe Buhler. Những người khác đã giúp đỡ tôi rất nhiều
bao gồm Tom Rike, Sam Vandervelde, Mark Saul, Tatiana Shubin, Tom Davis, Josh
Zucker, và đặc biệt, Zvezdelina Stankova.

ix
Machine Translated by Google
x

Và cuối cùng nhưng không kém phần quan trọng, tôi muốn tiếp tục viết từ ấn bản đầu tiên, và cầu

xin vợ và hai con của tôi tha thứ cho sự thiếu chú ý thiếu ngủ của tôi. Tôi dành tặng cuốn sách này, với
tình yêu, cho họ.

Paul Zeitz San Francisco, tháng 6 năm 2006

Lời nói đầu của Ấn bản đầu tiên

Tại sao lại là cuốn sách này?

Đây là cuốn sách về giải toán dành cho người mới học trình độ đại học. Ý tôi là những người thông minh

biết một số toán học (lý tưởng nhất là một số phép tính giải tích), những người yêu thích toán học,

những người có ít nhất một khái niệm mơ hồ về chứng minh, nhưng đã dành phần lớn thời gian của họ để làm

bài tập hơn là các vấn đề.

Bài tập là một câu hỏi kiểm tra mức độ thành thạo của học sinh đối với một kỹ thuật tập trung hẹp,

thường là một kỹ thuật gần đây đã được "đề cập". Các bài tập có thể khó hoặc dễ, nhưng chúng không bao

giờ khó hiểu, vì nó luôn rõ ràng ngay lập tức để tiến hành. Có được giải pháp có thể liên quan đến công

việc kỹ thuật phức tạp, nhưng con đường hướng tới giải pháp luôn rõ ràng. Ngược lại, một vấn đề là một

câu hỏi không thể trả lời ngay lập tức.

Các vấn đề thường có kết thúc mở, nghịch lý và đôi khi không thể giải quyết được, và cần phải điều tra

trước khi người ta có thể đi đến giải pháp. Vấn đề và giải quyết vấn đề là trọng tâm của toán học. Các

nhà toán học nghiên cứu không làm gì khác ngoài giải quyết vấn đề mở. Trong ngành công nghiệp, việc có

thể giải quyết một vấn đề chưa được xác định rõ ràng đối với nhà tuyển dụng quan trọng hơn nhiều so với

khả năng đảo ngược một ma trận. Máy tính có thể làm điều thứ hai, nhưng không thể làm điều thứ nhất.

Một người giải quyết vấn đề tốt không chỉ được tuyển dụng nhiều hơn. Một người học cách giải quyết

các vấn đề toán học đi vào văn hóa chính thống của toán học; anh ấy hoặc cô ấy phát triển sự tự tin

tuyệt vời và có thể truyền cảm hứng cho người khác. Hơn hết, những người giải quyết vấn đề có niềm vui;

người giải quyết vấn đề lão luyện biết làm thế nào để chơi với toán học, và hiểu và đánh giá cao toán

học đẹp.

Một phép tương tự: Một học sinh toán trung bình (không biết giải) giống như một người đến phòng tập

thể dục ba lần một tuần để lặp lại nhiều lần với mức tạ thấp trên nhiều máy tập khác nhau. Ngược lại,

người giải quyết vấn đề sẽ phải trải qua một chuyến đi dài và vất vả. Cả hai người đều trở nên mạnh mẽ

hơn. Người giải quyết vấn đề trở nên nóng, lạnh, ướt, mệt và đói. Người giải quyết vấn đề bị lạc và phải

tìm đường. Người giải quyết vấn đề bị phồng rộp. Người giải quyết vấn đề leo lên đỉnh núi, nhìn thấy

những khung cảnh không mơ mộng cho đến nay. Người giải quyết vấn đề đến những nơi có vẻ đẹp tuyệt vời và

trải nghiệm sự sung sướng được khuếch đại bởi nỗ lực đã bỏ ra để đến được đó. Khi người giải quyết vấn

đề trở về nhà, họ được tiếp thêm sinh lực cho cuộc phiêu lưu và không thể ngừng trào dâng về trải nghiệm

tuyệt vời. Trong khi đó, chú chuột tập thể dục đã khỏe mạnh dần lên, nhưng không có nhiều niềm vui và ít

chia sẻ với những người khác.

Mặc dù phần lớn học sinh toán Mỹ không phải là những người biết giải quyết vấn đề, nhưng vẫn tồn

tại một nền văn hóa giải quyết vấn đề ưu tú. Các thành viên của nó đã được lớn lên với các câu lạc bộ

toán học, và thường tham gia vào các cuộc thi toán học, và học các bài toán "văn học dân gian" quan trọng và
Machine Translated by Google
xi

ý tưởng mà hầu hết các nhà toán học coi là đương nhiên. Văn hóa này phổ biến ở các vùng của Đông Âu
và tồn tại trong các túi nhỏ ở Hoa Kỳ. Tôi lớn lên ở thành phố New York và theo học tại trường

trung học Stuyvesant, nơi tôi là đội trưởng đội toán học, và do đó, tôi có trình độ học vấn về
chuyên gia giải quyết vấn đề. Tôi đã và đang tham gia rất sâu vào các cuộc thi giải quyết vấn đề. Ở

trường trung học, tôi là thành viên của đội Hoa Kỳ đầu tiên tham gia Olympic Toán học Quốc tế (lMO)

và hai mươi năm sau, với tư cách là một giáo sư đại học, tôi đã huấn luyện một số đội IMO gần đây
nhất, bao gồm một đội vào năm 1994 đã đạt được thành tích chỉ có hiệu suất hoàn hảo nhất trong lịch

sử của IMO.
Nhưng hầu hết mọi người không lớn lên trong nền văn hóa giải quyết vấn đề này. Kinh nghiệm của
tôi với tư cách là một giáo viên trung học và đại học, chủ yếu là với những học sinh không lớn lên

như những người giải quyết vấn đề, đã thuyết phục tôi rằng giải quyết vấn đề là điều dễ dàng đối
với bất kỳ học sinh giỏi toán nào. Là một nhà truyền giáo cho nền văn hóa giải quyết vấn đề, Nghệ

thuật và Thủ công Giải quyết vấn đề là một ước tính đầu tiên trong nỗ lực truyền bá phúc âm của tôi.
Tôi quyết định viết cuốn sách này bởi vì tôi không thể tìm thấy bất kỳ văn bản phù hợp nào phù hợp

với sinh viên của tôi tại Đại học San Francisco. Có rất nhiều cuốn sách hay với rất nhiều toán học
hay, nhưng tôi thấy rằng bản thân toán học là chưa đủ. Nghệ thuật và Thủ công giải quyết vấn đề được
hướng dẫn bởi một số nguyên tắc:

• Giải quyết vấn đề có thể được dạy và có thể học


được. • Thành công khi giải quyết vấn đề phụ thuộc cốt yếu vào yếu tố tâm lý.
Các đặc tính như sự tự tin, sự tập trung và lòng dũng cảm là cực kỳ quan trọng. • Điều

tra không cấm ít nhất cũng quan trọng như lập luận chặt chẽ. • Các khía cạnh phi tâm lý của

việc giải quyết vấn đề là sự kết hợp của các bí quyết chiến lược, các phương pháp tiếp cận

chiến thuật tập trung hơn và các công cụ kỹ thuật được xác định hẹp. • Kiến thức về văn hóa

dân gian (ví dụ: nguyên tắc chuồng chim bồ câu hoặc Conway's

Vấn đề kiểm tra) cũng quan trọng như việc thành thạo các công cụ kỹ thuật.

Đọc sách này

Do đó, mặc dù cuốn sách này được sắp xếp giống như một cuốn sách giáo khoa toán tiêu chuẩn, nhưng
giọng điệu của nó ít trang trọng hơn nhiều: nó cố gắng đóng vai trò của một huấn luyện viên thân

thiện, giảng dạy không chỉ bằng cách trình bày, mà bằng cách khuyến khích, ví dụ và thách thức. Chỉ

có một số điều kiện tiên quyết là giả định và trong khi đối tượng mục tiêu của tôi là sinh viên
chuyên ngành toán đại học, cuốn sách chắc chắn có thể tiếp cận được với học sinh trung học phổ thông

và những người tự đọc, đặc biệt là giáo viên (ở bất kỳ cấp độ nào).
Cuốn sách được chia thành hai phần. Phần I là tổng quan về phương pháp giải quyết vấn đề, và

là cốt lõi của cuốn sách. Phần II bao gồm bốn chương có thể được đọc độc lập với nhau và phác thảo
đại số, tổ hợp, lý thuyết số và giải tích theo quan điểm của người giải quyết vấn đề.
Tôi

Để giữ cho độ dài của cuốn sách có thể

quản lý được, không có chương hình học. Các ý tưởng hình học được phổ biến xuyên suốt cuốn sách và
tập trung ở một vài nơi (ví dụ, Phần 4.2). Tuy nhiên,

Để bảo tồn các trang, ấn bản thứ hai không còn sử dụng nhãn chính thức "Phần I" và "Phần II" . Tuy nhiên,
Tôi

cuốn sách có cùng cấu trúc logic, với một chương bổ sung về hình học. Để biết thêm thông tin về cách đọc cuốn
sách, hãy xem Phần 1.4.
Machine Translated by Google
xii

cuốn sách là một chút nhẹ về hình học. May mắn thay, một số cuốn sách hình học tuyệt vời đã được
viết. Ở cấp tiểu học, Hình học được nhìn lại [6] và Hình học và trí tưởng tượng [2 1] không có giá
trị bằng.
Cấu trúc của mỗi phần trong mỗi chương rất đơn giản: giải thích, ví dụ và các vấn đề-rất nhiều

và rất nhiều-một số dễ, một số khó, một số rất khó. Mục đích của cuốn sách là dạy cách giải quyết
vấn đề và điều này chỉ có thể đạt được bằng cách vật lộn với nhiều vấn đề, giải quyết một số vấn
đề và học hỏi từ những người khác rằng không phải mọi vấn đề đều có nghĩa là phải giải quyết được
và bất kỳ thời gian nào dành cho việc suy nghĩ thành thật về một vấn đề đều là thời gian tốt đã
dành.
Mục tiêu của tôi là đọc cuốn sách này và giải quyết một số vấn đề trong số 660 vấn đề của nó
sẽ giống như chuyến đi ba lô được mô tả ở trên. Người đọc chắc chắn sẽ bị lạc trong một lúc nào
đó, và sẽ rất đau. Nhưng khi kết thúc chuyến đi, người đọc sẽ thấy khỏe khoắn, vui vẻ và sẵn sàng
cho những cuộc phiêu lưu nhiều hơn nữa.
Và anh ấy hoặc cô ấy sẽ học được rất nhiều về toán học - không phải là một ngành cụ thể của
toán học, mà là toán học, thuần túy và đơn giản. Thật vậy, một chủ đề lặp đi lặp lại xuyên suốt
cuốn sách là sự thống nhất của toán học. Nhiều meth ods giải quyết vấn đề cụ thể liên quan đến ý
tưởng lặp lại từ nhánh toán học này sang nhánh toán học khác; ví dụ, một giải thích hình học của
một bất đẳng thức đại số.

Dạy bằng cuốn sách này

Trong một khóa học kéo dài một học kỳ, hầu như tất cả Phần I nên được học, mặc dù không phải tất
cả chúng sẽ được học thành thạo. Ngoài ra, giảng viên có thể chọn các phần đã chọn từ Phần II. Ví

dụ: một khóa học ở cấp độ sinh viên năm nhất hoặc năm hai có thể tập trung vào Chương 1-6, trong
khi các lớp nâng cao hơn sẽ bỏ qua phần lớn Chương 5 (ngoại trừ phần cuối cùng) và Chương 6, thay
vào đó tập trung vào Chương 7 và 8.
Cuốn sách này dành cho sinh viên mới bắt đầu và tôi cũng không cho rằng người hướng dẫn là
chuyên gia. Sổ tay Tài nguyên dành cho Người hướng dẫn chứa các bản phác thảo giải pháp cho hầu
hết các vấn đề cũng như một số ý tưởng về cách giảng dạy một khóa học giải quyết vấn đề. Để biết
thêm thông tin, vui lòng truy cập www. wi ley. com / col lege / zeitz.

Sự nhìn nhận

Deborah Hughes Hallet đã là thiên thần hộ mệnh trong sự nghiệp của tôi trong gần hai mươi năm. Nếu
không có lòng tốt và sự khuyến khích của cô ấy, cuốn sách này sẽ không tồn tại, tôi cũng không thể
trở thành một giáo viên dạy toán. Tôi nợ bạn, Deb. Thanks!
Tôi đã có may mắn được làm việc tại Đại học San Francisco, nơi tôi được bao quanh bởi các
đồng nghiệp và nhân viên thân thiện và hỗ trợ, những sinh viên yêu thích học tập và các quản trị
viên luôn cố gắng giúp đỡ giảng viên. Đặc biệt, tôi muốn chọn ra một vài người để cảm ơn chân thành:

• Trưởng khoa của tôi, Stanley Nel, đã giúp đỡ tôi một cách hào phóng theo những cách cụ
thể, với việc nâng cấp com puter và tài trợ đi lại. Nhưng quan trọng hơn, anh ấy đã quan
tâm tích cực đến công việc của tôi ngay từ những ngày đầu tiên. Sự nhiệt tình của anh ấy
và bí quyết mà anh ấy hỗ trợ cho những nỗ lực của tôi đã giúp tôi tiếp tục phát triển trong
bốn năm qua.
Machine Translated by Google
xiii

• Tristan Needham đã là người cố vấn, đồng nghiệp và là bạn của tôi kể từ khi tôi đến USF năm 1992.
Tôi không bao giờ có thể hoàn thành cuốn sách này nếu không có lời khuyên và sự chăm chỉ của anh

ấy thay cho tôi. Trí tuệ của Tristan trải dài từ IbTp nhỏ nhất (chi tiết đến những hiểu biết sâu

sắc về lịch sử và nền tảng của toán học. Theo nhiều cách mà tôi vẫn chỉ mới bắt đầu hiểu, Tristan

đã dạy tôi hiểu ý nghĩa của việc thực sự hiểu một chân lý toán học).

• Nancy Campagna, Marvella Luey, Tonya Miller và Laleh Shahideh đã giúp tôi giải quyết
các vấn đề hành chính rất nhiều lần và theo nhiều cách đến mức tôi không biết bắt
đầu từ đâu. Chỉ cần nói rằng nếu không có sự giúp đỡ và tình bạn của họ, cuộc sống
của tôi tại USF thường sẽ trở nên nghiệt ngã và hỗn loạn.

• Không một ngày nào trôi qua mà không có Wing Ng, thư ký bộ phận đa tài của chúng tôi,
giúp tôi giải quyết các vấn đề liên quan đến những thứ như máy photocopy nhập sai
cách cài đặt phần mềm cho bố cục trang. Sự khéo léo và lòng vị tha của cô ấy đã nâng
cao năng suất làm việc của tôi.

Nhiều ý tưởng cho cuốn sách này xuất phát từ kinh nghiệm của tôi khi dạy học sinh trong hai lĩnh vực

khác nhau rất nhiều: một hội thảo giải quyết vấn đề tại USF và chương trình đào tạo cho đội Hoa Kỳ cho IMO.

Tôi cảm ơn tất cả các học sinh của tôi đã cho tôi cơ hội để chia sẻ toán học.

Các đồng nghiệp của tôi trong các cuộc thi toán học trên thế giới đã dạy tôi rất nhiều về cách giải

quyết vấn đề. Đặc biệt, tôi muốn cảm ơn Titu Andreescu, Jeremy Bem, Doug Jun greis, Kiran Kedlaya, Jim

Propp và Alexander Soifer về nhiều cuộc trò chuyện hữu ích.

Bob Bekes, John Chuchel, Dennis DeTurk, Tim Sipka, Robert Stolarsky, Agnes Tuska và Graeme West đã xem

xét các phiên bản trước của cuốn sách này. Họ đã đưa ra nhiều nhận xét hữu ích và tìm thấy nhiều sai sót.

Cuốn sách được cải thiện nhiều vì họ đọc cẩn thận. Bất cứ sai sót nào còn lại, tôi tất nhiên xin chịu mọi

trách nhiệm.

Cuốn sách này được viết trên máy tính Macintosh, sử dụng IbTp (chạy trên chương trình Textures tuyệt

vời, đi trước hàng km so với bất kỳ hệ thống TP nào khác. Tôi khuyến khích bất kỳ ai đang dự định viết

sách bằng TP (hoặc IbTp) hãy xem xét chương trình này (www. bluesky.com). Một phần mềm khác đã giúp tôi rất

nhiều là chương trình lập chỉ mục của Eric Scheide, chương trình này tự động hóa phần lớn IbTp (quy trình

lập chỉ mục. Chương trình của anh ấy dễ dàng giúp tôi tiết kiệm được một tuần. Liên hệ scheide@usfca.edu để

biết thêm thông tin.

Ruth Baruth, biên tập viên của tôi tại Wiley, đã giúp tôi biến một ý tưởng mơ hồ thành sách trong một

thời gian ngắn đáng ngạc nhiên, bằng cách kết hợp một cách thành thạo những lời động viên hào phóng, những

gợi ý sáng tạo và sự thúc đẩy nhẹ nhàng. Tôi xin chân thành cảm ơn sự giúp đỡ của cô ấy, và mong sẽ có
nhiều cuốn sách hơn nữa trong tương lai.

Vợ và con trai tôi đã phải chịu đựng rất nhiều trong quá trình viết cuốn sách này. Đây không phải là

nơi để tôi cảm ơn họ vì sự kiên nhẫn của họ, mà để xin lỗi vì sự bỏ bê của tôi. Đúng là tôi đã có thể hoàn

thành nhiều việc hơn và làm công việc mà tôi đã làm với ít cảm giác tội lỗi hơn, nếu tôi không bị gia đình

yêu cầu về thời gian của mình. Nhưng nếu không có gia đình tôi, chẳng có gì - thậm chí là vẻ đẹp của toán

học - sẽ chẳng có ý nghĩa gì cả.

Paul Zeitz San Francisco, tháng 11 năm 1998


Machine Translated by Google
Machine Translated by Google

Nội dung

Chương 1 Cuốn sách này nói về điều gì và cách đọc cuốn sách 1 1 .1
"Bài tập" so với "vấn đề" 1 1.2 Ba cấp độ của giải quyết

vấn đề 3 1.3 Một bộ lấy mẫu vấn đề 6 1.4 Cách đọc cuốn
sách này 11

Chương 2 Các chiến lược giúp tôi giải quyết vấn đề 13 2.1
Các chiến lược tâm lý 14 Sự bền bỉ về tinh thần:
Học hỏi từ chú chuột Polya 14 Sự sáng tạo 17

2.2 Các chiến lược để bắt đầu 25 Bước đầu tiên:


Định hướng 25 Tôi là người có định hướng.
Giờ thì sao? 26

2.3 Phương pháp lập luận 39 Viết tắt


phổ biến và quy ước phong cách 40 Suy giảm và lôgic

tượng trưng 41 Lập luận theo mâu thuẫn 41 Quy nạp toán

học 45

2.4 Các chiến lược quan trọng khác 52


Vẽ một bức tranh! 53

Hình ảnh không hữu ích? Khắc phục sự cố theo các cách khác! 54

Thay đổi quan điểm của bạn 58

Chương 3 Các chiến thuật để giải quyết vấn đề 61


3.1 Đối xứng 62 Đối xứng hình học 63
Đối xứng đại số 67 3.2 Nguyên lý
cực trị 73 3.3 Nguyên lý chuồng
chim 84

Chuồng bồ câu cơ bản 84

Chuồng bồ câu trung gian 86

Chuồng bồ câu nâng cao 87

xv
Machine Translated by Google

NỘI DUNG xvi

3.4 Bất biến 92


Chẵn lẻ 94

Số học và Tô màu mô-đun 100


Đơn biến số 102

Chương 4 Ba I mportant Crossover Tactics 109 4.1 Lý


thuyết đồ thị 109 Kết nối và chu kỳ 111 Đường
Eulerian và Hamilton 113

Hai người đàn ông của Tây Tạng 116

4.2 Số phức 120 Các phép toán


cơ bản 120 Nguồn gốc
của sự thống nhất 126
Một số ứng dụng 127 4.3
Tạo hàm 132 Ví dụ giới thiệu 133
Quan hệ lặp lại 134

Phân vùng 136

Chương 5 Đại số 143 5.1


Bộ, Số và Hàm 143 Bộ 143

Chức năng 145

5.2 Thao tác đại số được xem lại 147 Chiến


thuật nhân tố 148
Thao tác bình phương 149

Thay thế và đơn giản hóa 150

5.3 Tổng và Sản phẩm 156


Kí hiệu 156

Chuỗi số học 157

Chuỗi hình học và Công cụ kính thiên văn 158


Dòng Infinite 160

5,4 Đa thức 164 Phép


toán đa thức 164 Các
Zeros của một đa thức 165
5,5 Bất đẳng thức 173
Ldeas cơ bản 174

Bất bình đẳng AM-GM 176

Xoa bóp, Cauchy-Schwarz, và Chebyshev 181


Machine Translated by Google
NỘI DUNG xvii

Chương 6 Tổ hợp 188 6.1 Giới

thiệu về phép đếm 188


Hoán vị và Kết hợp 188

Đối số kết hợp 191

Tam giác Pascal và Định lý nhị thức 192

Các chiến lược và chiến thuật đếm 195

6.2 Các phân vùng và các phân giác 196 Đếm

các tập con 196 Quản lý thông tin

199 Bi trong Urns và các mã hóa cổ

điển khác 202 6.3 Nguyên tắc bao gồm-loại trừ 207

Đếm phần bổ sung 207 PIE với các bộ 207

PIE với các chức năng chỉ báo 212

6.4 Lặp lại 214 Xếp gạch và

Fibonacci Tái phát 215 Tái diễn Catalan 217

Chương 7 Lý thuyết số 222 7.1 Số nguyên

tố và phép chia hết 222 Định lý cơ bản


của số học 222

GCD, LCM và Thuật toán phân chia 224 7.2 Tính

đồng dư 230 Có gì tốt về số nguyên tố? 231

Định lý nhỏ Fermat 232

7.3 Các hàm số lý thuyết 235

Số chia 235

Phi và Mu 236

7.4 Phương trình Diophantine 240


Chiến lược và chiến thuật chung 240

7.5 Các ví dụ hướng dẫn khác 247 Một đa thức có thể luôn

tạo ra số nguyên tố không? Chương 247 nếu bạn đếm

được, đó là số nguyên 249 Chứng minh tổ hợp của


Định lý nhỏ Fermat 249

Tổng của hai hình vuông 250

Chương 8 Hình học cho người Mỹ 256 8.1 Ba vấn đề

"Dễ" 256 8.2 Hình học sống sót I 258 Điểm,

Đường, Góc và Hình tam giác 259


Machine Translated by Google

xviii NỘI DUNG

Các đường song song 260

Hình tròn và góc 264 Hình

tròn và hình tam giác 266 8.3

Hình học sống sót II 270 Diện tích 270

Các tam giác đồng dạng

274 Lời giải cho ba bài toán "dễ" 275 8.4 Sức mạnh

của Hình học sơ cấp 282

Điểm concyclic 283

Khu vực, Cevians, và các dòng đồng thời 286

Các tam giác tương tự và các điểm thẳng hàng 289


Điểm bóng ma và dòng đồng thời 292

8.5 Biến đổi 296

Đối xứng được duyệt lại 296

Chuyển động và Vectơ cứng nhắc 298

Homothety 305
Đảo ngược 307

Chương 9 Giải tích 315 9.1

Định lý cơ bản của Giải tích 315 9.2 Hội tụ và liên tục

317 Hội tụ 318 Tính liên tục 323 Tính liên tục đồng nhất
324 9.3 Phân biệt và tích hợp 328 Vẽ gần đúng và
đường cong 328 Định lý giá trị trung bình 331

Một công cụ hữu ích 334

Tích hợp 336


Phép đối xứng và phép biến đổi 338
9,4 Chuỗi lũy thừa và Toán Eulerian 342

Đừng lo lắng! Dòng

Taylor 342 với phần còn lại 344


Toán học Eulerian 346

Vẻ đẹp, sự đơn giản và sự đối xứng: Nhiệm vụ cho một bức màn chuyển động 350

Tài liệu tham khảo và Đọc thêm 356

Mục lục 360


Machine Translated by Google

Chương 1

Cuốn sách này là gì và làm thế nào để


Đọc nó

1.1 "Bài tập" so với "Vấn đề"

Đây là một cuốn sách về giải quyết vấn đề toán học. Chúng tôi đưa ra ba giả định về bạn, độc giả
của chúng tôi:

• Bạn thích toán học.

• Bạn biết khá tốt môn toán trung học, và ít nhất đã bắt đầu nghiên cứu "toán học cao cấp" như giải

tích và đại số tuyến tính.

• Bạn muốn trở nên giỏi hơn trong việc giải quyết các vấn đề toán học.

Đầu tiên, một vấn đề là gì? Chúng tôi phân biệt giữa vấn đề và bài tập. Một bài tập là một
câu hỏi mà bạn biết cách giải quyết ngay lập tức. Bạn có làm đúng hay không phụ thuộc vào mức độ
thành thạo của bạn áp dụng các kỹ thuật cụ thể, nhưng bạn không cần phải phân vân kỹ thuật nào
để sử dụng. Ngược lại, một vấn đề đòi hỏi nhiều suy nghĩ và sự tháo vát trước khi tìm ra cách
tiếp cận phù hợp. Ví dụ, đây là một bài tập.

Ví dụ 1.1.1 Tính toán 54363 mà không cần máy tính bỏ túi.

Bạn không nghi ngờ gì về cách tiến hành - chỉ cần nhân lên một cách cẩn thận. Câu hỏi tiếp
theo là tinh tế hơn.

Ví dụ 1.1.2 Viết

Tôi Tôi Tôi 1


+ + + ... + 3 · 4
""}: 2 2. 3 99 . 100

dưới dạng một phân số trong điều kiện thấp nhất.

Thoạt nhìn, đây là một bài tập tẻ nhạt khác, vì bạn có thể cộng lại tất cả 99 thuật ngữ một cách
cẩn thận và hy vọng rằng bạn sẽ có câu trả lời đúng. Nhưng một cuộc điều tra nhỏ mang lại một
điều gì đó hấp dẫn. Thêm một số thuật ngữ đầu tiên và đơn giản hóa, chúng tôi phát hiện ra rằng

1 1 2
=
""}: 2+ 2. 3 3 "'

1
Machine Translated by Google
2 CHƯƠNG 1 SÁCH NÀY GIỚI THIỆU GÌ VÀ CÁCH ĐỌC

1 1 1 3

+ + =
1. 2 2. 3 3 .4 1
4 '

1 1 1
4

+ + + 1 · 2 2 · 3 3 · =
4 4 · 5 5 '

dẫn đến phỏng đoán rằng với tất cả các số nguyên dương n,

1 1 l Tôi N
- + - + - +. . . + n (n + l) 1 = -.
· 4 · 2 2 · 3 3 n + l

Vì vậy, bây giờ chúng ta phải đối mặt với một vấn đề: phỏng đoán này có đúng không, và nếu vậy, làm thế nào

để chúng ta chứng minh rằng nó là đúng? Nếu chúng ta có kinh nghiệm về những vấn đề như vậy, đây vẫn chỉ là

một bài tập đơn thuần, trong kỹ thuật quy nạp toán học (xem trang 45). Nhưng nếu chúng ta chưa có kinh

nghiệm thì đó là một vấn đề, không phải là một bài tập. Để giải quyết nó, chúng ta cần dành một chút thời

gian, thử các cách tiếp cận khác nhau. Vấn đề càng khó, chúng ta càng cần nhiều thời gian. Thường thì cách

tiếp cận đầu tiên không thành công. Đôi khi hàng tá cách tiếp cận đầu tiên không thành công!

Đây là một câu hỏi khác, "Vấn đề người kiểm tra dân số" nổi tiếng. Một vài người

có thể coi đây là một bài tập, nhưng đối với hầu hết, nó là một vấn đề.

Ví dụ 1. 1.3 Một nhân viên điều tra dân số gõ cửa và hỏi người phụ nữ bên trong
có bao nhiêu đứa con và chúng bao nhiêu tuổi.
"Tôi có ba cô con gái, tuổi của chúng là số nguyên, và tích của tuổi là 36" , người mẹ nói.

"Đó là không đủ thông tin," người tham gia điều tra dân số trả lời.

"Tôi sẽ cho bạn biết tổng số tuổi của họ, nhưng bạn vẫn còn bối rối."

"Tôi ước gì bạn nói cho tôi biết thêm điều gì đó."

"Được rồi, con gái lớn Annie của tôi thích chó."

Tuổi của ba cô con gái là bao nhiêu?

Sau lần đọc đầu tiên, điều đó dường như là không thể - không có đủ thông tin để xác định độ tuổi. Đó

là lý do tại sao nó là một vấn đề, và một điều thú vị, ở đó. (Câu trả lời có ở cuối chương này, trên trang

1 2, nếu bạn cảm thấy bối rối.)

Nếu Bài toán Người kiểm tra dân số quá dễ, hãy thử bài toán tiếp theo (xem trang 75 để biết cách giải

quyết):

Ví dụ 1.1.4 Tôi mời 10 cặp đôi đến dự tiệc tại nhà tôi. Tôi hỏi tất cả những người có mặt, kể cả vợ tôi, họ

đã bắt tay với bao nhiêu người. Hóa ra là tất cả mọi người đều thắc mắc - tôi không hề thắc mắc về bản thân

mình, tất nhiên - đã bắt tay với một số người khác. Nếu chúng ta giả sử rằng không có ai bắt tay đối tác của

mình, thì vợ tôi đã bắt tay với bao nhiêu người? (Tôi đã không tự hỏi mình bất kỳ câu hỏi nào.)

Một vấn đề hay là bí ẩn và thú vị. Nó là bí ẩn, bởi vì ban đầu bạn không biết làm thế nào để giải

quyết nó. Nếu nó không thú vị, bạn sẽ không nghĩ về nó nhiều. Tuy nhiên, nếu nó thú vị, bạn sẽ muốn dành

nhiều thời gian và nỗ lực để hiểu nó.

Cuốn sách này sẽ giúp bạn điều tra và giải quyết vấn đề. Nếu bạn là một người không có khả năng giải

quyết vấn đề, bạn thường có thể nhanh chóng bỏ cuộc. Điều này xảy ra cho một số
lý do.

• Bạn có thể không biết cách bắt đầu.


Machine Translated by Google

1 .2 BA CẤP ĐỘ CỦA P ROBLEM GIẢI QUYẾT 3

• Bạn có thể đạt được một số tiến bộ ban đầu, nhưng sau đó không thể tiến xa hơn.

• Bạn thử một vài cách, không có gì hiệu quả, vì vậy bạn bỏ cuộc.

Ngược lại, một người giải quyết vấn đề có kinh nghiệm hiếm khi gặp khó khăn trong việc bắt đầu giải
l
quyết vấn đề như thế nào. Anh ấy hoặc cô ấy tự tin thử một số cách tiếp cận để bắt đầu.

Điều này có thể không giải quyết được vấn đề, nhưng một số tiến bộ đã đạt được. Sau đó, các niques

công nghệ cụ thể hơn sẽ phát huy tác dụng. Cuối cùng, ít nhất một thời gian, sự cố đã được giải quyết.
Người giải quyết vấn đề có kinh nghiệm hoạt động trên ba cấp độ khác nhau:

Chiến lược: Các ý tưởng toán học và tâm lý để bắt đầu và theo đuổi vấn đề.

Chiến thuật: Các phương pháp toán học đa dạng hoạt động trong nhiều môi trường khác nhau.

Công cụ: Các kỹ thuật và "thủ thuật" tập trung hẹp cho các tình huống cụ thể.

1.2 Ba cấp độ giải quyết vấn đề

Một số nhánh của toán học có lịch sử rất dài, với nhiều ký hiệu và từ ngữ tiêu chuẩn. Giải quyết vấn
2
đề không phải là một trong số đó. Chúng tôi sử dụng thuật ngữ chiến lược,

chiến thuật và công cụ để biểu thị ba cấp độ giải quyết vấn đề khác nhau. Vì đây không phải là các
định nghĩa tiêu chuẩn, điều quan trọng là chúng ta phải hiểu chính xác ý nghĩa của chúng.

Phép tương tự về người leo núi Bạn

đang đứng ở chân núi, hy vọng có thể leo lên đỉnh. Chiến lược đầu tiên của bạn có thể là thực hiện

một số chuyến đi nhỏ đến nhiều đỉnh núi dễ dàng hơn gần đó, để quan sát ngọn núi mục tiêu từ các góc

độ khác nhau. Sau đó, bạn có thể cân nhắc một chiến lược tập trung hơn, có lẽ là thử leo núi qua một

rặng núi nhỏ. Bây giờ bắt đầu cân nhắc chiến thuật : làm thế nào để thực sự đạt được chiến lược đã

chọn. Ví dụ: giả sử chiến lược đó đề xuất leo lên sườn phía nam của đỉnh núi, nhưng có những cánh

đồng tuyết và sông trên đường đi của chúng ta. Các chiến thuật khác nhau là cần thiết để thương lượng
từng trở ngại này. Đối với cánh đồng tuyết, chiến thuật của chúng tôi có thể là di chuyển vào sáng

sớm, khi tuyết rơi cứng. Đối với dòng sông, chiến thuật của chúng tôi có thể là dò tìm các bờ để có

nơi vượt qua an toàn nhất. Cuối cùng, chúng tôi chuyển sang cấp độ tập trung chặt chẽ nhất, đó là

các công cụ: các kỹ thuật cụ thể để hoàn thành các nhiệm vụ chuyên biệt. Ví dụ, để băng qua cánh
đồng tuyết, chúng tôi có thể thiết lập một hệ thống dây thừng cụ thể để đảm bảo an toàn và đi bộ

bằng trục băng.

Việc vượt sông có thể yêu cầu cả nhóm phải cởi đồ từ thắt lưng trở xuống và nắm tay nhau để giữ thăng
bằng. Đây là tất cả các công cụ. Chúng rất cụ thể. Bạn sẽ không bao giờ tóm tắt rằng, "Để leo núi,
chúng ta phải cởi quần ra và nắm tay nhau", bởi vì đây là một thành phần nhỏ nhưng thiết yếu của

toàn bộ cuộc leo núi. Mặt khác, chiến lược gic và đôi khi các ý tưởng chiến thuật thường được mô tả

trong phần tóm tắt của bạn: "Chúng tôi quyết định đến đỉnh núi qua sườn núi phía nam và phải băng

qua một cánh đồng tuyết khó khăn và một con sông nguy hiểm để đến được sườn núi."

Tôi

Do đó, chúng tôi sẽ tránh việc xây dựng "anh ấy hoặc cô ấy" khó xử bằng cách xen kẽ giới tính trong các chương tiếp theo.

2 Trên thực tế, thậm chí không tồn tại một tên gọi tiêu chuẩn nào cho lý thuyết giải quyết vấn đề, mặc dù George P6lya và

những người khác đã cố gắng phổ biến thuật ngữ heuristics (xem ví dụ, [32]).
Machine Translated by Google
4 CHƯƠNG 1 SÁCH NÀY GIỚI THIỆU GÌ VÀ CÁCH ĐỌC

Khi leo núi, chúng ta có thể gặp chướng ngại vật. Một số trở ngại này rất dễ thương lượng, vì chúng

chỉ là bài tập (tất nhiên điều này phụ thuộc vào khả năng và kinh nghiệm của người leo núi). Nhưng một

chướng ngại vật có thể đưa ra một bài toán khó thu nhỏ, mà giải pháp của nó sẽ dọn đường cho toàn bộ hành

trình leo núi. Ví dụ, con đường dẫn đến tổng mit có thể dễ dàng đi bộ, ngoại trừ một đoạn băng dốc dài 1

foot. Các nhà leo núi gọi việc đàm phán trở ngại chính là bước di chuyển mấu chốt. Chúng tôi cũng sẽ sử

dụng thuật ngữ này cho các vấn đề toán học. Một bước đi mấu chốt có thể diễn ra ở cấp chiến lược, chiến

thuật hoặc công cụ; một số vấn đề có một số di chuyển mấu chốt; nhiều người không có.

Từ leo núi đến Toán học

Hãy tiếp cận các vấn đề toán học với những ý tưởng leo núi này. Khi đối mặt với một vấn đề, bạn không thể

giải quyết nó ngay lập tức, ngược lại, nó không phải là một vấn đề mà chỉ là một bài tập đơn thuần. Bạn

phải bắt đầu một quá trình điều tra. Điều này trong nghi vấn có thể có nhiều hình thức. Một phương pháp,

không có nghĩa là khủng khiếp, là chỉ thử ngẫu nhiên bất cứ thứ gì nảy ra trong đầu bạn. Nếu bạn có một

trí tưởng tượng phong phú, một kho phương pháp tốt và nhiều thời gian rảnh rỗi, cuối cùng bạn có thể giải

quyết được vấn đề rắc rối. Tuy nhiên, nếu bạn là người mới bắt đầu, tốt nhất nên trau dồi cách tiếp cận có

tổ chức hơn.

Đầu tiên, hãy suy nghĩ một cách chiến lược. Đừng cố gắng giải quyết vấn đề ngay lập tức mà hãy nghĩ về nó

ở mức độ ít tập trung hơn. Mục tiêu của tư duy chiến lược là đưa ra một kế hoạch có thể chỉ có nội dung

toán học, nhưng dẫn đến một phương án "cải tiến", không khác gì chiến lược của người leo núi, "Nếu chúng

ta đến sườn núi phía nam, có vẻ như chúng ta sẽ có thể lên đến đỉnh. "

Các chiến lược giúp chúng tôi bắt đầu và giúp chúng tôi tiếp tục. Nhưng chúng chỉ là những phác thảo

mơ hồ về công việc thực tế cần phải làm. Các nhiệm vụ cụ thể để hoàn thành các kế hoạch chiến lược của

chúng tôi được thực hiện ở cấp độ chiến thuật và công cụ thấp hơn.

Dưới đây là một ví dụ cho thấy ba cấp độ đang hoạt động, từ một người Hungary năm 1926
Cuộc thi.

Ví dụ 1.2. 1 Chứng minh rằng tích của bốn số tự nhiên liên tiếp không thể là bình
phương của một số nguyên.

Giải pháp: Chiến lược ban đầu của chúng tôi là làm quen với nhận định của vấn đề, tức là để có được

định hướng. Trước tiên, chúng tôi lưu ý rằng câu hỏi yêu cầu chúng tôi chứng minh điều gì đó. Vấn đề thường

có hai loại - những vấn đề yêu cầu bạn chứng minh điều gì đó và những vấn đề yêu cầu bạn tìm kiếm điều gì

đó. Bài toán Điều tra dân số-Taker (Ví dụ 1.1.3) là một ví dụ của loại sau.

Tiếp theo, hãy quan sát vấn đề đang yêu cầu chúng ta chứng minh rằng thứ gì đó không thể bẻ được cây

bút. Chúng tôi chia vấn đề thành giả thuyết (còn được gọi là "cái đã cho") và kết luận (bất kể vấn đề gì

yêu cầu bạn tìm hoặc chứng minh). Giả thuyết là:

Cho n he là một số tự nhiên.

Kết luận là:

n (n + 1) (n + 2) (n + 3) không thể là bình phương của một số nguyên.

Việc hình thành giả thuyết và kết luận không phải là chuyện tầm thường, vì nhiều vấn đề không thể hiện

chúng một cách chính xác. Trong trường hợp này, chúng tôi phải giới thiệu một số ký hiệu. Đôi khi của chúng tôi
Machine Translated by Google

1 .2 BA CẤP ĐỘ CỦA P ROBLEM GIẢI QUYẾT 5

sự lựa chọn ký hiệu có thể rất quan trọng.

Có lẽ chúng ta nên tập trung vào kết luận: làm thế nào để bạn chứng minh rằng một cái gì
đó không thể là hình vuông? Chiến lược này, cố gắng suy nghĩ về điều gì sẽ dẫn đến kết luận
của vấn đề một cách trung gian, được gọi là nhìn vào bước áp chót . hình vuông. Vì vậy, chúng
tôi thử một chiến lược khác, một trong những chiến lược tốt nhất để bắt đầu đối với bất kỳ
vấn đề nào: làm bẩn tay. Chúng tôi thử cắm một số con số để thử nghiệm. Nếu may mắn, chúng ta
có thể nhìn thấy một con nhạn biển. Hãy thử một vài giá trị khác nhau cho n. Đây là một bảng.
Chúng ta sử dụng cách viết tắt f (n) = n (n + 1) (n + 2) (n + 3).

I f (n) I 2! I 1 2� I 36� I 84� I 1 68� I 171! � I

Nhận thấy bất cứ điều gì? Vấn đề liên quan đến hình vuông, vì vậy chúng tôi rất nhạy
cảm để tìm kiếm hình vuông. Mọi người đều nhận thấy rằng hai giá trị đầu tiên của f (n) nhỏ
hơn một bình phương hoàn hảo. Kiểm tra nhanh xác minh thêm rằng,

f (3) = 1 92 - 1, f (4) = 292 - 1, f (5) = 4 12 - 1, f (1O) = 1312 - I.

Chúng tôi tự tin phỏng đoán rằng f (n) nhỏ hơn một bình phương hoàn hảo với mọi n.
Vì phỏng đoán này là bước áp chót mà chúng tôi đang tìm kiếm, bởi vì một số nguyên
dương nhỏ hơn một hình vuông hoàn hảo không thể là một hình vuông hoàn hảo vì dãy 1,4,
9, 1 6, ... của các hình vuông hoàn hảo chứa không có số nguyên liên tiếp (khoảng cách
giữa các ô vuông liên tiếp ngày càng lớn hơn). Chiến lược mới của chúng tôi là chứng
minh phỏng đoán.
Để làm như vậy, chúng tôi cần trợ giúp ở cấp chiến thuật / công cụ. Chúng ta muốn chứng
minh rằng với mỗi n, tích n (n + 1) (n + 2) (n + 3) nhỏ hơn một bình phương hoàn hảo. Nói cách
khác, n (n + 1) (n + 2) (n + 3) + 1 phải là một hình vuông hoàn hảo. Làm thế nào để chứng tỏ
rằng một biểu thức đại số luôn luôn bằng một hình vuông hoàn hảo? Một chiến thuật: yếu tố biểu hiện!
Chúng ta cần thao tác với biểu thức, luôn ghi nhớ mục tiêu của chúng ta là nhận được hình
vuông. Vì vậy, chúng tôi tập trung vào việc ghép các bộ phận gần giống nhau. Chú ý rằng tích
của n và n + 3 gần giống như tích của n + I và n + 2, trong đó hai số hạng đầu tiên của chúng
đều là n2 + 3n. Sau khi tập hợp lại, chúng tôi có

2
[n (n + 3)] [(n + l) (n + 2)] + 1 = (n2 + 3n) (n + 3n + 2) + I. (1)

Thay vì nhân hai thuật ngữ gần như giống hệt nhau, chúng tôi giới thiệu một chút symme cố
gắng đưa các hình vuông vào tiêu điểm:

2
(N
+ 3n) (n2 + 3n + 2) + 1 = ( (n2 + 3n + 1) - 1 ) ( ( n2 + 3n + 1) + 1 ) + 1.

Bây giờ chúng ta sử dụng phép tính thừa số "chênh lệch của hai bình phương" (một công cụ!) Và chúng ta có

((n 2 + 3n + 1) - 1 ) ((n2 + 3n + 1) + 1 ) + 1 = (n2 + 3n + 1) 2 - 1 + 1


2
= (n + 3n + 1) 2.

3Từ "áp chót" có nghĩa là "bên cạnh las !."


Machine Translated by Google
6 CHƯƠNG 1 SÁCH NÀY GIỚI THIỆU GÌ VÀ CÁCH XEM

Chúng ta đã chứng minh rằng f (n) nhỏ hơn một bình phương hoàn hảo với mọi số nguyên n, cụ thể là f (n)

= (n2 + 3n + 1) 2 - 1 ,

và chúng tôi đã hoàn thành.



Hãy cùng chúng tôi nhìn lại và phân tích vấn đề này theo ba cấp độ. Chiến lược đầu
tiên của chúng tôi là định hướng, đọc kỹ vấn đề và phân loại vấn đề một cách sơ khai. Sau
đó, chúng tôi quyết định một chiến lược để xem xét bước áp chót mà ban đầu không hiệu quả,
nhưng chiến lược thử nghiệm số đã dẫn đến một phỏng đoán. Suc chứng minh điều này một cách
thành công liên quan đến chiến thuật bao thanh toán, cùng với việc sử dụng đối xứng và công
cụ nhận dạng một thừa số hóa phổ biến.
Cấp độ quan trọng nhất là chiến lược. Bắt đầu phỏng đoán là bước đi mấu chốt. Tại thời
điểm này, vấn đề đã biến thành một bài tập! Vì ngay cả khi bạn không nắm chắc chiến thuật
tốt, bạn vẫn có thể vượt qua được. Một phương pháp tốt là thay thế: Cho u = n2 + 3n trong
phương trình (1). Khi đó vế phải trở thành u (u + 2) + 1 = u2 + 2u + 1 = (u + 1) 2. Một
phương pháp khác là mUltiply out (ugh!). chúng tôi

4
n (n + 1) (n + 2) (n + 3) + 1 = n + 6n3 + 11 n 2 + 6n + 1.

Nếu đây là bình phương của một thứ gì đó, nó sẽ là bình phương của đa thức bậc hai n2 + an + 1 hoặc n2 + an

- 1. Thử trường hợp đầu tiên, chúng ta tương đương + 6n + 1 = (n2 + an + 1 ) 2 = n

2 4
4 n + 6n3 + 11 n + 2an3 + (a2 + 2) n2 + 2an + 1

và chúng ta thấy rằng a = 3 hoạt động; tức là, n (n + l) (n + 2) (n + 3) + 1 = (n2 + 3n + 1) 2. Cách này

kém thanh lịch hơn một chút so với cách đầu tiên chúng tôi giải quyết vấn đề, nhưng nó là một phương pháp tốt.
Thật vậy, nó dạy chúng ta một công cụ hữu ích : phương pháp hệ số không xác định.

1.3 Trình lấy mẫu vấn đề

Các vấn đề trong cuốn sách này được phân loại thành ba nhóm lớn: giải trí, cuộc thi và kết
thúc mở. Trong mỗi gia đình, các bài toán chia thành hai loại cơ bản: bài toán "tìm" và bài
toán "chứng minh. ,, 4 Bài toán" tìm "yêu cầu một thông tin cụ thể, trong khi bài toán"
chứng minh "yêu cầu một lập luận tổng quát hơn. Đôi khi sự phân biệt bị mờ. Ví dụ, Ví dụ
1.1.4 ở trên là một bài toán "cần tìm", nhưng giải pháp của nó có thể liên quan đến một lập
luận rất chung chung.
Những gì sau đây là một mô tả mẫu mô tả của mỗi gia đình.

Các vấn đề giải trí

Còn được gọi là "những câu đố hóc búa", những vấn đề này thường ít liên quan đến các tài liệu toán học

chính thức, mà thay vào đó dựa vào việc sử dụng sáng tạo các nguyên tắc chiến lược cơ bản. Chúng rất

xuất sắc để làm việc, bởi vì không cần kiến thức đặc biệt và bất kỳ thời gian nào đã dành để suy nghĩ về một

4Hai điều khoản này là do George P61ya [32].


Machine Translated by Google

1 .3 MẪU VẤN ĐỀ 7

Bài toán giải trí sẽ giúp bạn sau này với các lỗi xác suất phức tạp hơn về mặt toán
học. Bài toán Điều tra dân số-Taker (Ví dụ 1.1.3) là một ví dụ điển hình về bài toán
giải trí. Một mỏ vàng cho các bài toán giải trí xuất sắc là công trình của Martin Gard
ner, người đã biên tập bộ phận "Trò chơi Toán học" của tạp chí Scientific American trong
nhiều năm. Nhiều bài báo của ông đã được sưu tầm thành sách. Hai trong số những cái đẹp
nhất có lẽ là [12] và [1 1].

1.3.1 Một nhà sư leo núi. Anh ấy bắt đầu lúc 8 giờ sáng và đến đỉnh vào buổi trưa.

Anh ấy dành cả đêm trên đỉnh núi. Sáng hôm sau, anh ta rời đỉnh lúc 8 giờ sáng và đi
xuống bằng cùng một tuyến đường mà anh ta đã sử dụng vào ngày hôm trước, đến đáy vào
buổi trưa. Chứng minh rằng có một khoảng thời gian từ 8 giờ sáng đến giữa trưa mà nhà

sư đã ở đúng một điểm trên núi vào cả hai ngày. (Lưu ý rằng chúng tôi không nói rõ bất
cứ điều gì về tốc độ mà nhà sư đi. Ví dụ, anh ta có thể chạy với tốc độ lOOO dặm / giờ
trong vài phút đầu tiên, sau đó ngồi yên trong nhiều giờ, sau đó đi lùi lại, v.v. Nhà
sư cũng không để di chuyển với cùng tốc độ đi lên khi đi xuống.)

1.3.2 Bạn đang ở sảnh dưới của một ngôi nhà. Có ba công tắc, tất cả đều ở vị trí "tắt".
Ở tầng trên, có một căn phòng có bóng đèn đang tắt. Một và chỉ một trong ba công tắc
điều khiển bóng đèn. Bạn muốn khám phá công tắc nào điều khiển bóng đèn, nhưng bạn chỉ
được phép lên cầu thang một lần. Bạn làm như thế nào?
(Không được phép sử dụng dây, kính thiên văn, v.v.

1.3.3 Bạn rời khỏi ngôi nhà của mình, đi một dặm về phía nam, sau đó đi một dặm về phía
đông, sau đó đi một dặm về phía bắc. Bây giờ bạn đã trở lại ngôi nhà của bạn! Ban song o
dau? Có nhiều hơn một giải pháp; tìm càng nhiều càng tốt.

Vấn đề cuộc thi

Các bài toán này được viết cho các kỳ thi chính thức có giới hạn thời gian, thường đòi
hỏi các công cụ ized đặc biệt và / hoặc sự khéo léo để giải. Một số kỳ thi ở cấp trung
học và đại học liên quan đến toán học phức tạp và thú vị.

Kỳ thi Toán trung học Hoa Kỳ (AHSME) Được thực hiện bởi hàng trăm học sinh
trung học phổ thông tự chọn mỗi năm, bài kiểm tra trắc nghiệm này có các câu hỏi
tương tự như các bài toán khó và thú vị nhất trong Kỳ thi Toán học kỳ vọng Hoa

Kỳ SAT.5 (AIM E) Hơn 2000 người đạt điểm cao nhất trong AHSME đủ điều kiện cho
bài kiểm tra câu hỏi I S kéo dài ba giờ này. Cả AHSME và AI ME đều có các vấn đề
"cần tìm" vì các bài kiểm tra này được chấm điểm bằng máy.

Olympic Toán học Hoa Kỳ (USAMO) 150 thí sinh AIME hàng đầu tham gia vào kỳ thi
luận năm câu hỏi kéo dài 3 tiếng rưỡi đồng hồ ưu tú này, gồm các bài toán chủ
yếu là thử thách "để chứng minh."

Liên đoàn Toán học Khu vực Hoa Kỳ (ARML) Hàng năm, ARML tổ chức một cuộc thi
quốc gia giữa các đội học sinh trung học trong khu vực. Một vài

5 Hiện tại, kỳ thi này đã được thay thế bằng các kỳ thi AMC-8, AMC-IO và AMC-1 2 , cho các cấp lớp mục
tiêu khác nhau.
Machine Translated by Google
8 CHƯƠNG 1 GIỚI THIỆU SÁCH LÀ GÌ VÀ CÁCH XEM

trong số các bài toán khá khó và thú vị, gần tương đương với các câu hỏi khó hơn trong
AHSME và AIME và các bài toán USAMO dễ hơn.

Olympic quốc gia và khu vực khác Nhiều quốc gia khác tiến hành các cuộc thi giải quyết
vấn đề đình đám khác nhau. Đông Âu nói riêng có một truyền thống thi cử rất phong phú,
bao gồm các cuộc thi cấp thành phố rất thú vị, chẳng hạn như Olympic Toán học Leningrad.6
Gần đây Trung Quốc và Việt Nam đã triển khai các kỳ thi rất sáng tạo và đầy thử thách.

Olympic Toán học Quốc tế (IMO) Những người đạt điểm cao nhất của USAMO được mời tham gia
một chương trình đào tạo, sau đó sẽ chọn ra đội Hoa Kỳ gồm sáu thành viên tham gia cuộc
thi quốc tế này. Đây là một kỳ thi viết luận dài chín giờ, sáu câu hỏi, trải dài trong
hai ngày.1 IMO bắt đầu vào năm 1959, và diễn ra ở một quốc gia khác nhau mỗi năm. Ban
đầu, đây là một sự kiện nhỏ chỉ giới hạn ở các quốc gia của Bức Màn Sắt, nhưng gần đây
sự kiện này đã trở nên khá toàn diện, với 75 quốc gia đại diện trong 1996.

Kỳ thi Putnam Cuộc thi giải quyết vấn đề quan trọng nhất dành cho sinh viên đại học Hoa
Kỳ, một kỳ thi kéo dài 1 2 câu hỏi, kéo dài 6 giờ được thực hiện bởi vài nghìn stu
dents vào tháng 12 hàng năm. Điểm trung bình thường bằng không.

Vấn đề trên tạp chí Một số tạp chí toán học có bộ phận giải quyết vấn đề, trong đó độc
giả được mời đề xuất vấn đề và / hoặc gửi thư theo cách này. Các giải pháp thú vị nhất
được công bố, cùng với danh sách những người đã giải quyết được vấn đề. Một số vấn đề
trong số này có thể cực kỳ khó khăn và nhiều vấn đề vẫn chưa được giải quyết trong
nhiều năm. Các tạp chí có các bộ phận giải quyết vấn đề tốt, theo thứ tự độ khó tăng
dần, là Math Horizons, Tạp chí Toán học Đại học, Tạp chí Toán học, và The American Toán
học hàng tháng.
Tất cả những điều này đều được xuất bản bởi Hiệp hội Toán học Hoa Kỳ. Ngoài ra còn có
một tạp chí hoàn toàn dành cho các vấn đề thú vị và giải quyết vấn đề, Crux
Mathematicorum, được xuất bản bởi Hiệp hội Toán học Canada.

Các vấn đề của cuộc thi là rất khó. Đó là một thành tựu đáng kể để giải quyết một vấn đề
như vậy, ngay cả khi không có giới hạn thời gian. Các mẫu dưới đây bao gồm các bài toán thuộc
mọi cấp độ khó.

1.3.4 (AHSME 1 996) Trong mặt phẳng xy, độ dài của đường đi ngắn nhất từ (0,0) đến (12, 1 6)
không đi vào bên trong đường tròn (x - 6) 2 + ( y - 8) 2 = 25?

1.3.5 (AHSME 1 996) Cho rằng x2 + y2 = 14x + 6y + 6, giá trị lớn nhất mà 3x + 4y có thể có là
bao nhiêu?

1.3.6 (AHSME 1 994) Khi tung n con xúc xắc sáu mặt tiêu chuẩn, xác suất thu được tổng 1 994 lớn
hơn 0 và bằng xác suất thu được tổng S. Giá trị nhỏ nhất có thể là bao nhiêu. giá trị của S?

6 Olympic Toán học Leningrad được đổi tên thành SI. Olympic Thành phố Petersberg vào giữa những năm 1 990.

7 Bắt đầu từ năm 1996, USAMO đã áp dụng một hình thức tương tự: sáu câu hỏi, được thực hiện trong hai phiên họp buổi
sáng và buổi chiều kéo dài ba giờ.
Machine Translated by Google

1 .3 MẪU VẤN ĐỀ 9

1.3.7 (AIME 1 994) Tìm số nguyên dương n mà

llog2 1j + llog2 2j + llog2 3j +. . . + llog2 nj = 1 994,

trong đó lx j biểu thị số nguyên lớn nhất nhỏ hơn hoặc bằng x. (Ví dụ: l n j = 3.)

1.3.8 (AIME 1 994) Với bất kỳ dãy số thực A = (a 1, là dãy (a2 - ai, a3 a2 ,a3 , . . . ), định
- a2 ,a4 - a3 , nghĩa LlA ...) có tenn thứ n là một + 1 - an. Cho rằng
rằng tất cả các phần mười của dãy Ll (LlA) là 1 và rằng a 19 = a94 = O. Tìm a I.

1.3.9 (USAMO 1 989) 20 thành viên của một câu lạc bộ quần vợt địa phương đã lên lịch
chính xác 14 trận đấu dành cho hai người với nhau, với mỗi thành viên chơi ít nhất một
trận. Chứng minh rằng trong lịch trình này phải có một bộ sáu trò chơi với 12 người
chơi khác nhau.

1.3. 10 (USAMO 1 995) Máy tính bị hỏng để các phím duy nhất vẫn hoạt động là sin, cos,
tan, sin-I , cos-tôi , bất kỳ số hữu nút.
Tôi

tỉ dương
Ban đầu
q nào,
màn hãy
hìnhchứng
hiển tỏ
thịrằng
và tan
việcO.nhấn
Với

một dãy hữu hạn các nút sẽ thu được q. Giả sử rằng máy tính thực hiện các phép tính số thực với độ chính

xác vô hạn. Tất cả các hàm đều tính bằng ten radian.

1.3.11 (Russia 1 995) Giải phương trình

cos (cos (cos (cosx))) = sin (sin (sin (sinx))).

1.3.12 (IMO 1 976) Detennine, với bằng chứng, số lớn nhất là tích của các số nguyên
dương có tổng là 1 976.

1.3.13 (Putnam 1 978) Gọi A là một tập hợp bất kỳ gồm 20 số nguyên phân biệt được chọn từ cấp số cộng

1,4, 7 có tổng là 1 04. , ... , 100. Chứng minh rằng trong A phải tồn tại hai số nguyên phân biệt

1.3.14 (Putnam 1 994) Cho (an) là một dãy các số thực dương sao cho với mọi n, an : S a2n + a2n

+ I · Chứng minh rằng : L: = 1 phân kỳ.

1.3.15 (Putnam 1 994) Tìm giá trị dương của m sao cho diện tích trong phần tư thứ nhất được bao bởi hình
2
elip x 2/9 + y = 1, trục x và đường thẳng y = 2x13 bằng = 1, trục y và
2
diện tích trong góc phần tư đầu tiên được bao bởi hình elip x 2/9 + y
đường thẳng y = mx.

1.3. 16 (Putnam 1 990) Hãy xem xét một cú đấm giấy có thể được căn giữa tại bất kỳ
điểm nào của mặt phẳng và khi được vận hành, nó sẽ loại bỏ chính xác những điểm có
khoảng cách từ tâm của nó ra khỏi mặt phẳng là không hợp lý. Cần bao nhiêu cú đấm để
loại bỏ mọi điểm?

Các vấn đề kết thúc mở

Đây là những câu hỏi toán học đôi khi được diễn đạt một cách mơ hồ và có thể không có lời giải thực tế

(không giống như hai dạng bài toán được mô tả ở trên). Các vấn đề kết thúc mở có thể rất thú vị để giải

quyết, bởi vì bạn không biết kết quả sẽ như thế nào. Một vấn đề kết thúc mở tốt giống như một cuộc đi bộ

đường dài (hoặc thám hiểm!) Trong một vùng chưa được khám phá. Thường thì các giải pháp từng phần là tất

cả những gì bạn có thể nhận được. (Tất nhiên, một phần


Machine Translated by Google

10 CHƯƠNG 1 SÁCH NÀY GIỚI THIỆU GÌ VÀ CÁCH ĐỌC

các giải pháp luôn ổn, ngay cả khi bạn biết rằng vấn đề bạn đang giải quyết là một vấn đề của cuộc
thi chính thức có một giải pháp hoàn chỉnh.)

1.3.17 Đây là một vài hàng đầu tiên của Tam giác Pascal.

2
3 3
4 4
6
S S
10 10
TÔI,

trong đó các phần tử của mỗi hàng là tổng của các cặp phần tử liền kề của hàng trước đó. Ví dụ: 10
= 4 + 6. Hàng tiếp theo trong tam giác sẽ là

1, 6, IS, 20, IS, 6, 1.

Có rất nhiều mẫu thú vị trong Tam giác Pascal. Khám phá càng nhiều chim nhạn biển và các mối

quan hệ càng tốt, và chứng minh càng nhiều càng tốt. Đặc biệt, bằng cách nào đó bạn có thể trích

xuất các số Fibonacci (xem bài toán tiếp theo) từ Trian gle của Pascal (hoặc ngược lại) không? Một
câu hỏi khác: có một mẫu hoặc quy tắc nào cho tính chẵn lẻ (chẵn hoặc lẻ) của các phần tử của Tam

giác Pascal không?

1.3.18 Các số Fibonacci Trong được xác định bởi 10 = 0, II = 1 và In = In-I + In-2 Ví
quanh với .
dãy
dụ,số
h =
này;
1, 13
cố =
gắng
2,14
khám
= 3,
phá
15càng
= S,nhiều
16 = 8,
mẫuhcàng
= 13,
tốt
Isvà
= 21.
cố gắng
Chơichứng
với n>
minh
1 xung
những phỏng đoán của bạn tốt nhất có thể. Đặc biệt, hãy nhìn vào thực tế đáng kinh ngạc
này: với n 2: 0,
"

f "� � {( I +2 2
v'5 ) - ( 1 - v'5 )}

Bạn sẽ có thể chứng minh điều này bằng quy nạp toán học (xem trang 4S-S0 và Bài toán 2.3.37), nhưng

câu hỏi thú vị hơn là, công thức này đến từ đâu? Hãy nghĩ về điều này và những điều khác xuất hiện

khi bạn nghiên cứu dãy Fibonacci.

1.3. 19 Chữ "ell" là một viên gạch hình chữ L được ghép từ ba hình vuông 1 x I, như hình dưới đây.

Eb
Với các số nguyên dương a, b nào thì có thể gạch hoàn toàn một hình chữ nhật ax b chỉ sử dụng các

ells? ("Xếp gạch" có nghĩa là chúng tôi bao phủ chính xác hình chữ nhật bằng các ô, không có ô
thừa.) Ví dụ, rõ ràng là bạn có thể xếp hình chữ nhật 2 x 3 bằng các ô, nhưng (vẽ hình) bạn không

thể xếp ô 3 x 3 với ells. Sau khi bạn hiểu hình chữ nhật, hãy khái quát
Machine Translated by Google
1 .4 CÁCH ĐỌC SÁCH NÀY 11

theo hai hướng: hình elip có hình dạng phức tạp hơn, hình dạng ốp bằng những thứ khác với hình dạng khác
với hình ells.

1.3.20 Hãy tưởng tượng một hình chữ nhật dài I x L , với L là một số nguyên. Rõ ràng, người ta có thể đóng

gói hình chữ nhật này với L hình tròn đường kính I, và không hơn thế nữa. (Theo "pack", chúng tôi có nghĩa

là chạm vào là được, nhưng chồng chéo thì không.) Mặt khác, không rõ ràng ngay lập tức rằng hình tròn 2L là

số lượng tối đa có thể đóng gói một hình chữ nhật 2 x L. Kiểm tra điều này và tổng quát hóa thành hình chữ

nhật m x L.

1.4 Cách đọc cuốn sách này

Cuốn sách này không phải để đọc từ đầu đến cuối, mà là để đọc một cách "phi tuyến tính". Cuốn sách được

biên soạn nhằm giúp bạn nghiên cứu hai chủ đề: phương pháp giải quyết vấn đề và các ý tưởng toán học cụ

thể. Bạn sẽ dần dần học toán nhiều hơn và cũng trở nên thành thạo hơn trong "giải quyết vấn đề", và sự tiến

bộ trong lĩnh vực này sẽ kích thích thành công trong lĩnh vực kia.

Cuốn sách được chia thành hai phần, với một chương "cây cầu" ở giữa. Chương 1-3 giới thiệu tổng quan

về các chiến lược và chiến thuật. Mỗi chiến lược hoặc chiến thuật được thảo luận trong một phần bắt đầu

bằng các ví dụ đơn giản nhưng kết thúc bằng các vấn đề phức tạp.

Tại một số điểm, bạn có thể thấy rằng văn bản khó hiểu hơn, vì nó đòi hỏi nhiều kinh nghiệm toán học hơn.

Bạn nên đọc kỹ phần đầu của mỗi phần, nhưng sau đó bắt đầu đọc lướt (hoặc bỏ qua) vì nó ngày càng khó hơn.

Bạn có thể (và nên) đọc lại sau.

Các chương 5-9 được dành cho các ý tưởng toán học ở cấp độ chiến thuật hoặc công cụ, được chủ đề toán

học hóa và phát triển đặc biệt theo quan điểm của người giải quyết vấn đề. Tùy thuộc vào sở thích và nền

tảng của bạn, bạn sẽ đọc tất cả hoặc chỉ một số chương này.

Chương 4 là cầu nối giữa việc giải quyết vấn đề chung và các chủ đề toán học cụ thể. Nó xem xét chi

tiết ba chiến thuật "giao nhau" quan trọng kết nối các nhánh khác nhau của toán học. Một số tài liệu trong

chương này khá nâng cao, nhưng chúng tôi đưa nó vào đầu sách để cung cấp cho người đọc một lộ trình nhanh

chóng đến những ý tưởng phức tạp có thể được áp dụng rất rộng rãi.

Khi bạn nâng cao kiến thức toán học của mình (từ Chương 5-9), bạn có thể muốn quay lại các chương

trước đó để đọc lại các phần mà bạn có thể đã đọc lướt trước đó. Ngược lại, khi bạn nâng cao kỹ năng giải

quyết vấn đề của mình từ những chương đầu, bạn có thể đọc lại (hoặc đọc lần đầu tiên) một số chương sau đó.

Lý tưởng nhất là bạn sẽ đọc mỗi trang của cuốn sách này ít nhất hai lần, và đọc, nếu chưa giải quyết được

mọi vấn đề trong đó.

Xuyên suốt cuốn sách, các thuật ngữ mới và tên chiến lược, chiến thuật và công cụ cụ thể có trong

in đậm. Theo thời gian,

Khi một điểm quan trọng được thực hiện, nó được thụt vào và in nghiêng.
như thế này.

Điều đó có nghĩa là, "chú ý!" Để biểu thị sự hoàn thành thành công của một giải pháp, chúng tôi
Machine Translated by Google
12 CHƯƠNG 1 SÁCH NÀY GIỚI THIỆU GÌ VÀ CÁCH XEM

số 8

sử dụng biểu tượng "Halmos", một hình vuông được điền đầy đủ thông tin.
Chúng tôi đã sử dụng Halmos khi kết thúc Bài kiểm tra

xin 1.2.1 trên trang 6, và dòng này kết thúc bằng một.
_

Vui lòng đọc bằng bút chì và giấy bên cạnh bạn và / hoặc viết vào lề! Tin học toán học có nghĩa là để học tập một cách tích

cực. Ngoài ra - điều này đòi hỏi bạn phải hết sức kiềm chế - hãy cố gắng giải từng ví dụ khi bạn đọc nó, trước khi đọc lời giải trong

văn bản. Ít nhất, hãy dành một vài phút để suy ngẫm về vấn đề. Đừng để bị lôi cuốn vào việc xem xét ngay giải pháp. Bạn càng tích cực

tiếp cận tài liệu trong cuốn sách này, bạn càng nhanh chóng nắm vững nó. Và bạn sẽ có nhiều niềm vui hơn.

Tất nhiên, một số vấn đề được trình bày khó hơn các vấn đề khác. Ở cuối mỗi phần (hoặc tiểu mục), chúng ta có thể thảo luận về

một vấn đề "cổ điển", một vấn đề thường quá khó để người đọc mới bắt đầu có thể giải quyết một mình trong một khoảng thời gian hợp lý.

Những tác phẩm kinh điển này được bao gồm vì một số lý do: chúng minh họa những ý tưởng quan trọng; chúng là một phần của cái mà chúng

tôi coi là "kho" cần thiết cho mọi nhà toán học trẻ; và, quan trọng nhất, chúng là những tác phẩm nghệ thuật tuyệt đẹp, cần được suy

ngẫm và thưởng thức. Cuốn sách này có tên là Nghệ thuật và Thủ công của Giải quyết Vấn đề, và trong khi chúng tôi dành nhiều trang hơn

cho khía cạnh thủ công của việc giải quyết vấn đề, chúng tôi không muốn bạn quên rằng giải quyết vấn đề, tốt nhất, là một nỗ lực đầy

đam mê và thẩm mỹ. Nếu bạn sẽ làm chúng tôi thích thú với một phép loại suy khác, hãy giả vờ rằng bạn đang học cách ứng tấu piano jazz.

Điều quan trọng là bạn phải luyện tập các thang âm và tự ứng biến, nhưng bạn cũng cần sự hướng dẫn và nguồn cảm hứng đến từ việc nghe

một số bản thu âm tuyệt vời.

Giải pháp cho vấn đề người điều tra dân số

Tích của độ tuổi là 36, vì vậy chỉ có một vài bộ ba độ tuổi có thể xảy ra. Dưới đây là một bảng của tất cả các khả năng, với tổng của

các độ tuổi dưới mỗi bộ ba.

Aha! Bây giờ chúng ta thấy những gì đang xảy ra. Câu nói thứ hai của người mẹ ("Tôi muốn cho bạn biết tổng số tuổi của họ, nhưng bạn

vẫn còn bối rối") cung cấp cho chúng ta thông tin quý giá. Nó cho chúng ta biết rằng tuổi là (1, 6,6) hoặc (2,2, 9), trong tất cả các

trường hợp khác, kiến thức về tổng sẽ cho chúng ta biết rõ ràng tuổi là bao nhiêu! Đầu mối cuối cùng bây giờ có ý nghĩa; nó cho chúng

ta biết rằng có một người con gái lớn nhất, loại bỏ bộ ba (1, 6,6). Các cô con gái đã 2, 2 và 9 tuổi.

8 Được theo đuổi Paul Halmos. một nhà toán học và nhà văn đã phổ biến việc sử dụng nó.
Machine Translated by Google

Chương 2

Các chiến lược điều tra vấn đề

Như chúng ta đã thấy, giải quyết một vấn đề không giống như leo núi. Và đối với những người leo
núi thiếu kinh nghiệm, nhiệm vụ này có vẻ khó khăn. Núi dốc quá! Không có dấu vết! Bạn thậm chí
không thể nhìn thấy hội nghị thượng đỉnh! Nếu là ngọn núi đáng để leo, nó sẽ cần nỗ lực, kỹ năng
và có lẽ là cả may mắn. Có thể cần một số nỗ lực hủy bỏ (thường được gọi là "chuyến đi do thám")
trước khi hội nghị thượng đỉnh diễn ra.
Tương tự như vậy, một bài toán hay, thú vị và đáng giải, sẽ không tự giải được. Bạn phải
dành nhiều nỗ lực để khám phá sự kết hợp của các chiến thuật toán học phù hợp với các chiến lược
thích hợp. "Chiến lược" thường phi toán học. Một số chiến lược giải quyết vấn đề sẽ hoạt động
trên nhiều loại vấn đề, không chỉ toán học
những cái.

Đặc biệt, đối với người mới bắt đầu, chiến lược là rất quan trọng. Khi đối mặt với một vấn
đề mới và dường như khó khăn, bạn thường không biết bắt đầu từ đâu. Các chiến lược tâm lý có thể
giúp bạn đi đúng tâm trí. Các chiến lược khác giúp bạn bắt đầu quá trình điều tra. Khi bạn đã
bắt đầu công việc, bạn có thể cần một khuôn khổ chiến lược tổng thể để tiếp tục và hoàn thành
giải pháp của mình.
Chúng tôi bắt đầu với các chiến lược tâm lý áp dụng cho hầu hết mọi vấn đề. Đây là những ý
tưởng đơn giản thông thường. Điều đó không có nghĩa là chúng dễ dàng để làm chủ. Nhưng một khi
bạn bắt đầu nghĩ về chúng, bạn sẽ nhận thấy sự cải thiện nhanh chóng trong khả năng giải các bài
toán. Lưu ý rằng chúng tôi không hứa hẹn cải tiến trong việc giải quyết vấn đề. Điều đó sẽ đến
với thời gian. Nhưng trước hết bạn phải học cách làm việc thực sự .
Sau các chiến lược tâm lý, chúng tôi xem xét một số chiến lược giúp bạn bắt đầu điều tra.
Đây cũng là những ý tưởng rất đơn giản, dễ áp dụng và thường thú vị. Ban đầu, chúng có thể không
giúp bạn giải quyết nhiều vấn đề, nhưng chúng sẽ giúp bạn đạt được những tiến bộ đáng khích lệ.

Giải pháp cho mọi vấn đề bao gồm hai phần: cuộc điều tra, trong đó bạn khám phá những gì
đang xảy ra và lập luận, trong đó bạn thuyết phục người khác về khám phá của mình. Chúng tôi
thảo luận về phương pháp phổ biến nhất trong số nhiều phương pháp lập luận chính thức được đề
cập trong chương này. Chúng tôi kết thúc bằng một nghiên cứu về các chiến lược linh tinh có thể
được sử dụng ở các giai đoạn khác nhau của một cuộc điều tra toán học.

13
Machine Translated by Google

14 CHƯƠNG 2 CHIẾN LƯỢC ĐỐI VỚI CÁC VẤN ĐỀ ĐIỀU TRA VẤN ĐỀ

2.1 Chiến lược tâm lý

Những người giải quyết vấn đề hiệu quả nổi bật giữa đám đông. Bộ não của họ dường như hoạt động khác

nhau. Chúng cứng hơn, nhưng cũng nhạy cảm và linh hoạt hơn. Rất ít người sở hữu những thuộc tính đáng

khen ngợi này, nhưng rất dễ dàng để bắt đầu có được chúng.

Sự dẻo dai về tinh thần: Học hỏi từ Chuột của Polya

Chúng tôi sẽ tóm tắt các ý tưởng của chúng tôi bằng một câu chuyện nhỏ, "Chuột và đàn ông," được kể bởi

George P6lya, nhà toán học vĩ đại và là giáo viên giải quyết vấn đề ([33], trang 75).

Bà chủ nhà vội vã vào sân sau, đặt cái bẫy chuột xuống đất (là loại bẫy kiểu cũ,

lồng có cửa sập) và gọi con gái đến bắt con mèo. Con chuột trong bẫy dường như không

hiểu ý chính của những thủ tục này; nó chạy điên cuồng trong lồng, ném mình dữ dội

vào song sắt, lúc này bên này rồi bên kia, và trong giây phút cuối cùng nó đã thành

công lách mình qua và biến mất trong cánh đồng của người hàng xóm. Chắc hẳn ở phía đó

có một khe hở hơi rộng hơn giữa các thanh của tôi, tôi thầm chúc mừng con chuột. Anh

ấy đã giải được một cái bẫy chuột tuyệt vời. . . vấn đề và đưa ra một ví dụ tuyệt vời.

Đó là cách giải quyết vấn đề. Chúng tôi phải thử và thử lại cho đến khi cuối

cùng chúng tôi nhận ra sự khác biệt nhỏ giữa các lỗ mở khác nhau mà mọi thứ phụ thuộc

vào. Chúng ta phải thay đổi các thử nghiệm của mình để có thể khám phá tất cả các mặt

của vấn đề. Thật vậy, chúng ta không thể biết trước bên nào là khe hở duy nhất có thể

thực hiện được mà chúng ta có thể lách qua.

Phương pháp cơ bản của chuột và đàn ông là giống nhau: thử, thử lại, và thay đổi

các thử nghiệm để chúng ta không bỏ lỡ một vài khả năng thuận lợi. Đúng là đàn ông

thường giỏi giải quyết vấn đề hơn chuột. Một người đàn ông không cần phải ném mình

vào vật cản, anh ta có thể làm như vậy về mặt tinh thần; một người có thể thay đổi

các thử nghiệm của mình nhiều hơn và học được nhiều điều hơn từ thất bại trong các
thử nghiệm của mình hơn là một con chuột.

Tất nhiên, đạo đức của câu chuyện là một người giải quyết vấn đề tốt sẽ không từ bỏ.

Tuy nhiên, cô ấy không chỉ tiếp tục đập đầu vào tường (hoặc lồng!) Một cách ngu ngốc, mà thay vào đó,

thay đổi mỗi lần thử. Nhưng điều này quá đơn giản. Nếu mọi người không bao giờ từ bỏ các vấn đề, thế

giới sẽ là một nơi rất kỳ lạ và khó chịu. Đôi khi bạn không thể giải quyết một vấn đề. Bạn sẽ phải từ

bỏ, ít nhất là tạm thời. Tất cả những người giải quyết vấn đề giỏi đôi khi thừa nhận thất bại. Một phần

quan trọng trong nghệ thuật của người giải quyết vấn đề là biết khi nào nên từ bỏ.

Nhưng hầu hết những người mới bắt đầu đều bỏ cuộc quá sớm, bởi vì họ thiếu các thuộc tính dẻo dai

về tinh thần là sự tự tin và tập trung. Thật khó để giải quyết một vấn đề nếu bạn không tin rằng bạn có

thể giải quyết được nó và không thể tiếp tục làm việc vượt qua "ngưỡng thất vọng" của bạn. Người mới học

phải cải thiện sự dẻo dai về tinh thần của mình song song với các kỹ năng toán học để đạt được tiến bộ

đáng kể.
Machine Translated by Google

2.1 CHIẾN LƯỢC ICAL TÂM LÝ I ES 15

Không khó để có được một mức độ dẻo dai tinh thần khiêm tốn. Khi mới bắt đầu, bạn rất có thể thiếu tự

tin và sức mạnh tập trung, nhưng bạn có thể tăng cả hai điều này đồng thời. Bạn có thể nghĩ rằng xây dựng

sự tự tin là một điều khó khăn và thiếu tế nhị, nhưng chúng ta không nói ở đây về lòng tự trọng, tình dục

hay bất cứ điều gì sâu xa trong tâm hồn bạn. Các vấn đề toán học dễ giải quyết hơn. Bạn đã khá tự tin về

khả năng toán học của mình nếu không bạn sẽ không đọc nó. Bạn xây dựng sự tự tin sẵn có của mình bằng cách

giải quyết các vấn đề "dễ" lúc đầu, trong đó "dễ" có nghĩa là bạn có thể giải quyết nó sau khi bỏ ra một

nỗ lực khiêm tốn. Miễn là bạn tập trung vào các vấn đề hơn là các bài tập, bộ não của bạn sẽ được tập luyện

và tiềm thức của bạn sẽ quen với thành công. Sự tự tin của bạn tự động tăng lên.

Khi sự tự tin của bạn tăng lên, ngưỡng thất vọng của bạn cũng sẽ tăng lên, nếu bạn tăng dần “tải” trí

tuệ. Bắt đầu với những vấn đề dễ dàng, để khởi động, nhưng sau đó làm việc với những vấn đề khó hơn và khó

hơn liên tục thách thức và kéo bạn đến giới hạn. Miễn là các vấn đề còn đủ thú vị, bạn sẽ không ngại làm

việc lâu hơn và lâu hơn về chúng. Lúc đầu, bạn có thể sẽ thất bại sau 15 phút suy nghĩ căng thẳng.

Cuối cùng, bạn sẽ có thể làm việc hàng giờ đồng hồ chỉ để giải quyết một vấn đề và giữ cho các vấn đề khác

âm ỉ trong tâm trí bạn trong nhiều ngày hoặc nhiều tuần.

Thats tất cả để có nó. Có một bí quyết: phát triển sự dẻo dai về tinh thần cần có thời gian và duy

trì nó là nhiệm vụ cả đời. Nhưng còn gì vui hơn là nghĩ về những vấn đề khó khăn càng thường xuyên càng

tốt?

Đây là một vấn đề đơn giản và thú vị, thực sự được sử dụng trong một cuộc phỏng vấn việc làm phần mềm,
1
điều đó cho thấy tầm quan trọng của sự tự tin trong việc tiếp cận những điều chưa biết.

Ví dụ 2.1.1 Hãy xem xét sơ đồ sau. Bạn có thể kết nối mỗi hộp nhỏ ở trên cùng với chữ cái giống nhau của

nó ở dưới cùng bằng các đường dẫn không cắt nhau, cũng như không rời khỏi ranh giới của hộp lớn không?

Giải pháp: Làm thế nào để tiến hành? Có thể hoặc không. Nhân sự của công ty phần mềm ở đây khá xảo quyệt;

họ muốn xem ai đó sẽ bỏ cuộc nhanh như thế nào. Chắc chắn, nó không có vẻ khả thi. Mặt khác, sự tự tin

quyết định rằng

Chỉ bởi vì một vấn đề dường như không thể không có nghĩa là nó là khả thi.
Không bao giờ thừa nhận thất bại sau một cái nhìn lướt qua. Hãy bắt đầu một cách
lạc quan; cho rằng vấn đề có thể được giải quyết. Chỉ sau một số lần thất bại tại

Tôi

Chúng tôi cảm ơn Denise Hunter đã cho chúng tôi biết về vấn đề này.
Machine Translated by Google
16 CHƯƠNG 2 CHIẾN LƯỢC ĐỐI VỚI CÁC VẤN ĐỀ ĐIỀU TRA VẤN ĐỀ

cám dỗ bạn nên cố gắng chứng minh là không thể. Nếu bạn không thể làm như
vậy, thì đừng thừa nhận thất bại. Quay lại vấn đề sau.

Bây giờ chúng ta hãy cố gắng giải quyết vấn đề. Sẽ rất hữu ích nếu bạn cố gắng nới lỏng
và không lo lắng về các quy tắc hoặc ràng buộc. Ước mơ luôn vui vẻ và thường hữu ích. Ví dụ,
trong bài toán này, khó khăn chính là các hộp trên cùng có nhãn A và C nằm ở những vị trí
"sai". Vậy tại sao không di chuyển chúng xung quanh để làm cho vấn đề trở nên dễ dàng một
cách tầm thường? Xem sơ đồ tiếp theo.

Chúng tôi đã áp dụng chiến lược hết sức quan trọng là làm cho nó trở nên dễ dàng hơn :

Nếu vấn đề đã cho quá khó, hãy giải một bài toán dễ hơn.

Tất nhiên, chúng tôi vẫn chưa giải quyết được vấn đề ban đầu. Hay có chúng tôi? Chúng ta
có thể cố gắng "đẩy" các hộp nổi trở lại vị trí ban đầu của chúng, từng hộp một. Đầu tiên
hộp A :

Bây giờ là hộp C,

và đột nhiên vấn đề được giải quyết!



Machine Translated by Google

2.1 CÁC CHIẾN LƯỢC ICAL CỦA TÂM LÝ 17

Tất nhiên, có một đạo lý trong câu chuyện. Hầu hết mọi người, khi đối mặt với vấn đề này, ngay lập tức

tuyên bố rằng nó là không thể. Những người giải quyết vấn đề tốt thì không, bao giờ. Hãy nhớ rằng, không có

áp lực về thời gian. Có thể cảm thấy tốt khi nhanh chóng "xử lý" một vấn đề, bằng cách giải quyết nó hoặc

tuyên bố nó là không thể giải quyết được, nhưng tốt hơn hết là bạn nên dành thời gian để hiểu một vấn đề.

Tránh tuyên bố ngay lập tức về sự bất khả thi; họ không trung thực.

Chúng tôi đã giải quyết vấn đề này bằng cách sử dụng hai nguyên tắc chiến lược. Đầu tiên, chúng tôi sử

dụng chiến lược chọn lọc tâm hồn để nuôi dưỡng một thái độ lạc quan, cởi mở. Thứ hai, chúng tôi đã sử dụng

chiến lược thú vị để giải quyết vấn đề dễ dàng hơn. Chúng tôi đã may mắn, vì hóa ra vấn đề ban đầu gần như

ngay lập tức tương đương với phiên bản dễ sửa đổi hơn. Điều đó xảy ra vì một lý do toán học: vấn đề là một

"tôpô". Thủ thuật biến đổi một sơ đồ thành một sơ đồ "tương đương về mặt cấu trúc liên kết" rất đáng ghi nhớ.

Nó không phải là một chiến lược, mà là một công cụ, theo ngôn ngữ của chúng tôi.

Sự sáng tạo

Hầu hết các nhà toán học là "nhà theo thuyết Platon", tin rằng toàn bộ chủ đề của họ đã sẵn sàng "tồn tại"

và nhiệm vụ của các nhà nghiên cứu con người là "khám phá" ra nó, thay vì tạo ra nó. Đối với người theo chủ

nghĩa Platon, giải quyết vấn đề là nghệ thuật nhìn thấy giải pháp đã sẵn sàng ở đó. Do đó, người giải quyết

vấn đề giỏi rất cởi mở và dễ tiếp thu những ý tưởng đang trôi nổi trong tầm nhìn đơn giản, nhưng lại vô hình

đối với hầu hết mọi người.

Khả năng tiếp thu những ý tưởng mới khó nắm bắt này được chúng tôi gọi là sự sáng tạo. Quan sát nó trong

hành động giống như xem một buổi biểu diễn ảo thuật, nơi những điều kỳ diệu xảy ra theo những cách đáng ngạc

nhiên, khó giải thích. Đây là một ví dụ về một bài toán đơn giản với một giải pháp đáng yêu, chưa được giải

đáp, một bài toán đã xuất hiện trước đó là Bài toán 1 .3. 1 ở trang 7. Hãy suy
khinghĩ
đọc về
lờivấn đề một chút trước
giải!

Ví dụ 2.1.2 Một nhà sư leo núi. Anh ấy bắt đầu lúc 8 giờ sáng và đến đỉnh vào buổi trưa. Anh ấy dành cả đêm

trên đỉnh núi. Sáng hôm sau, anh ta rời đỉnh lúc 8 giờ sáng và đi xuống bằng cùng một tuyến đường mà anh ta

đã sử dụng vào ngày hôm trước, đến đáy vào buổi trưa. Chứng minh rằng có một khoảng thời gian từ 8 giờ sáng
đến giữa trưa mà nhà sư đã ở đúng một điểm trên núi vào cả hai ngày. (Lưu ý rằng chúng tôi không nói rõ bất

cứ điều gì về tốc độ mà nhà sư đi. Ví dụ, anh ta có thể đua với tốc độ 1000 dặm / giờ trong vài phút đầu tiên,

sau đó ngồi yên trong nhiều giờ, sau đó đi lùi lại, v.v. Nhà sư cũng không có. để di chuyển với cùng tốc độ

đi lên khi đi xuống.)

Giải pháp: Hãy để nhà sư leo lên núi bằng mọi cách. Ngay lúc anh ta bắt đầu đi xuống vào sáng hôm sau,

có một nhà sư khác bắt đầu đi bộ đường dài từ phía dưới, du hành giống hệt như nhà sư đầu tiên đã làm vào

ngày hôm trước. Đến một lúc nào đó, hai nhà sư sẽ gặp nhau trên con trai l. Đó là thời gian và địa điểm chúng

tôi muốn!
_

Điều phi thường của giải pháp này là sự sáng suốt bất ngờ khi phát minh ra một nhà sư thứ hai. Ý tưởng

dường như đến từ hư không, nhưng nó giải quyết vấn đề ngay lập tức, theo một cách rất dễ chịu. (Xem trang 53

để có giải pháp "thông thường" hơn cho vấn đề này.)

Đó là sự sáng tạo trong hành động. Phản ứng tự nhiên khi nhìn thấy một giải pháp có nguồn gốc từ tưởng

tượng, tuyệt vời như vậy là nói, "Chà! Làm thế nào mà cô ấy nghĩ ra điều đó? Tôi không bao giờ có thể làm được
Machine Translated by Google
18 CHƯƠNG 2 CHIẾN LƯỢC I ES ĐỐI VỚI CÁC VẤN ĐỀ ĐIỀU CHỈNH

"Đôi khi, trên thực tế, việc nhìn thấy một giải pháp sáng tạo có thể gây ức chế, vì mặc dù
ngưỡng mộ nó, chúng ta có thể không nghĩ rằng chúng ta có thể tự mình làm được. Mặc dù đúng là
một số người dường như tự nhiên hơn sáng tạo hơn những người khác. , "nhưng thay vào đó hãy
nghĩ," Ý tưởng hay! Nó tương tự như những cái tôi đã có. Hãy bắt tay vào công việc! "

Học cách thích hợp một cách hổ thẹn những ý tưởng mới và biến chúng thành của riêng bạn.

Không có gì sai với điều đó; những ý tưởng không được cấp bằng sáng chế. Nếu chúng là những ý
tưởng hay, bạn nên hào hứng làm chủ chúng và sử dụng chúng thường xuyên nhất có thể, và cố gắng
kéo dài chúng đến giới hạn bằng cách áp dụng chúng theo những cách mới lạ. Luôn chú ý đến những
ý tưởng mới. Mỗi vấn đề mới mà bạn gặp phải cần được phân tích về nội dung "ý tưởng mới lạ" của
nó. Bạn càng quen với việc chiếm đoạt và vận dụng các ý tưởng, bạn càng có thể nảy ra nhiều ý
tưởng mới của riêng mình.
Một cách để nâng cao khả năng tiếp thu của bạn đối với những ý tưởng mới là luôn "thả
lỏng", trau dồi một loại tầm nhìn ngoại vi về mặt tinh thần. Các tế bào thụ cảm trong võng mạc
của con người tập trung dày đặc nhất ở gần trung tâm, nhưng các thụ thể nhạy cảm nhất lại nằm ở
ngoại vi. Điều này có nghĩa là vào một ngày sáng sủa, bất cứ thứ gì bạn nhìn vào bạn đều có thể
nhìn thấy rất rõ. Tuy nhiên, nếu trời tối, bạn sẽ không thể nhìn thấy những vật mà bạn nhìn
trực tiếp, nhưng bạn sẽ cảm nhận được, mặc dù mờ ảo, những vật ở ngoại vi của trường thị giác
của bạn (thử Bài tập 2. 1.1 0). Tương tự như vậy, khi bạn bắt đầu một cuộc điều tra giải quyết
vấn đề, bạn đang ở trong bóng tối. Nhìn thẳng vào mọi thứ sẽ không giúp ích được gì. Bạn cần
thư giãn tầm nhìn và lấy ý tưởng từ ngoại vi. Giống như con chuột của P6lya, hãy liên tục đề
phòng những khúc quanh co và những mánh khóe. Đừng bị khóa vào một phương pháp. Cố gắng phá vỡ
hoặc bẻ cong các quy tắc một cách có ý thức.

Dưới đây là một vài ví dụ đơn giản, nhiều trong số đó là kinh điển cũ. Như mọi khi, đừng
nhảy ngay đến giải pháp. Cố gắng giải quyết hoặc ít nhất là suy nghĩ về từng vấn đề trước!

Bây giờ là thời điểm tốt để gấp một nửa tờ giấy hoặc lấy một thẻ mục lục lớn để ẩn các

giải pháp để bạn không bị cám dỗ và đọc chúng trước khi bạn suy nghĩ về vấn đề!

Ví dụ 2. 1.3 Nối tất cả chín điểm dưới đây bằng một đường thẳng gồm bốn đoạn thẳng không bị
đứt đoạn.

Giải pháp: Vấn đề này là không thể, trừ khi bạn giải phóng mình khỏi ranh giới nghệ thuật
của chín điểm. Khi bạn quyết định vẽ các đường kéo dài qua đoạn này
Machine Translated by Google

2.1 CÁC CHIẾN LƯỢC ICAL CỦA TÂM LÝ 19

ranh giới, nó là khá dễ dàng. Để dòng đầu tiên nối ba điểm và đảm bảo rằng mỗi dòng mới
nối thêm hai điểm.

-
Ví dụ 2.1.4 Pat muốn mang một thanh kiếm dài 5,5 mét lên tàu hỏa, nhưng người dẫn
đường không cho phép nó làm hành lý xách tay. Và hành lý sẽ không lấy bất kỳ vật phẩm
nào có kích thước lớn nhất vượt quá 1 mét. Pat phải làm gì?

Giải pháp: Điều này là không thể giải quyết được nếu chúng ta giới hạn bản thân trong không gian hai

chiều. Sau khi được giải phóng khỏi Flatland, chúng tôi nhận được một giải pháp hay: Thanh kiếm nằm gọn trong

một chiếc hộp 1 x 1 x I -meter , với đường chéo dài là VI 2 + 12 + 12 = J3 > 1,69 mét. _

Ví dụ 2.1.5 Chữ cái tiếp theo trong dãy 0, T, T, F, F, S, S, E là gì. . . ?

Giải pháp: Dãy là danh sách các chữ cái đầu tiên của các chữ số một, hai, ba,
bốn, ...; câu trả lời là "N" cho "chín". _

Ví dụ 2.1.6 Điền vào cột tiếp theo của bảng.

1 9 11 18 19 17 24 34 29
13 27 55
3 2 6 2 15 21 23 30
3 1 5 3 7 12 5 8 13 21

Giải pháp: Cố gắng tìm ra bảng này từng hàng một là khá khó khăn.
Các giá trị tăng, giảm, lặp lại, v.v., không có mẫu rõ ràng. Nhưng ai nói rằng các mẫu
phải được xếp thành hàng? Nếu bạn sử dụng tầm nhìn ngoại vi để quét toàn bộ bảng , bạn
sẽ nhận thấy một số con số quen thuộc. Ví dụ, có rất nhiều bội số của ba. Trên thực tế,
một số bội số đầu tiên của ba, theo thứ tự, được ẩn trong bảng.

1 3 2 11 18 13 15 8 19 27
6 3 1 9 2 13 21 21 55 17 24 34 29 23 30
5 3 7 12 5

Và một khi chúng ta thấy rằng các mô hình là đường chéo, chúng ta sẽ dễ dàng phát hiện ra một dãy số khác,

các số nguyên tố!

1 3 9 2 6 11 18 13 19 27 55 15 8 17 24 34
2 3 1 5 12 3 7 29 13 21 21 23 30 5
Machine Translated by Google

20 CHƯƠNG 2 CÁC CHIẾN LƯỢC CHO I CÁC VẤN ĐỀ NÂNG CAO

Dãy còn lại tất nhiên là các số Fibonacci (Bài toán 1 .3.18).

Vì vậy, cột tiếp theo của bảng là 31, 33, 89.



Ví dụ 2.1.7 Tìm thành viên tiếp theo trong chuỗi này.

1 , 11, 2 1 , 121 1, 1 1 1 221, ...

Giải pháp: Nếu bạn diễn giải các phần tử của dãy số dưới dạng số lượng, dường như
không có mẫu rõ ràng. Nhưng ai đã nói rằng chúng là những con số? Nếu bạn nhìn vào
mối quan hệ giữa một phần tử và phần tử tiền nhiệm của nó và tập trung vào nội dung
"tượng trưng", chúng ta sẽ thấy một khuôn mẫu. Mỗi phần tử "mô tả" phần tử trước đó.
Ví dụ, phần tử thứ ba là 21, có thể được mô tả là "một 2 và một 1", tức là
, phần
121 1,
tử là
thứ
tư. Điều này có thể được mô tả là "một 1, một 2 và hai tôi", tức là, 1 1 1 221.
Vì vậy, thành viên tiếp theo là 3122 1 1 ("ba 1 s, hai 2 và một 1").

Ví dụ 2.1.8 Ba phụ nữ đăng ký vào một phòng trọ có quảng cáo mức giá 27 đô la một đêm.
Mỗi người đưa 10 đô la cho người khuân vác, và yêu cầu cô ấy mang về ba tờ đô la.
Người khuân vác quay trở lại bàn làm việc, nơi cô biết rằng căn phòng thực sự chỉ có 25 đô la một đêm. Cô

đưa 25 đô la cho nhân viên bàn của nhà nghỉ, trả phòng và trả lại cho khách mỗi người một đô la, quyết định

không cho họ biết về mức giá thực tế. Như vậy người khuân vác đã bỏ túi 2 đô la, trong khi mỗi người khách

đã tiêu 10 - 1 = 9 đô la, tổng cộng 2 + 3 x 9 = 29 đô la.

Điều gì đã xảy ra với đồng đô la khác?

Giải pháp: Vấn đề này là cố tình đánh lừa người đọc nghĩ rằng lợi nhuận mà người khuân vác kiếm được

cộng với số tiền mà khách chi tiêu sẽ lên đến $ 30. Ví dụ, hãy thử kéo dài mọi thứ một chút: điều gì sẽ xảy

ra nếu giá phòng thực tế là $ O? Sau đó, người khuân vác sẽ bỏ túi 27 đô la và khách sẽ chi 27 đô la, con

số này tăng lên đến 54 đô la! "Bất biến" thực tế ở đây không phải là 30 đô la, mà là 27 đô la, số tiền mà

khách chi tiêu và số tiền này sẽ luôn bằng số tiền mà người khuân vác đã lấy (2 đô la) cộng với số tiền đã

đến bàn (25 đô la).


Mỗi ví dụ đều có một chủ đề chung: Đừng để sự áp đặt bản thân, những hạn chế không cần thiết hạn chế

suy nghĩ của bạn. Bất cứ khi nào bạn gặp một vấn đề, bạn nên dành một phút (hoặc hơn) để đặt câu hỏi, "Tôi

có đang áp đặt các quy tắc mà tôi không cần thiết phải làm không? Tôi có thể thay đổi hoặc bẻ cong các quy

tắc để có lợi cho mình không?"

Những người tử tế có thể về đích cuối cùng hoặc có thể không, nhưng

Những bé trai, bé gái ngoan, biết nghe lời sẽ giải quyết được ít vấn đề hơn những đứa trẻ
nghịch ngợm và nghịch ngợm.

Phá vỡ hoặc ít nhất là bẻ cong một vài quy tắc. Nó sẽ không làm hại ai cả, bạn sẽ vui vẻ và bạn sẽ bắt đầu

giải quyết những vấn đề mới.

Chúng tôi kết thúc phần này với "Vấn đề hành động khẳng định" đáng yêu, được đặt ra ban đầu (ở một

hình thức khác) bởi Donald Newman. Trong khi về mặt toán học nhiều sophisti hơn

2Chúng tôi cảm ơn Derek Vadala đã lưu ý chúng tôi vấn đề này. Nó xuất hiện trong [421. P. 277.
Machine Translated by Google

2.1 CÁC CHIẾN LƯỢC ICAL CỦA TÂM LÝ 21

so với bài toán về nhà sư, nó cũng sở hữu một giải pháp "một trong những" rất ngắn gọn và giàu
trí tưởng tượng. Giải pháp mà chúng tôi trình bày là do Jim Propp.

Ví dụ 2.1.9 Xét một mạng lưới gồm rất nhiều quả bóng, một số quả bóng được nối với nhau bằng
dây. Chúng ta sẽ tô màu các quả bóng là đen và trắng, và gọi một mạng là "tích hợp" nếu mỗi quả
bóng trắng có ít nhất nhiều đen bằng các hàng xóm trắng và ngược lại. Ví dụ dưới đây cho thấy
hai chất tạo màu khác nhau của cùng một mạng. Bóng bên trái không được tích hợp, vì bóng a có
hai hàng xóm trắng (c, d) và chỉ một hàng xóm đen (b). Mạng bên phải được tích hợp.

b e ---- {) 8

C <l--->;) d

Với bất kỳ mạng nào, có màu sắc nào tích hợp nó không?

Giải pháp: Câu trả lời là "có." Chúng ta hãy gọi một sợi dây là "cân bằng" nếu nó nối hai
quả bóng có màu khác nhau. Ví dụ, dây nối a và b trong mạng đầu tiên ở trên là cân bằng, trong
khi dây nối a và c thì không. Sau đó, giải pháp một dòng của chúng tôi là

Tối đa hóa các dây cân bằng!

Bây giờ chúng ta cần giải thích giải pháp thông minh của chúng ta! Xem xét tất cả các chất
tạo màu khác nhau có thể có của một mạng nhất định. Có rất nhiều chất tạo màu, vì vậy phải có
một chất tạo màu (có thể nhiều hơn một) để tạo ra số lượng dây cân bằng tối đa. Chúng tôi khẳng
định rằng màu này được tích hợp . Ngược lại, giả định rằng nó không được tích hợp. Sau đó, phải
có một quả bóng nào đó, gọi là A, có màu trắng (không làm mất đi tính đồng bộ của giới tính)
màu trắng, có nhiều hàng xóm da trắng hơn các hàng xóm da đen. Nhìn vào dây dẫn phát ra từ A.
Dây cân bằng duy nhất là dây nối A với các quả cầu màu đen. Nhiều dây hơn phát ra từ A không
cân bằng hơn là cân bằng. Tuy nhiên, nếu chúng ta tô lại màu đen cho A , thì nhiều dây sẽ được
cân bằng hơn là không cân bằng.

Vì việc đổi màu A chỉ ảnh hưởng đến các dây phát ra từ A, chúng tôi đã chỉ ra rằng việc đổi màu
A dẫn đến màu sắc với các dây cân bằng hơn trước. Điều đó đánh lừa giả định của chúng tôi rằng
màu của chúng tôi đã tối đa hóa số lượng dây cân bằng!

Tóm lại, chúng tôi đã chỉ ra rằng nếu một màu không được tích hợp, thì nó không thể tối đa
hóa dây cân bằng. Vì vậy, một màu tối đa hóa dây cân bằng phải được tích hợp! _

Những ý tưởng mới trong giải pháp này là gì? Điều đó phụ thuộc vào kinh nghiệm của bạn,
tất nhiên, nhưng chúng tôi chắc chắn có thể cô lập động thái mấu chốt tuyệt vời: ý tưởng tối đa
hóa số lượng dây cân bằng. Ý tưởng cơ bản, nguyên tắc cực đoan, thực sự là một chiến thuật "dân
gian" phổ biến được sử dụng bởi những người giải quyết vấn đề có kinh nghiệm (xem Phần 3.2 bên
dưới). Lúc đầu, nhìn thấy nguyên tắc cực đoan trong hành động giống như xem một
Machine Translated by Google

22 CHƯƠNG 2 CÁC CHIẾN LƯỢC CHO I CÁC VẤN ĐỀ NỔI BẬT

Chuyên gia karate phá ván bằng sức mạnh dường như không cần nỗ lực. Nhưng một khi bạn thành thạo nó để

sử dụng cho riêng mình, bạn sẽ phát hiện ra rằng việc phá vỡ ít nhất một số bảng không phải là quá khó.

Một đặc điểm đáng chú ý khác của giải pháp này là việc sử dụng khéo léo lập luận bằng mâu thuẫn. Một lần

nữa, đây là một phương pháp chứng minh khá tiêu chuẩn (xem Phần 2.3 bên dưới).

Điều này không có nghĩa là giải pháp của Jim Propp không thông minh. Thật vậy, nó là một trong

những lập luận gọn gàng nhất mà chúng tôi từng thấy. Nhưng một phần sức hấp dẫn của nó là sự đơn giản

trong các thành phần của nó, giống như origami, nơi một hình vuông chỉ bằng giấy biến hình thành những

hình dạng đẹp đến ngỡ ngàng. Hãy nhớ rằng tiêu đề của cuốn sách này là Nghệ thuật và Thủ công của Giải

quyết Vấn đề. Thủ công còn phải trải qua một chặng đường dài, và đây là lộ trình chúng tôi nhấn mạnh, vì

nếu không phát triển thủ công trước tiên, thì nghệ thuật tốt không thể thành hiện thực. Tuy nhiên, cuối

cùng, kinh nghiệm giải quyết vấn đề là một kinh nghiệm thẩm mỹ, như vấn đề Hành động khẳng định cho

thấy. Những vấn đề thú vị nhất thường là đẹp nhất; các giải pháp của họ đẹp như một bài thơ hay một bức

tranh.

OK, quay lại Trái đất! Làm thế nào để bạn trở thành Bậc thầy về giải quyết vấn đề bằng cách bẻ

bảng, gấp giấy, nghệ thuật và thủ công? Đáp án đơn giản:

Mạnh mẽ lên, thả lỏng và luyện tập.

Hãy củng cố bằng cách tăng dần số lượng và độ khó của công việc giải quyết vấn đề của bạn. Thả
lỏng bằng cách cố tình phá vỡ các quy tắc và mở lòng một cách có ý thức với những ý tưởng mới (bao
gồm cả việc chiếm đoạt chúng một cách đáng xấu hổ!). Đừng ngại chơi xung quanh và cố gắng đừng để
thất bại làm bạn ức chế. Giống như con chuột của P6lya, một vài lần thử thất bại cũng hoàn toàn
ổn, miễn là bạn tiếp tục thử các cách tiếp cận khác. Và không giống như con chuột của P6lya, bạn
sẽ không chết nếu bạn không giải quyết được vấn đề. Điều quan trọng là phải nhớ điều đó. Giải
quyết vấn đề không dễ dàng, nhưng nó sẽ rất thú vị, ít nhất là trong hầu hết thời gian!
Cuối cùng, hãy thực hành bằng cách làm việc với rất nhiều và rất nhiều và rất nhiều vấn đề. Giải

quyết chúng không quan trọng bằng. Sẽ rất tốt cho sức khỏe khi có một số vấn đề chưa được giải quyết xảy

ra xung quanh tâm trí tỉnh táo và vô thức của bạn. Dưới đây là một số để giúp bạn bắt đầu.

Các vấn đề và bài tập Phần đầu

tiên (2. 1.1D-2. 1. 1 2) là các bài tập rèn luyện tinh thần. Bạn không cần phải làm tất cả, nhưng vui
lòng đọc từng cái và làm một vài việc (một số trong số chúng đòi hỏi chi phí thời gian và năng lượng
liên tục, và bạn có thể cân nhắc viết nhật ký để giúp bạn theo dõi). Phần còn lại của các vấn đề chủ yếu
là những câu đố vui về não bộ, được thiết kế để giúp bạn thoải mái hơn, xen lẫn với một vài câu hỏi mở
để khơi dậy tinh thần của bạn.

2.1.10 Dưới đây là hai thí nghiệm thú vị mà bạn có thể từ từ di chuyển một thẻ có một chữ cái được viết trên đó vào

thực hiện để thấy rằng thị lực ngoại vi của bạn vừa kém vùng ngoại vi của trường thị giác của bạn. Bạn sẽ nhận thấy sự

nhạy bén lại vừa nhạy hơn thị lực trung tâm. chuyển động của thẻ rất lâu trước khi bạn có thể đọc được chữ

cái trên đó.


1. Vào một đêm trời quang, hãy nhìn vào Pleiades constella tion,

còn được gọi là Bảy chị em vì nó có bảy ngôi sao nổi bật. Thay 2.1.11 Nhiều vận động viên được hưởng lợi từ "đào tạo
vì nhìn thẳng vào chòm sao, hãy thử lướt qua Pleiades bằng tầm chéo", luyện tập thường xuyên trong một môn thể thao
nhìn ngoại vi của bạn; tức là, cố gắng "để ý" nó, trong khi khác để nâng cao thành tích trong môn thể thao mục tiêu.
không hoàn toàn nhìn vào nó. Bạn sẽ có thể nhìn thấy nhiều ngôi Ví dụ, vận động viên đua xe đạp có thể nâng tạ hoặc chạy bộ.

sao hơn! Mặc dù chúng tôi ủng hộ việc dành phần lớn năng lượng của bạn cho các

vấn đề toán học , nhưng việc đa dạng hóa có thể hữu ích. Đây là một vài

gợi ý.
2. Nhìn thẳng về phía trước vào một bức tường trong khi một người bạn
Machine Translated by Google

2.1 CHIẾN LƯỢC TÂM LÝ HỌC IES 23

(a) Nếu tiếng Anh là tiếng mẹ đẻ của bạn, hãy thử giải các câu chuyện về vụ đắm tàu ở Nam Cực và sự dẻo dai về tinh

câu đố từ. Nhiều tờ báo hàng ngày đưa ra câu đố Jumble, thần cần thiết để tồn tại).

trong đó bạn giải mã đảo chữ cái của ble (hoán vị các chữ
2.1.12 Nếu bạn khó tập trung trong khoảng thời gian dài,
cái của một từ). Ví dụ, djauts là điều chỉnh. Cố gắng đi
hãy thử làm bài tập sau: dạy cho mình một số phép tính nhẩm.
đến điểm mà các cụm từ đảo chữ cái tự biến mất một cách
Đầu tiên, tính các ô vuông 2 từ 1
vô thức, gần như ngay lập tức. Điều này khai thác khả năng đến 322. Ghi nhớ danh sách này. Sau đó, sử dụng iden -i
tuyệt vời của tâm trí bạn để tạo ra những liên tưởng phức 2
với x = (x - y) (x + y) để tính bình phương một cách nhanh chóng
tạp. Bạn cũng có thể thấy rằng việc đọc ngược lại, đảo
trong đầu bạn. Ví dụ, để tính toán 872 , chúng tôi lý luận

ngược hoặc thậm chí ar của các cụm từ đảo ngữ ban đầu theo như sau:
hình tam giác sẽ rất hữu ích, có lẽ vì hành động "chỉnh

sửa" lại vấn đề này khiến bạn mất hứng thú. 872 - 32 = (87 - 3) (87 + 3)

= 84 · 90

= 1 0 (80 · 9 + 4 · 9)
(b) Một câu đố chữ vui nhộn khác là mật mã, trong đó bạn phải
= 7560.
giải mã một đoạn văn đã được mã hóa bằng mã thay thế một

chữ cái (ví dụ: A thành L, B thành G, v.v.) Nếu bạn luyện Do đó 872 = 7560 + 32 = 7569. Thực hành phương pháp này cho đến
thi những điều này cho đến khi bạn có thể thực hiện câu
khi bạn có thể bình phương một cách tin cậy bất kỳ số có hai chữ
đố mà không cần viết ngắn gọn hoặc không cần viết lại,
số nào một cách nhanh chóng và chính xác trong đầu. Sau đó, hãy
bạn sẽ kích thích khả năng liên tưởng và nâng cao sức mạnh
thử tay của bạn với các số có ba chữ số. Ví dụ,
suy luận và sự tập trung của bạn.
5772 = 600. 554 + 23 2 = 332400 + 529 = 332929.

(c) Các câu đố ô chữ tiêu chuẩn là OK, nhưng không được khuyến
Điều này thực sự sẽ gây ấn tượng với bạn bè của bạn! Đây có vẻ là
khích nhiều, vì chúng tập trung vào các liên tưởng khá
một bài tập ngớ ngẩn, nhưng nó sẽ buộc bạn phải thực hiện và nỗ
đơn giản nhưng với các sự kiện khá bí truyền. Điều tương
lực dựa vào khả năng hình dung hoặc trí nhớ thính giác của trí óc
tự cũng xảy ra với các câu đố sudoku, vì chúng liên quan
bạn có thể kích thích khả năng tiếp thu của bạn khi bạn làm việc
đến logic khá chuẩn. Không bao giờ sai, chúng rất tốt cho
với những vấn đề nghiêm trọng hơn.
việc xây dựng kỹ năng tập trung và logic, đặc biệt nếu

bạn tập trung vào việc thử tìm các chiến lược giải pháp
2.1.13 Không quan trọng khi bạn giải quyết các vấn đề, miễn
mới. Nhưng đừng nghiện những câu đố này; có rất nhiều thứ
là bạn dành nhiều thời gian cho chúng, nhưng hãy lưu ý về
khác để suy nghĩ!
các thói quen của bạn. Bạn có thể học được rằng, ví dụ, bạn

suy nghĩ tốt nhất khi tắm vào buổi sáng, hoặc có lẽ thời
(d) Học cách chơi một trò chơi chiến lược, chẳng hạn như cờ vua
gian tốt nhất của bạn là sau nửa đêm khi nghe nhạc lớn, v.v.
hoặc cờ vây. Nếu bạn chơi bài, hãy bắt đầu tập trung ghi
Tìm một thói quen phù hợp và sau đó thực hiện nó. (Bạn có
nhớ các ván bài khi chúng được chơi.
thể phát hiện ra rằng đi bộ hoặc chạy có lợi cho suy nghĩ.
(e) Lấy một nhạc cụ, hoặc nếu bạn đã từng chơi, hãy bắt đầu Hãy thử điều này nếu bạn chưa từng làm.)
luyện tập lại.

(f) Học một hoạt động thể chất "thiền định", chẳng hạn như
2.1.14 Bây giờ bạn đã thiết lập một thói quen, oc tạm thời
yoga, thái cực quyền, aikido, v.v. Các môn thể thao phương
phá vỡ nó. Ví dụ, nếu bạn có xu hướng suy nghĩ vào buổi sáng
Tây như gôn và bowling cũng được. (g) Đọc những câu chuyện
ở một nơi yên tĩnh, hãy cố gắng tập trung vào một vấn đề tại
hư cấu và có thật nổi tiếng về cách giải quyết vấn đề và sự dẻo một buổi hòa nhạc vào ban đêm, v.v.
dai về tinh thần. Một số tác phẩm yêu thích của chúng tôi
Đây là hệ quả của quy tắc "phá vỡ các quy tắc" ở trang 20.
là The Gold Bug, của Edgar Allan Poe (một câu chuyện về

sự phá mã); bất kỳ cuộc phiêu lưu nào của Sher lock
2.1.15 Đây là một bài tập thú vị về nới lỏng: chọn một đối
Holmes, của Arthur Conan Doyle (những câu chuyện tuyệt vời
tượng phổ biến, ví dụ, một viên gạch, và liệt kê càng nhanh
về suy luận và tính tự phụ); Zen in the Art of Archery,
càng tốt càng nhiều mục đích sử dụng cho đối tượng này càng
của Eugen Herrigel (một Westemer đến Nhật Bản để học bắn
tốt. Cố gắng không bị cấm đoán và ngớ ngẩn.
cung, và anh ấy thực sự học được cách thuần thục);

Endurance, bởi Alfred Lansing (một sự thật 2.1.16 Một nguồn thú vị để giải trí thú vị là những câu đố

"tư duy bên". Ví dụ, hãy xem


Machine Translated by Google

24 CHƯƠNG 2 CHIẾN LƯỢC ĐỐI VỚI CÁC VẤN ĐỀ ĐIỀU TRA VẤN ĐỀ

[37]. Những câu đố này chủ yếu sử dụng những ý tưởng hàng chứa một lít cà phê đen. Đổ một lượng nhỏ từ B vào A, trộn

ngày, nhưng luôn đòi hỏi một tầm nhìn ngoại vi tốt để giải đều, sau đó đổ lại từ A vào B cho đến khi cả hai bình lại

quyết. Rất khuyến khích cho công việc khởi động. chứa một lít chất lỏng. Mối quan hệ giữa phần cà phê trong A

và phần sữa trong B là gì?


2.1.17 Trong ví dụ 2. 1 .9 ở trang 21, chúng tôi đã chứng
minh rằng bất kỳ màu nào tối đa hóa số lượng dây cân
bằng sẽ được tích hợp. Kết quả này có "sắc nét không;" 2.1.23 Indiana Jones cần băng qua một cây cầu dây mỏng manh

tức là, phải có màu tối đa hóa dây cân bằng để được tích qua một hẻm núi dài hàng dặm. Trời tối đến mức không thể đi

hợp? qua cầu mà không có đèn pin. Cây cầu quá yếu nên chỉ có thể

nâng đỡ sức nặng của hai người. Nhóm chỉ có một đèn flash, có
2.1.18 Các số nguyên không âm được chia thành ba nhóm như
chùm sáng yếu, vì vậy bất cứ khi nào hai người đi ngang qua
sau:
nhau, họ bị hạn chế đi cùng nhau, với tốc độ của người chậm

A = {0, 3. 6, 8, 9 . . .. }. B = {I, 4, 7, 1 1 ,1 4 , . . . }, hơn . Indiana Jones có thể qua cầu trong năm phút nữa. Bạn

gái của anh ấy có thể vượt qua trong 10 phút nữa. Cha của anh

ấy cần 20 phút, và phụ tá của cha anh ấy cần 25 phút. Họ cần


1 3,
C = {2. 5, 1 0 , . . . }. đưa mọi người qua đường an toàn trong một giờ để thoát khỏi

kẻ xấu. Họ có thể làm được không?


Giải thích.

2.1.19 Bạn bị nhốt trong một căn phòng rộng 50 x 50 x 50 foot

nằm trên những chiếc cà kheo dài 1 mét. Có một cửa sổ mở ở


2.1.24 Bạn đã làm việc một chút với Tam giác Pas cal
cuối phòng, gần sàn nhà, với một cái móc chắc chắn được gắn
(Bài toán 1 .3.17 trang 10). Tìm cách lấy số Fibonacci
chặt vào sàn bên cạnh cửa sổ. Vì vậy, nếu bạn có một sợi dây
(xem Bài 1 .3. 18 trang 10) từ Tam giác Pascal.
I oo-foot, bạn có thể buộc một đầu vào móc và leo xuống dây

để tự do. (Không thể vào nhà sàn từ cửa sổ.) There are two 50-

2.1.25 Ví dụ 1.1.2 liên quan đến phỏng đoán

chiều dài chân của sợi dây, từng sợi dây gắn chặt vào trần
l N
-II
Tôi

nhà, cách nhau khoảng 1 feet, gần tâm trần. Bạn là một vận + - + - + "' + - = -. · 2 2
1 · 3 3 · 4 n + I n (n + I)
động viên leo dây mạnh mẽ, nhanh nhẹn, thắt nút tốt và bạn có

một con dao sắc bén. Bạn không có dụng cụ nào khác (thậm chí Thử nghiệm và sau đó phỏng đoán hình thức tổng quát hơn cho
không có quần áo). Sợi dây đủ chắc để giữ trọng lượng của bạn, các tổng trong đó mẫu số có tích của ba số hạng. Sau đó khái
nhưng không đủ nếu nó bị cắt theo chiều dọc. Bạn có thể sống quát hóa thêm.
sót khi rơi không quá 10 feet. Làm thế nào để bạn thoát ra còn
2.1.26 Có thể vẽ hình A dưới đây mà không cần nhấc bút
sống?
chì lên theo cách mà bạn không bao giờ vẽ cùng một đường
2.1.20 Soạn vấn đề "người kiểm tra dân số" của riêng bạn. hai lần. Tuy nhiên, dù bạn có cố gắng thế nào đi chăng
Phát minh ra một câu đố liên quan đến thông tin số và manh mối
nữa thì việc vẽ hình B theo cách này dường như là không thể.
dường như không phải là manh mối.
Bạn có thể tìm thấy các tiêu chí cho phép bạn nhanh chóng xác

định xem bất kỳ hình nào đã cho có thể hoặc không thể được
2.1.21 Một nhóm giáo sư ghen tuông bị nhốt trong phòng.
vẽ theo cách này không?
Trong phòng không có gì khác ngoài những cây bút chì và
một mẩu giấy nhỏ cho mỗi người. Các sors hồ sơ muốn xác
định mức lương trung bình của họ (trung bình, không phải
tôi dian) để mỗi người có thể hả hê hoặc đau buồn về hoàn
cảnh cá nhân của mình so với các đồng nghiệp của họ.
Tuy nhiên, họ là những người kín tiếng, không muốn tiết lộ

thông tin lương bổng cá nhân cho bất kỳ ai khác. Liệu họ có

thể xác định mức lương trung bình theo cách mà không một giáo
Một B
sư nào có thể phát hiện ra bất kỳ sự thật nào về mức lương

của bất kỳ ai ngoài chính cô ấy? Ví dụ, ngay cả những thông

tin như "ba người kiếm được hơn 40.000 đô la" hoặc "không ai
2.1.27 Câu hỏi mẹo. Tất cả các vấn đề sau đây dường
kiếm được hơn 90.000 đô la" cũng không được phép.
như đều có lời giải rõ ràng, nhưng cách giải rõ ràng

2.1.22 Bình A chứa một lít sữa và bình B


là không đúng. Hãy suy ngẫm và giải quyết!
Machine Translated by Google

2.2 CHIẾN LƯỢC CHO G ETTING BẮT ĐẦU 25

(a) Một ngày nọ, Martha nói, "Tôi đã sống trong suốt hoặc dron (một tứ diện bao gồm bốn mặt của ấu trùng hình

một phần năm thập kỷ." Sắp tròn một năm, cô ấy trẻ tam giác). Giả sử chúng ta dán hai khối đa diện lại

nhất có thể là bao nhiêu? với nhau dọc theo một mặt hình tam giác (sao cho các

mặt được dán chính xác trùng nhau). Vật rắn mới có bao
nhiêu mặt?
(b) Trong tất cả các cuốn sách ở một thư viện nào đó, nếu
bạn đọc ngẫu nhiên một cuốn, thì 90% khả năng nó có
(e) Hình bên cho thấy một hình vuông được cắt thành hai đa
hình ảnh minh họa. Trong tất cả các hình minh họa
giác đồng dạng và một hình vuông khác được cắt thành
trong tất cả các cuốn sách, nếu bạn chọn ngẫu nhiên
bốn đa giác đồng dư.
một hình, thì có 90% khả năng là nó có màu. Nếu thư

viện có 1 0.000 cuốn sách, thì số sách tối thiểu phải


có hình minh họa col ored là bao nhiêu?

(c) Bạn phải lật đồng xu công bằng ít nhất bao nhiêu lần

trước khi có ít nhất 50% xác suất bạn nhận được ít

nhất ba mặt?

(d) Ta có hai khối đa diện (tức là chất rắn có các mặt là

đa giác), tất cả các cạnh của chúng đều có độ dài bằng Với n số nguyên dương nào thì một hình vuông có thể

1: một khối chóp có đáy là hình vuông và khối tứ diện. cắt thành n đa giác đồng dư?

2.2 Các chiến lược để bắt đầu

Những ý tưởng tâm lý được trình bày ở trên có vẻ quá mơ hồ. Có lẽ bạn hỏi, "Làm thế nào tôi có
thể học cách chăm chỉ giải quyết vấn đề nếu tôi thậm chí không thể bắt đầu?" Bạn đã thấy bốn
chiến lược rất thực tế giải quyết vấn đề này: bước áp chót và nắm bắt các chiến lược bẩn thỉu
trong Ví dụ 1 .2.1, và mơ tưởng và thực hiện các chiến lược dễ dàng hơn trong Ví dụ 2. 1.1. Còn
nhiều điều để nói về những điều này và những ý tưởng khác giúp bạn bắt đầu điều tra giải quyết
vấn đề.
Như chúng tôi đã nói trước đó, có hai phần cho bất kỳ giải pháp thành công nào: điều tra và
lập luận. Thông thường, cuộc điều tra bị che khuất bởi lập luận giải pháp chính thức bóng bẩy.
Nhưng hầu như luôn luôn, điều tra là trọng tâm của giải pháp.
Các cuộc điều tra thường quanh co, đầy rẫy những ngã rẽ và những ngộ nhận ngớ ngẩn. Một khi vấn
đề được giải quyết, bạn có thể dễ dàng xem lại cuộc điều tra kéo dài của mình và tự hỏi tại sao
bạn mất nhiều thời gian để nhìn thấy ánh sáng. Nhưng đó là bản chất của việc giải quyết vấn đề
đối với hầu hết tất cả mọi người: bạn sẽ không nhận được phần thưởng bằng sự sáng suốt cho đến
khi bạn đã trả giá bằng sự vất vả kéo dài, đôi khi không có kết quả. Vì vậy,

Bất cứ điều gì kích thích điều tra là tốt.

Dưới đây là một số gợi ý cụ thể.

Bước đầu tiên: Định hướng

Một vài điều cần được thực hiện khi bắt đầu mọi vấn đề.

• Đọc kỹ vấn đề. Chú ý đến các chi tiết như tích cực so với nega
tive, hữu hạn so với vô hạn, v.v.
Machine Translated by Google

26 CHƯƠNG 2 CHIẾN LƯỢC I ES ĐỐI VỚI CÁC VẤN ĐỀ ĐIỀU CHỈNH

• Bắt đầu phân loại: đó là vấn đề "để tìm" hay "chứng minh"? Vấn đề có giống
với những người khác mà bạn đã thấy không? • Xác định cẩn thận giả thuyết
và kết luận. • Thử một số động não sơ bộ nhanh:

Hãy suy nghĩ về ký hiệu thuận tiện.

Một phương pháp lập luận cụ thể (xem Phần 2.3) có hợp lý không?

Bạn có thể đoán một giải pháp khả thi không? Hãy tin vào trực giác của bạn!

Có những từ khóa hoặc khái niệm có vẻ quan trọng không? Ví dụ, các số nguyên tố hoặc bình phương hoàn

hảo hoặc dãy vô hạn có thể đóng một vai trò quan trọng không?

Khi bạn hoàn thành việc này (và đừng vội vàng!), Hãy quay lại và làm lại. Nó trả tiền để đọc lại một vấn đề

nhiều lần. Khi bạn suy nghĩ lại về phân loại, giả thuyết và kết luận, hãy tự hỏi bản thân xem bạn có thể trình

bày lại những gì bạn đã xây dựng hay không. Ví dụ, có vẻ như giả thuyết này thực sự tầm thường và bạn chỉ cần lặp

lại nguyên văn từ trạng thái của vấn đề. Nhưng nếu bạn cố gắng trình bày lại nó, bạn có thể khám phá ra thông tin

mới.

Đôi khi chỉ cần định dạng lại giả thuyết và kết luận với ký hiệu mới sẽ giúp ích (ví dụ, Ví dụ 1 .2. 1 trên trang

4). Ngoài ra, hãy lưu ý cách trình bày lại giúp một người giải quyết vấn đề Người kiểm tra dân số (Ví dụ 1.1.3

trên trang 2). Tinh tế hơn, hãy nhớ lại Ví dụ 2. 1 .7 ở trang 20, liên quan đến dãy số I 2 1, 121 1, 1 1 1

,1 1 ,đối
221 ,. . . . Thông thường, một người đọc thầm một vấn đề. Nhưng cáchvới nhiều
trình bàyngười,
lại đủviệc đọc to cảm
để truyền dãy hứng
số chỉ
cholàgiải
một

pháp chính xác (miễn là một số như "121 1" được đọc là "một-hai-một-một," không phải "một nghìn, hai trăm mười một

").

Khi nhìn vào kết luận của vấn đề, đặc biệt là của một vấn đề "cần tìm", đôi khi sẽ giúp bạn "tưởng tượng" ra

một câu trả lời. Chỉ cần tạo ra một cái gì đó, và sau đó đọc lại vấn đề. Câu trả lời tưởng tượng của bạn rất có

thể là sai và việc đọc lại vấn đề với câu trả lời này trong đầu có thể giúp bạn biết tại sao câu trả lời sai, điều

này có thể chỉ ra một số hạn chế quan trọng hơn của vấn đề.

Đừng dành quá nhiều thời gian cho việc định hướng. Bạn đã hoàn thành khi bạn đã có một ý tưởng rõ ràng về

những gì vấn đề yêu cầu và những gì đã cho là gì. Những phỏng đoán đầy hứa hẹn về câu trả lời hoặc phương pháp

luận là phần thưởng, và bạn không nên mong đợi điều gì. Thông thường họ yêu cầu điều tra chuyên sâu hơn.

Tôi là người có định hướng. Giờ thì sao?

Tại thời điểm này, bạn hiểu vấn đề đang đặt ra là gì và bạn có thể có một số ý tưởng về việc cần làm tiếp theo.

Thường xuyên hơn không, điều này liên quan đến một hoặc nhiều trong bốn chiến lược "khởi động" cơ bản mà chúng ta

đã thấy, bước áp chót, làm bẩn tay, mơ mộng và làm cho nó dễ dàng hơn. Hãy thảo luận chi tiết hơn về những điều

này.

Lấy tay bẩn: Điều này rất dễ dàng và thú vị. Hãy thả lỏng và thử nghiệm.
Cắm nhiều số. Tiếp tục chơi xung quanh cho đến khi bạn nhìn thấy một mẫu.
Sau đó, chơi xung quanh một số nữa và cố gắng tìm ra lý do tại sao mô hình bạn thấy lại xảy ra. Đó là

một bí mật được giữ kín rằng nhiều nghiên cứu toán học cấp cao là kết quả của các phương pháp "plug and

chug" công nghệ thấp. Carl Gauss vĩ đại, rộng rãi


Machine Translated by Google

2.2 CÁC CHIẾN LƯỢC CHO G ETTING BẮT ĐẦU 27

được coi là một trong những nhà toán học vĩ đại nhất trong lịch sử (xem trang 67), là một fan

hâm mộ lớn của phương pháp này. Trong một lần điều tra, anh ấy đã tính toán tỉ mỉ số nghiệm

nguyên cho .x2 + y2 ::; 90,000.3 Bước cuối cùng: Khi bạn biết kết luận mong muốn là gì, hãy tự

hỏi bản thân, "Điều gì sẽ mang lại kết luận trong một bước duy nhất?" Đôi khi, bước áp chót là

"hiển nhiên" khi bạn bắt đầu tìm kiếm. Và bạn càng có nhiều kinh nghiệm thì các bước thực hiện

càng rõ ràng. Ví dụ: giả sử A và B là những biểu thức kỳ lạ, xấu xí dường như không có mối liên

hệ nào, nhưng bạn phải chứng minh rằng A = B. Một bước cuối cùng là tranh luận riêng rằng A 2 B

VÀ B 2 A. Có lẽ bạn muốn Thay vào đó chỉ ra rằng A I- B. Một bước áp chót sẽ cho thấy rằng A luôn

chẵn, trong khi B luôn lẻ. Luôn dành thời gian suy nghĩ thấu đáo về những bước áp chót có thể

xảy ra. Tất nhiên, đôi khi, việc tìm kiếm bước áp chót không thành công và đôi khi nó giúp người

ta lập kế hoạch chiến lược chứng minh (xem Phần 2.3 bên dưới).

Suy nghĩ đầy ước mơ và biến nó trở nên dễ dàng hơn: Những chiến lược này kết hợp giữa tâm lý học

và toán học để giúp phá vỡ những bế tắc ban đầu trong công việc của bạn. Hãy tự hỏi bản thân, "Đó

là điều gì về vấn đề khiến nó trở nên khó khăn?" Sau đó, làm cho khó khăn biến mất! Bạn có thể

không thể làm điều này một cách hợp pháp, nhưng ai quan tâm? Tạm thời tránh phần khó của một vấn

đề sẽ cho phép bạn đạt được tiến bộ và có thể làm sáng tỏ những khó khăn. Ví dụ, nếu bài toán liên

quan đến những con số lớn, xấu, hãy làm cho chúng nhỏ và xinh. Nếu một vấn đề liên quan đến các

phân số hoặc gốc đại số phức tạp, hãy thử xem xét một vấn đề tương tự mà không có các thuật ngữ

như vậy.

Tốt nhất, giả vờ rằng khó khăn không có ở đó sẽ dẫn đến một giải pháp táo bạo, như trong Ví dụ

2. 1.1 trên trang 15. Tệ nhất, bạn sẽ buộc phải tập trung vào khó khăn chính của vấn đề và có

thể hình thành một phương án trung gian. thắc mắc, người có câu trả lời sẽ giúp bạn giải quyết

vấn đề trong tầm tay. Và loại bỏ phần khó của một vấn đề, dù chỉ là tạm thời, sẽ cho phép bạn

vui vẻ và nâng cao sự tự tin của mình. Nếu bạn không thể giải quyết vấn đề như đã viết, ít nhất

bạn có thể đạt được tiến bộ với người anh em họ dễ dàng hơn của nó!

Dưới đây là một vài ví dụ minh họa việc sử dụng các chiến lược này. Chúng tôi sẽ không tập trung

vào việc giải quyết các vấn đề ở đây, chỉ cần đạt được một số tiến bộ ban đầu. Điều quan trọng cần ghi

nhớ là mọi tiến trình đều ổn. Đừng bao giờ vội vàng để giải quyết một vấn đề! Quá trình điều tra cũng

quan trọng như vậy. Bạn có thể không phải lúc nào cũng tin vào điều này, nhưng hãy thử:

Thời gian dành để suy nghĩ về một vấn đề luôn là thời gian xứng đáng. Ngay
cả khi bạn dường như không có tiến bộ gì cả.

Ví dụ 2.2.1 (Russia, 1 995) Dãy aO, a1, a2 , ... thỏa mãn

1
am + n + am-n = + a2 n) (1)
"2 (a2 m

với mọi số nguyên không âm m và n với m 2 n. Nếu a1 = 1, xác định a1995.

Lời giải một phần: Phương trình (1) rất khó hiểu nếu không có thử nghiệm.

Chúng ta hãy cố gắng xây dựng một số giá trị của an. Đầu tiên, chúng tôi giữ mọi thứ đơn giản và thử m = n = 0,

3Câu trả lời là 2 82, 6 97, trong trường hợp bạn quan tâm. Xem [2 1], tr. 33.
Machine Translated by Google

28 CHƯƠNG 2 CÁC CHIẾN LƯỢC CHO I CÁC VẤN ĐỀ NỔI BẬT

kết quả là ao + ao = ao, do đó ao = O. Bây giờ sử dụng các chỉ số 0, 1 đã biết: cắm m = 1, n = o vào

(1) và chúng ta có 2al = i (a2 + ao) = az l2, do đó a2 = 4al = 4. Tổng quát hơn, nếu chúng ta chỉ cần
sửa n = 0, chúng ta có 2am = a2m / 2, hoặc a2m = 4am. Bây giờ hãy cắm m = 2, n = 1:

1
=
aUI + a2 -1
2 (a4 + a2 ) .

Vì a4 = 4a2 = 4 · 4 = 16 nên chúng ta có a3 + al = i (16 + 4), do đó a3 = 9. Tại thời điểm này,


2
chúng ta đã sẵn sàng để mạo hiểm phỏng đoán rằng an = n với mọi n = 1,2,3 .... Nếu phỏng đoán này

là đúng, một phương pháp chứng minh có thể sẽ là quy nạp toán học. Xem trang 45 dưới đây để biết sơ
lược về kỹ thuật này và trang 47 để biết phần tiếp tục của vấn đề này.

Ví dụ 2.2.2 (AIME 1985) Các số trong dãy

101, 104, 109, 1 16, ...

2,
có dạng an = 100 + n trong đó n = 1,2, 3 , .... Với mỗi n, để dn lớn nhất Tìm giá
4 ước chung cho các của một và một + I. trị lớn nhất của dn khi n dãy qua

số nguyên dương.

Lời giải một phần: Chỉ cần viết ra một số số hạng đầu tiên của an sẽ khiến chúng ta suy đoán
rằng giá trị lớn nhất của dn là 1, vì các số hạng liên tiếp của dãy dường như là số nguyên tố tương

đối. Nhưng trước tiên, chúng ta nên xem xét các trường hợp đơn giản hơn. Có lẽ không có gì đặc biệt
về số 100 trong định nghĩa của an, ngoại trừ có lẽ rằng 100 = 102.
2
Hãy xem xét các số được xác định bởi an = u + n , nơi u được cố định. Làm một cái bàn.

u al a2 a3 a4 a5 a6 a7 10 17
2 5 26 37 50 11
5118
1227
1938
28
1 3 6 39 52
2 3 4 7

Chúng tôi đánh dấu bằng chữ in đậm cặp số hạng liên tiếp trong mỗi hàng có GCD lớn nhất (ít

nhất là lớn nhất trong bảy số hạng đầu tiên của hàng đó). Để ý rằng khi u = 1 thì a2 và a3 có GCD
là 5. Khi u = 2 thì a4 và a5 có GCD là 9 và khi u = 3 thì a6 và a7 có GCD là 13. Có một mô hình rõ

ràng: chúng ta phỏng đoán rằng nói chung, với bất kỳ số nguyên dương u cố định nào, thì a2u và a2u

+ 1 sẽ có GCD là 4u + 1. Chúng ta có thể khám phá phỏng đoán này với một chút đại số: = 4u2 + u = u

(4u + 1),

2
a2u = u + (2u)
trong khi

2
a2u + 1 = u + (2u + 1) = 4u2 + 4u + 1 + u = 4u2 + 5u + 1 = (4u + l) (u + 1),

và, chắc chắn, a2u và a2u + 1 có chung nhân tử 4u + 1.

Đây là một tiến bộ đáng khích lệ, nhưng chúng tôi vẫn chưa thực hiện được. Chúng tôi chỉ đơn
thuần chỉ ra rằng 4u + 1 là nhân tử chung của a2u và a2u + I, nhưng chúng tôi muốn chứng minh rằng

đó là nhân tử chung lớn nhất . Và chúng tôi cũng cần chứng minh rằng giá trị 4u + 1 là giá trị lớn nhất

4Xem 3.2.4, 3.2. 17 và Phần 7. 1 để biết thêm thông tin về ước số chung lớn nhất.
Machine Translated by Google
2.2 CHIẾN LƯỢC CHO G ETTING BẮT ĐẦU 29

giá trị GCD có thể có đối với các cặp số hạng liên tiếp. Cả hai mục này đều khó chứng
minh nếu bạn biết một số công cụ lý thuyết số đơn giản. Chúng ta sẽ tiếp tục vấn đề này
trong Ví dụ 7. 1 .8 trên trang 227.

Ví dụ 2.2.3 Các tủ khóa trong một hàng được đánh số 1,2, ,. . . ,1 000. Lúc đầu, tất cả các tủ khóa

3 được đóng. Một người đi ngang qua và mở mọi tủ đựng đồ khác, bắt đầu với tủ đựng đồ số 2.
Do đó, các tủ khóa 2, 4, 6, ..., 998, 1 000 được mở. Một người khác đi ngang qua và thay
đổi "trạng thái" (nghĩa là đóng tủ khóa nếu nó đang mở, mở tủ nếu nó đang đóng) của mỗi tủ
khóa thứ ba, bắt đầu với tủ khóa số 3. Sau đó, một người khác thay đổi trạng thái của mỗi
tủ khóa thứ tư, bắt đầu với số 4, v.v. Quá trình này tiếp tục cho đến khi không thể thay
đổi được tủ khóa nào nữa. Tủ khóa nào sẽ bị đóng?

Giải pháp một phần: Rất có thể, không có gì đặc biệt về số 1 000 trong bài toán này. Hãy
để chúng tôi làm cho nó dễ dàng hơn: Đơn giản hóa vấn đề bằng cách giả sử một số lượng tủ khóa
nhỏ hơn nhiều, chẳng hạn như 1 0, để bắt đầu. Bây giờ hãy làm bẩn bàn tay của bạn bằng cách làm
"
một cái bàn, sử dụng ký hiệu "0 cho mở và "x" cho đóng. Ban đầu (bước 1) cả 10 đều đóng. Dưới
đây là bảng trạng thái của từng tủ đựng đồ ở mỗi lần đi qua. Chúng tôi dừng lại ở bước 1 0,
vì các bước tiếp theo sẽ không ảnh hưởng đến tủ khóa.

bươ c chân
Khóa #
2 3 4 5 6 7 8 9 10
xxx xxxx xx x
1 x 0 x 0 x 0 x 0 x 0
2 x 0 0 0 xxx 0 0 0
3 x 0 0 xxxx x 0 0
4 x 0 0 x 0 xx x 0 x
5 x 0 0 x 0 0 x x 0 x
6 x 0 0 x 0 0 0 x 0 x
7 x 0 0 x 0 0 0 0 0 x
8 x 0 0 x 0 0 0 0 x x
9 10 x 0 0 x 0 0 0 0 x 0

Ta thấy các ngăn tủ đóng được đánh số 1,4, 9; một phỏng đoán hợp lý là chỉ những
tủ khóa số vuông hoàn hảo mới được đóng cửa nói chung. Chúng tôi không thể chứng minh
phỏng đoán ngay bây giờ, nhưng chúng tôi có thể đạt được tiến bộ đáng kể bằng cách
nhìn vào bước áp chót. Điều gì xác định xem một tủ khóa đang mở hay đóng? Sau khi điền
vào bảng, bạn biết câu trả lời: tính chẵn lẻ (chẵn hoặc lẻ) của số lần trạng thái của
tủ khóa thay đổi. Tủ khóa được đóng hoặc mở tùy theo số lần thay đổi trạng thái là lẻ
hoặc chẵn. Áp dụng ý tưởng bước áp chót một lần nữa: điều gì gây ra sự thay đổi trạng
thái? Khi nào thì tủ đựng đồ bị chạm? Đơn giản hóa mọi thứ trong giây lát và chỉ tập
trung vào một tủ khóa, hãy nói # 6. Nó đã được thay đổi ở các bước 1, 2, 3 và 6, tổng
cộng bốn lần (một số chẵn, do đó tủ vẫn mở). Nhìn vào tủ đồ # 10. Nó đã được thay đổi
ở các bước 1, 2, 5 và 10. Bây giờ thì rõ ràng:

Khoá #n được thay đổi ở bước k nếu và chỉ khi k chia cho n.
Machine Translated by Google

30 CHƯƠNG 2 CHIẾN LƯỢC ĐỐI VỚI I CÁC VẤN ĐỀ ĐIỀU CHỈNH

Do đó, chúng tôi đã trình bày lại phỏng đoán của mình thành một dạng khác nhau không thể nhận ra, nhưng
tương đương:

Chứng minh rằng một số nguyên có một số ước là lẻ nếu và chỉ khi nó là một
hình vuông hoàn hảo.

Có nhiều cách để suy nghĩ về vấn đề cụ thể này. Xem thảo luận trên p. 68 và Bài toán 6. 1
.21 cho hai cách tiếp cận hoàn toàn khác nhau.

Ví dụ 2.2.4 (LMO 1 988) Có 25 người ngồi quanh bàn và mỗi người có hai thẻ. Một trong
các số 1, 2, 3 25 được viết trên mỗi thẻ và mỗi , ...
mộtsốtín
xuất
hiệu,
,những
hiệnmỗi
thẻ
trên
người
của
đúng

chuyển
hai
ấy -thẻ.
một
thẻ Theo
trong

số nhỏ hơn-cho người hàng xóm bên phải của cô ấy. Chứng minh rằng, sớm hay muộn, một
trong hai người chơi sẽ có hai thẻ có cùng số.

Giải pháp một phần: Lúc đầu, vấn đề này dường như không thể vượt qua. Làm thế nào
bạn có thể chứng minh một cái gì đó như thế này? Liều lượng nhỏ để làm bẩn tay của
chúng ta và làm cho nó dễ dàng hơn trong một chặng đường dài. Hãy giúp sự hiểu biết
của chúng tôi bằng cách làm cho vấn đề dễ dàng25,
hơn.
mặc
Códù
lẽchúng
khôngtôi
có gì
ngay
đặc
lập
biệt
tứcvề
được
số
cảnh báo về cả hình vuông và số lẻ. Hãy thử một ví dụ với hai người. Nếu mỗi người có
thẻ đánh số 1 và 2, chúng ta thấy rằng mô hình là tuần hoàn: mỗi người chỉ chuyển số
1 cho cô hàng xóm, và mỗi người luôn giữ số 1 và 2.
Vì vậy kết luận không đúng với hai người. Chẵn lẻ (chẵn hay lẻ) có quan trọng không,

sau đó? Có lẽ! Sử dụng ký hiệu - + - để chỉ một người cầm thẻ được đánh số a

và b. Hãy xem xét bốn người, ban đầu giữ

Chúng ta thấy rằng ở mỗi lượt, mọi người sẽ giữ 1 hoặc 2 ghép với 3 hoặc 4. Vì vậy, một lần
nữa, kết luận là không đúng, và đối với bất kỳ số chẵn nào, chúng ta có thể chắc chắn rằng
nó không bao giờ đúng. Ví dụ: nếu có 10 = 2 · 5 người, chỉ cần bắt đầu bằng cách đưa cho
mỗi người một thẻ được chọn từ {I , 10}. , ,2,
5} ...
và một thẻ được chọn từ {6, 7 , . ..
Sau đó, ở mỗi lượt, mỗi người cầm một thẻ được đánh số trong phạm vi 1-5 ghép với một thẻ
trong phạm vi 6-1 0, vì vậy không ai được cầm hai thẻ có cùng số.
Bây giờ chúng ta chuyển sự chú ý của chúng ta đến trường hợp có một số lượng người lẻ.
Đây là một ví dụ liên quan đến 5 người.

TÔI ! Tôi tôi tôi; Tôi � tôi � tôi

Chúng tôi sắp xếp bàn sao cho các thẻ hàng trên cùng nhỏ hơn hàng dưới, để chúng tôi
biết rằng đây là những thẻ sẽ được chuyển ở lượt tiếp theo. Sau đó, chúng tôi sắp xếp lại
chúng để cấp cao nhất chứa các số nhỏ hơn:
Machine Translated by Google
2.2 CHIẾN LƯỢC CHO G ETTING BẮT ĐẦU 31

1 � 1 � 1 ; 1 ; 1 � 1 (�) 1 j 1! 1; 1; 1 ; 1

(��91 � 1! 1; 1; 1 ; I ·

Tại thời điểm này, chúng ta có thể lặp lại quá trình, nhưng trước khi làm như vậy, chúng ta quan

sát thấy rằng số lớn nhất ở hàng trên cùng lớn nhất bằng số nhỏ nhất ở hàng dưới cùng. Vì vậy, sau khi

chúng tôi thay đổi các số hàng đầu, chúng tôi không còn phải sắp xếp nữa. Chúng ta thấy rằng cuối cùng

số 3 ở trên cùng sẽ nối với số 3 ở dưới cùng:

1 � 1! 1 ; 1; 1 ; 1 (�) 1! 1 � 1 ; 1; 1 ; 1

(�) 1 � 1! 1 ; 1 � 1 ; 1 (�) 1 � 1 � 1 ; 1 ; 1 ; TÔI·

Vì vậy, những gì thực sự đã xảy ra? Chúng tôi có thể làm cho hai số 3 trùng nhau, bởi vì hàng trên

và dưới đã ngừng "trộn" và có một số 3 ở trên cùng và 3 ở dưới cùng. Điều này sẽ luôn xảy ra? Bạn có thể

đưa ra một lập luận chung không? Và sự chẵn lẻ của 5 đã đóng vai trò gì? Một vài thử nghiệm nữa, có lẽ

với 7 người, sẽ giúp bạn hoàn thành việc này.

Trong ví dụ tiếp theo, chúng ta không chỉ không giải được bài toán, mà còn khám phá ra một điều

chắc chắn mà chúng ta biết là sai! Tuy nhiên, chúng tôi đạt được tiến bộ từng phần: chúng tôi phát triển

sự hiểu biết về cách thức hoạt động của vấn đề, ngay cả khi chúng tôi không đạt được giải pháp hoàn chỉnh.

Ví dụ 2.2.5 (Putnam 1 983) Cho f (n) = n + L v'nJ. Chứng minh rằng, với mọi số nguyên

dương m, dãy

m, f (m), f (f (m)), f (f (f (m))), ...

chứa bình phương của một số nguyên.5 Giải Một phần: Lúc đầu, nó có vẻ

khá khó khăn. Hàm f (n) có một định nghĩa kỳ lạ và kết luận mong muốn cũng khó hiểu. Đầu tiên

chúng ta hãy định hướng: bài toán yêu cầu chúng ta chỉ ra rằng điều gì đó liên quan đến f (m) và bình

phương là đúng với mọi số nguyên dương m. Cách duy nhất để tiếp tục là làm bẩn tay. Chúng ta cần hiểu

cách hoạt động của hàm f (m) . Vì vậy, chúng tôi bắt đầu thử nghiệm và lập bảng.

9 12

5
Nhớ lại rằng lx J là số nguyên lớn nhất nhỏ hơn hoặc bằng x. Để biết thêm thông tin, hãy xem trang 1 46.
Machine Translated by Google
32 CHƯƠNG 2 CHIẾN LƯỢC I ES ĐỐI VỚI CÁC VẤN ĐỀ ĐIỀU CHỈNH

Mô hình có vẻ đơn giản: f (m) tăng 1 khi m tăng, cho đến khi m là một hình vuông hoàn hảo, trong

trường hợp đó f (m) tăng 2 (chữ đậm). Bất cứ khi nào bạn quan sát một hình mẫu, bạn nên thử xem tại sao

nó lại đúng. Trong trường hợp này, không khó để xem những gì đang xảy ra.

Ví dụ, if 9 :::; m < 16 thì đại lượng l vrn J có giá trị không đổi là 3. Do đó f (m) = m + 3 for 9 :::; m < 16.

Tương tự, f (m) = m + 4 for 16 :::; m < 25, tính đến "bỏ qua" xảy ra tại các hình vuông hoàn hảo.

Bây giờ chúng ta đã hiểu khá rõ về f , đã đến lúc xem xét các lần lặp đi lặp lại của

Chức năng này. Một lần nữa, hãy thử nghiệm và lập bảng! Chúng ta sẽ sử dụng ký hiệu r (m) = f (f (-

· · f (m) ··.)), (2) trong đó phía bên tay phải chứa r fs, và chúng ta sẽ

chỉ ra các hình vuông có nét đậm.


Lưu ý rằng chúng ta không bận tâm đến việc thử các giá trị của m là bình phương, vì như vậy chúng ta sẽ

hoàn thành một cách đáng kể. Thay vào đó, chúng ta bắt đầu với các giá trị của m lớn hơn một hình vuông

hoàn hảo, v.v. Ngoài ra, ngay cả khi chúng ta đạt được một hình vuông hoàn hảo, chúng ta vẫn tiếp tục

điền vào bảng. Đây là một thói quen làm việc quan trọng:

Đừng bỏ qua thử nghiệm! Tiếp tục lộn xộn cho đến khi bạn nghĩ rằng bạn
hiểu chuyện gì đang xảy ra. Sau đó lộn xộn xung quanh một số nữa.

mf (m) F (m) p (m) f4 (m) p (m) r (m) 5 1 2 6 13 7 15 8 16 50 72 51 73 101 1 32 102


103 15 18 22

9 16 20 24

7 10 18 22 26

8 1 20 24 28

9 2 80 88 97

10 57 58 13 81 90 99

III 64 65 121 143 154 166

1 1 1 1 33 144 156 168

2 1 13 22 123 1 34 145 157 169

Bây giờ có nhiều mẫu hơn xuất hiện. Dường như nếu m có dạng n2 + 1, với n là số nguyên, thì f2 (m)

là một hình vuông hoàn hảo. Tương tự, nếu m có dạng n2 + 2, thì r (m) dường như là một hình vuông hoàn

hảo. Đối với các số có dạng n2 + 3, chẳng hạn như 7 và 103, điều đó không rõ ràng hơn: f 6 (103) là một

hình vuông hoàn hảo, nhưng f (7) là một hình vuông hoàn hảo trong khi f 6 (7) thì không.

Vì vậy, chúng ta biết rằng "phỏng đoán" sau đây không hoàn toàn đúng: lfm = n2 + b,

thì f2b (m) là một hình vuông hoàn hảo.

Tuy nhiên, chiến lược mơ tưởng đòi hỏi chúng ta ít nhất phải xem xét tuyên bố này. Sau cùng, chúng

ta muốn chứng minh rằng với m bất kỳ, sẽ có r sao cho r (m) là một hình vuông hoàn hảo. Trước khi đi sâu

vào vấn đề này, chúng ta hãy tạm dừng và xem xét: khó khăn chính của vấn đề này là gì? Đó là thuật ngữ l

vrnJ gây bối rối . Vì vậy, trước tiên chúng ta nên tập trung vào biểu thức này. Một khi chúng ta hiểu nó,

chúng ta sẽ thực sự hiểu cách hoạt động của f (m) .

g (n2 + b) = n. Chúng ta có thể Khi đó g làm


dễ dàng (n2 cho
+ b)điều
bằngnày
baochính
nhiêu? Nếu
xác b là
hơn, "đủcách
bằng nhỏ"thử
thìnghiệm
Xác định g chỉ
hoặc (m) cần
= l suy
vrnJ.

nghĩ rõ ràng về đại số. Với những giá trị nào của m thì g (m) = n? Câu trả lời:

2 n :::; m < (n + 1 f = n2 + 2n + 1 .
Machine Translated by Google

2.2 CHIẾN LƯỢC TÔI THỰC HIỆN CHO G ETTING BẮT ĐẦU 33

Nói cách khác,

.
g (n2 + b) = n nếu và chỉ khi 0 : S b < 2n + 1

Bây giờ chúng ta hãy xem xét "phỏng đoán." Ví dụ, hãy xem xét trường hợp b = I.
Khi đó m = n2 + 1 và g (m) = n và f

(m) = m + g (m) = n2 + 1 + n.

Lặp lại hàm f một lần nữa, chúng ta có

f2 (m) = f (n2 + n + 1) = n2 + n + 1 + g (n2 + n + I)

= n2 + n + 1 + n = n2 + 2n + 1 = (n + 1 f,

vì vậy thực sự "phỏng đoán" là đúng với b = 1.


Bây giờ hãy xem b = 2. Khi đó m = n2 + 2 và g (m) = n với điều kiện 2 < 2n + 1,
điều này đúng với mọi số nguyên dương n. Do đó, f (m) = m

+ g (m) = n2 + 2 + n,

f2 (n2 + 2) = n2 + n + 2 + g (n2 + n + 2).

Vì n + 2 < 2n + 1 đúng với n> 1 nên ta có g (n2 + n + 2) = n và do đó, nếu n> 1,

Bây giờ chúng ta đã khám phá ra một điều thú vị: nếu m nhiều hơn 2 một hình vuông hoàn hảo
và m> 12 + 2 = 3, thì hai phép lặp của f tạo ra một số lớn hơn 1 hình vuông hoàn hảo. Chúng
tôi biết từ công việc trước đó của chúng tôi rằng hai lần lặp lại của f sau đó sẽ cho chúng
ta một hình vuông hoàn hảo. Ví dụ, cho m = 62 + 2 = 38. Khi đó f (m) = 38 + 6 = 44 và f (44)
4
= 44 +6 = 50 và f2 (50) = 64 (từ bảng). Vì vậy, f (38) = 64. Số duy nhất
của dạng n2 + 2 không hoạt động là 12 + 2 = 3, nhưng trong trường hợp này, f (3) = 4, vì vậy chúng
ta đã hoàn thành.

Chúng tôi đã đạt được một phần tiến bộ đáng kể. Chúng ta đã chứng minh rằng nếu m = n2 + 1
hoặc m = n2 + 2, thì có vô số lần lặp lại của f sẽ thu được một hình vuông hoàn hảo. Và chúng tôi
có một hướng tốt để làm việc. Mục tiêu của chúng tôi là có được những hình vuông hoàn hảo. Cách
chúng tôi đo lường sự tiến bộ từng phần để đạt được mục tiêu này là viết các số của chúng tôi
dưới dạng n2 + b, trong đó 0 : S b < 2n + 1. Nói cách khác, b là "phần còn lại". Bây giờ một
phỏng đoán hấp dẫn hơn là

Nếu m có dư b thì f2 (m) có dư b - 1.


Nếu chúng ta có thể thiết lập phỏng đoán này, thì chúng ta đã hoàn thành, vì cuối cùng, phần
còn lại sẽ trở thành số không. Thật không may, phỏng đoán này không hoàn toàn đúng. Ví dụ:
nếu m = 7 = 22 + 3, thì f2 (7) = 12 = 32 + 3. Mặc dù "phỏng đoán mơ mộng" này là sai, nhưng
phân tích cẩn thận sẽ phát hiện ra điều gì đó rất giống sự thật và điều này sẽ dẫn đến đến
một giải pháp đầy đủ. Chúng tôi để lại phân tích này cho bạn.
Machine Translated by Google
34 CHƯƠNG 2 CÁC CHIẾN LƯỢC CHO I CÁC VẤN ĐỀ NỔI BẬT

Ví dụ 2.2.6 (Putnam 1 991) Cho A và B là các ma trận n x n khác nhau với số lần thử en thực. Nếu A3 =
B3 và A2B = B2 A thì A2 + B2 có khả nghịch không?

Giải pháp: Đây là dạng bài toán mà hầu hết học sinh đều né tránh, ngay cả những em học
giỏi môn đại số tuyến tính. Nhưng nó thực sự không khó chút nào khi tiếp cận với sự tự tin.
Trước hết, chúng ta lưu ý rằng: A = I- B, A3 = B3 và A2B = B2A.
Kết luận là xác định xem C = A2 + B2 có khả nghịch hay không. Hoặc C là khả nghịch hoặc không. Làm cách

nào để chứng minh rằng một ma trận là khả nghịch? Một cách là chỉ ra rằng định thức của nó là khác
không. Điều đó có vẻ khó, vì ma trận là n x n, trong đó n là tùy ý, và công thức cho định thức rất phức

tạp khi n � 3. Một cách khác để chứng minh rằng ma trận C là khả nghịch là chứng tỏ rằng Chj = I- 0

cho mỗi vectơ cơ sở . Vậy là được rồi; bây giờ chúng ta nghĩ về tính không đảo ngược. Điều đó hóa ra
dễ dàng hơn ,một ...,hn.
h 2,chút: tất Nhưng
cả những
điềugìđó
chúng
cũngtakhó,
cần vì
làmchúng
là tìm
tamột
cầnvectơ
phải vtìm
khác
ra0 cơ
saosở.
cho Cv = O. Bây

giờ, đó là bước áp chót có thể quản lý được. Nó sẽ hoạt động? Chúng tôi không có cách nào để biết,

nhưng những ý tưởng chiến lược của sự mơ tưởng và làm cho nó trở nên dễ dàng hơn yêu cầu chúng tôi

nghiên cứu con đường này.

Chúng ta cần sử dụng giá trị đã cho, bắt đầu bằng C, và bằng cách nào đó nhận được số không. Một
3 3
suy nghĩ cho chúng ta biết để nhìn vào các công trình xây dựng như A - B3, kể từ A lần nữa, wishful = B3.
2
C = A Bắt đầu với + B2, cách tiếp cận trực tiếp nhất để lấy số hạng bậc ba có vẻ hiệu quả (nhớ lại

rằng phép nhân ma trận không có tính chất giao hoán, do đó B2 A không nhất thiết phải bằng AB2): (A2 +
B2) (A - B) = A3 - A2B + B2A _ B3 = A3 - B3 + B2A _ A2B = O.

Bây giờ chúng ta đã hoàn thành! Vì A = I- B nên ma trận A - B = I- O. Do đó tồn tại


vectơ u sao cho (A - B) u = I- O. Bây giờ chỉ cần đặt v = (A - B) u và chúng ta có

Cv = ((A2 + B2) (A - B)) u = Ou = O.


2
Như vậy A
+ B2 luôn không thể đảo ngược.

Ví dụ 2.2.7 (Olympic Toán học Leningrad 1 988) Cho p (x) là một đa thức
với hệ số thực. Chứng minh rằng nếu
ili
p (x) - p '(x) - p "(x) + p (x) � 0

với mọi x thực thì p (x) � 0 với mọi x thực.

Giải pháp một phần: Nếu bạn chưa bao giờ gặp vấn đề kiểu này trước đây, thì đó là một vấn đề khá

khó hiểu. Đạo hàm phải làm gì với việc một hàm không âm hay không? Và tại sao điều quan trọng là p (x)

là một đa thức?

Chúng ta phải đơn giản hóa vấn đề. Phần khó nhất là gì? Rõ ràng, biểu thức phức
tạp p (x) - p '(x) - p "(x) + p"' (x) . Bao thanh toán là một chiến thuật đại số quan
trọng (xem trang 148). Được thúc đẩy bởi quá trình phân tích nhân tử

l -x-x2 + x3 = (l -x) (I - �),

chúng tôi viết

ili
p (x) - p '(x) - p "(x) + p (x) = (p (x) - p "(x)) - (p (x) - p" (x)) '.
Machine Translated by Google

2.2 CÁC CHIẾN LƯỢC CHO G ETTING BẮT ĐẦU 35

Nói cách khác, nếu chúng ta cho q (x) = p (x) - p "(x) , sau đó

p (x) - p ' (x) - p " (x) + pili (x) = q (x) - q' (x) .

Vì vậy, bây giờ chúng ta có một vấn đề đơn giản hơn để kiểm tra:

Nếu q (x) là một đa thức và q (x) - q ' (x) 2 0 với mọi x thực, ta có thể
nói gì về q (x)?

Có thể nào rằng q (x) 2 0 với mọi x thực không? Điều này có thể đúng hoặc có thể không đúng, và nó có thể không giải

quyết được vấn đề ban đầu, nhưng nó chắc chắn có giá trị điều tra. Suy nghĩ mơ mộng đòi hỏi chúng ta phải xem xét điều

này.

Bất phương trình q (x) - q '(x) 2 0 tương đương với q' (x) ::; q (x). Do đó, nếu q (x) < 0, thì q '(x) cũng phải

âm. Do đó, nếu đồ thị của y = q (x) bao giờ giảm xuống dưới trục x (đi từ trái sang phải), thì nó phải ở dưới trục x,

vì hàm q (x) sẽ luôn giảm! Chúng tôi có ba trường hợp.

• Đồ thị của y = q (x) đi qua trục x. Bằng cách lập luận trên, nó chỉ phải giao nhau một lần, đi từ dương sang

âm (vì một khi nó là âm thì nó vẫn âm). Hơn nữa, vì q (x) là một đa thức, chúng ta biết rằng

lim q (x) = +00 và lim q (x) = - 00,


x�-oo x� + oo

1
vì bất kỳ đa thức q (x) = anXZ + an-1 XZ- +. . . + ao bị chi phối bởi số hạng cấp
cao nhất của nó đối với x đủ lớn (dương hoặc âm). Vì vậy, q (x) phải có bậc lẻ n
và một < O. Ví dụ, đồ thị của đa giác q (x) = -x7 + x2 + 3 có hành vi thích hợp.

1 0

-2 -TÔI 2 3

-5

Tuy nhiên, đa thức này không thỏa mãn bất đẳng thức q ' (x) ::; q (x): Ta có q (x) = _x7 + x2 + 3 và q ' (x)

= -7x6 + 2x. Cả hai đa thức đều bị chi phối bởi số hạng cao nhất của chúng. Khi x là một số dương lớn, cả q

(x) và q ' (x) sẽ âm, nhưng q (x) sẽ lớn hơn về giá trị tuyệt đối, vì số hạng trội của nó là bậc 7 trong khi

số hạng trội của q' ( x) là bậc 6. Nói cách khác, với x dương đủ lớn, ta sẽ có q (x) < q ' (x) . Chắc chắn

lập luận này là một lập luận tổng quát: nếu đồ thị của y = q (x) đi qua trục x, thì bất đẳng thức q ' (x) ::;

q (x) sẽ không đúng với mọi x. Vì vậy trường hợp này là không thể.
Machine Translated by Google

36 CHƯƠNG 2 CÁC CHIẾN LƯỢC CHO I CÁC VẤN ĐỀ NỔI BẬT

• Đồ thị của y = q (x) nằm bên dưới (hoặc vừa chạm) trục x. Khi đó, vì q (x) là một đa thức nên

nó phải có bậc chẵn và một hệ số âm.

- 200 sẽ có đúng loại đồ thị. Tuy nhiên, lập


số 8

Ví dụ: q (x) = -5x

luận trước đó vẫn được áp dụng: với x dương đủ lớn, chúng ta sẽ có


'
q (x) < q(x). Vì vậy trường hợp này cũng không được.
• Đồ thị của y = q (x) nằm trên (hoặc vừa chạm) trục x, tức là q (x) 2 O.
Trường hợp này phải đúng, vì chúng tôi đã loại bỏ các khả năng khác! Làm thế nào để hiểu

được lý do tại sao lập luận trước đó không dẫn đến một sự lừa gạt là điều có thể hiểu
được. Bây giờ q (x) phải có bậc chẵn với hệ số hàng đầu là dương, + lO có dạng đồ thị phù
Z
hợp. Nhưng bây giờ, q '(x) = 2x. ví dụ: q (x) = x
Khi x là một số dương lớn thì q (x) > q '(x) vì các hệ số đứng đầu đều dương. Đó là chìa khóa.

Dù sao, chúng tôi đã cố gắng chứng minh một khẳng định rất hay:

Nếu q (x) là đa thức với hệ số thực thỏa mãn q (x) 2 q '(x) với mọi x thực thì q
(x) có bậc chẵn với hệ số dẫn dương và luôn không âm.

Thực tế này sẽ cung cấp cho chúng tôi sự tự tin để kết thúc vấn đề ban đầu. Chúng ta biết rằng
q (x) = p (x) - p "(x) có bậc chẵn với hệ số dẫn dương, do đó điều này cũng đúng với p (x).
Vì vậy, chúng tôi đã giảm vấn đề ban đầu thành một cách có vẻ dễ dàng hơn
một:

Chứng minh rằng nếu p (x) có hoành độ chẵn với hệ số dẫn dương và p (x) - p "(x)

2 Trong số hoặc tất cả x thực thì p (x) 2 Trong số hoặc tất cả x thực.

Ví dụ 2.2.8 (Putnam 1 990) Tìm tất cả các hàm f chức năng phân biệt liên tục có giá trị thực trên
dòng thực sao cho với mọi x,
X
2 2
(f (x)) = fo [(f (t) f + (f '(t)) ] dt + 1 990.

Giải pháp từng phần: Điều tồi tệ nhất của vấn đề này là gì? Nó chứa đựng cả sự khác biệt và tích

hợp. Phương trình vi phân đã đủ tệ, nhưng phương trình vi phân tích phân còn tệ hơn! Vì vậy, chiến

lược là hiển nhiên: làm cho nó dễ dàng hơn bằng cách đưa ra cả hai vế của phương trình đối với x:

2 2
[(f (t))
:) f (x)) = : x (fo x + (f '(t)) 2] dt + 1 990) .

Vế trái chỉ là 2f (x) f '(x) (theo Quy tắc Chuỗi) và vế phải trở thành (f (x))
2 2
+ (f '(x)) (đạo hàm của hằng số 1 990 biến mất).
Bây giờ chúng tôi đã rút gọn vấn đề thành một phương trình vi phân,

2 2
2f (x) f '(x) = (f (x)) + (f '(x)) .

Điều này không đẹp (chưa), nhưng đẹp hơn nhiều so với những gì chúng tôi bắt đầu. Bạn có thấy phải làm gì
tiếp theo không?
Machine Translated by Google
2.2 CÁC CHIẾN LƯỢC CHO G ETTING BẮT ĐẦU 37

Các vấn đề và bài tập Đối với

hầu hết những điều này, nhiệm vụ của bạn chỉ là thử nghiệm và làm bẩn tay và đưa ra các phỏng đoán. Đừng lo lắng
về việc chứng minh những phỏng đoán của bạn tại thời điểm này. Ý tưởng là hãy thả lỏng và không bị gò bó, làm
quen với việc động não. Một số câu hỏi yêu cầu bạn phỏng đoán một công thức hoặc một thuật toán. Ở phần sau,
chúng tôi muốn nói đến một thủ tục tính toán không phải là một công thức đơn giản, nhưng tuy nhiên khá dễ giải
thích và thực hiện. Ví dụ, f (n) = VnT là một công thức nhưng sau đây là một thuật toán:

Tính tổng của mọi chữ số thứ ba của khai triển cơ số 3 của n. Nếu tổng là số chẵn thì
đó là f (n). Nếu không, hãy bình phương nó, và đó là f (n).

Chúng tôi sẽ trở lại nhiều vấn đề sau và phát triển các cách chứng minh chặt chẽ hơn. Nhưng điều quan trọng là
bạn phải làm bẩn bàn tay của mình ngay bây giờ và bắt đầu nghĩ về chúng. Chúng tôi xin nhắc lại: việc thực sự
“giải quyết” các vấn đề lúc này không quan trọng. Có một loạt các bài toán đã được giải quyết một phần, có thể
là những phỏng đoán đúng trong tâm trí bạn không chỉ là OK mà còn rất lý tưởng. Các vấn đề "backburner" lên men
một cách vui vẻ, khiến não bạn say mê với những ý tưởng. Một số quá trình lên men này là có ý thức, một số thì không.
Một số ý tưởng sẽ hoạt động, những ý tưởng khác sẽ thất bại. Càng nhiều ý tưởng, càng tốt! (Nhân tiện, một số
vấn đề đã được cố tình chọn để tương tự với một số ví dụ. Hãy nhớ rằng, một trong những chiến lược định hướng
của bạn là hỏi, "Có vấn đề tương tự không?")

2.2.9 Định nghĩa f (x) = 1 / (1 - x) và biểu thị r lần lặp của hàm mô hình và phỏng đoán một quy tắc kết nối ba số này.

f theo r [xem phương trình (2) ở trang 32].

Máy tính f I999 (2000).

2.2.10 (Putnam 1990) Để

To = 2, TI = 3, T2 = 6,

và cho n :::: 3,
2.2.13 Mặt phẳng được chia bởi n đường thẳng ở vị trí chung (không

có hai đường thẳng song song; không có ba đường thẳng nào gặp nhau

tại một điểm) thành bao nhiêu miền?


Một số điều khoản đầu tiên là
2.2.14 Đường tròn lớn là đường tròn được vẽ trên một mặt cầu là

2,3,6, 1 4, 40, 1 52, 784, 5168, 40576,363392. "đường xích đạo;" tức là tâm của nó cũng là tâm của mặt cầu. Có n

hình tròn lớn trên một hình cầu, không có ba hình tròn nào gặp nhau
Tìm công thức cho T " có dạng 'ft, = A" + B ", trong đó (An) và
tại bất kỳ điểm nào. Họ chia khối cầu thành bao nhiêu vùng?
(B ,, ) là các dãy đã biết.

2.2.11 Gọi N là các số tự nhiên


2.2.15 Với mỗi số nguyên n> I, tìm các số nguyên dương x và y khác
{1,2, 3, 4 ,. . . }. Xét một hàm f thỏa mãn f (l) = 1, / (2n) = f nhau sao cho
(n) và f (2n
đơn
+ 1)
giản
= f
cho
(2n)
f (+ N).
1 với
Thuật
mọi toán
n E N.
củaTìm
bạnmột
chỉthuật
nên dài
toán
tối
III - + -
đa một câu. = -. xyn

2.2.12 Nhìn, vẽ hoặc dựng một số (ít nhất tám) khối đa diện, tức 2.2.16 Với mỗi số nguyên dương n, tìm nghiệm nguyên dương XI

là các khối rắn có các mặt là đa giác. Dưới đây là hai ví dụ: một ,X2 ,... ,Xn vào phương trình

hình hộp và một hình "ell" ba chiều. Đối với mỗi hình đa diện, hãy
II - Tôi

đếm số lượng đỉnh (hình sao), mặt và cạnh. Ví dụ, hình hộp có tám + - + . . . + - + XI = 1 .
X2 x "X IX2" 'X "
đỉnh, sáu mặt và 12 cạnh, trong khi hình ell có 12 đỉnh, tám mặt

và 18 cạnh. Tìm một 2.2.17 Xét một tam giác vẽ trên mặt phẳng tọa độ, tất cả các đỉnh

của chúng đều là điểm mạng (điểm


Machine Translated by Google

38 CHƯƠNG 2 CÁC CHIẾN LƯỢC CHO I CÁC VẤN ĐỀ NỔI BẬT

với tọa độ nguyên). Gọi A, B và I lần lượt là biểu thị với n là bất kỳ số nguyên dương nào.

diện tích, số điểm mạng biên và số điểm mạng bên trong của (c) Thực nghiệm và phỏng đoán sự tổng quát hóa trên:
tam giác này. Ví dụ, tam giác với các đỉnh tại (0,0), Với mỗi số nguyên dương k, có một công thức đẹp cho
(2,0), (1,2) có (xác minh!) A = 2, B = 4, 1 = I.

Tôi k + 2k + 3 * + ·· · + nk?
Bạn có thể tìm thấy mối quan hệ đơn giản giữa A, B và I
phù hợp với bất kỳ tam giác nào có các đỉnh là các điểm
2.2.22 Định nghĩa s (n) là số cách viết số nguyên dương n
mạng không?
dưới dạng tổng có thứ tự của ít nhất một số nguyên dương.

2.2.18 (Olympic Toán học Anh 1 996) Xác định Ví dụ,

4 = 1 + 3 = 3 + 1 = 2 + 2 = 1 + 1 + 2

l �J J (n = I, 2, ...). q (n) = l = 1 + 2 + 1 = 2 + 1 + 1 = 1 + 1 + 1 + 1 ,

Xác định (kèm theo chứng minh) tất cả các số nguyên


do đó s (4) = 8. Phỏng đoán một công thức tổng quát.
dương n mà q (n) > q (n + I).
2.2.23 Tìm vô số nghiệm nguyên dương của phương
2.2.19 Bay Area Rapid Food bán gà cốm.
trình
Bạn có thể mua gói bảy hoặc gói II.
Số nguyên n lớn nhất sao cho không có cách nào để mua
chính xác n cốm là bao nhiêu? Bạn có thể khái quát điều này?
2.2.24 Lưu ý rằng
2.2.20 Cho n là số nguyên dương.

I. Viết ra tất cả các cặp số nguyên có thứ tự (a, b),


ở đâu và

• I Sa < b S n;

• a + b > n;
Điều tra, khái quát hóa, phỏng đoán.
• a và b tương đối nguyên tố (chia sẻ không có com
2.2.25 (Thổ Nhĩ Kỳ 1 996) Để
số chia mon ngoài I).
1996
2. Với mỗi cặp có thứ tự (a, b), tính
nghịch đảo của tích hai số, nói cách J] (I + nx3n) = I + a1x * '+ a2x * 2 +. .. + amx * m,

khác, tính 1 / trên 3. Cộng tất cả các


ở đâu al, a2 , ..., am là số khác và kl < k 2 <... <
phân số này. km · Tìm một năm 1996.

Ví dụ: nếu n = 6, các cặp có thứ tự là 2.2.26 Vấn đề về bộ sưu tập. Các bức tường của một phòng

trưng bày bảo tàng tạo thành một đa giác với n cạnh, không
(1,6), (5,6), (2,5), (3,5), (4,5), (3,4),
nhất thiết phải đều đặn hoặc thậm chí là lồi. Các vệ binh

và tổng tương ứng là được đặt tại các điểm lo cố định bên trong phòng trưng
bày. Giả sử rằng lính canh có thể quay đầu lại, nhưng
Tôi Tôi Tôi Tôi Tôi Tôi Tôi

=
+ + (; 30 + + + 15 20 không đi lại, thì số lượng lính canh tối thiểu cần thiết
10 12 2:.
để đảm bảo rằng mỗi inch tường đều có thể quan sát được?
Điều tra điều gì xảy ra với các giá trị khác của n và Khi n = 3 hoặc n = 4, rõ ràng là chỉ cần một người bảo vệ
phỏng đoán điều gì đó. là đủ. Điều này cũng đúng với n = 5, mặc dù phải mất một

2.2.21 vài hình ảnh để thuyết phục (trường hợp không lồi là sự
quan tâm). Nhưng với n = 6, người ta có thể tạo các phòng
(a) Tìm một công thức đơn giản hay cho
trưng bày yêu cầu hai người bảo vệ. Đây là hình ảnh của

1 + 2 + 3 + ··· + n, trường hợp n = 5 và n = 6. Các dấu chấm chỉ ra vị trí của lính canh.

với n là bất kỳ số nguyên dương nào.

(b) Tìm một công thức đơn giản hay cho

13 + 23 + 33 +. . . + n3 ,
Machine Translated by Google

2.3 CÁC PHƯƠNG PHÁP ARG U MENT 39

Bạn có thể khám phá ra một biểu đồ chung cho số lính số x. Ví dụ: {3.I} = 3 và {4,7} = 5.

canh, như một hàm của n không? Bây giờ hãy xác định fen) : = n + {y'n}. Chứng minh rằng,
với mọi số nguyên dương m, dãy
2.2.27 Vấn đề Josephus. Một nhóm gồm n người đang đứng

trong một vòng tròn, được đánh số liên tiếp theo kim đồng
f (m), f (f (m)), f (f (f (m))) ,. ..
hồ từ I đến n. Bắt đầu với người số 2, chúng tôi di chuyển

từng người khác, tiếp tục theo chiều kim đồng hồ. Ví dụ, không bao giờ chứa bình phương của một số nguyên. (So sánh

nếu n = 6, những người được loại bỏ theo thứ tự 2,4,6,3, I điều này với Ví dụ 2.2.5 ở trang 31.)

và người cuối cùng còn lại là # 5. Gọi j (n) là người cuối


2.2.33 Hoàn thành giải pháp bắt đầu trong Ví dụ 2.2.7 trên
cùng còn lại.
trang 34.

(a) Tính j (n) cho n = 2,3, ... 25. (b)


2.2.34 Hoàn thành giải pháp bắt đầu trong Ví dụ 2.2.8 trên
Tìm cách tính j (n) cho bất kỳ số nguyên dương nào n > trang 36.
I. Bạn có thể không nhận được "tốt" cho mula , nhưng
2.2.35 Các câu chuyện cảnh báo. Nó rất dễ bị dụ dỗ bởi sự
hãy cố gắng tìm một thuật toán thuận tiện dễ tính
dễ dàng của thử nghiệm-phỏng đoán. Nhưng đây chỉ là một
toán bằng tay hoặc máy.
phần của điều tra toán học. Đôi khi, một cuộc điều tra
2.2.28 Gọi g (n) là số mười lẻ trong hàng của Tam giác tương đối không cẩn thận sẽ khiến chúng ta lạc lối.
Pascal bắt đầu bằng I, n . . .. Đối với đề thi, g (6) = 4, Đây là hai ví dụ. Có rất nhiều kỳ thi khác xin vui lòng
kể từ hàng như thế này; xem [17J để có một cuộc thảo luận tuyệt vời.
2
1,6, 15,20, 15,6, I (a) Cho fen) : = n + n +41. Fen) là một số nguyên tố cho tất cả

số nguyên dương n?
chứa bốn số lẻ. Phỏng đoán một fonnula (hoặc một cách tính

toán dễ dàng) g (n) . (b) Gọi mười) là số diện tích tối đa mà bạn có thể chia

một hình tròn thành khi bạn đặt n điểm trên chu vi
2.2.29 (Putnam 1991) Với mỗi số nguyên n 2 ': 0, đặt Sen) = n
và vẽ tất cả các đoạn thẳng có thể nối các điểm.
- m2 , với m là số nguyên lớn nhất với m2 : S
Thật dễ dàng để kiểm tra (xác minh!)
n. Xác định một dãy số (ak); = O bởi ao = A và = ak + S
ak + (ak) với k 2 ': O. Với số nguyên dương nào
1 gers A cuối cùng dãy này là hằng số? t (I) = I, t (2) = 2, t (3) = 4, t (4) = 8, t (5) = 16.

2.2.30 Hoàn thành giải pháp bắt đầu trong Đề thi 2.2.4. n 1
Phỏng đoán rằng ten) = 2 thực tế là không

thể tránh được. Tuy nhiên, t (6) bằng 31, không phải 32 (một
2.2.31 Hoàn thành giải pháp bắt đầu Trong kỳ thi 2.2.5. lần nữa, hãy xác minh!), Vì vậy một cái gì đó khác đang xảy ra.

Dù sao, bạn có thể suy ra fonnula chính xác cho mười)

2.2.32 Gọi {x} biểu thị số nguyên gần nhất với số thực không?

2.3 Phương pháp lập luận

Như chúng tôi đã nói trước đó, giải pháp cho mọi vấn đề bao gồm hai phần: cuộc điều tra,
trong đó bạn khám phá những gì đang xảy ra và lập luận, trong đó bạn thuyết phục người khác
(hoặc có thể chỉ chính bạn!) Về khám phá của bạn. Điều tra ban đầu của bạn có thể gợi ý một
phương pháp lập luận dự kiến. Tất nhiên, đôi khi một vấn đề chia thành các trường hợp hoặc
vấn đề con, mỗi vấn đề có thể yêu cầu các phương pháp lập luận hoàn toàn khác nhau.

Lập luận phải chặt chẽ và rõ ràng. Tuy nhiên, "chặt chẽ" và "rõ ràng" đều là những
thuật ngữ chủ quan. Chắc chắn, bạn nên tránh để lộ ra những sai sót hoặc khoảng trống logic
trong cách hát của bạn. Tất nhiên, điều này nói thì dễ hơn làm. Lập luận càng phức tạp thì
càng khó xác định xem nó có đúng về mặt logic hay không. Tương tự như vậy, tất nhiên, bạn
nên tránh những tuyên bố mơ hồ mơ hồ, nhưng sự rõ ràng cuối cùng của lập luận của bạn phụ thuộc vào
Machine Translated by Google

40 CHƯƠNG 2 CÁC CHIẾN LƯỢC CHO I CÁC VẤN ĐỀ NỔI BẬT

nhiều đối tượng dự định của nó hơn bất kỳ thứ gì khác. Ví dụ, nhiều nhà toán học
chuyên nghiệp sẽ chấp nhận "Tối đa hóa dây cân bằng!" như một giải pháp đầy đủ và rõ
ràng cho vấn đề Hành động thích hợp (Ví dụ 2. 1 .9 trên trang 21).
Cuốn sách này quan tâm nhiều đến quá trình điều tra và khám phá hơn là với lập
luận toán học bóng bẩy. Tuy nhiên, một ý tưởng tuyệt vời sẽ vô dụng nếu nó không thể
được truyền đạt cho bất kỳ ai khác. Furthennore, sự thông thạo trong lập luận toán
học sẽ giúp bạn định hướng và sửa đổi điều tra của mình.6 Ít nhất, bạn nên cảm thấy
thoải mái với ba kiểu lập luận riêng biệt: suy luận thẳng thắn (còn được gọi là
"chứng minh trực tiếp"), lập luận bằng mâu thuẫn và quy nạp toán học. Chúng ta sẽ khám
phá chúng dưới đây, nhưng trước tiên, một vài lưu ý ngắn gọn về phong cách.

Các từ viết tắt phổ biến và các quy ước về phong cách 1.

Hầu hết các lập luận toán học tốt đều bắt đầu bằng những tuyên bố rõ ràng về dấu
ngoặc kép và kết luận. Kết thúc thành công của đối số thường được đánh dấu
bằng một ký hiệu. Chúng tôi sử dụng biểu tượng Halmos, nhưng một số lựa chọn
khác là ký hiệu ab

QED cho từ tiếng Latinh quod erat biểu tình ("đã bị quỷ đeo bám") hoặc
tiếng Anh "được thực hiện khá thanh lịch"; AWD cho "và chúng tôi đã hoàn

thành"; W5 cho "đó là những gì chúng tôi muốn."

2. Giống như giải trình thông thường, các lập luận toán học phải đầy đủ sen
danh từ và động từ. Các động từ toán học phổ biến là

¥, �, 2, <,>, E, C, = *, �.
(Bốn giá trị trung bình cuối cùng "là một phần tử của," "là một tập con của",
"ngụ ý" và "tương đương với".)

3. Các phương trình phức tạp phải luôn được hiển thị trên một dòng và được dán
nhãn nếu được tham chiếu sau. Ví dụ:
2
dx = Vi, (3)
1: người yêu cũ

4. Thông thường, khi bạn khám phá bước cuối cùng của một lập luận (hoặc lập luận
phụ), bạn muốn đánh dấu điều này cho khán giả rõ ràng. Các chữ viết tắt TS và
ISTS ("hiển thị" và "đủ để hiển thị") đặc biệt hữu ích cho mục đích này.

5. Một chút ký hiệu hay, được mượn từ khoa học máy tính và dần trở nên phổ biến
hơn trong toán học, là ": =" for "được định nghĩa là." Ví dụ, A : = B + C giới
thiệu một biến mới A và định nghĩa nó là tổng của các biến đã được xác định B
và C. Hãy nghĩ về dấu hai chấm như điểm của một mũi tên; chúng tôi luôn phân
biệt giữa trái và phải. Điều ở bên trái của

6 Phần này cố tình ngắn gọn. Nếu bạn muốn một cách xử lý dễ dàng hơn đối với lập luận logic và
các phương pháp chứng minh, bao gồm quy nạp toán học, chúng tôi đề xuất Chương 0 và 4.1 của (15).
Machine Translated by Google
2.3 CÁC ĐẠO ĐỨC CỦA ARG U MENT 41

cái " : = " là định nghĩa mới (thường là một biến đơn giản) và điều ở bên

phải là một biểu thức sử dụng các biến đã được xác định. Xem Ví dụ 2.3.3
để biết ví dụ về ký hiệu này.
6. Một lập luận chính thức nghiêm ngặt có thể giải quyết nhiều trường hợp tương tự về mặt
logic. Đôi khi, việc chỉ ra một trường hợp hoặc ví dụ minh họa cũng rõ ràng. Khi điều
này xảy ra, chúng tôi luôn cảnh báo khán giả bằng WLOG ("không mất thông tin chung").
Chỉ cần đảm bảo rằng bạn thực sự có thể lập luận cái cụ thể và thực sự chứng minh được
cái chung. Ví dụ, giả sử bạn định chứng minh rằng 1 + 2 + 3 + ... + n = n (n + 1) / 2
với mọi số nguyên dương n. Sẽ là sai nếu lập luận, "WLOG, cho n = 5. Khi đó 1 + 2 + 3 +
4 + 5 = 15 = 5 · 6/2. QED." Lập luận này chắc chắn không phải là chung chung!

Khấu trừ và Logic tượng trưng "Khấu trừ"

ở đây không liên quan gì đến Sherlock Holmes. Còn được gọi là "bằng chứng trực tiếp", nó chỉ là
hình thức lập luận đơn giản nhất về mặt logic. Một đối số suy diễn có dạng "Nếu P, thì Q" hoặc
"P === :::;. Q" hoặc "P ngụ ý Q." Đôi khi cấu trúc tổng thể của một lập luận là suy diễn, nhưng
các phần nhỏ hơn sử dụng các phong cách khác.
Nếu bạn đã cô lập bước áp chót, thì bạn đã giảm vấn đề thành câu suy luận đơn giản

Sự thật của bước áp chót === ::: ;. Kết luận.

Tất nhiên, việc thiết lập bước áp chót có thể liên quan đến các hình thức lập luận khác.
Đôi khi cả p === :::;. Q và Q === :::;. P đều đúng. Trong trường hợp này, chúng ta nói rằng P và Q tương

đương về mặt logic, hoặc P {= ::} Q. Để chứng minh tính tương đương, trước tiên chúng ta chứng minh một hướng

(giả sử P === * Q) và sau đó là Q nghịch đảo của nó === * P.

Theo dõi hướng của các hàm ý của bạn. Nhớ lại rằng p === :::;. Q không tương đương về mặt
logic với Q ngược lại của nó === * P. Ví dụ, hãy xem xét câu nói đúng "Chó là động vật có vú."
Điều này tương đương với "Nếu bạn là một con chó, thì bạn là động vật có vú."
Chắc chắn, câu chuyện "Nếu bạn là động vật có vú, thì bạn là chó" là không đúng!
Tuy nhiên, đồ thị của P === * Q là câu lệnh (không phải Q) === :::;. (không phải P), và hai
giá trị này tương đương về mặt logic. Kết cấu của câu nói rõ ràng đúng "Chó là động vật có vú"
là câu nói đúng "Động vật không có vú không phải là chó".
Nhìn chung, hầu hết các lập luận có cấu trúc suy diễn đơn giản. Nhưng cục bộ, các phần
riêng lẻ của một lập luận có thể có nhiều dạng. Bây giờ chúng ta đi đến hình thức phổ biến nhất
của những hình thức thay thế này, lập luận bằng mâu thuẫn.

Lập luận theo mâu thuẫn

Thay vì cố gắng chứng minh điều gì đó một cách trực tiếp, chúng ta bắt đầu bằng cách giả định
rằng điều đó là sai, và chỉ ra rằng giả định này dẫn chúng ta đến một kết luận vô lý. Một lập
luận mâu thuẫn thường hữu ích để chứng minh trực tiếp rằng điều gì đó không thể xảy ra. Đây là
một ví dụ lý thuyết số đơn giản.

Ví dụ 2.3.1 Chứng tỏ rằng

(4)
Machine Translated by Google

42 CHƯƠNG 2 CHIẾN LƯỢC ĐỐI VỚI CÁC VẤN ĐỀ ĐIỀU TRA VẤN ĐỀ

không có nghiệm nguyên dương.

Giải pháp: Chúng tôi muốn chứng minh rằng đẳng thức (4) không thể đúng. Vì vậy, giả sử, + b
2
ngược lại, điều đó (4) là đúng. Nếu b + 1 = a2 , 2 thì b < a, và a - b2 = b + 1. Như trong

Ví dụ 2.2.7 trên trang 34, chúng tôi sử dụng chiến thuật hữu ích của bao thanh toán, điều này mang lại

(a - b) (a + b) = b + l. (5)

Vì a > b 2 1 nên ta phải có a - b 2 1 và a + b 2 2 + b nên vế trái của (5) lớn hơn hoặc bằng 1 .

(b + 2), lớn hơn vế phải của (5). Đây là điều


là đúng,
không trên
thể xảy
thực
ra,
tế vì
phải
vậylà
giả
sai.
định ban đầu, rằng (4)

Đây là một bằng chứng bất khả thi khác, một lập luận cổ điển từ thời Hy Lạp cổ đại.

Ví dụ 2.3.2 Chứng tỏ rằng V2 không hữu tỉ.

Giải pháp: Tìm mâu thuẫn, giả sử V2 là hợp lý. Khi đó V2 có thể được biểu diễn dưới dạng
thương của hai số nguyên dương (không mất tính tổng quát, chúng ta có thể giả sử cả tử số và mẫu
số đều dương). Bây giờ chúng ta sẽ sử dụng nguyên tắc cực đoan: trong tất cả các cách có thể để
làm điều này, hãy chọn thương số mà mẫu số là nhỏ nhất.

Do đó chúng ta viết V2 = alb trong đó a, b EN, trong đó b càng nhỏ càng tốt. Điều này có
nghĩa là phân số alb ở "số hạng thấp nhất", vì nếu không, chúng ta có thể chia cả a và b cho một
số nguyên dương lớn hơn 1, làm cho cả a và b nhỏ hơn, mâu thuẫn với tính nhỏ nhất của b. Đặc
biệt, không thể có cả a và b đều.
2
Tuy nhiên, V2 = a I b ngụ ý rằng 2b2 = a2 , vì vậy a là chẵn (vì nó bằng 2
2
lần một số nguyên). Nhưng điều này ngụ ý rằng a cũng phải chẵn (vì nếu a là lẻ,
cũng sẽ là số lẻ), và do đó a bằng 2 lần một số nguyên. Vì vậy, chúng ta có thể viết a = 2t,
trong đó tEN. Thay thế, chúng tôi nhận được

2 2b
= a2 = (2t) 2 = 4t2 ,
2
2 nên b = 2t . Nhưng bây giờ bằng cách suy luận chính xác, chúng tôi kết luận rằng b cũng là số
chẵn! Điều này là không thể, vì vậy chúng tôi đã mâu thuẫn với giả định ban đầu, rằng V2
là hợp lý. _

Lập luận bằng sự mâu thuẫn cũng có thể được sử dụng để chứng minh những nhận định "tích cực".
Nghiên cứu ví dụ tiếp theo.

Ví dụ 2.3.3 (Hy Lạp 1 995) Nếu a, b, c, d, e là các số thực sao cho phương trình

ax2 + (c + b) x + (e + d) = 0

có nghiệm nguyên lớn hơn 1, chứng tỏ rằng phương trình


4 3 2 + bx
cái rìu + cx + dx + e = 0

có ít nhất một gốc thực.

Lời giải: Giả thuyết là P (x): = ax2 + (c + b) x + (e + d) = 0 có nghiệm nguyên lớn hơn 1 và
kết luận mong muốn là Q (x): = ax4 + bx3 + cx2 + dx + e = 0
Machine Translated by Google
2.3 CÁC ĐẠO ĐỨC CỦA BIỆN PHÁP 43

có ít nhất một gốc thực. Giả sử kết luận đó là sai, tức là Q (x) không có nghiệm
nguyên thực. Như vậy Q (x) luôn dương hoặc luôn âm với mọi x thực. Không mất tính
tổng quát, giả sử rằng Q (x) > 0 với mọi x thực, trong trường hợp đó a > O.
Bây giờ chiến lược của chúng tôi là sử dụng bất đẳng thức liên quan đến Q để tạo ra một mâu

thuẫn, có lẽ bằng cách sử dụng giả thuyết về P theo một cách nào đó. Hai polynomi als có liên quan
như thế nào? Chúng tôi có thể viết

3 2 +
Q (x) = ax4 + bx dx + e + cx
4
= cái rìu
+ (c + b) x2 + (e + d) + bx3 - b2 + dx - d,

vì thế

Q (x) = P (2) + (x - l) (bx2 + d). (6)

Bây giờ cho Y là một nghiệm nguyên của P. Theo giả thiết, y > 1. Do đó, nếu chúng ta đặt u:

= ..; Y, chúng ta có u > 1 và P (u2) = O. Thay x = u vào (6) thu được

2
Q (u) = P (u2) + (u - 1) (bu2 + d) = (u - 1) (bu + d).

Nhớ lại rằng chúng ta đã giả sử Q luôn dương, do đó (u - 1) (bu2 + d) > O. Nhưng chúng ta cũng có
thể cắm x = -u vào (6), và chúng tad).
nhận được Q (-u) = P ( u2) + (-u - 1) (bu2 + d) = (-u - 1) (bu2 +

2 2
Vì vậy, bây giờ chúng ta phải có cả hai (u - 1) (bu + d) > 0 và (-u - 1) (bu + d) > O. Nhưng
điều này là không thể, vì u - 1 và -u - 1 lần lượt là dương và âm (hãy nhớ, u > 1).
Chúng tôi đã đạt được mâu thuẫn của mình, vì vậy giả định ban đầu của chúng tôi rằng
Q luôn luôn là số dương phải là sai. Ta kết luận rằng Q phải có ít nhất một gốc thực. _

Tại sao sự mâu thuẫn lại hoạt động trong ví dụ này? Chắc chắn, có nhiều cách khác để chứng minh

rằng một đa thức có ít nhất một căn thực. Điều đã giúp chúng tôi trong vấn đề này là thực tế là sự

phủ định của kết luận tạo ra một cái gì đó rất dễ thực hiện. Một khi chúng ta giả định rằng Q không
có gốc thực, chúng ta có một bất đẳng thức tốt đẹp mà chúng ta có thể giải quyết một cách hiệu quả.

Khi bạn bắt đầu suy nghĩ về một vấn đề, điều đáng hỏi luôn là

Điều gì xảy ra nếu chúng ta phủ định kết luận? Chúng ta sẽ có thứ gì đó mà
chúng ta có thể làm việc dễ dàng?

Nếu câu trả lời là "có", thì hãy thử tranh luận bằng cách mâu thuẫn. Nó sẽ không hoạt động luôn
luôn, nhưng đó là bản chất của cuộc điều tra. Để trở lại sự tương tự về hoạt động leo núi cũ của
chúng tôi, chúng tôi đang cố gắng leo lên. Đôi khi kết luận có vẻ giống như một bức tường kính thẳng

đứng, nhưng sự phủ định của nó có rất nhiều chỗ đứng. Khi đó, phủ định dễ điều tra hơn kết luận. Tất

cả đều nằm trong cùng một nguyên tắc chiến lược cơ hội cơ bản:

Bất cứ điều gì làm phiền quá trình điều tra của bạn đều đáng làm.

Ví dụ tiếp theo liên quan đến một số lý thuyết số cơ bản, một chủ đề mà chúng tôi phát triển
chi tiết hơn trong Chương 7. Tuy nhiên, điều quan trọng là phải học ít nhất một lượng tối thiểu lý

thuyết số "tồn tại cơ bản" càng sớm càng tốt. Chúng ta sẽ thảo luận về một số
Machine Translated by Google

44 CHƯƠNG 2 CHIẾN LƯỢC CHO I CÁC VẤN ĐỀ ĐIỀU CHỈNH

các vấn đề lý thuyết trong chương này và chương tiếp theo. Đầu tiên chúng ta hãy giới thiệu một số ký

hiệu cực kỳ hữu ích và quan trọng.

Gọi m là số nguyên dương. Nếu a - b là bội của m, ta viết

a = b (mod m)

(đọc "a là đồng dư với b modulo m"). Ví dụ,

10 = 1 (mod 3), 17 = 102 (mod S ) , 2 = - 1 (mod 3), 32 = 0 (mod 8).

Ký hiệu này, do Gauss phát minh, rất tiện lợi. Dưới đây là một số sự kiện mà bạn nên xác minh ngay lập

tức:

• Nếu bạn chia a cho b và nhận phần dư là r thì a = r (mod b).


• Câu lệnh a = b (mod m) tương đương với việc nói rằng tồn tại số nguyên
k sao cho a = b + mk. • Nếu a = b (mod m) và c = d (mod m) thì a + c =

b + d (mod m) và ac = bd (mod m).

Ví dụ 2.3.4 Chứng minh rằng nếu p là số nguyên tố thì modulo p, mọi số khác không đều có một nghịch

đảo nhân duy nhất ; tức là, nếu x không phải là bội của p thì tồn tại một y duy nhất E {1, 2, 3, ...,

p -tương
1} sao cho
ứng làxy = 1 (mod p)
1,4,5,2,3,6 .]. [Ví dụ: nếu p = 7, các nghịch đảo nhân (mod 7) của 1, 2,3, 4, 5,6

Lời giải: Cho x E {I , p của


- I}p.
làBây
modulo
giờ hãy
khácxem
không
xét p;
các
tức
số là
( p
, -
2,I)
3 ,. .. , x không phải là mUltiple

x. x, 2x, 3x, ...


, (p - 1)

Ý tưởng mấu chốt: chúng tôi khẳng định rằng những con số này đều là modulo p riêng biệt. Chúng tôi sẽ

cho thấy điều này bằng sự mâu thuẫn. Ngược lại, giả định rằng chúng không khác biệt. Vậy thì chúng ta
phải có

ux = vx (mod p) , (7)

đối với u, v E {I, 2, 3, ... , p - I} với u I- v. Nhưng (7) ngụ ý rằng

ux - vx = (u - v) x = O (mod p) ;

nói cách khác, (u - v) x là bội số của p. Nhưng x không phải là bội số của p theo giả thuyết, và giá

trị của u - v khác 0 và nhiều nhất là p - 2 ở giá trị tuyệt đối (vì sự khác biệt lớn nhất sẽ xảy ra

nếu một trong số x, y là 1 và giá trị kia là p - 1). Do đó u - v cũng không thể là bội số của p . Vì p

là một số nguyên tố nên không thể có tích (u - v) x là bội số của p. Đó là mâu thuẫn mà chúng tôi mong

muốn: chúng tôi có, (p - I) x là môđun riêng biệt p. chứng minh rằng x, 2x, 3x, ...

Vì p - 1 số phân biệt đó cũng khác không modulo p (tại sao?), Chính xác một trong số chúng phải

bằng 1 modulo p. Do đó tồn tại một y duy nhất trong tập {1, 2, 3 , . .. , p- l } sao cho .xy = 1 (mod

p) .
_
Machine Translated by Google

2.3 CÁC ĐẠO ĐỨC CỦA BIỆN PHÁP 45

Quy nạp toán học

Đây là một phương pháp rất mạnh để chứng minh các khẳng định được "lập chỉ mục" bởi các số nguyên; Ví

dụ:

• Tổng các góc trong của n-gon bất kỳ là 1 80 (n - 2) độ. • Bất đẳng thức n! > 2
N
đúng với mọi số nguyên dương n ; :::: 4.

Mỗi khẳng định có thể được đặt dưới dạng

P (n) là true / hoặc tất cả các số nguyên n; :::: không,

trong đó P (n) là một câu lệnh liên quan đến số nguyên n và không là "điểm bắt đầu". Có hai hình thức

cảm ứng, tiêu chuẩn và mạnh mẽ.

Cảm ứng tiêu chuẩn

Đây là cách hoạt động của cảm ứng tiêu chuẩn:

1. Xác lập chân lý của P (không). Đây được gọi là "trường hợp cơ sở" và nó thường là
bài tập dễ dàng.

2. Giả sử rằng P (n) đúng với một số nguyên n tùy ý. Đây được gọi là giả thuyết cảm ứng. Sau
đó chứng tỏ rằng giả thuyết quy nạp ngụ ý rằng P (n + 1) cũng đúng.

Điều này đủ để chứng minh P (n) với mọi số nguyên n; :::: không, vì P (không) là đúng bởi (1) và (2)

ngụ ý rằng P (không + 1) là đúng, và bây giờ (2 ) ngụ ý rằng P (không + 1 + 1) là đúng, v.v.
Đây là một phép loại suy. Giả sử bạn sắp xếp vô số dominos trên một dòng, cor phản ứng với các

câu P (1), P (2) , ... Nếu bạn làm cho quân cờ domino đầu tiên rơi về bên

phải, thì bạn có thể chắc chắn rằng tất cả quân cờ dominos sẽ đổ, với điều kiện là bất cứ khi nào một

quân cờ domino rơi xuống, nó sẽ hạ gục người hàng xóm ở bên phải.

2 3 4 n n + l

Gõ cái đầu tiên xuống giống như thiết lập trường hợp cơ sở. Việc chỉ ra rằng quân cờ domino rơi
xuống đánh gục người hàng xóm của nó tương đương với việc chỉ ra rằng P (n) ngụ ý P (n + 1) với
mọi n; :::: 1.
Hãy sử dụng quy nạp để chứng minh hai ví dụ trên.

Ví dụ 2.3.5 Chứng minh rằng tổng các góc trong của n-gon bất kỳ là 1 80 (n - 2) độ.

Lời giải một phần: Trường hợp cơ sở (không = 3) là thực tế nổi tiếng rằng tổng các góc trong của

bất kỳ tam giác nào là 1 80 độ (xem 8.2.7 để biết các gợi ý về cách chứng minh).

Bây giờ giả sử rằng định lý đúng với n-gons đối với một số n; :::: 3. Chúng ta sẽ chỉ ra rằng
điều này ngụ ý sự thật đối với (n + 1 ) -gons; tức là, tổng các góc trong của (n + 1 ) -gon
bất kỳ là 1 80 (n + 1 - 2) = 1 80 (n - 1) độ.
Machine Translated by Google

46 CHƯƠNG 2 CHIẾN LƯỢC I ES ĐỐI VỚI CÁC VẤN ĐỀ ĐIỀU CHỈNH

Gọi S là một giác (n + 1 ) tùy ý với các đỉnh VI, V2, "., Vn + l. Phân tích S thành hợp của
tam giác T với các đỉnh VI, V2, V3 và n-gon U với các đỉnh VI, V3
,. •• , VII + I ·

Tổng các góc trong của S bằng tổng các góc trong của T (là 1 80 độ), cộng với tổng các góc trong
của U [là 1 80 (n - 2) theo giả thuyết quy nạp ]. Do đó, tổng là 1 80+ 1 80 (n - 2) = 1 80 (n -
1), chỉ
những gì chúng ta muốn.

Chúng tôi gọi đây là "giải pháp từng phần" vì lập luận của chúng tôi có một lỗ hổng nhỏ. Làm
thế nào để chúng ta biết rằng chúng ta có thể phân tích đa giác như trên để trích xuất một hình
tam giác "trên đường viền?" Không có vấn đề gì nếu đa giác là lồi, nhưng nếu nó là lõm, như trong
ví dụ, không phải lúc nào cũng có thể thực hiện được điều này. Xem Vấn đề 3.2. 1 1 cho một gợi ý.

1
Ví dụ 2.3.6 Chứng minh rằng nếu n là số nguyên lớn hơn 3 thì n! > 21 .

4
Lời giải: Trường hợp cơ sở, không = 4, rõ ràng là đúng: 4! > 2 . Bây giờ giả sử rằng
N! > 2 N
cho một số n. Chúng tôi muốn sử dụng điều này để chứng minh trường hợp "tiếp theo"; tức là chúng tôi

muốn chứng minh rằng (n + I)! > 2 n .


+ lHãy suy nghĩ một cách chiến lược: vế trái của giả thuyết quy nạp

là n !, và vế trái của "mục tiêu" là (n + I) !. Làm thế nào để đi từ cái này sang cái kia? Tất
nhiên là nhân cả hai vế của giả thuyết quy nạp với n + 1 .
Nhân cả hai vế của một bất đẳng thức với một số dương không thay đổi chân lý của nó, vì vậy chúng
ta nhận được
N
(n + I)! > 2 (n + 1).
N
Đây gần như là những gì chúng tôi muốn, vì phía bên phải của "mục tiêu" là 211+ 1 n = 2 · 2 và

+ 1 chắc chắn lớn hơn 2; tức là, (n + I)! > 21 1 (n + 1) > 2


N
I. 2 = 2
n +
.

Lập luận quy nạp có thể khá tinh tế. Đôi khi không rõ cái gì đóng vai trò của "chỉ mục" n.
Đôi khi bước di chuyển mấu chốt là một lựa chọn thông minh của biến này và / hoặc một phương pháp

di chuyển gọn gàng giữa P (n) và P (n + 1). Đây là một ví dụ.

Ví dụ 2.3.7 Mặt phẳng được chia thành các vùng bởi các đường thẳng. Chứng tỏ rằng luôn có
thể tô màu các vùng bằng hai màu để các vùng liền kề không bao giờ có cùng màu (như bàn cờ).

Giải pháp: Phát biểu của bài toán không liên quan trực tiếp đến số nguyên. Đã bao giờ,
khi chúng tôi thử nghiệm, chúng tôi tự nhiên bắt đầu với một dòng, sau đó là hai, v.v., vì vậy
Machine Translated by Google
Machine Translated by Google
48 CHƯƠNG 2 CHIẾN LƯỢC CHO I CÁC VẤN ĐỀ ĐIỀU TRA

Bây giờ giả thuyết quy nạp mạnh hơn của chúng tôi cho phép chúng tôi sử dụng chân lý của cả P (u) và
P (u - l), vì vậy

au + l + (u - 1) 2 = 2u2 + 2,

và do đó


Ví dụ trước cần chân trị của P ( u) và P ( u - I). Ví dụ tiếp theo sử dụng chân trị của P (k)
cho hai giá trị tùy ý.

Ví dụ 2.3.9 Một phân vùng của một tập hợp là một sự phân rã của tập hợp thành các tập con rời rạc.

Tam giác của một đa giác là sự phân chia đa giác thành các tam giác, tất cả các đỉnh của chúng đều

là đỉnh của đa giác ban đầu. Một đa giác đã cho có thể có nhiều tam giác khác nhau. Đây là hai phép
đo tam giác khác nhau của một chiếc 9 gon.

Cho một tam giác, gọi hai đỉnh là kề nhau nếu chúng được nối bởi cạnh của một tam giác. Giả sử

chúng ta quyết định tô màu các đỉnh của một đa giác tam giác. Chúng ta cần sử dụng bao nhiêu màu để
đảm bảo rằng không có hai đỉnh liền kề nào có cùng màu? Chắc chắn chúng ta cần ít nhất ba màu, vì
điều đó chỉ cần cho một hình tam giác duy nhất. Sự thật đáng ngạc nhiên là

Ba màu luôn đủ cho bất kỳ tam giác nào của một đa giác!

Chứng minh "Chúng ta sẽ quy về n, số đỉnh của đa giác. Phát biểu P (n) mà chúng ta muốn chứng

minh với mọi số nguyên n 2: 3 là Đối với bất kỳ tam giác nào của một n-gon, có thể là 3 tô màu các

đỉnh (tức là tô màu các đỉnh bằng cách sử dụng ba màu) để không có hai sắc thái cạnh nhau nào
có cùng màu.

Trường hợp cơ sở P (3) rõ ràng là đúng. Giả thuyết quy nạp là

P (3), P (4), ... , P (n)

tất cả đều đúng. Chúng tôi sẽ chỉ ra rằng điều này ngụ ý P (n + 1). Cho một tam giác (n + 1)

gon, chọn một cạnh bất kỳ và xét tam giác T với các đỉnh x, y, z chứa cạnh này. Tam giác này cắt (n +

1 ) -gon thành hai đa giác tam giác nhỏ hơn L và


Machine Translated by Google
Machine Translated by Google

50 CHƯƠNG 2 CHIẾN LƯỢC ĐỐI VỚI I CÁC VẤN ĐỀ ĐIỀU CHỈNH

Trích xuất đại lượng trong dấu ngoặc lớn và sử dụng giả thuyết quy nạp, sẽ đủ để chứng minh

1
:S '
y3n 2n+ +2
1 (2n l ) J3n + 3

Thật không may, sự bất bình đẳng này là sai! Ví dụ: nếu bạn cắm vào n = I, bạn nhận được

:S
1 4(3)
v'3 có 1 J6 '

nghĩa là V2 : s 4/3, điều này rõ ràng là vô lý.

Chuyện gì đã xảy ra thế? Chúng tôi đã sử dụng những suy nghĩ mơ mộng và bị đốt cháy. Nó xảy ra
mọi lúc. Bất đẳng thức mà chúng ta muốn chứng minh, trong khi đúng, là rất yếu (tức là khẳng định

rất ít), đặc biệt là đối với n nhỏ. Giả thuyết khởi đầu của P (I) quá yếu dẫn đến P (2), Giải pháp:
củng cố giả thuyết và chúng
ngay ta đãHãy
từ đầu. chết.
để chúng tôi thay thế 3n bằng

3n + I. Biểu thị câu lệnh

(�) (�) . . . (2�: I ) : s �

bởi Q (n). Chắc chắn, Q (I) là đúng; trên thực tế, nó là một đẳng thức (1/2 = 1 / v4), sắc nét như

một bất đẳng thức có thể có! Vì vậy, chúng ta hãy thử chứng minh Q (n + I) bằng cách sử dụng Q (n)

làm giả thuyết quy nạp. Như trước đây, chúng tôi thử đại số hiển nhiên và hy vọng rằng chúng tôi có

thể chứng minh bất đẳng thức

1
:S .
J3n 1+ (2n
1 2n
+ +
l 2
) J3n + 4

Bình phương và nhân chéo làm giảm điều này thành bất đẳng thức (3n

+ 4) (4n2 + 4n + I) : s 4 (n2 + 2n + 1) (3n + I),

làm giảm (sau một số phép nhân tẻ nhạt) xuống 19n : s 20n, và điều đó chắc chắn đúng.
Vậy là chúng ta đã hoàn thành. _

Các vấn đề và bài tập

2
2.3.11 Cho a, b, c là các số nguyên thỏa mãn a2 + b2 = c • trong cùng một lập luận như trước, lập luận chia nhỏ

Đưa ra hai chứng minh khác nhau rằng abc phải là số ở đâu?

chẵn, (a) bằng cách xem xét các trường hợp chẵn lẻ 2.3.13 Chứng minh rằng không tồn tại số thực dương
nhỏ nhất.
khác nhau; (b) sử dụng lập luận mâu thuẫn.

2.3.12 Hãy chắc chắn rằng bạn hiểu Ví dụ 2.3.2 per fectly 2.3.14 Chứng minh rằng log 10 2 là vô tỉ.

bằng cách làm các bài tập sau. (a) Chứng minh rằng J3 là vô 2.3.15 Chứng minh rằng v'2 + J3 là vô tỷ.

tỷ. (b) Chứng minh rằng v'6 là vô tỷ. (c) Nếu bạn cố gắng 2.3.16 Các số phức có thể được sắp thứ tự không? Nói cách

chứng minh J49 là không hợp lý bởi chúng tôi khác, liệu có thể xác định khái niệm "bất đẳng thức" để hai

số phức a + bi và c + di bất kỳ có thể


Machine Translated by Google
2.3 CÁC ĐẠO ĐỨC CỦA BIỆN LUẬN 51

được so sánh và nó có thể được quyết định rằng một "lớn hơn" 2.3.25 Nếu bạn vẫn chưa giải quyết được, hãy xem Vấn đề 2.2.

hoặc một "nhỏ hơn" hoặc cả hai đều bằng nhau? (Sử dụng hàm 13 ở trang 37. Câu trả lời đúng là (n2 + n + 2) / 2. Chứng

định mức Ix + iyl = J X2 + y 2 là "gian lận", vì minh điều này bằng quy nạp.
điều này chuyển đổi mỗi số phức thành một số thực và do đó
2.3.26 Dễ dàng chứng minh rằng tích của ba số nguyên liên
loại bỏ câu hỏi liệu có thể so sánh bất kỳ hai số phức nào
tiếp luôn chia hết cho 6, có ít nhất một trong ba số nguyên
hay không.)
là chẵn và ít nhất một số chia hết cho 3. Xong! Đây là một
2.3.17 Chứng minh rằng nếu a là hữu tỉ và b là vô tỉ thì a + chứng minh rất dễ dàng, nhưng như một bài tập, hãy chứng minh
b là vô tỉ. khẳng định bằng cách sử dụng quy nạp. Nó ít vui hơn, nhưng

thực hành tốt.


2.3.18 Đúng hay sai và tại sao: Nếu a và b là vô tỉ thì ab
là vô tỉ. 2.3.27 Chứng minh rằng một tập hợp có n phần tử thì có 2n tập

2.3.19 Chứng minh các phát biểu được đưa ra trong cuộc thảo luận về hợp con, bao gồm cả tập rỗng và tập chính nó. Ví dụ, tập {a,

b, c} có tám tập con


ký hiệu đồng dư trên trang 44.

2.3.20 Chứng minh sự tổng quát sau của Đề thi 2.3.4 0, {a}, {b}, {c}, {a, b}, {a, e}, {b, c}, {a, b, c}.

trang 44:
2.3.28 Chứng minh công thức tính tổng của một chuỗi hình
Gọi m là số nguyên dương và đặt S học:
là tập các số nguyên dương nhỏ hơn
m tương đối nguyên tố với m, tức là , một n I + an-2 + ... + 1 = - .
không có chung nhân tử nào với m (một
- Tôi ) a- Tôi

khác J. Khi đó với mỗi xES, có một 2.3.29 Chứng minh rằng giá trị tuyệt đối của tổng sev

y E m duy nhất như vậy mà xy = - I (mod m). eral số thực hoặc số phức nhiều nhất bằng tổng các giá trị

Ví dụ, nếu m = 1 2, thì S = { 1,5, 7, II}. Mô-đun "nhân tuyệt đối của số đó. Lưu ý: trước tiên bạn sẽ cần xác minh

nghịch đảo" 12 của mỗi phần tử xES hóa ra là x: 5 · 5 = - I chân trị của tam giác bằng nhau, trong đó nói rằng la + bl :

(mod 1 2), 7 · 7 = - I (mod 12), v.v. S lal + Ibl cho bất kỳ số thực hoặc số phức a, b nào.

2.3.21 Có vô hạn số nguyên tố. Trong số rất nhiều bằng chứng 2.3.30 Chứng minh rằng độ lớn của tổng một số vectơ

về sự thật quan trọng này, có lẽ là bằng chứng lâu đời nhất trong mặt phẳng nhiều nhất bằng tổng độ lớn của các vectơ.

được người Hy Lạp cổ đại biết đến và được viết ra bởi Eu

clid. Đó là một lập luận cổ điển bởi mâu thuẫn. Chúng ta bắt
2.3.31 Chứng tỏ rằng 7n - I chia hết cho 6 với mọi số nguyên
đầu bằng cách giả sử rằng chỉ có vô hạn số nguyên tố PI, P2, posi tive n.
P3, ..., PN. Bây giờ (bước di chuyển mấu chốt khéo léo!) Hãy
2.3.32 (Đức 1 995) Cho x là số thực sao cho là số nguyên.
xem xét số Q: = (PIP2 P3 ··· PN) + I. Tôi N
Chứng minh rằng � + x là an mà x + x
Hoàn thành bằng chứng!
số nguyên, với mọi số nguyên dương n.
2.3.22 (Putnam 1 995) Gọi S là tập hợp các số thực đóng dưới
2.3.33 Chứng minh Bất đẳng thức Bernoulli, trong đó nói rằng
phép nhân (nghĩa là nếu a và b thuộc S thì ab cũng vậy). Gọi
nếu x > -I, x i = 0 và n là một số nguyên dương lớn hơn I, thì
T và U là các tập con rời rạc của S có liên hợp là S. Cho

rằng tích của ba phần tử bất kỳ (không nhất thiết phải khác

biệt) của T thuộc T và tích của ba phần tử bất kỳ của U


( 1 + xt > I + nx.
thuộc U, chứng tỏ rằng tại ít nhất một trong hai tập con T,
U được đóng dưới phép nhân. 2.3.34 Sau khi nghiên cứu Vấn đề 2.2. 1 1 ở trang 37, bạn có

thể đã kết luận rằng f (n) bằng số đơn vị trong biểu diễn nhị

phân (cơ số 2) của n. [Ví dụ, f ( 13) = 3, vì 13 là 1 1 0 I


2.3.23 (Nga 1 995) Có thể đặt các số tự nhiên khác nhau
trong hệ nhị phân.] Chứng minh đặc điểm này của f (n) bằng
1 995 dọc theo một đường tròn sao cho bất kỳ trong hai
quy nạp.
số này, tỉ số của số lớn nhất và số nhỏ nhất là một số
nguyên tố? 2.3.35 (Olympic Toán học Vùng Vịnh, 2006)
Giả sử rằng n ô vuông của một lưới ô vuông vô hạn có màu xám,
2.3.24 Hoàn thành giải pháp bắt đầu trong Ví dụ 1.1.2
và các ô còn lại có màu trắng. Ở mỗi bước, một lưới ô vuông
trên trang I.
mới được thu thập dựa trên
Machine Translated by Google
Machine Translated by Google

2.4 CHIẾN LƯỢC QUAN TRỌNG KHÁC IES 53

tâm trong bất kỳ cuộc điều tra nào. Chúng ta cũng sẽ thảo luận về các chiến lược nâng cao hơn trong các

chương sau.

Vẽ một bức tranh!

Trọng tâm của thái độ cởi mở của một người giải quyết vấn đề "sáng tạo" là nhận thức rằng các vấn đề có thể

và cần được cải tổ theo những cách khác nhau. Thông thường, chỉ cần chuyển một cái gì đó sang dạng hình ảnh

là điều kỳ diệu. Ví dụ, bài toán về nhà sư (Đề thi phần 2. 1 .2 trên trang 17) có một giải pháp sáng tạo

tuyệt vời. Nhưng điều gì sẽ xảy ra nếu chúng ta chỉ giải thích tình huống bằng một biểu đồ khoảng cách-thời

gian đơn giản?

Triệu hồi .....,. ------------- ... "

Con đường ngày đầu tiên

Căn cứ� ------------- �


BAM Không bật

Rõ ràng là bất kể hai con đường được vẽ như thế nào, chúng phải giao nhau ở đâu!

Bất cứ khi nào một vấn đề liên quan đến một số biến đại số, điều đáng cân nhắc là liệu một số trong số

chúng có thể được hiểu là tọa độ hay không. Ví dụ tiếp theo sử dụng cả vectơ và điểm mạng. (Xem Bài 2.2. 17

ở trang 37 và Bài 2.3.38 ở trang 52 để thực hành với các điểm mạng tinh thể.)

Ví dụ 2.4.1 Có bao nhiêu cặp số thực (s, t) có thứ tự với 0 < s, t < 1 sao cho cả 3s + 7t và 5s
+ t đều là số nguyên?

Giải pháp: Người ta có thể bị cám dỗ để giải thích (s, t) là một điểm trong mặt phẳng, nhưng điều đó

không giúp ích được gì nhiều. Một cách tiếp cận khác là xem (3s + 7t, 5s + t) như một điểm. Đối với bất kỳ
s, t chúng tôi có

(3s + 7t, 5s + t) = (3,5) s + (7, 1) t.

Điều kiện 0 < s, t < 1 có nghĩa là (3s + 7t, 5s + t) là điểm cuối của một vectơ nằm bên trong
hình bình hành với các đỉnh (0, 0), (3,5), (7, 1) và (3,5) + (7, 1) = (1 0,6).
Hình dưới đây minh họa tình huống khi s = 0,4, t = 0,7.
Machine Translated by Google

54 CHƯƠNG 2 CHIẾN LƯỢC I ES ĐỐI VỚI CÁC VẤN ĐỀ ĐIỀU CHỈNH

: : : : : ::: :
(10,6)

.
: : (3; 5): : :

..
. •. J: �p: fFJ •• ··


'' ' '
� (1, 1) 1:

(0,0)

Vì cả 3s + 7t và 5s + t đều là số nguyên nên (3s + 7t, 5s + t) là một điểm mạng. Việc đếm các cặp có thứ

tự (s, t) với 0 < s, t < 1 sao cho cả 3s + 7t và 5s + t đều là số nguyên tương đương với việc đếm các điểm

mạng bên trong hình bình hành.

Điều này rất dễ dàng để làm bằng tay; câu trả lời là 31.

Bài toán này có thể được khái quát một cách độc đáo bằng cách sử dụng Định lý Pick (Bài toán 2.3.38 trang

52). Xem Vấn đề 2.4.22 bên dưới.

Hình ảnh không hữu ích? Khắc phục sự cố theo các cách khác!

Ý tưởng mạnh mẽ về việc chuyển đổi một vấn đề từ lời nói thành hình ảnh chỉ là một khía cạnh của chiến lược

tầm nhìn ngoại vi cơ bản. Mở rộng tâm trí của bạn với những cách khác để giải thích lại các vấn đề. Một ví

dụ mà bạn đã gặp là Ví dụ 2. 1 .7 ở trang 20, nơi những gì có vẻ là một dãy số thực sự là một dãy mô tả các

số. Một ví dụ khác là bài toán khóa (Ví dụ 2.2.3 trang 29), trong đó một câu hỏi tổ hợp được biến đổi thành

một bổ đề lý thuyết số. "Kết hợp � Lý thuyết số" là một trong những lý thuyết phổ biến nhất và sản xuất

tương tự như "crossover", nhưng có nhiều khả năng khác. Một số tiến bộ chiến thuật đáng chú ý nhất trong

toán học xảy ra khi ai đó khám phá ra một định dạng mới.

Ví dụ, ý tưởng của Descartes về việc lặp lại các câu hỏi hình học ở dạng số / đại số đã dẫn đến sự phát

triển của hình học giải tích, sau đó dẫn đến giải tích.

Ví dụ đầu tiên của chúng tôi là một bài toán kinh điển.

Ví dụ 2.4.2 Loại bỏ hai ô vuông đối diện theo đường chéo của bàn cờ vua.
Có thể gạch hình dạng này bằng 31 "dominos" 2 x 1 không? (Nói cách khác, mọi hình vuông đều được che phủ và

không có tượng domino nào chồng lên nhau.)

Giải pháp: Lúc đầu, đây có vẻ giống như một bài toán hình học / tổ hợp với nhiều trường hợp
và trường hợp con. Nhưng nó thực sự chỉ là một câu hỏi về việc đếm màu sắc. Hai sao chổi bị loại
bỏ đều có màu trắng (không làm mất đi tính tổng quát), vì vậy hình dạng mà chúng ta quan tâm có
chứa 32 hình vuông màu đen và 30 hình vuông màu trắng. Tuy nhiên, bất kỳ domino nào, một khi nó
Machine Translated by Google

2.4 CHIẾN LƯỢC KHÁC CỦA IMPO RTANT I ES 55

được đặt, sẽ chiếm đúng một hình vuông màu đen và một hình vuông màu trắng. Do đó, 31 dominos yêu cầu 31

hình vuông màu đen và 31 hình vuông màu trắng, vì vậy việc lát gạch là không thể.
_

Ý tưởng giới thiệu màu để định dạng lại một vấn đề là khá cũ. Không bao giờ là không, phải mất vài

năm trước khi có người nghĩ sử dụng phương pháp này cho Bài toán Thư viện (Bài toán 2.2.26 ở trang 38). Vấn

đề này lần đầu tiên được đề xuất bởi Victor Klee vào năm 1973, và được giải quyết ngay sau đó bởi Vaclav

Chvatal. Tuy nhiên, bằng chứng của anh ta khá phức tạp. Vào năm 1 978, S. Fisk đã phát hiện ra lập luận tô

màu trang nhã mà chúng tôi trình bày dưới đây.7 Nếu bạn chưa nghĩ đến vấn đề này, vui lòng thực hiện trước

khi đọc lời giải bên dưới.

Ví dụ 2.4.3 Lời giải cho Bài toán Thư viện: Nếu chúng ta đặt g (n) biểu thị số bảo vệ tối thiểu cần thiết

cho một phòng trưng bày n mặt, chúng ta nhận được g (3) = g (4) = g (5) = 1 và g (6) = 2 bằng thực nghiệm

dễ dàng. Cố gắng hết sức để vẽ những phòng tranh với những căn phòng "ẩn", dường như không thể có được một

phòng tranh 7 mặt hoặc 8 mặt cần nhiều hơn hai người bảo vệ, nhưng chúng ta có thể sử dụng ý tưởng về phòng

tranh 6 cạnh, hai cạnh. gallery ở trang 38 để tạo ra một gallery 9 cạnh, hình như cần ba người bảo vệ. Dưới

đây là các ví dụ về phòng trưng bày 8 mặt và 9 mặt, với các dấu chấm cho biết người bảo vệ.

Nếu mô hình này vẫn tiếp diễn, chúng ta có phỏng đoán dự kiến rằng g (n) = Ln / 3J. Tuy

nhiên, một khó khăn chính của vấn đề này là ngay cả khi chúng ta vẽ một phòng tranh, cũng khó có thể chắc

chắn rằng cần có bao nhiêu người bảo vệ. Và khi n trở nên lớn, các phòng trưng bày có thể trở nên khá phức

tạp.

Một cách định dạng lại màu sắc được đưa ra để giải cứu: Tam giác hóa đa giác thư viện.

Nhớ lại rằng chúng ta đã chứng minh, trong Ví dụ 2.3.9 ở trang 48, rằng chúng ta có thể tô màu các đỉnh của

tam giác này với ba màu sao cho không có hai đỉnh liền kề nào cùng màu. Bây giờ, chọn một màu, và bảo vệ

trạm ở tất cả các đỉnh có màu đó. Những người bảo vệ này sẽ có thể xem toàn bộ phòng trưng bày, vì mọi tam

giác trong tam giác đều được đảm bảo có một người bảo vệ tại một trong các đỉnh của nó! Đây là một ví dụ về

phép ghép tam giác của 1 bộ sưu tập 5 mặt.

7
Xem [22] để biết một cuộc thảo luận hay về bằng chứng của Chvatal và [3 1] để biết cách xử lý đầy đủ về vấn đề này và các vấn

đề liên quan.
Machine Translated by Google
56 CHƯƠNG 2 CHIẾN LƯỢC I ES ĐỐI VỚI CÁC VẤN ĐỀ ĐIỀU CHỈNH

Quy trình này hoạt động cho cả ba màu. Một màu phải được sử dụng tối đa ở
l n / 3 đỉnh J (vì nếu không, mỗi màu được sử dụng trên hơn l n / 3 đỉnh J,
điều này sẽ thêm tối đa n đỉnh). Chọn màu đó, và chúng tôi đã chỉ ra rằng cần
tối đa l n / 3 J bảo vệ.
Do đó, g (n) :::; l n / 3 J. Để thấy rằng g (n) = l n / 3 J, chúng ta chỉ cần đưa ra một ví
dụ, với mỗi n, yêu cầu l n / 3 J bảo vệ. Điều đó rất dễ thực hiện; chỉ cần điều chỉnh cấu trúc
_
được sử dụng cho 9 gon ở trên. Nếu l n / 3 J = r, chỉ cần tạo r "gai", v.v.

Ví dụ tiếp theo (được sử dụng để đào tạo đội 1 996 Hoa Kỳ cho IMO) là khá giả.
Thoạt nhìn, nó có vẻ là một bài toán đại số xấu xí. Nhưng nó thực sự là một thứ khác,
được ngụy trang một cách thô thiển (ít nhất là đối với những người nhớ tốt về lượng giác).
Ví dụ 2.4.4 Tìm giá trị của x thỏa mãn đẳng thức 5

(VI = X + v'I + x) = 6x + 8�.

Giải: Chú ý các hằng số 5,6, 8 và VI - x Số hạng 2 và VI ± x. Tất cả trông

giống như hình tam giác 3-4-5 (có thể 6-8-10 hình tam giác) và hình tam giác. Nhắc lại các công thức cơ bản:
.
sin2 e + cos2 e = 1,
!! -
.- _ J I - cos e !! -
.- _ J I + cose
Tội cos ·
2 2 ' 2 2

Thực hiện phép thay thế trig x = cos e. Chúng ta chọn cosin hơn là sin, vì v'I ±
cos e tham gia vào công thức nửa góc, nhưng v'I ± sin e thì không. Nhóm thay thế
này trông khá đẹp, vì chúng ta ngay lập tức nhận được v'I - x2 = sin e, và cũng

v'I ± x = VI ± cos e = r,:; J 1 ± cos e


v

2 .
2

Do đó, phương trình ban đầu trở thành

(9)
5 v'2 ( sin � + cos �) = 6 cos e + 8 sin e.
Machine Translated by Google

2.4 CHIẾN LƯỢC QUAN TRỌNG KHÁC IES 57

Bây giờ chúng tôi giới thiệu một công cụ trig đơn giản : Cho biểu thức có dạng a cos 0 + b sin
O, viết

một

b cosO + va +
2 2
a cos O + b sin o = va2 + b2 ( 2 va + b b 2 tội lỗi o) .

một

Điều này rất hữu ích, và b lần lượt là sin và côsin va2
vì va2 + + b2
b2 của góc a: = arctan (a / b).

một

Do đó,
acos 0 + b sinO = Va2 + b2 (sin a cos 0 + cos a sin 0)

= Va2 + b2 sin (a + O).

Đặc biệt, chúng tôi có

sinx + cosx = v'2sin (x + �).

Áp dụng điều này, phương trình (9) trở thành (lưu ý rằng V62 + 82 = 10)

5 v'2v'2 sin ( � + �) = 10 (� cos 0 + � sin 0 ) .

Do đó, nếu a = arctan (3/4), chúng ta có

sin (� + �) = sin (a + 0).

Các góc bằng nhau tạo ra 0 = 1r / 2 - 2a. Như vậy

x = cos O = sin (2a) = 2 sin a cos a = 2 (�) (�) = ��. •


Chuyển đổi một vấn đề sang dạng hình học hoặc hình ảnh thường hữu ích, nhưng trong một
số trường hợp, điều ngược lại là đúng. Tất nhiên, ví dụ cổ điển là hình học giải tích,
chuyển đổi hình ảnh thành đại số. Đây là một ví dụ kỳ lạ hơn: một vấn đề là chỉ số địa lý
trên bề mặt, nhưng không phải là cốt lõi của nó.

Ví dụ 2.4.5 Chúng ta có n hành tinh trong không gian, với n là một số nguyên dương. Mỗi hành tinh
là một khối cầu hoàn hảo và tất cả các hành tinh đều có cùng bán kính R. Gọi một điểm trên bề mặt
của một hành tinh là riêng nếu nó không thể được nhìn thấy từ bất kỳ hành tinh nào khác. (Bỏ qua
những thứ như chiều cao của người trên hành tinh, mây, phối cảnh, v.v. Ngoài ra, hãy giả sử rằng
Machine Translated by Google

58 CHƯƠNG 2 CHIẾN LƯỢC I ES CHO I CÁC VẤN ĐỀ ĐIỀU TRA

các hành tinh không chạm vào nhau.) Dễ dàng kiểm tra rằng nếu n = 2 thì tổng diện tích riêng là
4nR2 , mà chỉ là tổng diện tích của một hành tinh. Bạn có thể nói gì về n
= 3? Các giá trị khác của n?

Giải pháp một phần: Một chút thử nghiệm thuyết phục chúng ta rằng nếu n = 3, tổng
diện tích riêng cũng bằng tổng diện tích của một hành tinh. Chơi với n lớn hơn cho kết quả
tương tự. Chúng tôi phỏng đoán rằng tổng diện tích riêng luôn bằng chính xác với diện tích
của một hành tinh, bất kể hành tinh đó nằm ở vị trí nào. Đây có vẻ là một vấn đề khó chịu
trong hình học rắn, nhưng phải không? Các khái niệm về "tư nhân" và "công cộng" dường như
được liên kết với một loại tính hai mặt; có lẽ vấn đề thực sự không phải là hình học mà
là logic. Chúng ta cần một số "ký hiệu." Chúng ta hãy giả sử rằng có một hệ tọa độ phổ
quát, chẳng hạn như kinh độ và vĩ độ, để chúng ta có thể tham chiếu đến cùng một vị trí
trên bất kỳ hành tinh nào. Ví dụ, nếu các hành tinh là những quả bóng nhỏ lơ lửng trong
một căn phòng, thì vị trí "cực bắc" có nghĩa là điểm trên hành tinh gần trần nhà nhất.

Với một hệ tọa độ phổ quát như vậy, chúng ta có thể nói gì về một hành tinh P có một
điểm riêng tại vị trí x? Không mất tính tổng quát, hãy đặt x ở "cực bắc". Rõ ràng, tâm của
tất cả các hành tinh khác phải nằm ở phía nam của mặt phẳng "xích đạo" P. Nhưng điều đó
làm cho cực bắc của các hành tinh này là công khai, vì cực bắc của chúng có thể nhìn thấy
từ một điểm ở bán cầu nam của P (hoặc từ bán cầu nam của bất kỳ hành tinh nào nằm giữa).
Nói cách khác, chúng tôi đã chỉ ra khá dễ dàng rằng

Nếu vị trí x là riêng tư trên một hành tinh, thì nó là công khai trên tất cả các hành tinh khác.

Sau khám phá tuyệt vời này, bước cuối cùng đã rõ ràng: để chứng minh rằng Với

bất kỳ vị trí x nào, nó phải là riêng tư trên hành tinh nào đó.8

Chúng tôi để đây như một bài tập (vấn đề?) Cho người đọc.

Các ví dụ trên chỉ là bề mặt của cơ thể rộng lớn của các ý tưởng giao nhau.
Mặc dù khái niệm tái định dạng một vấn đề mang tính chiến lược, nhưng việc thực hiện nó lại mang
tính quyết định, thường đòi hỏi kiến thức chuyên môn. Chúng ta sẽ thảo luận chi tiết về một số ý
tưởng chéo khác trong Chương 4.

Thay đổi quan điểm của bạn

Thay đổi quan điểm chỉ là một biểu hiện khác của tầm nhìn ngoại vi. Đôi khi
một vấn đề khó chỉ vì chúng ta chọn "sai" quan điểm. Bỏ ra vài phút tìm kiếm
quan điểm “thuận theo tự nhiên” có thể trả cổ tức lớn. Đây là một ví dụ cổ
điển.

Ví dụ 2.4.6 Một người lặn từ trên cầu xuống sông rồi bơi ngược dòng nước trong
thời gian 1 giờ với vận tốc không đổi. Sau đó cô ấy quay lại và bơi xuôi dòng
trong nước với cùng một tốc độ. Khi vận động viên bơi lội qua cầu,

Người đọc sành sỏi có thể nhận thấy rằng chúng ta đang phủ nhận một kỹ thuật: có thể có những điểm đặc biệt
không tuân theo quy tắc này. Tuy nhiên, những điểm này tạo thành một tập hợp số không đo, sẽ không ảnh hưởng đến
kết quả.
Machine Translated by Google

2.4 CHIẾN LƯỢC QUAN TRỌNG KHÁC IES 59

người ngoài cuộc nói với cô ấy rằng chiếc mũ của cô ấy đã rơi xuống sông khi cô ấy lặn
lúc đầu. Người bơi tiếp tục xuôi dòng với tốc độ như cũ, đuổi kịp chiếc mũ ở một cây
cầu khác cách cây đầu tiên một dặm về phía hạ lưu . Tốc độ của dòng điện tính bằng dặm
trên giờ là bao nhiêu?

Giải pháp: Chắc chắn có thể giải điều này theo cách thông thường, bằng cách cho x
bằng cường độ dòng điện và y bằng vận tốc của vận động viên bơi lội, v.v ... Nhưng nếu
chúng ta nhìn mọi thứ từ quan điểm của chiếc mũ thì sao? Chiếc mũ không nghĩ rằng nó di
chuyển. Theo quan điểm của nó, người bơi từ bỏ nó, và sau đó bơi đi trong một giờ với
một tốc độ nhất định (cụ thể là tốc độ của dòng điện cộng với tốc độ của người bơi).
Sau đó người bơi quay đầu lại và đi với tốc độ như cũ. Do đó, vận động viên bơi sẽ lấy
lại chiếc mũ trong đúng một giờ sau khi quay lại. Vì vậy, toàn bộ cuộc phiêu lưu kéo
dài hai giờ, trong đó chiếc mũ đi được một dặm về phía hạ lưu.
_
Vậy tốc độ của dòng điện là ! dă m mô t giờ.

Để có một ví dụ khác về sức mạnh của quan điểm "tự nhiên", hãy xem bài toán "Bốn
lỗi" (Ví dụ 3. 1 .6 trên trang 65). Bài toán cổ điển này kết hợp một quan điểm thông
minh với chiến thuật cơ bản của đối xứng.

Các vấn đề và bài tập

2.4.7 Pat làm việc trong thành phố và sống ở ngoại ô với Sal. đạt đến A lúc II: 15PM. Mặt trời mọc khi nào?

Mỗi buổi chiều, Pat lên một chuyến tàu chạy ở ga ngoại ô lúc
2.4.10 Một con bọ đang bò trên mặt phẳng tọa độ từ (7, II) đến
5 giờ chiều. Sal rời khỏi nhà trước 5 giờ và lái xe với tốc độ
(- 1 7, -3). Con bọ di chuyển với tốc độ không đổi một đơn vị
không đổi để đến ga tàu lúc 5 giờ chiều đón Pat. Lộ trình mà
mỗi giây ở mọi nơi trừ góc phần tư II (tọa độ x- và y dương),
Sal lái xe không bao giờ thay đổi.
nơi nó di chuyển tại
� đơn vị trên giây. Con bọ nên đi theo con đường nào để hoàn
Một ngày nọ, thói quen này bị gián đoạn, vì có sự cố mất thành hành trình của mình trong thời gian tối thiểu? Tổng quát hóa!
điện tại nơi làm việc. Pat đi sớm và bắt một chuyến tàu đến ga
2.4.11 Lần đầu tiên sau 12 giờ kim giờ và kim phút gặp nhau là
ngoại ô lúc 4 giờ chiều. Thay vì gọi điện cho Sal để yêu cầu
bao nhiêu? Đây là một bài tập đại số rất hay và có độ khó vừa
đón sớm hơn, Pat quyết định tập thể dục một chút, và bắt đầu
phải, rất đáng để làm nếu bạn chưa từng làm nó trước đây. Tuy
đi bộ về nhà dọc theo con đường mà Sal lái xe, biết rằng cuối
nhiên, vấn đề này có thể được giải quyết trong vài giây trong
cùng Sal sẽ chặn được Pat, và sau đó sẽ tạo ra một cú V-tum,
đầu bạn nếu bạn tránh đại số lộn xộn và chỉ xem xét quan điểm
và họ sẽ cùng nhau về nhà trên xe. Đây thực sự là những gì sẽ
"tự nhiên". Cứ liều thử đi!
xảy ra, và Pat về nhà sớm hơn 10 phút so với ngày thường. Giả

sử rằng vận tốc đi bộ của Pat không đổi, vận tốc V-tum không

mất thời gian và vận tốc lái xe của Sal là không đổi, thì Pat 2.4.12 Sonia đi lên thang cuốn đang đi lên.
đã đi bộ trong bao nhiêu phút? Khi cô ấy đi bộ với tốc độ một bước mỗi giây, cô ấy phải mất

20 bước để lên đến đỉnh. Nếu cô ấy đi bộ với tốc độ hai bước

mỗi giây, cô ấy phải mất 32 bước để lên đến đỉnh. Cô ấy không

bao giờ bỏ qua bất kỳ bước nào. Thang cuốn có mấy bậc?
2.4.8 Chứng minh, không tính đại số, rằng tổng của n số nguyên

dương lẻ đầu tiên là n2 •

2.4.13 Các số tam giác là tổng của các số nguyên bí mật, bắt
2.4.9 Hai thị trấn A và B được nối với nhau bằng một con đường.
đầu bằng I. Một vài số tam giác đầu tiên là
Lúc mặt trời mọc, Pat bắt đầu đạp xe từ A đến B dọc theo con

đường này, trong khi Dana bắt đầu đạp xe từ B đến A. Mỗi người

đạp xe với tốc độ không đổi và họ băng qua đường vào buổi

trưa. Pat đến B lúc 5 giờ chiều trong khi Dana


Tôi
, 1 + 2 = 3, 1 + 2 + 3 = 6, 1 + 2 + 3 + 4 = 10 .....
Machine Translated by Google

60 CHƯƠNG 2 CHIẾN LƯỢC ĐỐI VỚI I CÁC VẤN ĐỀ ĐIỀU CHỈNH

Chứng minh rằng nếu T là một số tam giác thì 8T + 1 là một hình bất kể giá trị nào được chọn cho chiếc áo dài.

vuông hoàn hảo. Tất nhiên, bạn có thể làm điều này với đại số,
2.4.21 Đối với số nguyên dương n, xác định S " là giá trị nhỏ
nhưng hãy thử vẽ một bức tranh. nhất của tổng

2.4.14 Một lần nữa, bạn có thể dễ dàng sử dụng quy nạp để
chứng minh một thực tế rất thú vị rằng tổng của n hình khối �I V (2k - I) 2 + af,
đầu tiên bằng bình phương của số tam giác thứ n, nhưng thay

vào đó bạn có thể làm điều đó với một bức tranh không? như al, a2, ... ,tất cả các phạm vi thông qua tất cả các giá trị dương

sao cho
2.4.15 Chứng minh rằng tất cả các số nguyên dương ngoại trừ số

nguyên của hai số đều có thể được viết dưới dạng tổng của ít

nhất hai số nguyên dương liên tiếp. Một lần nữa, đại số có thể

được sử dụng, nhưng hình ảnh cũng vậy! Tìm SIO.

2.4.16 Hoàn thành lời giải của bài toán Các hành tinh, 2.4.22 (Đài Loan 1 995) Cho a, h, c, d là các số nguyên sao cho

bắt đầu trong Ví dụ 2.4.5 trên trang 57. ad - bc = k > 0 và

2.4.17 (Putnam 1 984) Tìm giá trị nhỏ nhất của GCD (a, h) = GCD (e, d) = 1.

Chứng minh rằng tồn tại đúng k cặp số thực (XI,


(u - v) 2 + (J2 - u2 - �r X2) có thứ tự sao cho 0 : S XI, X2 < I và aXI +
hX2 và eXI + dX2 đều là các số nguyên.
cho 0 < u < v'2 và v > o.
2.4.23 Có bao nhiêu số hạng phân biệt khi
2.4.18 Một con bọ nằm trên một ô của một khối đơn vị và muốn bò

đến ô đối diện theo đường chéo. Nếu con bọ có thể chui qua khối (I + x7 + xI 3) \ OO

lập phương, khoảng cách tất nhiên sẽ là 0. Nhưng con bọ phải ở

trên bề mặt của khối lập phương. Độ dài của đường đi ngắn nhất được nhân ra và đơn giản hóa?

là bao nhiêu? 2.4.24 Một số viên bi được đặt trên một đường tròn có chu
vi một mét. Chiều rộng của đường ray và bán kính của viên

2.4.19 Cho a và h là các số nguyên lớn hơn một số không có ước bi không đáng kể. Mỗi viên bi được định hướng ngẫu nhiên,

chung. Chứng minh rằng theo chiều kim đồng hồ hoặc ngược chiều kim đồng hồ. Tại
thời điểm không, mỗi viên bi bắt đầu chuyển động với vận

tốc một mét trên phút, trong đó hướng chuyển động phụ

thuộc vào phương hướng. Bất cứ khi nào hai viên bi va chạm

vào nhau, chúng sẽ bật trở lại với tốc độ không thay đổi,
và tìm giá trị của tổng chung này.
tuân theo định luật va chạm không đàn hồi.
2.4.20 Cho ao là bất kỳ số thực nào lớn hơn 0 và nhỏ hơn 1. Sau Bạn có thể nói gì về vị trí có thể có của các viên bi sau
đó xác định dãy a I, a2, a3 , ... bởi = vr = a ;; - cho n = 0, một phút, đối với vị trí ban đầu của chúng? Có ba yếu tố cần
a ,, + I 1,2 , . . . . Cho thấy xem xét: số lượng viên bi, vị trí ban đầu của chúng và hướng
đầu ti của chúng.
. 0-1 Iim a ,,
= --- ,
11- + 00
2
Machine Translated by Google

Chương 3

Chiến thuật giải quyết vấn đề

Bây giờ chúng ta chuyển sang cấp độ chiến thuật của việc giải quyết vấn đề. Nhớ lại rằng chiến thuật là

những phương pháp toán học có thể áp dụng rộng rãi thường đơn giản hóa các vấn đề. Chiến lược một mình

hiếm khi giải quyết được vấn đề; chúng ta cần sức mạnh tập trung hơn của các chiến thuật (và thường là

các công cụ được chú trọng hóa cao ) để hoàn thành công việc. Trong số nhiều chiến thuật khác nhau, chương

này sẽ khám phá một số chiến thuật quan trọng nhất có thể được sử dụng trong nhiều cài đặt ical toán học

khác nhau.

Hầu hết các ý tưởng chiến lược trong Chương 2 đều là lẽ thường tình. Ngược lại, các ý tưởng chiến

thuật trong chương này, mặc dù dễ sử dụng, nhưng ít "tự nhiên" hơn, như ít người nghĩ đến. Hãy quay lại

sự tương tự về hoạt động leo núi của chúng ta trong giây lát. Một chiến thuật leo núi quan trọng là ý

tưởng không rõ ràng (có nghĩa là được hiểu theo nghĩa đen):

Chọc mông ra!


Người mới tập leo núi điển hình ôm chặt vào mặt đá mà anh ta đang cố gắng leo lên, vì việc đẩy ra khỏi

tảng đá là không trực quan. Tuy nhiên, một khi anh ta nghiến răng và đẩy đuôi xe ra, điều kỳ diệu sẽ xảy

ra: thành phần của trọng lực vuông góc với tảng đá tăng lên, làm tăng lực ma sát lên bàn chân của anh ta

và ngay lập tức có được một tư thế an toàn hơn.

Trước Sau

Tương tự như vậy, bạn có thể thấy rằng một số ý tưởng chiến thuật bên dưới là peCUliar. Nhưng một khi

bạn nắm vững chúng, bạn sẽ nhận thấy sự cải thiện đáng kể trong việc giải quyết vấn đề của mình.

Nhiều chiến thuật giải quyết vấn đề cơ bản liên quan đến việc tìm kiếm trật tự. Thông thường các vấn

đề khó vì chúng có vẻ "hỗn loạn" hoặc mất trật tự; chúng dường như bị bỏ sót các phần (dữ kiện, biến, mẫu)

hoặc các phần dường như không kết nối với nhau. Tìm (và

61
Machine Translated by Google

62 CHƯƠNG 3 CHIẾN THUẬT GIẢI QUYẾT VẤN ĐỀ

sử dụng) đặt hàng có thể nhanh chóng đơn giản hóa các vấn đề như vậy. Do đó, chúng ta sẽ bắt đầu

bằng cách nghiên cứu các chiến thuật giải quyết vấn đề giúp chúng ta tìm ra hoặc áp đặt trật tự mà

dường như không có. Hình thức trật tự ấn tượng nhất là chủ đề đầu tiên của chúng tôi, đối xứng.

Đối xứng liên quan đến việc tìm kiếm hoặc áp đặt trật tự một cách cụ thể, ví dụ, bằng cách phản
ánh. Các thủ pháp khác tìm hoặc khai thác trật tự, nhưng theo những cách trừu tượng hơn, gần như là

"ical ẩn dụ". Chúng ta sẽ thảo luận về ba phương pháp "giả đối xứng". Tất cả chúng đều dựa trên

những quan sát rất đơn giản mà đôi khi mang lại những thông tin hữu ích đáng kinh ngạc.
Chiến thuật đầu tiên, nguyên tắc cực đoan, mà chúng ta đã gặp trong bài toán Hành động khẳng định

(Ví dụ 2. 1 .9 trên trang 21), hoạt động bằng cách tập trung vào các thực thể lớn nhất và nhỏ nhất
trong một vấn đề. Tiếp theo, nguyên tắc chuồng bồ câu bắt nguồn từ quan sát gần như bỏ trống rằng

nếu bạn có nhiều khách hơn số phòng trống, một số khách sẽ phải chia sẻ phòng. Chiến thuật cuối cùng

của chúng tôi, khái niệm bất biến, cho thấy lượng thông tin đến theo cách của bạn khi bạn hạn chế sự

chú ý vào một khía cạnh hẹp của vấn đề không thay đổi (chẳng hạn như tính chẵn lẻ). Đây là một ý

tưởng cực kỳ mạnh mẽ, nền tảng của toán học, làm nền tảng cho nhiều chiến thuật và công cụ dường

như khác nhau.

3.1 Đối xứng

Tất cả chúng ta đều có một ý tưởng trực quan về sự đối xứng; ví dụ, mọi người đều hiểu rằng các vòng

tròn là đối xứng. Tuy nhiên, sẽ rất hữu ích nếu định nghĩa đối xứng theo cách chính thức, nếu chỉ vì

điều này sẽ mở rộng quan niệm của chúng ta về nó. Chúng ta gọi một đối tượng là đối xứng nếu có một

hoặc nhiều "hành động" không tầm thường khiến đối tượng không thay đổi. Chúng tôi gọi các hành động
Tôi

thực hiện điều này là các đối xứng của đối tượng.

Chúng tôi đã hứa một cái gì đó chính thức, nhưng định nghĩa trên có vẻ khá mơ hồ. Chúng tôi có

nghĩa là gì bởi một "hành động"? Gần như mọi thứ ! Chúng tôi đang cố tình mơ hồ, bởi vì mục đích của

chúng tôi là để bạn thấy được sự đối xứng trong càng nhiều tình huống càng tốt. Dưới đây là một vài
ví dụ.

Ví dụ 3.1.1 Một hình vuông là đối xứng khi phản xạ qua một đường chéo. Hình phản xạ là một trong số
các phép đối xứng của hình vuông. Các phép đối xứng khác bao gồm xoay 90 độ theo chiều kim đồng hồ

và phản xạ về một đoạn thẳng nối các điểm giữa của hai cạnh đối diện.

Ví dụ 3.1.2 Một vòng tròn có vô số đối xứng, ví dụ, đồng hồ quay khôn ngoan theo độ với a bất kỳ .

Ví dụ 3.1.3 Dãy vô hạn kép

... ,3 , 1, 4, 3, 1,4, 3 , 1, 4, ...

đối xứng với hành động "chuyển mọi thứ ba nơi sang phải (hoặc trái)."

1
Chúng tôi cố tình tránh ngôn ngữ chuyển đổi và tự động hóa mà sẽ được yêu cầu bởi
định nghĩa chính xác về mặt toán học.
Machine Translated by Google

3.1 TỔNG HỢP 63

Tại sao tính đối xứng lại quan trọng? Bởi vì nó cung cấp cho bạn thông tin "miễn phí". Nếu bạn
biết rằng một vật nào đó, chẳng hạn, đối xứng với góc quay 90 độ về một điểm nào đó, thì bạn chỉ cần

nhìn vào một phần tư của vật đó. Và bạn cũng biết tâm vòng quay là một điểm “đặc biệt”, đáng được khảo

sát chặt chẽ. Bạn sẽ thấy những ý tưởng này đang hoạt động dưới đây, nhưng trước khi bắt đầu, hãy để

chúng tôi đề cập đến hai điều cần ghi nhớ khi bạn suy ngẫm về sự đối xứng:

• Các nguyên tắc chiến lược của tầm nhìn ngoại vi và phá vỡ quy tắc cho chúng ta biết phải tìm

kiếm sự đối xứng ở những nơi không chắc chắn, và đừng lo lắng nếu một cái gì đó gần như, nhưng

không hoàn toàn đối xứng. Trong những trường hợp này, điều khôn ngoan là nên tiến hành như

thể có sự đối xứng, vì chúng ta có thể sẽ học được điều gì đó hữu ích.

• Một định nghĩa thay thế không chính thức về tính đối xứng là "sự hài hòa". Điều này thậm chí

còn mơ hồ hơn định nghĩa "chính thức" của chúng tôi, nhưng nó không phải là không có giá trị.

Tìm kiếm sự hài hòa và vẻ đẹp, bất cứ khi nào bạn điều tra một vấn đề. Nếu bạn có thể làm điều

gì đó khiến mọi thứ trở nên hài hòa hơn hoặc đẹp đẽ hơn, ngay cả khi bạn không biết cách định

nghĩa hai thuật ngữ này như thế nào, thì bạn thường đang đi đúng hướng.

Đối xứng Hình học Hầu hết các

cuộc điều tra hình học đều thu được lợi nhuận bằng cách xem xét tính đối xứng. Hỏi những câu hỏi sau về

tính đối xứng:

• Nó có hiện tại

không? • Nếu không, nó có thể bị áp đặt

không? • Làm thế nào nó có thể được khai thác sau đó?

Đây là một ví dụ đơn giản nhưng nổi bật.

Ví dụ 3.1.4 Hình vuông nội tiếp hình tròn nội tiếp hình vuông. Tìm tỉ số diện
tích của hai hình vuông.

Giải pháp: Bài toán chắc chắn có thể được giải bằng đại số (đặt x bằng độ dài của hình vuông

nhỏ, sau đó sử dụng định lý Pitago, v.v.), nhưng có một phương pháp tốt hơn. Sơ đồ ban đầu có đầy đủ

các đối xứng. Chúng ta có thể tự do xoay và / hoặc phản chiếu nhiều hình dạng mà vẫn bảo toàn diện

tích của hai hình vuông. Làm thế nào để chúng tôi chọn từ tất cả những khả năng này? Chúng ta cần sử

dụng giả thuyết rằng các đối tượng nội tiếp với nhau. Nếu chúng ta xoay hình vuông nhỏ 45 độ, các

đỉnh của nó bây giờ thẳng hàng với các điểm tiếp tuyến giữa hình tròn và hình vuông lớn, và ngay lập

tức giải pháp xuất hiện.


Machine Translated by Google

64 CHƯƠNG 3 CHIẾN THUẬT GIẢI QUYẾT VẤN ĐỀ

Hình vuông nhỏ rõ ràng có một nửa diện tích của hình vuông lớn hơn!
-
Các đối xứng hình học đơn giản nhất là quay và phản xạ. Luôn kiểm tra xem liệu các phép quay
hoặc phản xạ có áp đặt trật tự cho vấn đề của bạn hay không. Ví dụ tiếp theo cho thấy sức mạnh của
áp đặt đối xứng phản xạ.

Ví dụ 3.1.5 Căn nhà của bạn cách một con suối chảy theo hướng đông tây hai dặm về phía bắc.
Cabin của bà bạn cách cabin của bạn 12 dặm về phía tây và một dặm về phía bắc. Mỗi ngày, bạn
đi từ cabin của mình đến nhà của bà, nhưng trước tiên hãy ghé thăm con suối (để lấy nước
ngọt cho bà). Chiều dài của tuyến đường với khoảng cách tối thiểu là bao nhiêu?

Giải pháp: Đầu tiên, hãy vẽ một bức tranh! Gắn nhãn vị trí của bạn theo Y và Bà theo G. Chắc
chắn, bài toán này có thể được thực hiện bằng phép tính, nhưng nó rất xấu (bạn cần phải phân biệt
tổng của hai căn, cho người mới bắt đầu). Vấn đề dường như không có sự đối xứng trong đó, nhưng
luồng thực tế đang cầu xin bạn phản ánh nó! Vẽ một đường dẫn mẫu (được hiển thị bên dưới là Y A
theo sau là AG) và nhìn vào hình ảnh phản chiếu của nó. Chúng tôi gọi các phản chiếu của ngôi nhà
của bạn và nhà của bà , tương ứng là y ' và G' .

Dòng

-------- �- = � ------ �� --------------

G '

Trong khi bạn đang mang nước cho Bà, bản sao của bạn trong một vũ trụ thay thế cũng làm như
vậy, chỉ ở phía nam của dòng suối. Lưu ý rằng AG = AG ', vì vậy độ dài con đường của bạn sẽ không
thay đổi nếu bạn đến thăm Bà ngoại được phản chiếu thay vì người thật. Vì khoảng cách ngắn nhất
giữa hai điểm là đường thẳng Y BG 'nên đường đi theo thời gian của op sẽ là Y B theo sau là BG.
Chiều dài của nó bằng với chiều dài của Y BG ', chỉ là cạnh huyền của một tam giác vuông có chân
là 5 và 12 dặm. Do đó, câu trả lời của chúng tôi là 13 dặm.
_

Khi cân nhắc một tình huống đối xứng, bạn nên luôn tập trung ngắn gọn vào các đối tượng "cố
định" mà đối xứng không thay đổi. Ví dụ, nếu một cái gì đó đối xứng với sự phản xạ về một trục,
thì trục đó là cố định và đáng được nghiên cứu (dòng trong bài toán trước đóng vai trò đó). Đây là
một ví dụ khác, một bài toán cổ điển khai thác tính đối xứng quay cùng với một điểm cố định quan
trọng.
Machine Translated by Google

3.1 SYMM ETRY 65

Ví dụ 3.1.6 Bốn lỗi nằm ở mỗi đỉnh của một hình vuông đơn vị. Đột nhiên, từng con bọ
bắt đầu đuổi theo người hàng xóm ngược chiều kim đồng hồ của nó. Nếu các con bọ di
chuyển với tốc độ 1 đơn vị mỗi phút, thì sau bao lâu để bốn con bọ đâm vào nhau?

Giải pháp: Tình hình là đối xứng luân phiên ở chỗ không có một lỗi nào
"sai lầm". Nếu cấu hình ban đầu của họ là hình vuông, thì họ sẽ luôn duy trì
cấu hình đó. Đây là cái nhìn sâu sắc quan trọng, tin hay không, và nó là một
cái rất có lợi!

................. �

... ·
.. .···· ..�
! \

.. .
·. · Z ·
o '0 "

.. : ; �.
. }
g.
· a 9:.
. e.:�·
· .'8 : .... �.,.o
.
# �..
: �. b�

: đường dẫn của lỗi


_________________________ _
Tôi

�: �

Theo thời gian, các lỗi tạo thành một hình vuông thu nhỏ xoay ngược đồng hồ một
cách khôn ngoan. Tâm của hình vuông không di chuyển. Do đó, trung tâm này là điểm
duy nhất "được đánh giá cao", vì vậy chúng tôi tập trung phân tích vào đó. Nhiều vấn
đề khó giải quyết khác trở nên dễ dàng một khi chúng ta chuyển trọng tâm sang hệ quy
chiếu tự nhiên; trong trường hợp này chúng ta nên xem xét một hệ quy chiếu xuyên
tâm , một hệ quy chiếu quay theo hình vuông. Ví dụ, chọn một trong các lỗi (không
quan trọng là lỗi nào!), Và nhìn vào đoạn thẳng từ tâm của hình vuông đến lỗi. Đoạn
này sẽ quay ngược chiều kim đồng hồ, và (quan trọng hơn) thu nhỏ lại. Khi nó đã bị
thu hẹp về 0, các lỗi sẽ đâm vào nhau. Nó co lại nhanh như thế nào? Quên thực tế là
dòng đang quay. Theo quan điểm của đường xuyên tâm này, con bọ luôn di chuyển theo
một góc 45 °. Vì con bọ di chuyển với tốc độ đơn vị, thành phần vận tốc hướng tâm
của nó chỉ là 1 · cos 45 ° = V2 / 2 đơn vị mỗi phút, tức là, đường hướng tâm co lại
ở tốc độ này. Vì độ dài ban đầu của đường hướng tâm là V2 / 2, nên sẽ chỉ mất 1 phút
để các lỗi xảy ra. _

Đây là một bài toán giải tích đơn giản mà chắc chắn có thể được giải một cách dễ dàng theo
cách thông thường hơn. Tuy nhiên, phương pháp của chúng tôi dưới đây minh họa sức mạnh của chiến
lược Vẽ một bức tranh cùng với tính đối xứng và có thể được áp dụng trong nhiều tình huống khó hơn.

Trong

Ví dụ 3.1.7 Tính toán 102 cos2 xdx trong đầu của bạn.

Giải pháp: Vẽ bằng tay các đồ thị sin và cosin từ 0 đến 11r, và bạn sẽ nhận thấy
rằng chúng đối xứng với nhau khi phản ánh về đường thẳng đứng x = i 1r.
Machine Translated by Google

66 CHƯƠNG 3 CÁC THUẬT TOÁN GIẢI QUYẾT VẤN ĐỀ

Như vậy

Do đó, giá trị mà chúng tôi tìm kiếm là


Vấn đề tiếp theo, từ IMO 1995 , khó hơn các bài khác, nhưng chỉ ở khía cạnh "kỹ thuật".
Để giải được nó, bạn cần phải làm quen với Định lý Ptolemy, định lý này phát biểu

Cho ABCD là một tứ giác nội tiếp, tức là ,một tứ giác có các đỉnh
nằm trên một vòng tròn. sau đó

AB · CD + AD · BC = AC · BD.

Xem Vấn đề 4.2.4 1, 8.4.30 và 8.5.49 để biết các ý tưởng khác nhau cho một bằng chứng. Đặc

điểm chính của tứ giác tuần hoàn là sự thật dễ dàng xác minh (xem trang 266)

Một tứ giác là hình tuần hoàn nếu và chỉ khi các góc đối diện của nó
là tâm nhỏ (cộng đến 180 độ).

Ví dụ 3.1.8 Cho hình lục giác lồi ABCDEF có AB = BC = CD và DE = EF = FA, sao cho L.BCD =
L.EFA = 1C / 3. Giả sử G và H là các điểm trong hình lục giác sao cho LAGB = L.DHE = 21C / 3.
Chứng minh rằng

AG + GB + GH + DH + HE 2: CF.

Giải pháp: Đầu tiên, đối với tất cả các bài toán hình học, hãy vẽ một sơ đồ
chính xác, sử dụng bút chì, compa và thước kẻ. Hãy tìm sự đối xứng. Chú ý rằng BCD
và EFA là các tam giác đều, sao cho BD = BA và DE = AE. Theo tính đối xứng của
hình, có vẻ như phản ánh về BE có lợi. Gọi C 'và F' lần lượt là điểm phản xạ của C và F.
Machine Translated by Google

3.1 SYMM ETRY 67

c C'

F '
F

Một thủ thuật thường áp dụng cho các bất đẳng thức hình học là tìm cách so sánh tổng của một

số độ dài với một độ dài duy nhất, vì khoảng cách ngắn nhất giữa hai điểm là một đường thẳng. Các
tứ giác AGBC ' và H EF' D là nội tiếp, vì các góc đối diện bằng 1 80 độ. Khi đó Định lý Ptolemy ngụ
ý rằng AG · BC '+ GB. AC '= C' G. AB. Vì ABC ' là cạnh đều, điều này ngụ ý rằng AG + GB = C' G.

Tương tự, DH + HE = H F ' . Đường ngắn nhất giữa hai điểm là một đường thẳng. Nó
theo sau đó

CF = C'F ' � C'G + GH + HF' = AG + GB + GH + DH + HE, bằng nhau

nếu và chỉ khi G và H cùng nằm trên C 'F'. •

Đối xứng Đại số Đừng

hạn chế khái niệm đối xứng của bạn đối với các đối tượng vật lý hoặc hình học. Ví
dụ, các chuỗi có thể có đối xứng, như hàng này của Tam giác Pascal:

1, 6, 15,20,20, 15,6, 1.

Đó chỉ là sự khởi đầu. Trong bất kỳ tình huống nào mà bạn có thể tưởng tượng ra việc "ghép nối"

mọi thứ lại với nhau, bạn có thể nghĩ đến sự đối xứng. Và suy nghĩ về sự đối xứng hầu như luôn mang
lại hiệu quả.

Công cụ ghép nối Gaussian

Carl Friedrich Gauss (1777-1855) chắc chắn là một trong những nhà toán học vĩ đại nhất mọi thời

đại. Nhiều câu chuyện ca ngợi sự thông minh và sức mạnh tinh thần phi thường của ông. Không ai biết
sự thật của những câu chuyện này như thế nào, bởi vì nhiều người trong số chúng chỉ được cho là do
chính Gauss. Giai thoại sau có nhiều dị bản. Chúng tôi chọn một trong những đơn giản nhất.
Khi Gauss 10 tuổi, giáo viên của cậu đã phạt lớp cậu bằng một khoản tiền có vẻ tẻ nhạt:

1 + 2 + 3 + ... + 98 + 99 + 100.
Machine Translated by Google

68 CHƯƠNG 3 CÁC THUẬT TOÁN GIẢI QUYẾT VẤN ĐỀ

Trong khi các học sinh khác từ từ cộng các con số, cậu bé Carl đã phát hiện ra một điểm
thiếu sót và ngay lập tức đến với đáp án là 5,050. Anh ấy là học sinh duy nhất tìm được tổng
chính xác. Sự hiểu biết sâu sắc của ông là nhận thấy rằng 1 có thể được ghép với 100, 2 với
99, 3 với 98, v.v. để tạo ra 50 tổng giống hệt nhau của 101. Do đó, câu trả lời là 101 · 50 = 5050.
Một cách khác, chính thức hơn để làm điều này là viết tổng trong câu hỏi hai lần, trước tiên chuyển tiếp, sau

đó lùi lại:

S = 1 + 2 +. .. + 99 + 100, S = 100 + 99
+. . . + 2 + 1.

Sau đó, tất nhiên, 2S = 100 · 101. Ưu điểm của phương pháp này là không quan
trọng số lượng các điều khoản là chẵn hay lẻ (lưu ý rằng đây là một vấn đề với
phương pháp ghép nối ban đầu).
Đây là một thủ thuật khá hay, đặc biệt là đối với al O-age, và nó có rất nhiều ứng dụng. Đừng bị giới

hạn số tiền. Tìm kiếm bất kỳ loại đối xứng nào trong một vấn đề và sau đó điều tra xem liệu việc ghép nối các

mục một cách thông minh có thể đơn giản hóa mọi thứ hay không. Đầu tiên, hãy giải quyết "Vấn đề về tủ khóa",

mà chúng ta gặp phải lần đầu tiên trong Ví dụ 2.2.3 trên trang 29.

Nhớ lại rằng chúng tôi đã giảm vấn đề này xuống

Chứng minh rằng d (n ) là số lẻ nếu và chỉ khi n là một hình vuông hoàn

hảo, trong đó d (n) biểu thị số ước của n, bao gồm 1 và n. Bây giờ chúng ta có thể sử dụng công cụ ghép nối

Gaussian. Sự đối xứng ở đây là người ta luôn có thể ghép một di visor d của n với njd. Ví dụ, nếu n = 28, là

"tự nhiên" để ghép số chia 2 với số chia 14. Vì vậy, khi chúng ta xem qua danh sách các ước của n, mỗi ước sẽ

có một "bạn đời" duy nhất trừ khi n là một số hoàn hảo . vuông, trong trường hợp đó Vn được ghép với chính nó.

Ví dụ, các ước của 28 là 1,2, 4, 7, 14,28, có thể được xếp sau thành các cặp (1, 28), (2, 14), (4,7), rõ ràng

là d ( 28) là số chẵn. Mặt khác, các ước của hình vuông hoàn hảo 36 là 1, 2,3,4,6,9, 1 2, 1 8, 36, ghép thành

(1,36), (2, 18), (3 , 1 2), (4,9),


lẻ (6,6).
(một sốLưu
cặpý số
rằng số cộng
đúng 6 được
vớighép vớichỉ
số 6, chính nó, được
có thể do đóđếm
số một
lượng ước thực sự là
lần).


2
Chúng ta có thể kết luận rằng d (n) là số lẻ nếu và chỉ khi n là một hình vuông hoàn hảo.

Lập luận được sử dụng có thể được lặp lại gần như giống hệt nhau, nhưng trong một bối cảnh hoàn toàn khác,

để chứng minh một định lý nổi tiếng trong lý thuyết số.

Ví dụ 3.1.9 Định lý Wilson. Chứng minh rằng với mọi số nguyên tố p,

(p - l)! == - 1 (mod p) .

Giải pháp: Đầu tiên hãy thử một ví dụ. Cho p = 1 3. Khi đó tích trong câu hỏi là 1 · 2 · 3 · 4 · 5 · 6 ·

7. 8 · 9 · 10 · 11. 1 2. Một cách để đánh giá sản phẩm này modulo 13 sẽ là nhân tất cả ra, nhưng đó không phải

là cách của người giải quyết vấn đề! Chúng ta nên tìm kiếm các sản phẩm con nhỏ hơn để dễ tính modulo p. Các số

dễ tính nhất là 0, 1 và -1. Vì p là số nguyên tố nên không thừa số 1, 2,3, ... p - 1 đồng dư với 0 modulo p.

Tương tự như vậy, không có sản phẩm con nào có thể đồng dư với O. Mặt khác, tính nguyên thủy của
mọip số
ngụ
khác
ý rằng
0 đều

có một

2A đối số hoàn toàn khác sử dụng các phương pháp đếm cơ bản và tính chẵn lẻ. Xem Vấn đề 6. 1 .21 trên trang 1 95.
Machine Translated by Google

3.1 TỔNG HỢP 69

môđun nghịch đảo nhân p; tức là, nếu x không phải là bội của p thì có một y duy nhất E {I, 2, 3 ,. . . , p - I}

sao cho xy = 1 (mod p). (Nhớ lại cách chứng minh khẳng định này trong Ví dụ 2.3.4 ở trang 44.)

Với thông tin này, chúng ta tiến hành như trong bài toán khóa: Ghép từng phần tử x
E {I, 2, 3, ... , p - I} với người bạn đời "tự nhiên" của nó là y E {I, 2, 3, .. . , p -
I} sao cho xy = 1 (mod p). Ví dụ, nếu p = 1 3, các cặp là

(1, 1), (2,7), (3, 9), (4, 10), (5, 8), (6, 1 1), (12, 12),

và chúng ta có thể viết lại

1 2! = 1 · 2 · 3 · 4 · 5 · 6 · 7 · 8 · 9 · 10 · 1 1 · 12

như

1 · (2 · 7) (3 · 9) (4 · 1 0) (5 · 8) (6 · 1 1) · 12 = 1 · 1 · 1 · 1 · 1 · 1 · 12 = - 1 (mod 13).

Lưu ý rằng 1 và 12 là các phần tử duy nhất được ghép nối với chính chúng. Cũng lưu ý rằng 12 = -1 (mod 13). Nói

chung, chúng ta sẽ hoàn thành việc đó nếu chúng ta có thể chỉ ra rằng x là modulo nghịch đảo cấp số nhân của

riêng nó nếu và chỉ khi x = ± 1. Nhưng điều này rất dễ dàng: Nếu J? = 1 (mod p), sau đó

2 x - 1 = (x - l) (x + 1) = 0 (mod p).

Vì p là số nguyên tố nên điều này ngụ ý rằng x - I hoặc x + 1 là bội số của p; tức là, x = ± 1 (mod p) là khả

năng duy nhất .


_

Đối xứng trong Đa thức và Bất đẳng thức

Các bài toán đại số với nhiều biến hoặc mức độ cao thường khó giải trừ khi có một số đối xứng cơ bản để khai

thác. Đây là một ví dụ đáng yêu.

Ví dụ 3.1.10 Giải x4 + .J + J? + x + 1 = o.

Giải pháp: Trong khi có nhiều cách khác để tiếp cận vấn đề này (xem trang 1 26), chúng tôi sẽ sử dụng

tính đối xứng của các hệ số như một điểm khởi đầu để áp đặt tính đối xứng hơn, về bậc của các số hạng. Đơn giản

là chia cho J? :

1 1
2 x = 0.
+ x + l +xx
- + z

Điều này trông không đơn giản hơn, nhưng lưu ý rằng bây giờ có nhiều đối xứng hơn, vì chúng tôi có thể thu thập
các thuật ngữ "thích" như sau:

� + xz 1 + x +x - + 1 = 0. (1)

Bây giờ thực hiện thay thế u: = x + !. Lưu ý rằng


x
Machine Translated by Google

70 CHƯƠNG 3 CHIẾN THUẬT GIẢI QUYẾT VẤN ĐỀ

do đó (1) trở thành u2 - 2 + u + 1 = 0, hoặc u2 + u - 1 = 0, có nghiệm

- 1 ± V5
U =
2

!
hoặc Giải ra x + = u, chúng ta nhận được x2 - ux + 1 = 0,
x

u ± Ju2 -4
x = -----
2

Kết hợp những thứ này lại với nhau, chúng ta có

± _ 4
2 2


- 1 ± 0 � ( _ 1 ± V5) - 1 ± V5 ± i \ 1t0 ± 2V5
x =
2 4

Vài bước cuối cùng chỉ là "chi tiết kỹ thuật". Hai bước đi mấu chốt là để
tăng tính đối xứng của bài toán và sau đó thực hiện phép thay thế đối xứng u =
x + x-1 •
Trong ví dụ tiếp theo, chúng tôi sử dụng đối xứng để giảm độ phức tạp của một bất đẳng thức.

Ví dụ 3. 1.11 Chứng minh rằng

(a + b) (b + c) (c + a) 2 8abc

Đúng với mọi số dương a, b và c, chỉ bằng nhau khi a = b = c.

Giải pháp: Quan sát rằng bất đẳng thức được cho là đối xứng, nó không thay đổi nếu chúng
ta hoán vị bất kỳ biến nào. Điều này gợi ý rằng chúng ta không nhân phần bên trái (hiếm khi
là một ý tưởng khôn ngoan!) Mà thay vào đó hãy xem xét các phần nhân tử, để biết chuỗi

a + b, b + c, c + a

có thể được suy ra bằng cách chỉ nhìn vào số hạng a + b và sau đó thực hiện chu kỳ cho mỗi đột biến

a t --- + b, b t --- + C, C t --- + a một lần và sau đó hai lần.


Phiên bản hai biến đơn giản của bất đẳng thức Số học-Hình học-Trung bình
(xem Phần 5.5 để biết thêm chi tiết) ngụ ý

a + b 2 2vah.

Bây giờ, chỉ cần thực hiện các hoán vị tuần hoàn

c + a 2 2vca ·

Bất đẳng thức mong muốn theo sau bằng cách nhân ba bất đẳng thức này.

Machine Translated by Google

3.1 TỔNG HỢP 71

Nó là giá trị khám phá khái niệm của hoán vị chu kỳ chi tiết hơn. Đưa ra một

biểu thức n-biến f (x \, X2, '", xn), chúng tôi sẽ biểu thị tổng chu kỳ bằng �f (X \, X2

,. .., Xn) : =
một

f (x \, X2, ···, xn) + f (X2, X3 ,'", xn, xd +... + f (xn, Xl,'", xII-d · Ví dụ:

nếu các biến của chúng ta là x, y và z, thì

� x3 = � + y3 + z3 và � xz2 = xz2 + yx2 + zi.


một một

Hãy để chúng tôi sử dụng ký hiệu này để nhân tử một bậc ba đối xứng trong ba biến.

Ví dụ 3.1.12 Hệ số a3 + b3 + e3 - 3abe .

Giải pháp: Chúng tôi hy vọng điều tốt nhất và tiến hành một cách ngây thơ, đảm bảo rằng các suy đoán

của chúng tôi luôn cân xứng. Dự đoán đơn giản nhất cho một thừa số sẽ là a + b + e, vì vậy chúng ta hãy thử nó.

Giải thích a + b + e bằng a2 + b2 + e2 sẽ cho chúng ta các số hạng bậc ba, với một số thuật ngữ lỗi. Cụ thể,

chúng tôi có

a3 + b3 + c3 - 3abc = (a + b + c) (a2 + b2 + (2) - � (a2h + h2a) - 3ahc =


một

(a + b + c) (a2 + b2 + e2) - � ( a2b + h2a + abe) =


một

(a + b + c) (a2 + b2 + e2) - � (ab (a + b + e ))


một

� (a + h + c) (i '+ b2 + c "- � ab)

= (a + b + e) (a2 + b2 + e2 - ab - he - ac).

Chú ý cách ký hiệu La tiết kiệm thời gian và khi bạn đã quen với nó, giảm khả năng mắc lỗi.

Các vấn đề và bài tập

3.1.13 Tìm độ dài của đường đi ngắn nhất từ điểm Bạn có thể muốn sử dụng hàm d (n) (được định nghĩa trong lời giải

(3,5) đến điểm (8, 2) tiếp xúc với trục x và cũng cho vấn đề Tủ khóa ở trang 68) trong công thức của mình.

tiếp xúc với trục y.

3.1.14 Trong ví dụ 1 .2. 1 ở trang 4, chúng ta thấy 3.1.16 Có bao nhiêu tập con của tập {I, 2,3,4. ...,
rằng tích của bốn số nguyên liên tiếp luôn nhỏ hơn 30} có tính chất là tổng các phần tử của tập hợp con
một hình vuông. Bạn có thể nói gì về tích của bốn lớn hơn 232?
số hạng liên tiếp của một cấp số cộng tùy ý, ví dụ:
3.1.17 (Putnam 1 998) Cho một điểm (a, b) với 0 < b
3. số 8 . 13. 1 8?
< a, hãy xác định chu vi nhỏ nhất của một tam giác
3.1.15 Tìm (và chứng minh) một công thức đẹp cho có một đỉnh tại (a, b), một trên trục x và một trên
tích các ước của bất kỳ số nguyên nào. Ví dụ, nếu đường thẳng y = x. Bạn có thể cho rằng tồn tại một
n = 1 2, tích các ước của nó là tam giác có chu vi nhỏ nhất.

1 · 2 · 3 · 4 · 6 · 12 = 1728.
Machine Translated by Google
72 CHƯƠNG 3 CHIẾN THUẬT GIẢI QUYẾT VẤN ĐỀ

3.1.18 Một đa thức trong một số biến được gọi là đối xứng (e) Có thể viết đa thức đối xứng nào trong ba biến ables

nếu nó không thay đổi khi các biến được hoán vị. Ví dụ, dưới dạng đa thức trong hàm đối xứng tary không?

2 (f) Có thể viết đa thức (không nhất thiết đối xứng ric)
J (x, y, z): = X + i + Z2 + xyz
ba biến dưới dạng đa thức trong hàm đối xứng sơ cấp

là đối xứng, vì không?

(g) Tổng quát hóa cho nhiều biến hơn. Nếu bạn không hiểu,
J (x, y, z) = J (X, z, y) = J (y, x, Z) = J (y, z, X)
hãy xem trường hợp hai biến (SI: = x + y, S2: = xy).
= J (Z, x, y) = J (Z, y, X).
(h) Mối quan hệ, nếu có, giữa các tổng tuần hoàn và

Một đa thức trong một số biến được gọi là đồng nhất bậc r các hàm đối xứng sơ cấp là gì?

nếu tất cả các số hạng đều là bậc r. Ví dụ: g (x, y): = x


2
+ 5xy là thuần nhất bậc 2. (Số hạng
3.1.19 Xem xét Ví dụ 3. 1 .6, Four Bugs prob lem. Khi những
5xy được coi là có bậc 2, vì nó là tích của hai số hạng
con bọ di chuyển, chúng "biến". Ví dụ: nếu một con bọ bắt
bậc-.) Nói chung, Xk ) là đồng nhất của đa thức k biến g (XI,
đầu quay mặt về hướng bắc, nhưng sau đó dần dần quay mặt về
r nếu với mọi t, chúng ta có X2 , ... neous độ
hướng tây, nó sẽ quay 90 °. Nó thậm chí có thể là các lỗi

quay hơn 360 °. Mỗi con bọ quay (tính bằng độ) bao nhiêu

trước khi chúng đâm vào nhau?


g (tx1, tx2, ···, txk) = t ' g (XI, X2, ··· , Xk) ·

Cho ba biến x, y, z, chúng ta xác định các hàm đối xứng cơ


3.1.20 Hãy xem xét trò chơi hai người sau đây. Mỗi người
bản
chơi lần lượt đặt một đồng xu trên bề mặt của một chiếc bàn

SI: = x + y + z, hình chữ nhật. Không một xu nào có thể đụng đến một xu đã có

sẵn trên bàn. Bảng bắt đầu không có ngòi bút. Người chơi
S2: = xy + yz + zx,
cuối cùng thực hiện một nước đi hợp pháp sẽ thắng.
S 3: = xyz.
Người chơi đầu tiên có chiến lược chiến thắng không?

Hàm đối xứng sơ cấp Sk là hàm đối xứng, thuần nhất bậc k và 3.1.21 Một quả bóng bi-a (đường kính vô phân số) đập

tất cả các hệ số của nó bằng I. Hàm đối xứng cơ bản có thể


vào tia Be tại điểm e, với góc tới a như hình vẽ. Quả
được xác định với bất kỳ số biến nào. Ví dụ: đối với bốn bóng bi-a tiếp tục đường đi của nó, bật ra khỏi các đoạn
biến x, y, z, w, chúng thẳng AB và Be theo quy tắc "góc tới bằng góc phản xạ."
Nếu AB = Be, xác định số lần quả bóng bật ra khỏi hai
đoạn thẳng (kể cả lần nảy đầu tiên, lúc e). Câu trả lời
SI: = x + y + z + w,
của bạn sẽ là một hàm của a và / 3.
S2: = xy + xz + xw + yz + yw + zw,

S 3: = xyz + xyw + xzw + yzw,

S4: = xyzw.

(a) Xác minh rằng

2
2 x + i + z2 = (x + y + z) - 2 (xy + yz + zx)

= si - 2S2,
B c
trong đó Si là các hàm chức năng đối xứng cơ bản
trong ba biến số.
3.1.22 Đạn được phóng từ chính giữa sàn của một căn phòng

(b) Tương tự, biểu diễn x3 + y3 + z3 dưới dạng đa thức hình chữ nhật rộng 40 feet với trần rất cao. Đạn chạm vào

trong các hàm đối xứng cơ bản. bức tường ở độ cao chính xác 10 feet so với mặt sàn, phản xạ

khỏi bức tường này (tuân theo quy tắc "góc tới tương đương
(c) Làm tương tự đối với (x + y) (x + z) (y + z).
với một tia phản xạ"), chạm vào bức tường đối diện và lại
(d) Làm tương tự với xl + yz4 + zx4 + xz4 + yx4 + lệch một lần nữa, cuối cùng hạ cánh trở lại chính xác nơi nó
zl · là
Machine Translated by Google

3.2 NGUYÊN TẮC MỞ RỘNG 73

phóng mà không chạm trần. Đây là điều có thể xảy ra bởi vì rằng mỗi điểm được chọn một cách độc lập so với sự phân bố

viên đạn không di chuyển theo đường thẳng mà di chuyển dọc đồng đều trên mặt cầu.)

theo các đoạn parabol do trọng lực. Khi đạn ở điểm cao nhất
3.1.30 Tính đối xứng trong xác suất. Hãy tưởng tượng bạn thả
thì nó ở độ cao bao nhiêu so với mặt sàn?
ba chốt ngẫu nhiên trên khoảng đơn vị [0, 1]. Họ tách khoảng

thời gian thành bốn phần. Chiều dài trung bình của mỗi mảnh là

3.1.23 Nhớ lại rằng hình elip được định nghĩa là quỹ tích của bao nhiêu? Rõ ràng là câu trả lời "nên" là 1/4 và điều này sẽ

tất cả các điểm trong mặt phẳng, tổng khoảng cách của chúng đúng nếu các phân phối xác suất (trung bình, độ lệch chuẩn,

đến hai điểm cố định (foci) là một hằng số. Chứng minh tính v.v.) cho mỗi độ dài trong số bốn độ dài là giống nhau. Và

chất phản xạ của hình elip: nếu bàn bi-a được xây bằng một bức trong chứng thư, điều này là đúng. Một cách để thấy điều này

tường hình elip và bạn bắn một quả bóng từ tiêu điểm này đến là tưởng tượng rằng chúng ta không thực sự thả ba chốt trên

bất kỳ điểm nào trên tường, bóng sẽ phản xạ khỏi tường và đi một đường thẳng, mà thay vào đó là thả bốn chốt trên một hình

thẳng đến tiêu điểm khác. tròn có chu vi 1 đơn vị. Bất cứ nơi nào điểm thứ tư tiếp đất,

hãy cắt vòng tròn ở đó và "mở" nó ra để tạo thành khoảng đơn


3.1.24 Nhớ lại rằng parabol được định nghĩa là điểm chung của
vị. Suy ngẫm về lập luận này cho đến khi nó có ý nghĩa.
tất cả các điểm trong mặt phẳng, sao cho khoảng cách đến một

điểm cố định (tiêu điểm) bằng khoảng cách đến một đường thẳng
Sau đó, hãy thử một số vấn đề tiếp theo!
cố định (ma trận). Chứng minh tính chất phản xạ của parabol:

nếu một chùm ánh sáng truyền theo phương vuông góc với ma (a) Một bộ bài bình thường gồm 52 lá với bốn quân át được

trận, chiếu vào bất kỳ điểm nào trên mặt lõm của gương xáo trộn, và sau đó các quân bài được rút từng lá một

parabol, chùm tia sẽ phản xạ ra khỏi gương và truyền thẳng cho đến khi quân át đầu tiên xuất hiện. Hỏi trung bình

đến tiêu điểm. có bao nhiêu quân bài được rút ra?

(b) (Jim Propp) Đưa ra một bộ bài 52 lá, trích ra 26


3.1.25 Một hành tinh hình cầu, ba chiều có cen ter tại (0,0,

0) và bán kính 20. Tại bất kỳ điểm nào (x, y, z) trên bề mặt ngẫu nhiên các quân bài theo một trong (��) các
của hành tinh này, nhiệt độ là T (x, y , z) : = (x + y) 2 + cách và đặt chúng lên đầu bộ bài theo thứ tự tương đối

(y - z) 2 độ. Nhiệt độ trung bình của bề mặt hành tinh này là như trước khi được chọn. Số lượng quân bài dự kiến

bao nhiêu? hiện chiếm cùng vị trí trong bộ bài như trước là bao

nhiêu?
3.1.26 (Putnam 1980) Đánh giá

2
r /

dx
· (c) Cho một dãy n số nguyên phân biệt bất kỳ, chúng ta
1 + (tanx) v'2
tính "số hoán đổi" của nó theo cách sau: Đọc từ trái
o

sang phải, bất cứ khi nào chúng ta gặp một số nhỏ hơn
3.1.27 (Hungary 1906) Gọi K, L, M, N là tâm của các hình vuông
số đầu tiên
được dựng ở bốn cạnh (mặt ngoài) của hình thoi. Chứng minh ber trong dãy, chúng ta hoán đổi vị trí của
rằng đa giác KLMN là hình vuông. nó với số đầu tiên trong dãy. Chúng tôi tiếp
tục theo cách này cho đến khi chúng tôi đi

3.1.28 Làm rõ vấn đề trên bằng cách chỉ ra rằng kết luận vẫn đến cuối trình tự. Số hoán đổi của dãy số là

đúng nếu hình thoi chỉ đơn thuần là một hình bình hành arbi
tổng số lần hoán đổi. Ví dụ, dòng thứ 3,4,2,

trary. tôi có số hoán đổi là 2, chúng ta hoán đổi 3


với 2 để được 2,4,3, 1 và sau đó chúng ta
3.1.29 (Putnam 1992) Bốn điểm được chọn tại ran dom trên bề
hoán đổi 2 với 1 để được 1,4,3, 2. Tìm giá
mặt của một quả cầu. Tính xác suất để tâm của mặt cầu nằm bên
trị trung bình của các số hoán đổi của 7! =
trong tứ diện có các đỉnh nằm ở bốn điểm là bao nhiêu? (Nó
5040 hoán vị khác nhau của các số nguyên 1,2,3,4,5,6,7.
được hiểu

3.2 Nguyên tắc Cực đoan

Khi bạn bắt đầu vật lộn với một vấn đề, một trong những khó khăn là có quá
nhiều thứ cần theo dõi và hiểu rõ. Một vấn đề có thể liên quan đến một chuỗi có
nhiều (có thể là vô hạn) phần tử. Một bài toán hình học có thể sử dụng nhiều
Machine Translated by Google

74 CHƯƠNG 3 CHIẾN THUẬT GIẢI QUYẾT VẤN ĐỀ

các đường khác nhau và các hình dạng khác. Một người giải quyết vấn đề giỏi luôn cố gắng sắp xếp
khối lượng công việc này. Một chiến thuật cơ bản là nguyên tắc cực đoan:

Nếu có thể, hãy giả sử rằng các yếu tố của vấn đề của bạn là "theo thứ tự."
Tập trung vào các yếu tố "lớn nhất" và "nhỏ nhất", vì chúng có thể bị bó buộc
theo những cách thú vị.

Điều này có vẻ gần như sáo mòn, nhưng nó hoạt động kỳ diệu trong một số tình huống (ví dụ, vấn đề
Hành động khẳng định trên trang 21). Đây là một ví dụ đơn giản.

Ví dụ 3.2.1 Gọi B và W lần lượt là tập hợp hữu hạn các điểm đen và trắng trong mặt phẳng, với tính
chất là mọi đoạn thẳng nối hai điểm cùng màu đều chứa một điểm khác màu. Chứng minh rằng cả hai
tập hợp phải nằm trên một đoạn thẳng duy nhất.

Giải pháp: Sau khi thực nghiệm, có vẻ như nếu tất cả các điểm không nằm trên một đường thẳng
thì không thể có tối đa bao nhiêu điểm trong số đó. Có thể chứng minh điều này bằng nhiều sơ đồ
phức tạp cho thấy rằng "bạn luôn có thể vẽ một điểm mới", nhưng điều này không dễ dàng. Nguyên tắc
cực đoan để giải cứu: Giả sử rằng tất cả các điểm không nằm trên một đường thẳng. Sau đó, chúng
tạo thành ít nhất một tam giác. Xét tam giác có diện tích nhỏ nhất. Hai trong số các đỉnh của nó
có cùng màu, vì vậy giữa chúng là một điểm có màu khác, nhưng điều này tạo thành một tam giác nhỏ
hơn - một sự mâu thuẫn!
_

Lưu ý cách nguyên tắc cực đoan ngay lập tức giải quyết vấn đề. Nó dễ dàng đến mức gần như
giống như gian lận. Cấu trúc của lập luận được sử dụng là một điển hình: Làm việc theo mâu thuẫn;
cho rằng bất cứ điều gì bạn muốn chứng minh là không đúng.
Sau đó, nhìn vào phần tử tối thiểu (hoặc cực đại) và phát triển một đối số đặt
ra một phần tử nhỏ hơn (hoặc lớn hơn), đó là mâu thuẫn mong muốn. Miễn là một
tập hợp các số thực là hữu hạn, nó sẽ có một phần tử nhỏ nhất và một phần tử tối
đa. Đối với tập hợp vô hạn, có thể không có giá trị cực trị (ví dụ: coi tập vô
2 3
{I, 2, 1 / 2,2 , 1/22,hạn 23 , ... }, không có số el tối thiểu và tối đa
, 1/2
ement), nhưng nếu tập hợp bao gồm các số nguyên dương, chúng ta có thể sử dụng Nguyên tắc Sắp
xếp Tốt: Mọi tập hợp các số nguyên dương không rỗng đều có một phần tử nhỏ nhất.

Đây là một ví dụ đơn giản khác. Chiến thuật được sử dụng là nguyên tắc cực đoan; Mấu chốt
di chuyển là một cấu trúc hình học thông minh.

Ví dụ 3.2.2 (Hàn Quốc 1995) Xét có vô số điểm trong mặt phẳng sao cho nếu ta chọn ba điểm A, B, C
bất kỳ trong số đó thì diện tích tam giác ABC luôn là
Machine Translated by Google

3.2 NGUYÊN TẮC CỰC KỲ 75

nhỏ hơn 1. Chứng tỏ rằng tất cả các điểm này nằm trong bên trong hoặc trên ranh giới của một
tam giác có diện tích nhỏ hơn 4.

Giải: Cho tam giác ABC có diện tích lớn nhất trong tất cả các tam giác mà các đỉnh của
chúng được lấy từ tập hợp các điểm đã cho. Gọi [ABC] là diện tích tam giác ABC.
Khi đó [ABC] < 1. Cho tam giác LMN là tam giác có trung tuyến là ABC. (Nói cách khác,
A, B, C là trung điểm của các cạnh của tam giác LMN dưới đây.)

Khi đó [LMN] = 4 [ABC] < 4. Chúng tôi cho rằng tập hợp các điểm phải nằm trên biên hoặc bên
trong của LMN. Giả sử một điểm P nằm ngoài LMN. Khi đó ta có thể nối P với hai trong số các
đỉnh của ABC tạo thành một tam giác có diện tích lớn hơn ABC, suy ra tính cực đại của [ABC].


Luôn luôn lưu ý về thứ tự và mức tối đa / tối thiểu trong một vấn đề, và luôn chắc
chắn, nếu có thể, rằng các phần tử được sắp xếp theo thứ tự (chúng tôi gọi đây là đơn thức hóa).
Hãy coi đây là "thông tin miễn phí." Ví dụ tiếp theo minh họa nguyên tắc một lần nữa
rằng việc xem xét kỹ các yếu tố tối đa (và tối thiểu) thường mang lại hiệu quả. Lần
đầu tiên bạn gặp phải vấn đề này là Vấn đề 1.1.4 ở trang 2. Chúng tôi chia giải pháp
thành hai phần: phần điều tra, sau đó là phần viết lại chính thức.

Ví dụ 3.2.3 Tôi mời 10 cặp đôi đến dự tiệc tại nhà tôi. Tôi hỏi tất cả những người có mặt,
kể cả vợ tôi, họ đã bắt tay với bao nhiêu người. Hóa ra là tất cả mọi người đều thắc mắc -
tôi không hề thắc mắc về bản thân mình, tất nhiên - đã bắt tay với một số người khác. Nếu
chúng ta giả sử rằng không có ai bắt tay đối tác của mình, thì vợ tôi đã bắt tay với bao
nhiêu người? (Tôi đã không tự hỏi mình bất kỳ câu hỏi nào.)

Điều tra: Vấn đề này có vẻ khó chữa. Dường như không có đủ thông tin. Tuy nhiên, chúng
ta có thể làm cho nó dễ dàng hơn bằng cách xem xét một trường hợp đơn giản hơn, một trường
hợp, chẳng hạn, có hai cặp vợ chồng ngoài chủ nhà và nữ tiếp viên.
Người dẫn chương trình phát hiện ra rằng trong số năm người mà anh ta thẩm vấn, có
năm "số bắt tay" khác nhau. Vì những con số này bao gồm từ 0 đến 4 (không ai bắt tay
với đối tác của họ), năm số bắt tay được phát hiện là 0, 1,2,3 và 4. Hãy gọi những
người này lần lượt là PO, p},trong
..., sơ
P4 đồ
và của
hãy chúng
vẽ mộttabức tranh,
(với nhãn bao
H). gồm cả máy chủ
Machine Translated by Google
76 CHƯƠNG 3 CHIẾN THUẬT GIẢI QUYẾT VẤN ĐỀ

Po

Thật thú vị khi nhìn hai người có số bắt tay cực khủng, tức là Po và P4. Hãy xem xét Po, thành

viên "ít hòa đồng nhất" trong nhóm. Anh ấy hoặc cô ấy không bắt tay ai cả. Còn người "thích giao du

nhất", P4 thì sao? P4 bắt tay với tất cả mọi người có thể, có nghĩa là tất cả mọi người ngoại trừ đối

tác của P4 đều bắt tay P4 ! Vì vậy, tất cả những ai không phải là đối tác của P4 đều có số lần bắt tay

khác không. Vì vậy P4 và Po phải là đối tác của nhau!

Tại thời điểm này, chúng ta có thể thư giãn, vì chúng ta đã đạt được bước tiến then chốt. Bản năng

của chúng ta cho chúng ta biết rằng có lẽ P3 và PI cũng là đối tác của nhau. Làm thế nào chúng ta có

thể chứng minh điều này? Hãy cố gắng điều chỉnh lập luận mà chúng ta vừa sử dụng. P3 đã bắt tay tất cả,

trừ một người không phải đối tác của mình. PI chỉ bắt tay một người. Chúng tôi có thêm thông tin nào không?

Đúng: P4 là một người mà PI đã bắt tay và một trong những người mà P3 đã bắt tay
cùng. Nói cách khác, nếu chúng ta loại trừ P4 và đối tác của anh ấy hoặc cô ấy, Po,
thì PI và P3 đóng vai trò mà Po và P4 đã đóng; tức là, họ tương ứng là ít nhất và
mang tính cộng đồng nhất, và theo cùng một lý do, phải là đối tác của nhau (P3 bắt
tay với hai trong số ba người
P3, vì
PI,vậy
P2,người
H, vàduy
chúng
nhấttacóbiết
thể rằng
mà P3PIcókhông
thể hợp
bắt tác
tay PI).

với

Bây giờ chúng ta đã hoàn thành, vì chỉ còn lại hai người là P2 và H. Họ phải là đối tác, vì vậy P2

là bà chủ, và cô ấy bắt tay hai người. Có thể dễ dàng áp dụng lập luận này vào trường hợp chung của n

cặp vợ chồng.

Giải pháp chính thức: Chúng tôi sẽ sử dụng cảm ứng. Với mỗi số nguyên dương n, hãy xác định câu

lệnh P (n) là

Nếu người dẫn chương trình mời n cặp đôi và nếu không một người bắt tay đối
tác của mình và nếu mỗi người trong số 2n+ người được người dẫn chương trình
thẩm vấn bắt tay một số lượng khác nhau, thì người chủ nhà bắt tay thứ n.

Dễ dàng kiểm tra P ( 1) là đúng bằng cách vẽ sơ đồ và tìm ra khả năng hợp lý duy nhất. Chúng ta

cần chứng minh rằng P ( n) ngụ ý P ( n + 1), và chúng ta sẽ hoàn thành. Vì vậy, giả sử rằng P (n) đúng

với một số nguyên dương n. Bây giờ, hãy xem xét một bữa tiệc có n + 1 cặp đôi (không phải chủ nhà và nữ

tiếp viên) thỏa mãn các giả thuyết (không ai bắt tay đối tác; tất cả các số lần bắt tay đều khác nhau).

Sau đó, tất cả các số bắt tay từ 0 đến 2 (n + 1) = 2n + 2 bao gồm sẽ xảy ra trong số 2n + 3 người được

người dẫn chương trình thẩm vấn. Xét người X, người lắc nhiều nhất là 2n + 2 tay. Người này đã bắt tay

tất cả, trừ hai trong số 2n + 4 người trong bữa tiệc. Vì không ai bắt tay mình hoặc bạn đời của họ, nên

X bắt tay với tất cả mọi người có thể.

Vì vậy, người duy nhất có thể hợp tác với X phải là người Y, người không bắt tay, vì những người khác

đều bắt tay với X và do đó không đủ tư cách là


Machine Translated by Google
3.2 NGUYÊN TẮC MỞ RỘNG 77

cộng sự.

Bây giờ, chúng ta hãy xóa X và Y khỏi nhóm. Nếu chúng tôi không còn đếm số lần bắt tay liên quan

đến hai người này, chúng tôi sẽ giảm xuống còn một bữa tiệc với n cặp đôi được mời và số lần bắt tay

của ev eryone (ngoài người chủ trì, người mà chúng tôi không có thông tin) đã giảm xuống chính xác 1,

vì tất cả mọi người bắt tay với X và không ai bắt tay Y. Nhưng giả thuyết quy nạp P ( n) cho chúng ta

biết rằng bà chủ tại bữa tiệc "rút gọn" này đã bắt tay n . Nhưng trên thực tế, bà chủ đã bắt tay thêm

một cái của chữ X mà chúng tôi vừa loại bỏ. Vì vậy, trong bữa tiệc có n + 1 khách được mời, bà chủ bắt

tay n + 1 , thiết lập P ( n + 1).


Thông thường, các vấn đề liên quan đến nguyên tắc cực đoan cộng với một phong cách lập luận cụ

thể, chẳng hạn như mâu thuẫn hoặc quy nạp. Một chiến thuật phổ biến là sử dụng các cực đoan để "rút

gọn" một vấn đề thành một phiên bản nhỏ hơn, như trên. Đối số chính thức sau đó sử dụng quy nạp. Đây

là một ví dụ khác. Như trên, chúng tôi chia nhỏ giải pháp của mình thành một cuộc điều tra không chính

thức, sau đó là một cuộc điều tra chính thức.

Ví dụ 3.2.4 (Olympic Thành phố St.Petersburg năm 1996) Một số số nguyên dương được viết mười trên bảng

đen. Người ta có thể xóa bất kỳ hai số nguyên phân biệt nào và viết ước số chung lớn nhất và bội số

chung nhỏ nhất của chúng. Chứng minh rằng cuối cùng các số sẽ ngừng thay đổi.

Điều tra: Trước khi bắt đầu, chúng tôi đề cập đến một vài định nghĩa lý thuyết số đơn giản
hàng tấn. Xem Chương 7 để biết thêm chi tiết.

• Đối với các số nguyên a và b, ký hiệu alb có nghĩa là "a chia cho b"; tức là, tồn tại một

số nguyên m sao cho am = b.


• Ước chung lớn nhất của a và b được xác định là số nguyên g lớn nhất thỏa mãn cả gla và glb. Kí

hiệu được sử dụng là GCD (a, b). Đôi khi (a, b)


cũng được sử dụng.

• Bội số chung nhỏ nhất của a và b được xác định là số nguyên dương u nhỏ nhất thỏa mãn cả alu và

blu. Kí hiệu được sử dụng là LCM (a, b). Một số lần [a, b] cũng được sử dụng. Chúng ta phải

xác định rằng bội số phổ biến nhất là số dương, nếu không nó sẽ là -oo!

Bắt đầu với một ví dụ đơn giản về hai số như 1 0, 15. Số này ngay lập tức được chuyển thành 5,

30, sau đó không bao giờ thay đổi, kể từ 5130. Đây là một ví dụ phức tạp hơn. Chúng tôi sẽ viết đậm

hai phần tử bị xóa và thay thế,

11
16 30 72
11
2 240 72

2 240 792
1 1 2 24 7920,

và một lần nữa, trình tự sẽ không thay đổi sau đó, vì

1 12, 2124, 2417920.

Một vài thử nghiệm nữa (hãy thực hiện chúng!) Dễ dẫn đến phỏng đoán sau:
Machine Translated by Google
78 CHƯƠNG 3 CÁC THUẬT TOÁN GIẢI QUYẾT VẤN ĐỀ

Cuối cùng, trình tự sẽ tạo thành một chuỗi trong đó mỗi phần tử sẽ di
chuyển tiếp theo (khi được sắp xếp theo thứ tự). Hơn nữa, phần tử nhỏ
nhất và phần tử lớn nhất của chuỗi này lần lượt là ước chung lớn nhất và
bội chung nhỏ nhất của tất cả các số ban đầu.

Làm thế nào để chúng ta sử dụng nguyên tắc cực trị để chứng minh điều này? Tập
trung vào điểm ít nhất của trình tự ở mỗi giai đoạn. Trong ví dụ trên, các phần tử ít
tôi,
phần ,tử
các nhất
phần
nhỏlà
tử
nhất
1.
nàyTương
không
là 1 tự
tăng,
1, như
2, trong
1vậy,
72,240,792,7290.
hãy
khi xem
dãy các
các phần
phần
Chúng
tử
tử ta
lớn
lớnquan
nhất.
nhấtsát
không
Trong
thấygiảm.

rằng
dụ Tại
dãy
của sao
các
chúng
điều này lại đúng? Gọi một < h là hai phần tử bị xóa và thay thế bằng

GCD (a, h), LCM (a, h).

Chú ý rằng nếu alh, thì a = GCD (a, b) và h = LCM (a, h), do đó, dãy không thay đổi.
Nếu không, chúng ta có GCD ( a , h) <a và LCM (a, h)> h . Do đó, nếu x là phần tử nhỏ
nhất hiện tại, ở lượt tiếp theo,

• Chúng tôi xóa x và một phần tử khác y (tất nhiên chúng tôi giả sử rằng x không chia
y), thay thế x bằng GCD (x, y) < x, tạo ra một phần tử mới nhỏ hơn và nhỏ nhất. •

Chúng tôi xóa hai phần tử, không có phần tử nào bằng x. Trong trường hợp này, phần
tử nhỏ hơn trong hai phần tử mới được tạo ra hoặc nhỏ hơn x, tạo ra phần tử mới
nhỏ nhất hoặc lớn hơn hoặc bằng x, trong trường hợp này x vẫn là phần tử nhỏ nhất.

Một đối số hoàn toàn tương tự hoạt động cho phần tử lớn nhất, nhưng hiện tại, chúng
ta hãy chỉ tập trung vào phần tử nhỏ nhất. Chúng tôi biết rằng phần tử ít nhất sẽ được
giữ nguyên hoặc giảm đi mỗi khi chúng tôi xóa và thay thế. Cuối cùng, nó sẽ chạm đáy.
Tại sao? Giá trị lớn nhất có thể mà chúng ta có thể gặp phải là bội số chung nhỏ nhất
của các phần tử ban đầu, vì vậy chỉ có rất nhiều giá trị có thể có cho chuỗi của chúng
ta ở mỗi giai đoạn. Cuối cùng thì trình tự không thể thay đổi, trong trường hợp đó thì
chúng tôi đã hoàn thành - đây là điều chúng tôi muốn chứng minh - hoặc cuối cùng trình
tự sẽ lặp lại. Do đó, chúng ta có thể phân biệt một phần tử nhỏ nhất nhỏ nhất,,nhỏ
tức là
phần tử f ít nhất có thể từng xuất hiện khi dãy của chúng ta phát triển. Chúng tôi khẳng
định rằng f phải chia tất cả các phần tử khác trong dãy mà nó xuất hiện. Nếu không, thì
chúng ta sẽ có fIx cho một số phần tử x, và sau đó nếu chúng ta xóa-và thay thế cặp C,
x, phần tử được thay thế GCD (f, x) < f, mâu thuẫn với tính nhỏ nhất của f.
Tại sao điều này tốt? Bởi vì một khi đạt được phần tử tối giản f , nó có
thể bị bỏ qua, vì f chia tất cả các phần tử khác và do đó dãy sẽ không thay đổi
nếu chúng ta cố gắng xóa và thay thế hai phần tử, một trong số đó là f. Do đó
phần "hoạt động" của dãy đã được rút ngắn một phần tử.
Đây là đòn bẩy vừa đủ để vượt qua một bằng chứng quy nạp! Hãy để chúng tôi kết luận
bằng một lập luận chính thức.

Giải pháp chính thức: Chúng tôi sẽ chứng minh khẳng định bằng cách quy nạp vào số
n phần tử trong dãy. Trường hợp cơ sở cho n = 2 là hiển nhiên: a, h trở thành

GCD (a, h), LCM (a, h)

mà sau đó không còn thay đổi.


Machine Translated by Google

3.2 NGUYÊN TẮC MỞ RỘNG 79

Bây giờ giả thiết quy nạp rằng tất cả các dãy có độ dài n cuối cùng sẽ trở nên không thay đổi.
Hãy xem xét một (n + 1 ) trình tự tính toán

U \, U2 ,. .. , Un, Un + l ·

Chúng tôi sẽ thực hiện thao tác xóa và thay thế lặp đi lặp lại trên trình tự này, với sự hiểu biết
rằng chúng tôi chỉ thực hiện các thao tác tạo ra sự thay đổi và ở mỗi giai đoạn, chúng tôi sẽ sắp
xếp các điều khoản theo thứ tự tăng dần. Chúng tôi thực hiện một số quan sát đơn giản:

1. Phần tử ít nhất trong dãy ở bất kỳ giai đoạn nào có thể có ít nhất bằng
ước chung lớn nhất của tất cả các phần tử ban đầu.

2. Phần tử lớn nhất trong dãy ở bất kỳ giai đoạn nào có thể có ít nhất bằng mUltiple chung
nhất của tất cả các phần tử ban đầu.

3. Phần tử nhỏ nhất tại bất kỳ giai đoạn nhất định nào luôn nhỏ hơn hoặc bằng phần tử nhỏ nhất
phần tử ở giai đoạn trước.

4. Vì chúng ta sắp xếp các số hạng theo thứ tự tăng dần, các quan sát (1) và (2) ngụ ý rằng
chỉ có thể có nhiều dãy nhất có thể.

Khi chúng tôi thực hiện các thao tác xóa và thay thế đối với chuỗi, hãy để £ j là ít nhất
phần tử của dãy ở giai đoạn i. Có hai tình huống có thể xảy ra.

• Cuối cùng, chúng tôi sẽ không thể thay đổi trình tự, trong trường hợp đó là
dãy phần tử ít nhất £ 1, £ 2 ,£ 3,. .. chấm dứt.

• Tại một thời điểm nào đó, chẳng hạn giai đoạn k, chúng ta sẽ quay trở lại một trình tự mà
trước đó đã bị uốn cong (vì quan sát (4) ở trên). Sau đó, dãy có ít phần tử nhất £ I, £ 2
,£ 3, ... có thể là vô hạn, nhưng £ k = £ k + 1 = £ k + 2 = .. ”do quan sát (3) trên.

Do đó, đối với mỗi trình tự ban đầu U \, U2 , ... , Un, Un + l, tùy thuộc vào cách nó

phát triển, sẽ có một giai đoạn k và một số £ : = £ k là phần tử nhỏ nhất nhỏ nhất từng xuất hiện ở
bất kỳ giai đoạn nào.
Số £ này phải chia tất cả các phần tử khác của dãy ở giai đoạn k và tất cả các giai đoạn sau,
vì nếu không, chúng ta có thể xóa và thay thế £ bằng một phần tử khác để tạo ra một phần tử nhỏ
nhất, mâu thuẫn với số nhỏ nhất của £. Do đó, khi chúng ta đến giai đoạn k, phần tử £ ít nhất nằm

ngoài bức tranh-khả năng duy nhất để xóa và thay thế để tạo ra thay đổi là sử dụng n phần tử còn
lại. Nhưng giả thuyết quy nạp nói rằng cuối cùng, n phần tử này sẽ không còn thay đổi nữa.

Do đó, cuối cùng, trình tự gia tốc (n + 1 ) ban đầu của chúng ta sẽ không còn thay đổi nữa.
_

Trong các bài toán phức tạp hơn, không phải lúc nào cũng rõ ràng thực thể nào nên được đơn

điệu hóa và Nguyên tắc sắp xếp tốt không phải lúc nào cũng đúng với các tập hợp vô hạn. Trong các
tình huống liên quan đến tập hợp vô hạn, đôi khi các lập luận cực đoan có tác dụng, nhưng bạn cần
phải cẩn thận.

Ví dụ 3.2.5 Cho f (x) là đa thức với hệ số thực bậc n sao cho f (x) 20 với mọi x E �. Xác định g (x) :

= f (x) + f '(x) + ! ,, (x) ··· + f (n) (x). Chứng tỏ

rằng g (x) 2 0 với mọi x E �.


Machine Translated by Google

80 CHƯƠNG 3 CÁC MĨ THUẬT GIẢI QUYẾT VẤN ĐỀ

Giải pháp: Có một số điều mà chúng ta có thể áp dụng các lập luận cực đoan.
Vì f (x) 2 0, chúng ta có thể muốn xem giá trị của x mà tại đó f (x) là cực tiểu. Đầu tiên
chúng ta cần chứng minh rằng giá trị này thực sự tồn tại, tức là có một Xo E lR sao cho f (xo)
là cực tiểu. [Điều này không đúng với tất cả các hàm, chẳng hạn như f (x) = l / x.] Viết

f (x) = anx "+ an_ 1x" -1 + ... + alx + aO ,

trong đó mỗi ai E lR. Vì f (x) luôn luôn không âm nên hệ số đứng đầu an phải là số dương, vì
số hạng đứng đầu sẽ chi phối giá trị của f (x) khi x là một số lớn dương hoặc âm. Hơn nữa,
chúng ta biết rằng n là số chẵn. Vì vậy, chúng tôi biết rằng

lim f (x) = lim f (x) = +00,


x --- + - oo x --- ++ oo

và do đó, f (x) có giá trị nhỏ nhất. Để ý rằng g (x) có cùng số hạng đứng đầu với f (x) nên g
(x) cũng có giá trị nhỏ nhất. Thật vậy, g (x) là thứ chúng ta sẽ tập trung chú ý vào. Chúng
tôi muốn chứng minh rằng đa thức này luôn không âm, do đó, một chiến lược đầy hứa hẹn là mâu
thuẫn. Giả sử rằng g (x) < 0 với một số giá trị của x.
Xét giá trị nhỏ nhất của nó , đạt được khi x = Xo. Khi đó g (xo) < O. Bây giờ, mối quan hệ giữa
1
g (x) và f (x) là gì? Vì f (x) là bậc n nên r + g ' (x) = J' (x) + f "(x) +(x)
...= +0 fvà
(n) (x) =

g (x) - f (x).

Do đó,

g ' (xo) = g (xo) - f (xo) < 0,

vì g (xo) là hoàn toàn âm và f (xo) là không âm theo giả thuyết. Nhưng con số này chứng minh
rằng g (x) đạt được giá trị nhỏ nhất của nó tại xo, khi đó g ' (xo) phải bằng 0!
_

Trong ví dụ tiếp theo, ghép nối Gaussian cộng với đơn hóa giúp giải quyết một
vấn đề khó khăn, từ IMO 1994 tại Hồng Kông.

Ví dụ 3.2.6 Cho m và n là các số nguyên dương. Cho al, a2, ... của ,là các phần tử riêng

{I, 2, ... k,biệt


1 �, kn}�sao
m,cho
vớibất
ai cứ
+ aj
khi= nào
ak. ai
Chứng
+ aj minh
� n rằng
với một số i, j, 1 � i � j � m, thì tồn tại

al + a2 + ... + am n + I 2

m 2 ·

Giải pháp: Đây không phải là một vấn đề dễ dàng. Một phần của khó khăn là tìm ra tuyên bố
của vấn đề! Chúng ta hãy gọi những chuỗi có thuộc tính được mô tả trong bài toán là chuỗi
"tốt". Nói cách khác, một dãy tốt là một dãy gồm các phần tử riêng biệt của {I, 2, ..., n} sao
ai, a2, ..., cho bất cứ khi nào ai + aj � n với một �
số k
i,�j,m,
1 với
� iai
�+ jaj
�= mak.
thìBắt
tồnđầu
tạibằng
k, 1
cách nhập các giá trị dễ dàng cho m và n, giả sử m = 4, n = 100. Chúng
a4 ta
gồmcócác
dãysốal,
nguyên
a2, a3,
dương
riêng biệt có thể nằm trong khoảng từ 1 đến 100, bao gồm cả. Một dãy số có thể có 14,99.
là 5,93,
Đây có
phải là một trình tự tốt không? Lưu ý rằng al + a2 = 98 � 100, vì vậy chúng ta cần có một k
sao cho ak = 98. Than ôi, không có. Cải thiện trình tự ban đầu, chúng tôi thử 5,93,98,99. Bây
giờ, hãy kiểm tra các cặp có tổng số lên đến 100 hoặc ít hơn. Ở đó
Machine Translated by Google

3.2 NGUYÊN TẮC MỞ RỘNG 81

chỉ là một, (5,93), và chắc chắn, tổng của chúng là một phần tử của dãy. Vì vậy, trình tự của
chúng tôi là tốt. Bây giờ hãy tính giá trị trung bình và thực sự, chúng ta có 5 + 93 + 98 +

99 100+ 1 2
.
4 2

Nhưng tại sao? Hãy thử xây dựng một dãy tốt khác, với m = 6, n = 100. Bắt đầu bằng 1 1
78. Vì 11 + 78
,78, 89.= Chú
89 ::; 100,89phải
ý rằng + 11có= một
100 số
::;hạng
100,trong dãy100
vì vậy bằng 89.phải
cũng Bây nằm
giờ trong
ta có chuỗi
1 1,

của chúng ta. Bây giờ chúng ta có 1 1 78, 89, 100 và nếu chúng ta bao gồm thêm bất kỳ
,sẽ buộc
thuật ngữ nhỏ nào, chúng ta cần phải cẩn thận. Ví dụ,
dãy phải chứanếu
11 chúng
+ 5 = ta
1 6bao
và gồm
78 +5,5 điều
= 83 đó

89 + 5 = 94, điều này là không thể, vì chúng ta chỉ có sáu số hạng. Mặt khác, chúng
tôi có thể nối hai điều khoản lớn mà không gặp khó khăn. Một dãy có thể có là 1 1, 78,
89, 100,99,90. Và một lần nữa, chúng ta có 1 1 + 78 + 89 + 100 + 99 + 90 101

-.
6 > - 2
------: ------

Một lần nữa, chúng ta cần tìm hiểu lý do tại sao điều này lại xảy ra. Nhân với 6, ta được

101 11 + 78 + 89 + 100+ 99 + 90 2 6 · = 3. 101 ,


2

điều này thực sự gợi ý rằng chúng ta cố gắng ghép nối sáu số hạng để tạo thành ba tổng, mỗi tổng lớn hơn

101. Và thực sự, điều đó rất dễ thực hiện:

11 + 78 + 89 + 100 + 99 + 90 = (1 1 + 100) + (78 + 89) + (99 + 90).

Nói chung chúng ta có thể làm được điều này không? Chúng tôi có thể hy vọng như vậy. Lưu ý rằng bằng cách

hy vọng điều này, chúng tôi thực sự đang cố gắng chứng minh điều gì đó mạnh hơn một chút. Rốt cuộc, có

thể là các thuật ngữ không phải lúc nào cũng kết hợp đẹp, như chúng đã làm trong trường hợp này, nhưng

tuy nhiên, tổng của tất cả các thuật ngữ luôn đủ lớn. Tuy nhiên, không có hại gì khi cố gắng chứng minh

một tuyên bố mạnh mẽ hơn. Đôi khi những tuyên bố mạnh mẽ hơn lại dễ chứng minh hơn.

Hãy thử tìm ra một đối số xử lý tất cả các dãy có m = 6 và n = 100. Đơn thức đầu tiên ! Đây là một

sự đơn giản hóa quan trọng. Giả sử không mất đi tính tổng quát rằng trình tự của chúng ta tốt và

al < a2 < a3 < a4 < as < a6 ·

Chúng ta có những bất đẳng thức nghiêm ngặt ở trên bởi vì mỗi số hạng trong một dãy tốt là khác biệt, đó

là một trong những đặc điểm của dãy tốt.

Chúng ta muốn xem liệu chúng ta có thể ghép nối các số hạng sao cho tổng của mỗi cặp lớn hơn hoặc

bằng n + 1. Hãy tạm dừng một chút và suy nghĩ về chiến lược. Giả thuyết là dãy tốt (và đơn điệu), và kết

luận mà chúng ta mong muốn là một bộ ba bất phương trình. Thật khó để chứng minh trực tiếp ba bất đẳng

thức khác nhau (và nếu n lớn hơn thì sẽ có nhiều hơn). Một cách tiếp cận hứa hẹn hơn là mâu thuẫn. Vì

vậy, chúng ta chỉ cần giả sử rằng một trong các bất đẳng thức không thành công, và nếu điều đó có mâu

thuẫn, chúng ta đã hoàn thành. Và không chỉ vậy, nếu chúng ta giả sử rằng hai thứ có tổng nhỏ hơn n + 1,

điều đó có nghĩa là tổng là ::; n, liên quan đến định nghĩa của một chuỗi tốt - thực sự rất hứa hẹn! Vì
vậy, dựa vào tính đối xứng của dãy đơn nguyên, chúng ta sẽ giả sử rằng ít nhất một trong các tổng

al + a6 , a2 + dưới dạng, a3 + a4
Machine Translated by Google

82 CHƯƠNG 3 CHIẾN THUẬT GIẢI QUYẾT VẤN ĐỀ

nhỏ hơn hoặc bằng n. Hãy chọn al + a6. Nếu tổng này nhỏ hơn hoặc bằng n, độ tốt ngụ ý rằng một trong

các ak bằng al + a6. Nhưng điều này là không thể, bởi vì ai là tích cực và đơn điệu. Một sự mâu

thuẫn!

Bây giờ hãy thử giả sử rằng a2 + as ::; N. Khi đó tổng này bằng ak với k nào đó từ I đến 6.

Tổng này lớn hơn as, vì vậy chúng ta phải có a2 + as = a6. Cho đến nay, không có mâu thuẫn. Nhưng

chúng ta đã không cạn kiệt các giả thuyết về lòng tốt. Tổng al + as hoàn toàn nhỏ hơn a2 + as (theo

tính đơn điệu); do đó al + as = aj cho một số j giữa I và 6. Nhưng aj > as, buộc aj = a6. Nhưng điều
này không thể xảy ra, vì al + as hoàn toàn nhỏ hơn a2 + as = a6, vì tất cả các số hạng đều khác
biệt. Một mâu thuẫn khác.

Tương tự như vậy, nếu chúng ta giả sử rằng a3 + a4 ::; n, độ tốt sẽ ngụ ý rằng a3 + a4 bằng

hoặc bằng a6. Nhưng các tổng al + a4 và a2 + a4 cũng là ::; n, và lớn hơn a4, và khác biệt. Nhưng
đây là một mâu thuẫn: chúng ta có ba tổng phân biệt (a3 + a4, a2 + a4, và al + a4) với chỉ hai giá

trị có thể có (as và a6).


Cuối cùng, chúng ta có thể đưa ra một lập luận chung. Trước khi bạn đọc nó, hãy cố gắng viết nó

lên của riêng bạn. Lập luận sau giả sử rằng n chẵn. Bạn sẽ cần phải thay đổi nó một chút đối với

trường hợp n là số lẻ.


Cho ai, a2, ... ,tôi tốt, và không mất đi tính tổng quát, hãy giả sử rằng

Chúng ta sẽ chứng minh rằng al + a2 + ... + am 2 m (n + 1) 12 bằng cách chỉ ra kết quả mạnh hơn rằng

mỗi cặp al + an, a2 + an-l, ... an / 2, an / 2+ 1 lớn hơn hoặc bằng n + 1.

Giả sử là không. Sau đó, cho một số j ::; n12, chúng ta phải có aj + an-j ::; N. Lòng tốt ngụ ý rằng

đối với một số k, 1 ::; k ::; m, ta có aj + an-j = ak. Trong thực tế, k > n - j, vì các số hạng là
số dương và dãy số là đơn thức tăng. Tương tự như vậy, mỗi j
số tiền

al + an-j, a2 + an-j, ···, aj, an-j

nhỏ hơn hoặc bằng n và khác biệt, và mỗi giá trị lớn hơn an-j. Độ tốt ngụ ý rằng mỗi tổng này bằng
với at đối với một số I > n - j. Chỉ có j - 1 lựa chọn cho I, nhưng có j tổng khác nhau. Chúng tôi
đã đạt được mâu thuẫn của chúng tôi.
_

Các vấn đề và bài tập

3.2.7 Hãy tưởng tượng một bàn cờ vô hạn có chứa một số các đa giác trong mặt phẳng sao cho nội thất của các đa
nguyên dương trong mỗi ô vuông. Nếu giá trị trong mỗi ô giác không trùng nhau, mỗi đỉnh của một đa giác trùng với
vuông bằng giá trị trung bình của bốn nước láng giềng của một đỉnh của đa giác khác và không có điểm nào của mặt
nó ở phía bắc, nam, tây và đông, chứng tỏ các giá trị phẳng bị che khuất. Một đa giác đơn vị là một đa giác có
trong tất cả các ô vuông bằng nhau. tất cả các cạnh có độ dài là một.
Il khá dễ dàng để xếp mặt phẳng với vô số ô vuông
3.2.8 Có 2000 điểm trên một hình tròn, và mỗi điểm được
đơn vị. Tương tự như vậy, có thể dễ dàng xếp mặt phẳng
cho một số bằng trung bình cộng của hai số lân cận gần
với vô số tam giác đều đơn vị.
nhất của nó. Chứng tỏ rằng tất cả các số phải bằng nhau.

(a) Chứng minh rằng có một mặt phẳng lát gạch có vô


3.2.9 (Olympic Toán học Vùng Vịnh 2004) Một mặt phẳng lát
số hình vuông đơn vị và vô số hình tam giác đều
các đa giác bao gồm việc đặt
đơn vị trong cùng một mặt phẳng lát gạch.
Machine Translated by Google

3.2 NGUYÊN TẮC CỰC KỲ IPLE 83

(b) Chứng minh rằng không thể tìm thấy một mặt phẳng nào có vô bạn đã sẵn sàng để chứng minh phỏng đoán này. Sử dụng một lập luận

số hình vuông đơn vị và vô số (và ít nhất một) tam giác cực đoan. Hãy xem xét con đường "hợp pháp" dài nhất và cố gắng

đều đơn vị trong cùng một hình lát. tranh luận (bằng cách mâu thuẫn) rằng con đường này bao gồm tất cả

các đường trong hình của bạn.

3.2.10 Giả sử bạn được cho một tập hợp hữu hạn các đồng xu trong 3.2.17 Nguyên lý cực trị trong lý thuyết số. Trong ví dụ

mặt phẳng, tất cả đều có đường kính khác nhau. Chứng tỏ rằng một 3.2.4 ở trang 77, chúng ta đã gặp các khái niệm về ước số

trong các đồng xu tiếp xúc với nhiều nhất năm đồng xu khác. chung lớn nhất và bội số chung nhỏ nhất.

Ở đây chúng tôi đề cập đến một ý tưởng lý thuyết số đơn giản khác,
3.2.11 Sửa lỗi chứng minh trong Ví dụ 2.3.5 trên trang 45.
thuật toán chia:
Chứng tỏ rằng ngay cả một đa giác lõm cũng có một cạnh nhọn mà

chúng ta có thể sử dụng để "cắt" một hình tam giác. Tại sao chúng Cho a và h he là các số nguyên dương. h 2 a.

ta cần nguyên tắc cực trị cho điều này? Khi đó tồn tại các số nguyên q, r thỏa mãn q

2 I và 0 S r < a sueh mà
3.2.12 (Canada 1987) Trên một cánh đồng rộng lớn, bằng phẳng, n

người (n > I) được định vị sao cho đối với mỗi người thì thái độ h = qa + r.
của mỗi người là khác nhau. Mỗi người cầm một khẩu súng lục nước
Nói cách khác, nếu bạn chia h cho a, bạn nhận được thương số nguyên
và theo một tín hiệu nhất định sẽ bắn và bắn trúng người ở gần
dương q với phần dư r ít nhất bằng 0 (bằng 0 nếu alh) nhưng nhỏ hơn
nhất. Khi n lẻ, chứng tỏ rằng có ít nhất một người còn lại khô.
a. Thuật toán divi sion là một ý tưởng "hiển nhiên", một ý tưởng
Điều này có luôn đúng khi n chẵn không?
mà bạn đã thấy từ khi còn đi học. Nó thực sự là một hệ quả của

nguyên tắc sắp xếp tốt.


3.2.13 Giải pháp của chúng tôi cho vấn đề GCD-LCM (Ví dụ 3.2.4 ,

trang 77) đã bỏ qua một bằng chứng rằng cuối cùng dòng sẽ tạo
(a) Để khởi động, hãy chứng minh thuật toán chia một cách chặt
thành một chuỗi trong đó mỗi phần tử chia phần tiếp theo (khi được
chẽ, bằng cách xem xét giá trị không âm nhỏ nhất của h -
sắp xếp theo thứ tự tăng dần).
at khi t nằm trong khoảng các số nguyên dương.
Sửa đổi đối số để thực hiện việc này (nếu bạn hiểu đối số ban đầu,

bạn sẽ có thể thực hiện việc này chỉ với một chút công việc).

(b) Một khởi động khác: chứng minh (điều này sẽ mất hai giây!)

rằng nếu alh và a le, thì al (hx + cy) với mọi số nguyên
3.2.14 (Russia 1996) palindrome là một số hoặc từ giống nhau khi
đọc về phía trước và phía sau x, y (dương hoặc âm). (c) Bây giờ chỉ ra rằng nếu mục tiêu

phường, ví dụ, "176671" và "dân sự". Số nhận được bằng cách viết và anh ta, thì LCM (a, h) lm.

các số từ I đến n theo thứ tự (với một số n > I) có thể là một

palindrome không?
(d) Cuối cùng, chỉ ra rằng với mọi số nguyên a. h, ước số chung

3.2.15 Đặt các số nguyên 1,2,3, ... , cation)


n2 (không
theo thứ
trùng
tự lặp lớn nhất của a và h bằng giá trị nhỏ nhất của ax + hy, vì

bất kỳ lên bàn cờ n x n, với một số nguyên trên mỗi ô vuông . x và y nằm trong phạm vi tất cả các số nguyên (dương và

Chứng tỏ rằng tồn tại hai mục nhập liền kề quảng cáo có hiệu nega). Ví dụ, nếu a = 7, h = II chúng ta có GCD (7, II) =

số ít nhất là n + I. (Liền kề quảng cáo có nghĩa là liền kề I (vì 7 và II là các số nguyên tố, chúng không ,chia
chung
sẻ nào
ước

theo chiều ngang hoặc chiều dọc hoặc theo đường chéo.) ngoại trừ I), nhưng cũng có 1 = (-3) · 7 + 2 · 11.

3.2.16 Sau khi thử nghiệm với Vấn đề 2. 1 .26 ở trang 24, bạn có

thể đã đi đến kết luận rằng các cấu hình khả thi duy nhất là các 3.2.18 Cho P (x) = anx " + an_lx" - 1 +. .. + a () là đa thức với

cấu hình trong đó mức độ của mỗi điểm giao nhau (tức là số dòng hệ số nguyên và gọi q là số nguyên tố. Nếu q là nhân tử của mỗi -

xuất phát từ điểm giao nhau) là chẵn hoặc những nơi mà chính xác I. all-2 ..... al • a (), nhưng q không phải là nhân tử của tất cả,

hai điểm giao nhau có mức độ lẻ và trong trường hợp thứ hai này, và q
2 không phải là một thừa số

con đường phải bắt đầu tại một trong các điểm có mức độ lẻ và kết của a (), thì P (x) là bất khả quy đối với các số hữu tỉ; I E , P (�

thúc ở điểm khác. Bây giờ \) không thể biến thành hai đa thức không hằng số với hệ số hữu tỉ.
Machine Translated by Google

84 CHƯƠNG 3 CHIẾN THUẬT GIẢI QUYẾT VẤN ĐỀ

3.3 Nguyên tắc chuồng bồ câu

Chuồng bồ câu cơ bản

Nguyên tắc của chuồng chim bồ câu, 3 trong hiện thân đơn giản nhất của nó, phát biểu như sau:

Nếu bạn có nhiều chim bồ câu hơn lỗ chim bồ câu, và bạn cố gắng nhét
chim bồ câu vào các lỗ, thì ít nhất một lỗ phải chứa ít nhất hai
chim bồ câu.

Thật ngạc nhiên, ý tưởng tầm thường này lại được hầu hết các nhà toán học tôn sùng, vì nó ít nhất

cũng mạnh ngang với thủ thuật ghép đôi Gauss. Ví dụ, nguyên lý chuồng chim bồ câu đóng một vai trò quan

trọng trong lời giải của ít nhất một phần ba trong số 1 994 bài toán của kỳ thi Putnam. Một vài ví dụ

sẽ thuyết phục bạn về sức mạnh của nó.

Ví dụ 3.3. 1 Mọi điểm trên máy bay được tô màu đỏ hoặc xanh lam. Chứng minh rằng bất kể tô màu
được thực hiện như thế nào, phải tồn tại hai điểm, cách nhau đúng một dặm, có cùng màu.

Giải pháp: Chỉ cần lộn xộn sẽ nhanh chóng đưa ra giải pháp. Chọn một điểm, bất kỳ điểm nào.
Không mất tính tổng quát, nó có màu đỏ. Vẽ một vòng tròn bán kính một dặm với điểm này là tâm. Nếu bất

kỳ điểm nào trên chu vi của hình tròn này có màu đỏ, chúng ta đã hoàn thành. Nếu tất cả các điểm có màu

xanh lam, chúng ta vẫn hoàn thành, vì chúng ta có thể tìm thấy hai điểm trên chu vi cách nhau một dặm

(tại sao?).

Điều đó không khó, nhưng đó không phải là giải pháp cho chuồng bồ câu. Hãy xem xét điều này: chỉ

cần tưởng tượng các đỉnh của một tam giác đều với độ dài cạnh là một dặm. Có ba đỉnh, nhưng chỉ có hai

màu. Nguyên tắc chuồng chim bồ câu cho chúng ta biết rằng hai đỉnh phải cùng màu!
_

Đây là một ví dụ đơn giản khác.

Ví dụ 3.3.2 Cho một hình vuông đơn vị, chứng tỏ rằng nếu năm điểm được đặt ở bất kỳ đâu trong cạnh hoặc

trên hình vuông này, thì hai trong số chúng phải cách nhau nhiều nhất V2 / 2 đơn vị.

Giải: Chia ô vuông đơn vị thành bốn ô vuông 1 x 1 . Bởi chuồng chim bồ câu, một trong những hình
vuông nhỏ hơn này phải chứa ít nhất hai điểm. Vì đường chéo của mỗi hình vuông nhỏ là V2 / 2, đó là

khoảng cách lớn nhất giữa hai điểm.


_

Các giải pháp nhanh chóng và đẹp mắt là đặc trưng của các vấn đề về chuồng bồ câu. Ví dụ trên khá

đơn giản. Giải quyết hầu hết các vấn đề về chuồng bồ câu thường là một quá trình gồm ba phần:

1. Nhận ra rằng vấn đề có thể yêu cầu nguyên tắc chuồng chim bồ câu.
2. Tìm ra những con chim bồ câu sẽ như thế nào và những lỗ phải có. Đây là
động thái mấu chốt tự do.

'Nguyên tắc chuồng chim bồ câu đôi khi còn được gọi là Nguyên tắc Dirichlet để vinh danh vào thế kỷ thứ 9
nhà toán học Peter Dirichlet.
Machine Translated by Google

3.3 IPLE 85 của PIGEONHOLE PRINC

3. Sau khi áp dụng nguyên tắc chuồng bồ câu, thường có nhiều việc phải làm hơn.
Đôi khi nguyên tắc chuồng chim bồ câu chỉ mang lại "bước áp chót" và đôi khi chỉ là một kết
quả trung gian. Người giải quyết vấn đề có kỹ năng lập kế hoạch cho việc này khi nghĩ ra

chiến lược.

Đây là một bài toán đơn giản minh họa sự phối hợp của bước áp chót tốt với nguyên tắc chuồng

chim bồ câu. Bạn sẽ sử dụng vấn đề này như một khối xây dựng trong nhiều vấn đề khác sau này.

Ví dụ 3.3.3 Chứng tỏ rằng trong số n + 1 số nguyên dương bất kỳ, phải có hai số mà hiệu của chúng
là bội số của n.

Giải pháp: Tất nhiên, bước áp chót là nhận ra rằng hai ký tự số mong muốn phải có cùng phần dư

khi chia cho n. Vì chỉ có n phần dư có thể, chúng tôi đã hoàn thành.
_

Vấn đề tiếp theo này phức tạp hơn một chút. Cũng cần lưu ý đến yếu tố chiến lược của sự tự tin

có trong giải pháp. Đây không phải là một vấn đề quá khó đối với những người dũng cảm và ngoan cường.

Ví dụ 3.3.4 (IMO 1 972) Chứng minh rằng từ tập mười số có hai chữ số phân biệt (trong hệ thập phân),

có thể chọn hai tập con rời nhau mà các thành viên của chúng có tổng bằng nhau.

Giải pháp: Chúng ta muốn có hai tập con có tổng bằng nhau, do đó hợp lý để đặt các
tập con là chim bồ câu, và tổng là lỗ. Làm thế nào để chúng tôi làm điều này một cách
chính xác? Đầu tiên, chúng ta hãy nhìn vào các khoản tiền. Tổng nhỏ nhất có thể là 10
và tổng lớn nhất có thể là 99 + 98 + 97 + ... + 90. Sử dụng thủ thuật ghép đôi Gaussian
(để thực hành), đây là 1 89 · 5 = 945. Do đó, có 945 - 10 + 1 = 936 tổng khác nhau.
Bây giờ chúng ta cần đếm số lượng chim bồ câu. Số tập hợp con (xem Phần 6. 1) chỉ là 2 IO = 1
024. Vì 1024 > 936, có nhiều chim
tập con bồ có
phải câutổng
hơn bằng
lỗ, vì vậy chúng ta đã hoàn thành: hai trong số các
nhau.

Nhưng chúng ta đã xong chưa? Vẫn chưa hoàn toàn. Bài toán chỉ ra rằng hai tập hợp con là rời

rạc! Không hoảng loạn. Tất nhiên sẽ là một dạng giải quyết vấn đề tồi nếu từ bỏ giải pháp của chúng

tôi tại thời điểm này, vì chúng tôi đã đạt được giải pháp một phần: chúng tôi đã chỉ ra rằng có hai
tập con khác nhau (có lẽ không rời rạc) có cùng tổng. Chúng ta có thể sử dụng điều này để tìm hai

tập con rời rạc có cùng tổng không? Tất nhiên rồi. Gọi A và B là hai tập hợp. Chia thành các trường
hợp:

• A và B rời rạc. Xong ! • A và B

không rời rạc. Từ mỗi tập hợp, loại bỏ các phần tử mà chúng có điểm chung. Bây giờ chúng ta có

các bộ rời rạc, nhưng tổng vẫn bằng nhau (tại sao?), Vậy là chúng ta đã hoàn thành!
_

Bạn có thể tự hỏi, "Điều gì sẽ xảy ra nếu, sau khi loại bỏ các yếu tố chung, chúng tôi không có

ing left from one of the set? "Điều đó là không thể. Tại sao?
Machine Translated by Google
86 CHƯƠNG 3 CHIẾN THUẬT GIẢI QUYẾT VẤN ĐỀ

Chuồng bồ câu trung gian Đây là

phiên bản phức tạp hơn của nguyên tắc chuồng bồ câu, một phiên bản được sử dụng trong thực tế
thường xuyên hơn nguyên tắc chuồng bồ câu cơ bản được mô tả ở trên. [Kí hiệu r xl ( trần của
x) có nghĩa là số nguyên nhỏ nhất lớn hơn hoặc bằng x. Ví dụ: r 1r 1 = 4. Xem trang 146 để
biết thêm thông tin.]

Nếu bạn có p chim bồ câu và h lỗ. sau đó ít nhất một trong những lỗ
chứa ít nhất rp / h 1 chim bồ câu.

Chú ý rằng nguyên tắc cơ bản của chuồng chim bồ câu là một hệ quả của phát biểu này: chúng ta có
p > h, do đó đại lượng r p / h 1 ít nhất là 2.
Hãy chắc chắn rằng bạn hiểu tuyên bố của "chuồng chim bồ câu trung gian" này bằng cách
xem qua một số ví dụ. Hãy chắc chắn rằng bạn hiểu lý do tại sao trạng thái đề cập là đúng.

Nếu bạn may mắn, một ứng dụng đơn giản, đẹp đẽ của một kỹ thuật sẽ giải quyết được vấn
đề của bạn. Nhưng nhìn chung, chúng tôi không may mắn như vậy. Không có gì ngạc nhiên khi một
số vấn đề đòi hỏi nhiều ứng dụng của nguyên tắc chuồng chim bồ câu. Mỗi khi sử dụng nguyên tắc
chuồng bồ câu, thông tin sẽ thu được. Đây là một ví dụ sử dụng ứng dụng lặp đi lặp lại của
nguyên tắc chuồng chim bồ câu trung gian.

Ví dụ 3.3.5 (A. Soifer, S. Slobodnik) Bốn mươi mốt quân được đặt trên bàn cờ 10 x 10 . Chứng
minh rằng phải tồn tại năm con ngựa, không con nào tấn công nhau.
(Nhớ lại rằng các quân tấn công từng quân nằm trên hàng hoặc cột của nó.)

Giải pháp: Khi bạn nhìn thấy số 41 liền kề với 10, bạn nghi ngờ rằng nguyên tắc chuồng
chim bồ câu có thể có liên quan, vì số 41 chỉ là một hơn 4 · 10. Khi đã hài lòng với nguyên
tắc chuồng bồ câu, chúng ta không thể không nhận thấy rằng r 41/101 = 5, đáng khích lệ, vì đây
là số lượng xe mà chúng tôi tìm kiếm. Tất nhiên đây không phải là một giải pháp, nhưng nó gợi
ý rằng chúng ta nên thăm dò cẩn thận để tìm một cái bằng cách sử dụng nguyên tắc chuồng bồ câu.
Hãy để chúng tôi làm như vậy. Chúng tôi tìm kiếm năm con ngựa không tấn công nhau. Hai

quân không tấn công nếu chúng nằm trên các hàng khác nhau và các cột khác nhau. Vì vậy, chúng
ta cần tìm năm con ngựa, mỗi con sống ở một hàng khác nhau và mỗi con sống ở một cột khác
nhau. Một chiến lược mơ hồ: chúng ta cần năm hàng khác nhau, mỗi hàng có "rất nhiều" ô. Sau
đó, chúng ta có thể chọn một con từ một hàng, tìm một con ngựa khác ở một cột khác ở hàng kia,
v.v. Có 10 hàng và 41 con xe, vì vậy nguyên tắc chuồng chim bồ câu cho chúng ta biết rằng một
hàng phải chứa ít nhất r 41/101 = 5 rooks. Đây là một sự khởi đầu. Chúng ta có thể áp dụng lại
nguyên tắc chuồng chim bồ câu để có thêm thông tin không? Đúng! Điều gì đang xảy ra với các hàng khác?
Chúng tôi muốn tìm các hàng khác có nhiều ô. Chúng tôi đã cô lập một hàng với ít nhất năm

quân. Bây giờ loại bỏ nó! Tối đa, chúng tôi sẽ loại bỏ 10 ô trống. Điều đó để lại 31 ô trên

chín hàng còn lại. Nguyên tắc chuồng chim bồ câu cho chúng ta biết rằng một trong chín hàng
này phải chứa ít nhất r31 / 91 = 4 rooks.
Bây giờ chúng tôi đang trên một cuộn. Loại bỏ hàng này và "pigeonholing" một lần nữa,
chúng ta suy ra rằng có một hàng khác chứa ít nhất r2 1/81 = 3 rooks. Tiếp tục (xác minh!),
Chúng tôi thấy rằng một hàng khác phải có ít nhất hai ô và một hàng nữa phải chứa ít nhất một
ô.

Do đó, có năm hàng đặc biệt trên bàn cờ, chứa ít nhất 5, 4, 3, 2 và 1 quân tương ứng. Bây
giờ chúng ta có thể xây dựng "nhóm ngũ quan hòa bình": Bắt đầu bằng
Machine Translated by Google

3.3 IPLE PIG EONHOLE PRINC 87

chọn xe trong hàng có ít nhất một xe. Sau đó đi đến hàng có ít nhất hai ô. Ít nhất một trong các ô này
sẽ không ở cùng cột với ô đầu tiên của chúng ta, vì vậy hãy chọn nó làm ô thứ hai. Tiếp theo, nhìn vào

hàng có ít nhất ba xe. Một trong ba quân này không ở cùng cột với quân thứ nhất hoặc thứ hai.

Chọn đây làm ô thứ ba của chúng tôi, v.v., và chúng tôi đã hoàn tất!

Bài toán khá phức tạp này là một minh họa tốt cho cả ciple của ông hoàng chuồng bồ câu và chiến

lược mơ tưởng, tức là không bỏ cuộc. Khi bạn nghĩ rằng một vấn đề có thể bị tấn công với nguyên tắc chuồng

chim bồ câu, trước tiên hãy cố gắng thực hiện công việc một cách gọn gàng.

Hãy tìm cách xác định các lỗ và lỗ của chim bồ câu mang lại một giải pháp nhanh chóng. Nếu điều đó không

hiệu quả, đừng bỏ cuộc! Sử dụng nguyên tắc chuồng chim bồ câu một lần để đạt được chỗ đứng, sau đó cố

gắng sử dụng lại. Tiếp tục trích xuất thông tin!

Chuồng bồ câu nâng cao

Các ví dụ ở đây là những vấn đề khó hơn. Một số khó vì chúng đòi hỏi những ý tưởng toán học chuyên biệt

khác cùng với nguyên lý chuồng chim bồ câu. Các vấn đề khác chỉ yêu cầu chuồng bồ câu cơ bản, nhưng chuồng

bồ câu và / hoặc lỗ thì không rõ ràng.

Bài toán lý thuyết số dưới đây chỉ sử dụng chuồng chim bồ câu cơ bản, nhưng với những chú chim bồ

câu được chọn rất khéo léo.

Ví dụ 3.3.6 (Olympic Toán Colorado 1 986) Cho n là một số nguyên dương. Chứng tỏ rằng nếu bạn có n số

nguyên, thì một trong số chúng là bội của n hoặc tổng của một số chúng là bội của n.

Giải pháp: Chúng ta cần chỉ ra rằng một cái gì đó là bội số của n. Làm thế nào chúng ta có thể làm

điều này với nguyên tắc chuồng bồ câu? Theo ví dụ 3.3.3, bạn đã biết câu trả lời: hãy để các chuồng chim

bồ câu là n phần dư khác nhau có thể có khi chia cho n. Sau đó, nếu hai số nằm trong cùng một chuồng chim

bồ câu, hiệu của chúng sẽ là bội số của n. (Tại sao?)

Nhưng trong ví dụ 3.3.3, chúng ta có n + 1 con chim bồ câu để đặt vào n lỗ. Trong bài toán này,

chúng ta chỉ có n số. Làm thế nào để chúng ta tạo ra n + 1 hoặc nhiều chim bồ câu? Ngoài ra, làm thế nào

chúng ta có thể chọn chim bồ câu để thứ có kết thúc là bội số của n là một trong các số ban đầu hoặc tổng

của một số chúng?

Nếu chúng ta có thể trả lời cả hai câu hỏi này, chúng ta sẽ hoàn thành. Câu hỏi đầu tiên là khá bí

ẩn, vì chúng ta có quá ít để làm việc, nhưng câu hỏi thứ hai có một câu trả lời dễ hiểu: hãy để những con

chim bồ câu được tự tổng hợp; Nếu chúng ta chọn chúng một cách cẩn thận, sự khác biệt của những con chim

bồ câu vẫn sẽ là tổng. Làm thế nào điều đó có thể được thực hiện?
Gọi các số trong tập hợp của chúng ta là aI, a2, ... ,một. Xem xét trình tự

Pn tại + a2 +. . . + an ·

Chúng tôi đang sử dụng chữ P cho chim bồ câu; tức là, Pk biểu thị con chim bồ câu thứ k. Lưu ý rằng đối

với bất kỳ hai chỉ số khác biệt i < j, hiệu Pj - Pi sẽ bằng tổng ai + l + ai + 2 +
Machine Translated by Google
88 CHƯƠNG 3 CHIẾN THUẬT GIẢI QUYẾT VẤN ĐỀ

... + a j. Vì vậy, những con chim bồ câu của chúng tôi có hành vi đúng, nhưng không may, chỉ có n
trong số chúng. Nhưng đó không phải là xấu như nó có vẻ. Đôi khi (như trong Ví dụ 3.3.4) bạn có thể

giảm số lượng chuồng chim bồ câu cần thiết bằng cách chia thành các trường hợp.

Chúng ta có n con chim bồ câu ..., Pn. Có hai trường hợp.

PI, P2, • Một con chim bồ câu có số dư là 0 khi chia cho n, trong trường hợp đó chúng ta đã

hoàn thành! (Tại sao?) • Không có con chim bồ câu nào có phần dư là 0, vì vậy bây giờ chúng

ta chỉ có n - I lỗ chim bồ câu để xem xét. Với n con chim bồ câu, hai con trong số chúng phải
có số dư bằng nhau, vì vậy hiệu của chúng, tổng của một số số ban đầu, sẽ là một bội của n,

và chúng ta đã hoàn thành.


_

Ví dụ tiếp theo là một vấn đề nổi tiếng, do Paul Erd6s vĩ đại, 4 được tái

có thể đánh dấu được vì khó khăn là ở chuồng bồ câu, không phải ở bồ câu.

Ví dụ 3.3.7 Cho n là số nguyên dương. Chọn tập con (n + I) bất kỳ của {I, 2, ..., 2n}. Chứng tỏ

rằng tập hợp con này phải chứa hai số nguyên, một trong số đó chia
cai khac.

Điều tra: Ngôn ngữ của bài toán dẫn chúng ta đến việc thử tính toán của chuồng chim bồ câu,

sửa số n + 1 trong tập hợp con đã chọn là chim bồ câu. Chúng ta cần tạo nhiều nhất n chuồng chim bồ

câu, vì có n + 1 con chim bồ câu. Và chúng tôi muốn các chuồng chim bồ câu của chúng tôi được chọn
sao cho nếu hai số sống trong cùng một lỗ, thì một trong số chúng phải chia cho số kia. Khi đó, mỗi

chuồng chim bồ câu là một tập hợp các số nguyên với đặc tính là nếu a và b là hai phần tử bất kỳ

của tập hợp thì a là bội của b hoặc b là bội của a.

Chúng ta hãy cố gắng xây dựng một tập hợp như vậy. Nếu tập hợp chứa 7, thì tất cả các số khác

phải là thừa số của 7 hoặc mUltip là 7. Giả sử số tiếp theo trong tập hợp là 21.
Khi đó các số khác bây giờ phải là bội số của 21 hoặc thừa số của 7, v.v. Vì vậy, nếu 7 là số nhỏ

nhất trong tập hợp, tập hợp sẽ là danh sách các số có dạng sau: 7, 7a, 7ab, 7abc, 7abcd , . .. ,
trong đó a, b, c, d, v.v. là các số nguyên dương.

Khi đó, nhiệm vụ của chúng ta là phân vùng tập {I, 2, ..., 2n} thành nhiều nhất n tập con rời
rạc với thuộc tính trên. Điều này không dễ dàng; điều tốt nhất nên làm lúc này là thử nghiệm với các

giá trị nhỏ của n. Ví dụ, cho n = 5. Hãy thử phân vùng {1,2, 3, 4,5,6, 7, 8, 9, 1O} thành năm tập

con rời nhau với thuộc tính đặc biệt. Mỗi tập hợp có một phần tử nhỏ nhất, vì vậy chúng ta cần chọn
năm "hạt giống" như vậy. Khi phấn đấu cho một phương pháp chung --- một phương pháp có thể được sử
dụng cho các giá trị khác của n-bộ sưu tập "tự nhiên" duy nhất của năm hạt giống là 1,3,5,7, 9. (Danh
sách 2, 4,6, 8 , 10 không bao
danhgồm
sách
1 và
này1không
phải là
phải
phần
là tử
ứngtối
cử thiểu
viên "tự
củanhiên"
một trong
cho các
hạt tập
giống
hợp,
củavì
chúng
vậy
tôi.)

Chú ý rằng mỗi hạt là lẻ. Để có các số còn lại, chúng ta chỉ cần nhân các hạt với 2. Nhưng điều

đó không hiệu quả lắm, vì chúng ta sẽ không nhận được tất cả các số.
Tuy nhiên, nếu chúng ta tiếp tục nhân lên, chúng ta nhận được

phân vùng {1,2, 4, 8}; {3, 6}; {5, 1O}; {7}; {9}.

Erdos, qua đời năm 1996 ở tuổi 83, là nhà toán học xuất sắc nhất thời hiện đại, đã là tác giả
hoặc đồng tác giả của hơn 1.000 tài liệu nghiên cứu.
Machine Translated by Google

3.3 NGUYÊN TẮC HEO HEO 89

Tập hợp các chuồng chim bồ câu này thực hiện thủ thuật. Nếu chúng ta chọn bất kỳ sáu số nào từ

{1,2, ... , 1 O},

thì hai trong số chúng phải được chứa trong một trong năm bộ trên. Một số tập hợp (trong trường hợp này là

hai tập hợp) chỉ chứa một phần tử, vì vậy hai số "sống chung" trong một tập hợp không thể nằm trong các tập

hợp này. Do đó, hai người sống chung phải sống trong {1,2, 4, 8} hoặc {3, 6} hoặc {5, 1 0}, và sau đó chúng

ta hoàn thành, vì khi đó một trong hai người sống chung là bội số
của những thứ còn lại.

Bây giờ nó là dễ dàng để giải quyết vấn đề nói chung.

Lời giải chính thức: Mỗi phần tử của {I, 2, ..., 2n} có thể được viết duy nhất trong q, trong đó q là
dạng 2 r
số nguyên lẻ và r là số nguyên không âm. Mỗi số lẻ q khác nhau xác định một chuồng chim bồ câu, cụ

thể là tất cả các phần tử của {1,2 ,. . . , 2n} có


dạng 2 r
q với một số nguyên dương r. (Ví dụ: nếu n = 1 00, giá trị q = 11 sẽ xác định chuồng chim bồ

câu {II, 22, 44, 88, 1 76}.) Vì có chính xác n ký tự số lẻ từ 1 đến 2n, chúng tôi đã xác định n các tập hợp,

và các tập hợp này là rời rạc (chúng cần phải rời rạc; nếu không chúng không thể là "lỗ chim bồ câu".) Vì

vậy, chúng ta đã hoàn thành, vì theo nguyên tắc chuồng chim bồ câu, hai trong số n + 1 sẽ nằm trong một trong

n tập hợp của chúng ta, điều này sẽ buộc một trong hai số phải là bội số của số kia.

Vấn đề tiếp theo, từ kỳ thi Putnam 1 994 , liên quan đến một số đại số tuyến tính, khiến nó vốn đã trở

nên khá khó khăn. Nhưng phần thú vị là hai bước đi mấu chốt: xác định một hàm và sử dụng nguyên lý chuồng

chim bồ câu với các căn của một đa thức.

Cả hai ý tưởng đều có nhiều ứng dụng.

Ví dụ 3.3.8 Cho A và B là các ma trận 2 x 2 với các phần tử là số nguyên sao cho A, A + B, A +
2B, A + 3B và A + 4B đều là các ma trận khả nghịch mà phần nghịch đảo của chúng có các phần tử là
số nguyên. Chứng tỏ rằng A + 5B là khả nghịch và nghịch đảo của nó có các mục số nguyên.

Giải: Nếu X là một ma trận khả nghịch với các phần tử là số nguyên và nghịch đảo của nó cũng có các phần

tử số nguyên, thì detX = ± 1. Điều này xuất phát từ thực tế rằng detX và det (X - I ) đều là số nguyên và det

(X - \ ) = 1 / detX. Và ngược lại, nếu định thức của một ma trận có các

phần tử nguyên là ± 1, thì nghịch đảo cũng sẽ có các phần tử số nguyên (tại sao?).

Bây giờ hãy xác định hàm f (t) : = det (A + tB). Vì A và B là các ma trận 2 x 2 với các phần tử nguyên,

f (t) là một đa thức bậc hai theo t với các hệ số nguyên (xác minh!).

Vì A, A + B, A + 2B, A + 3B và A + 4B đều là các ma trận khả nghịch mà các nghịch đảo của chúng có các mục số

nguyên, chúng ta biết rằng năm số

f (O), f ( 1), f (2) , / (3), / (4)

chỉ nhận các giá trị 1 hoặc - 1. Theo nguyên tắc chuồng bồ câu, ít nhất ba trong số các số này phải có cùng

giá trị; Không mất tính tổng quát, giả sử rằng nó là 1. Khi đó đa thức bậc hai f (t) bằng 1 khi ba số khác

nhau được cắm vào cho t. Điều này có nghĩa là f (t) phải là đa thức hằng số; tức là, f (t) == 1. Có det (A +

5B) = f (5) = 1, do đó A + 5B là khả nghịch và nghịch đảo của nó có các mục là số nguyên.

-
Machine Translated by Google
Machine Translated by Google

3.3 NGUYÊN TẮC CON HEO 91

2. Tiếp theo, "nâng" từng vùng lên, với hình vuông mạng của nó. Hãy coi mỗi vùng như
một bức tranh được vẽ trên một hình vuông đơn vị. Xếp chồng các hình vuông này
thành một chồng, như hình minh họa. Bây giờ, hãy tưởng tượng rằng chúng ta dán
một chiếc ghim qua các ô vuông, sao cho chiếc ghim xuyên qua mỗi ô vuông ở cùng
một vị trí. Vì diện tích của S lớn hơn 1 nên phải tìm được điểm sao cho chốt qua
điểm đó đâm vào ít nhất hai vùng con. Trong ví dụ của chúng ta, chốt xuyên qua cả
a và c. thì tổng diện tích của S sẽ nhỏ hơn hoặc bằng diện tích của một hình vuông đơn vị.
Một cách khác để nghĩ về nó: hãy tưởng tượng cắt các tiểu vùng bằng một cái kéo
và cố gắng xếp tất cả chúng lại trong một hình vuông đơn vị, không có bất kỳ
chồng chéo nào. Điều đó là không thể, vì tổng diện tích lớn hơn 1. (Bằng cách lập
luận tương tự, nếu tổng diện tích lớn hơn số nguyên n, thì có thể tìm thấy một
điểm mà chốt xuyên qua ít nhất n + 1 tiểu vùng. )

3. Cuối cùng, chúng tôi theo dõi điểm mà ghim của chúng tôi xuyên qua các vùng con
và xây dựng lại S. Các điểm được biểu thị bằng các chấm đen. Vì hai điểm nằm ở
cùng một vị trí so với các đường mạng, nên bây giờ có thể dịch S (theo hướng của
mũi tên), để cả hai điểm trắng nằm trên các điểm mạng, và chúng ta đã hoàn thành !
_

Các vấn đề và bài tập

3.3.10 Chứng minh nguyên lý chuồng chim bồ câu trung gian, 3.3.14 Chứng tỏ rằng khai triển thập phân của một số hữu
sử dụng lập luận bằng mâu thuẫn. tỉ cuối cùng phải trở thành tuần hoàn.

3.3.11 Chứng tỏ rằng trong bất kỳ cuộc tụ tập hữu hạn 3.3.15 Bảy điểm được đặt bên trong một hình lục giác đều
người nào, có ít nhất hai người biết cùng một số người với độ dài cạnh 1. Chứng tỏ rằng có ít nhất hai điểm cách
tại cuộc tụ tập (giả sử rằng “biết” là mối quan hệ tương nhau nhiều nhất một đơn vị.
hỗ).
3.3.16 Bên trong một hình vuông 1 x 1, 101 điểm được đặt .

3.3.12 Sử dụng lập luận mâu thuẫn để chứng minh biến thể Chứng tỏ rằng một số ba trong số chúng tạo thành một tam giác có

hữu ích fol thấp hơn của nguyên tắc chuồng chim bồ câu: diện tích không quá 0,01.

al, a2, Cho ... , là số nguyên dương. Nếu


3.3.17 Chứng tỏ rằng trong số n + 2 số nguyên bất kỳ, có

(al + a2 + ... + an) -n + 1 hai số có hiệu là bội của 2n hoặc có hai số mà tổng của
chúng chia hết cho 2n.
Chim bồ câu được bỏ vào n ổ chim bồ câu, thì đối với thứ
3.3.18 Chọn bất kỳ tập con (n + 1 )-gia tốc nào từ {I, 2 , . ..,
i, phát biểu "Chuồng bồ câu thứ i chứa ít nhất ai chim bồ
2n}. Chứng tỏ rằng tập hợp con này phải chứa hai số nguyên tương
câu" phải đúng.
đối nguyên tố.
3.3.13 Hãy chú ý cách lập luận chuồng chim bồ câu đơn
3.3.19 Mọi người đang ngồi quanh một chiếc bàn tròn tại
giản như thế nào trong Ví dụ 3.3.2 ở trang 84. Điều gì
một nhà hàng. Thức ăn được đặt trên một bệ tròn ở giữa
sai với "giải pháp" sau đây?
bàn, và bệ tròn này có thể xoay (điều này thường thấy ở
Đặt bốn trong số các điểm trên các đỉnh của hình
các quán ăn Trung Quốc chuyên phục vụ tiệc chiêu đãi). Mỗi
vuông; theo cách đó chúng được tách biệt hoàn
người đặt hàng một món ăn khác nhau, và hóa ra không ai
toàn khỏi nhau. Khi đó, điểm thứ năm phải nằm
có món ăn chính xác trước mặt anh ta. Chứng tỏ rằng có thể
trong v'2 / 2 của một trong bốn điểm còn lại, đối
xoay nền để ít nhất hai người có được quyền thích hợp.
với lông ở góc xa nhất thì nó có thể là tâm, và

đó chính xác là v'2 / 2 đơn vị tính từ mỗi góc.

3.3.20 Xét một dãy gồm N số nguyên dương


Machine Translated by Google

92 CHƯƠNG 3 CÁC THUẬT TOÁN GIẢI QUYẾT VẤN ĐỀ

chứa n số nguyên phân biệt. Nếu N ; ::: 2n, chứng tỏ rằng có n / 6 - I của khoảng cách

một khối số nguyên liên tiếp có tích là một hình vuông hoàn

hảo. Bất đẳng thức này là tốt nhất có thể?

nhỏ hơn PiQ.


3.3.21 (Hàn Quốc 1995) Với bất kỳ số nguyên dương m nào, chứng

tỏ rằng tồn tại các số nguyên a, b thỏa mãn 3.3.27 Ví dụ 3.3.5 ở trang 86 đã sử dụng nguyên tắc lỗ địa

pi nhiều lần để thu được solu tion. Tồn tại một giải pháp

đơn giản tuyệt vờiJy chỉ sử dụng một ứng dụng của nguyên tắc
lal Sm, Ibl Sm,
lỗ chim bồ câu (trung gian). Bạn có thể tìm thấy nó không?

3.3.22 Chứng tỏ rằng với bất kỳ số nguyên dương n nào, có một


3.3.28 Các bài toán về Chèo được lấy cảm hứng từ Ví dụ rất dễ
bội số dương của n chỉ chứa các chữ số 7 và O.
dàng 3.3. 1. Tuy nhiên, không phải aU của các biến thể dưới

đây là dễ dàng. Chúc vui vẻ!


3.3.23 Để đảm bảo rằng bạn hiểu Đề thi 3.3.7, hãy xây dựng
(a) Tô màu máy bay bằng ba màu. Chứng minh rằng có hai điểm
các lỗ chim bồ câu một cách rõ ràng khi n = 25. Xác minh rằng
cùng màu cách nhau một đơn vị.
giải pháp có hiệu quả trong trường hợp này.

(b) Tô màu máy bay bằng hai màu. Chứng minh rằng một trong
3.3.24 Một kỳ thủ cờ vua chuẩn bị cho một giải đấu bằng cách
các màu này chứa các cặp điểm ở mọi khoảng cách tương
chơi một số trò chơi luyện tập trong khoảng thời gian tám
ứng.
tuần. Cô ấy chơi ít nhất một trò chơi mỗi ngày, nhưng không

quá II trò chơi mỗi tuần. Chứng tỏ rằng phải có một khoảng (c) Tô màu máy bay bằng hai màu. Chứng minh rằng luôn tồn

thời gian liên tục trong ngày mà cô ấy chơi đúng 23 trò chơi. tại một tam giác đều với aU các đỉnh của nó cùng màu.

3.3.25 (Putnam 1994) Chứng minh rằng các điểm của tam giác (d) Chứng tỏ rằng có thể tô màu mặt phẳng bằng hai màu sao

vuông có độ dài cạnh I không được tô màu trong bốn ô sao cho cho không thể tồn tại tam giác đều cạnh I với các đỉnh

không có hai điểm nào cách nhau ít nhất 2 - ,; z nhận cùng màu. aU cùng màu.

3.3.26 (Olympic Toán học Vùng Vịnh 2005) Cho n; ::: 12 là một (e) Tô màu máy bay bằng hai màu. Chứng tỏ rằng tồn tại một

số nguyên và đặt PI, P2, ... , Pn, Q là các hình chữ nhật aU có các đỉnh cùng màu.

điểm nằm trong mặt phẳng. Chứng minh rằng đối với một số tôi, ít nhất

3.4 Tôi không thay đổi

Cuộc thảo luận của chúng tôi về nguyên tắc cực đoan đã đề cập đến tầm quan trọng của việc trích
xuất thông tin quan trọng từ sự hỗn loạn mà hầu hết các vấn đề ban đầu xuất hiện. Chiến lược là
một số cách "giảm thiểu" vấn đề để tập trung vào một số thực thể thiết yếu nhất định. Trong
Phần 3.2, chiến thuật để thực hiện chiến lược này là tập trung vào các giá trị cực đoan. Có
những chiến thuật khác với cùng một chiến lược cơ bản. Ở đây chúng tôi sẽ giới thiệu chủ đề rất
phong phú về bất biến.

Một bất biến, như tên cho thấy, chỉ đơn thuần là một số khía cạnh của vấn đề - thường là
một đại lượng số - không thay đổi, ngay cả khi nhiều thuộc tính khác thay đổi.
Dưới đây là một vài ví dụ.

Ví dụ 3.4. 1 Nghịch lý Phòng trọ. Nhớ lại Ví dụ 2. 1 .8 ở trang 20, vụ án liên


quan đến ba người phụ nữ nhận phòng trong một phòng trọ. Gọi g, p, d lần lượt là
số tiền khách đã chi tiêu, số tiền người khuân vác tự bỏ túi và số tiền mà bàn
của nhà nghỉ nhận được. Sau đó, số lượng

g - p - d
Machine Translated by Google

3.4 KHÁCH HÀNG 93

là một bất biến, luôn luôn bằng không.

Ví dụ 3.4.2 Định lý lũy thừa của một điểm. Cho điểm P cố định và một đường tròn
cố định, kẻ đường thẳng qua P cắt đường tròn tại X và Y. lũy thừa của điểm P đối
với đường tròn này được xác định là đại lượng PX. PY.
Định lý Power of a Point (còn được gọi là POP) phát biểu rằng đại lượng này là bất
biến; tức là, nó không phụ thuộc vào đường được vẽ. Ví dụ, trong hình dưới đây,

'
PX · PY = PX · Py '.

Y '

Chắc chắn bạn đã học định lý này trong hình học sơ cấp, ít nhất là đối với trường hợp điểm
P nằm bên trong đường tròn. Để có bằng chứng, hãy xem Ví dụ 8.3.1 1.

Ví dụ 3.4.3 Công thức của Euler. Ban đầu bạn gặp phải vấn đề này là Vấn đề 2.2. 12 ở trang
37, yêu cầu bạn phỏng đoán mối quan hệ giữa số lượng đỉnh, cạnh và mặt của bất kỳ hình đa
diện nào. Hóa ra rằng nếu v, e và f biểu thị số đỉnh, cạnh và mặt của một hình đa diện
không có "lỗ" thì

v - e + f = 2

luôn luôn giữ; tức là, đại lượng v - e + f là một bất biến. Đây được gọi là Công thức của
Euler. Xem Bài toán 3.4.40 để biết một số gợi ý về cách chứng minh công thức này.

Ví dụ 3.4.4 Đối xứng. Mặc dù chúng tôi đã dành phần đầu tiên của chương này để nói về đối
xứng, nhưng về mặt logic, chủ đề này chứa đựng trong khái niệm bất biến. Nếu một đối tượng
cụ thể (hình học hoặc cách khác) chứa đối xứng, đó chỉ là một cách nói khác rằng bản thân
đối tượng là một bất biến đối với một số phép biến đổi hoặc tập hợp các phép biến đổi. Ví
dụ: một hình vuông là bất biến đối với các phép quay về tâm của nó là 0, 90, 1 80 và 270
độ.

Ở mức độ sâu hơn, phép thay thế u: = x + xI , đã giúp giải quyết

x 4 + x3 + .l + x + l = 0

trong Ví dụ 3. 1.1 0 có tác dụng, bởi vì u là bất biến đối với một hoán vị của một số gốc.5

5 Ý tưởng này là mầm mống của một trong những thành tựu vĩ đại nhất của toán học thế kỷ 9, lý thuyết Galois,
cùng với những thứ khác phát triển một cách có hệ thống để xác định đa thức nào có thể giải được bằng căn và không
thể. Để biết thêm thông tin, hãy tham khảo cuốn sách tuyệt vời của Herstein [20].
Machine Translated by Google

94 CHƯƠNG 3 CÁC THUẬT TOÁN GIẢI QUYẾT VẤN ĐỀ

Ví dụ 3.4.5 Phép chia hết cho Chín. Gọi s (n) là tổng các chữ số của biểu diễn cơ số mười của
số nguyên dương n. sau đó

n - s (n) luôn chia hết cho 9.

Ví dụ: nếu n = 1 36, thì

n - s (n) = 1 36 - (1 + 3 + 6) = 1 26 = 9 · 14.

Đây là một ví dụ về bất biến không phải là số, mặc dù chúng ta có thể đúc lại mọi thứ ở dạng số bằng

cách nói, chẳng hạn, phần còn lại khi chia cho 9 của n - s (n) là đại lượng bất biến bằng không.

Đây là một ví dụ khác về bất biến chia hết.

Ví dụ 3.4.6 Lúc đầu, một căn phòng trống. Mỗi phút, một người vào hoặc hai
1999 1000
người rời đi. Sau đúng 3 phút, căn phòng có thể chứa 3 + 2 người?

Bài giải: Nếu có n người trong phòng thì sau một phút sẽ có n + 1 hoặc n - 2 người. Sự
khác biệt giữa hai kết quả có thể có này là 3.
Tiếp tục trong thời gian dài hơn, chúng tôi thấy rằng

Tại bất kỳ thời điểm cố định t nào, tất cả các giá trị có thể có của dân số
trong phòng khác nhau một bội số của 3.

1999 1999
Trong 3
phút, sau đó, một dân số có thể có của phòng chỉ là 3 người
(giả sử rằng một người vào mỗi lần). Đây là bội số của 3, vì vậy tất cả các quần thể tìm vị trí
1000
cho căn phòng cũng phải là bội số của 3. Do đó 3 + 2 ý chí
không phải là một tập hợp hợp lệ.
_

Bất biến chia hết đơn giản nhất là tính chẵn lẻ, mà chúng ta đã sử dụng trong Ví dụ 2.2.3 trên

trang 29 và Ví dụ 2.2.4 ở trang 30. Hãy cùng chúng tôi khám phá khái niệm này chi tiết hơn.

Ngang bằng

Số nguyên được chia thành hai lớp chẵn lẻ: chẵn và lẻ. Các số nguyên chẵn chia hết cho 2, tỷ lệ cược là

không. Lưu ý rằng số 0 là số chẵn. Những sự kiện sau đây rất dễ chứng minh một cách chặt chẽ (hãy làm

điều đó!) Nhưng đáng lẽ bạn phải biết từ khi còn nhỏ.

• Tính chẵn lẻ của tổng tập hợp các số nguyên là lẻ nếu và chỉ khi số lẻ
phần tử là lẻ.

• Tính chẵn lẻ của một tích của một tập hợp các số nguyên là lẻ khi và chỉ khi không có phần tử
chẵn nào trong tập hợp đó.

Bạn có thể nghĩ rằng chẵn lẻ là một thứ khá thô thiển - xét cho cùng, có vô số số nguyên nhưng chỉ

có hai loại chẵn lẻ - nhưng kiến thức về chẵn lẻ đôi khi là tất cả những gì cần thiết, đặc biệt nếu

tính chẵn lẻ có liên quan đến tuyên bố của bài toán. Đây là một ví dụ cơ bản, một vấn đề đã xuất hiện

dưới nhiều hình thức. Phiên bản này đã xuất hiện trong Olympic Toán học Colorado Springs lần thứ 1 986 .
Machine Translated by Google

3.4 BÊN MỜI 95

Ví dụ 3.4.7 Nếu 127 người chơi trong một giải quần vợt đơn, hãy chứng minh rằng khi kết thúc giải
đấu, số người đã chơi một số trận lẻ là
thậm chí.

Giải pháp: Nhiều người tiếp cận vấn đề này không thành công bằng cách cho rằng giải đấu có
một cấu trúc cụ thể, chẳng hạn như loại trực tiếp đôi hoặc vòng tròn một lượt, v.v. Nhưng vấn đề
không chỉ rõ điều này! Hàm ý là bất kể giải đấu được cấu trúc như thế nào thì số người chơi một số
trò chơi là số lẻ phải là số chẵn. Ví dụ, một giải đấu có thể xảy ra sẽ là một giải đấu mà không
có ai chơi bất kỳ trò chơi nào. Khi đó số người đã chơi một số trò chơi lẻ là số không, là số
chẵn, theo yêu cầu.

Dường như có rất ít hạn chế. Có cái nào không? Có, mỗi trò chơi có đúng hai người chơi trong
đó. Nói cách khác, nếu A chơi với B thì trò chơi đó được tính hai lần: một lần thuộc phần A và một
lần thuộc phần B. Chính xác hơn, nếu chúng ta đặt gi là số trận đấu mà người chơi tôi đã chơi vào
cuối giải đấu, thì tổng

gl + g2 + g3 + ... + g127

phải là số chẵn, vì tổng này tính mọi trò chơi đã được chơi chính xác hai lần!
Lưu ý rằng tổng này luôn là số chẵn, không chỉ ở cuối giải đấu mà bất cứ lúc nào!

Bây giờ chúng tôi đã hoàn thành; tổng trên là chẵn và là tổng của một số lẻ (127) các phần
tử. Nếu một số lẻ trong số chúng là số lẻ, tổng sẽ không phải là số chẵn. Vì vậy, số gi là số lẻ

là số chẵn.
_

Vấn đề tiếp theo đến từ một cuộc thi 1 906 của Hungary. Chúng tôi sẽ trình bày hai lưu ý như
vậy; cách đầu tiên sử dụng tính chẵn lẻ một cách đơn giản và cách thứ hai khéo léo xây dựng một
bất biến khác trước.

Ví dụ 3.4.8 Cho al, a2, ..., an biểu diễn sự sắp xếp tùy ý của các số 1,2,3 , . . . , N. Chứng
minh rằng, nếu n lẻ, tích

(al - l) (a2 - 2) (a3 - 3) ··· (an - n)

là một số chẵn.

Giải pháp 1: Tất nhiên, sẽ hữu ích khi nhìn vào một trường hợp cụ thể, chẳng hạn, n = 1. 1
Tôi đang triển khai chiến lược bước áp chót, chúng tôi tự hỏi mình, điều gì sẽ thúc đẩy sản phẩm

(al - 1) (a2 - 2) (a3 - 3) ·· · (all - 1 1)

là chẵn? Rõ ràng, đủ để cho thấy rằng một trong những con số

al - 1, a2 - 2, a3 - 3, ..., tất cả - 11

là thậm chí. làm như thế nào? Một chiến lược tốt là thử một bằng chứng mâu thuẫn, vì chúng ta cần
chỉ ra rằng chỉ một trong các số ở trên là số chẵn và chúng ta không biết số nào. Nhưng nếu chúng
ta bắt đầu với giả định rằng tất cả các số đều là số lẻ, chúng ta có thông tin cụ thể tốt để làm
việc với. Vì vậy, giả sử rằng mỗi

al-l, a2 - 2, a3 - 3, ... ,al l - ll


Machine Translated by Google
96 CHƯƠNG 3 CÁC THUẬT TOÁN GIẢI QUYẾT VẤN ĐỀ

là số lẻ. Bây giờ chúng ta có thể khôi phục tính chẵn lẻ của aj ban đầu . Chúng tôi thấy rằng sáu trong số họ,

là đồng đều, trong khi năm phần còn lại,

đều kỳ quặc. Đây là một mâu thuẫn, bởi vì aj là một hoán vị của 1,2, 3, ... ,1 1,

và tập hợp này chứa năm phần tử chẵn và sáu phần tử lẻ. Rõ ràng lập luận này mở rộng cho trường
hợp chung, đối với thuộc tính đặc biệt duy nhất của 11 mà chúng tôi sử dụng là thực tế là nó kỳ
quặc. _

Giải pháp 2: Bước đi mấu chốt: xét tổng các số hạng. Chúng ta có

(a) - I) + (a2 - 2) + "'+ (an - n) =

(a) + a2 + ... + an) - (1 +2+ ··· + n) = (1 +

2 + ··· + n) - (1 +2+ ··· + n) = 0,

vì vậy tổng là một bất biến: nó bằng 0 không có vấn đề gì sắp xếp. Tổng của một số lẻ các số
nguyên bằng 0 (một số chẵn) phải chứa ít nhất một số chẵn!
_

Cả hai giải pháp đều tốt, nhưng giải pháp thứ hai đặc biệt thông minh. Cố gắng kết hợp
ý tưởng mới ở đây trong các vấn đề trong tương lai.

Hãy đề phòng / hoặc những điều bất biến "dễ dàng". Kiểm tra xem liệu bạn có thể sắp xếp

lại vấn đề của mình để nhận các số đơn giản như số không hoặc số một hay không.

Ví dụ 3.4.9 Cho p) , P2, ..., P) 997 là các điểm phân biệt trong mặt phẳng. Nối các điểm
với các đoạn thẳng P) P2, P2P3, P3P4 "" , P) 996P) 997, P) 997P). Người ta có thể vẽ

một đường thẳng đi qua phần bên trong của tất cả các đoạn này không?

Giải pháp: Ở đây không hiển nhiên là tính chẵn lẻ, nhưng người ta nên luôn ghi nhớ tính chẵn
lẻ.

Bất cứ khi nào một vấn đề liên quan đến số nguyên, hãy tự hỏi bản thân xem có

bất kỳ hạn chế nào về chẵn lẻ không. Thử nghiệm với các giá trị khác với giá trị
đã cho nếu cần.

Vấn đề này liên quan đến 1.997 điểm. Một vài thí nghiệm với số điểm nhỏ hơn nhiều sẽ thuyết
phục bạn dễ dàng (hãy làm điều đó!) Rằng có thể vẽ đường thẳng nếu và chỉ khi số điểm là số chẵn.
Vì vậy, tính chẵn lẻ dường như là quan trọng. Chúng ta hãy tìm một lập luận chặt chẽ cho một
trường hợp cụ thể, nói bảy điểm. Một lần nữa, chúng ta sẽ tranh luận bằng sự mâu thuẫn bởi vì giả
định rằng chúng ta có thể vẽ đường thẳng sẽ cung cấp cho chúng ta nhiều thông tin cụ thể mà chúng
ta có thể làm việc với. Vì vậy, giả sử có một đường thẳng L đi qua phần bên trong của mỗi đoạn.
Đường thẳng này cắt mặt phẳng thành hai vùng, chúng ta sẽ gọi là phía "trái" và "phải" của L.
Không mất tính tổng quát, p) nằm về phía bên trái của L. Điều này buộc P2 nằm về phía bên phải của
L, tính bằng tum buộc P3 phải nằm trong
Machine Translated by Google
Machine Translated by Google

98 CHƯƠNG 3 CÁC THUẬT TOÁN GIẢI QUYẾT VẤN ĐỀ

sao cho mỗi cặp chứa hai số có 9 bộ số chẵn lẻ giống hệt nhau.
Do đó, tích của các số trong mỗi cặp là một hình vuông hoàn hảo. Nói cách khác, nếu chúng ta
sau đó mọi số trong danh sách cho Ci : = aibi,

CI, C2 "", C736

là một hình vuông hoàn hảo. Do đó, mỗi

Fl ,,, foi, ··· , JC736

là một số nguyên không có thừa số nguyên tố nào lớn hơn 23. Sử dụng nguyên lý chuồng chim bồ câu
một lần nữa, chúng ta kết luận rằng ít nhất hai số trong danh sách trên có cùng 9 bộ số chẵn lẻ.

Không mất tính tổng quát, hãy gọi chúng là .JCk và JCj. Khi đó .JCkVCJ là một hình vuông hoàn hảo;
2 4
, cho một số nguyên n. Do đó CkCj = n .
tức là, .JCkVCJ = n
Nhưng CjCk = ajbjakbb vì vậy chúng tôi đã tìm thấy bốn số từ tập hợp ban đầu của chúng tôi gồm 1,
_
985 số nguyên có tích là lũy thừa thứ tư của một số nguyên.

Chúng tôi kết thúc cuộc thảo luận của chúng tôi về tính ngang bằng với một vấn đề nổi tiếng,
ban đầu do de Bruijn đưa ra (dưới một hình thức khác) [7]. Ít nhất mười bốn giải pháp khác nhau đã
được đề cập, một số trong số đó sử dụng bất biến theo những cách khác nhau (xem [46] để biết tài

khoản rất dễ đọc về các giải pháp này). Giải pháp mà chúng tôi trình bày, sử dụng tính chẵn lẻ, có
lẽ là phù hợp nhất và là do Andrei Gnepp. Đó là một giải pháp đẹp; chúng tôi mong bạn trước tiên
hãy suy nghĩ về vấn đề trước khi đọc nó. Bằng cách đó, bạn sẽ đánh giá được mức độ khó của vấn đề
và cách giải quyết thông minh của Gnepp!

Ví dụ 3.4.11 Một hình chữ nhật được lát bằng các hình chữ nhật nhỏ hơn, mỗi hình chữ nhật có ít
nhất một cạnh bằng chiều dài tích phân. Chứng minh rằng hình chữ nhật lát gạch cũng phải có ít nhất
một cạnh bằng chiều dài tích phân.

Giải pháp: Đây là một ví dụ. Hình chữ nhật lớn đã được đặt trên mặt phẳng với các đường lưới
được đánh dấu tại mỗi đơn vị, sao cho phần dưới bên trái của nó là một điểm mạng.
Lưu ý rằng mỗi hình chữ nhật nhỏ có ít nhất một cạnh có độ dài là số nguyên và hình chữ nhật lớn 4

x 2,5 cũng có tính chất này. (Các vòng tròn trong sơ đồ được sử dụng trong lập luận của chúng tôi;
chúng tôi sẽ giải thích chúng bên dưới.)

·
· .
. .
.... _ . . . . . . . . . . . ... _ . . . . . . . . . ...... .... _ . . . . ...... ...... _ . . . . . . . .... _ ....
.
. .
. .
.
.
·
· .
.
. .
. .
.
· . . .
Tôi Tôi

. . . . .... ...
..
__ ...
J.

__ �_-4 ............

... ----.... ...... ...-.... - +


..... _ . . . . . . . . . .

Tôi
Machine Translated by Google

3.4 LỜI MỜI 99

Cái nhìn sâu sắc đầu tiên là phát hiện ra một bước áp chót thích hợp. Chúng ta hãy gọi tính chất

của việc có ít nhất một mặt tích phân là "tốt". Làm thế nào để chứng tỏ rằng hình chữ nhật lớn là tốt?

Định hướng nó sao cho comer phía dưới bên trái là một điểm mạng là chìa khóa:

Nếu hình chữ nhật không tốt, thì nó sẽ chỉ có một góc điểm mạng. Nhưng nếu hình
chữ nhật là tốt, thì nó sẽ có hai góc điểm mạng (nếu một chiều là số nguyên) hoặc
bốn góc điểm mạng (nếu cả chiều dài và chiều rộng đều là số nguyên).

Nói cách khác, tính chẵn lẻ đóng một vai trò nào đó: một hình chữ nhật với điểm kết hợp bên trái

thấp hơn tại một điểm mạng là tốt nếu và chỉ khi số điểm mạng kết hợp là chẵn! Hãy để chúng tôi đếm

số điểm xuất hiện của mạng tinh thể, hy vọng sẽ cho thấy rằng số lượng điểm đến của mạng tinh thể cho

hình chữ nhật lớn phải là số chẵn.

Tất nhiên chúng ta phải sử dụng giả thuyết rằng mỗi hình chữ nhật nhỏ hơn là tốt.

Hãy xem xét các cạnh của một hình chữ nhật nhỏ. Nó có thể không có điểm kết tinh, hoặc có thể có hai

hoặc bốn, nhưng không bao giờ có thể chỉ có một hoặc ba, vì lòng tốt.

Do đó, nếu chúng ta đếm số điểm mạng kết hợp trên mỗi hình chữ nhật nhỏ và cộng chúng lại, tổng mà

chúng ta gọi là S, sẽ là số chẵn.

Nhưng chúng tôi đã vượt quá một số điểm mạng. Ví dụ, trong hình trên, tất cả các điểm mạng tinh

thể ở góc được biểu thị bằng các vòng tròn. Có mười hình chữ nhật với các điểm mạng chung, và mỗi

hình có đúng hai điểm mạng chung, do đó S = 20. Hình tròn màu trắng được đếm chính xác một lần, hình

tròn màu xám được đếm hai lần và hình tròn màu đen được đếm bốn lần. Vì vậy, một cách khác để đếm

tổng S là

S = 1. (# vòng tròn trắng) + 2. (# vòng tròn màu xám) + 4. (# vòng tròn đen)

= 1 · 2 + 2 · 7 + 4 · 1 = 20.

Nói chung, khi chúng ta tính tổng S, chúng ta sẽ tính quá nhiều ở một số điểm đến. Đây là hai

quan sát đơn giản dễ kiểm tra.

• Chúng tôi sẽ chỉ tính điểm đến một lần, hai lần hoặc bốn lần - không bao giờ ba lần. • Điểm

đến duy nhất được tính chính xác một lần là điểm đến của số điểm lớn

hình chữ nhật.

Tô màu các điểm đến của mạng tinh thể như trong ví dụ của chúng ta, chúng ta thấy rằng nếu w, g,

b biểu thị số lượng các vòng tròn trắng, xám và đen, thì

S = w + 2g + 4b.

Như vậy w = S - 2g - 4b là số điểm mạng kết hợp của hình chữ nhật lớn. Và vì S chẵn nên S - 2g - 4b

cũng chẵn: hình chữ nhật lớn phải là hình chữ nhật tốt!

Giải pháp này là khá hướng dẫn. Một phác thảo ngắn gọn mà một người giải quyết vấn đề có kinh

nghiệm có thể hiểu được có thể là, "Định hướng hình chữ nhật lớn sao cho một điểm kết hợp là điểm

mạng, sau đó xem xét tính chẵn lẻ của số điểm mạng kết hợp." Có hai bước đi mấu chốt đối với giải

pháp tuyệt vời này: đầu tiên cố định điểm comer phía dưới bên trái trên một điểm mạng (mang lại thông

tin "miễn phí") và sau đó suy ra quy tắc chẵn lẻ cho sự tốt đẹp. Phần còn lại là một lập luận khá

chuẩn (như bạn sẽ thấy trong Chương 6, chiến thuật đếm thứ gì đó theo hai hoặc nhiều cách khác nhau

là một phương pháp phổ biến).


Machine Translated by Google

1 00 CHƯƠNG 3 CÁC THUẬT TOÁN GIẢI QUYẾT VẤN ĐỀ

Số học và Tô màu mô-đun

Parity hoạt động tốt một cách đáng kinh ngạc, nhưng nó khá thô thiển. Rốt cuộc, chúng ta
đang thu nhỏ vũ trụ vô tận của các số nguyên thành một thế giới nhỏ bé chỉ có hai thực
thể "chẵn" và "lẻ" sinh sống. Đôi khi chúng ta cần khám phá một thế giới phức tạp hơn.
Ví dụ 3.4.5 và 3.4.6 đã sử dụng các bất biến tương ứng với chín và ba giá trị có thể.
Đây là cả hai ví dụ về số học mô-đun, nghĩa là, việc rút gọn quan điểm của chúng ta từ
tập vô hạn các số nguyên thành tập hữu hạn các phần dư có thể có modulo m, trong đó m
được chọn một cách khéo léo.
Về mặt thực hành, đây là một bằng chứng nhanh cho khẳng định trong Ví dụ 3.4.5 ở trên. Bạn
có thể muốn nhớ lại các tính chất cơ bản của sự đồng dư từ trang 44. Không mất tính tổng quát,
hãy gọi n là một số có 4 chữ số với biểu diễn thập phân được giảm bớt. Khi đó n = 103 a + 102 b

+ l Oe + d. chúng ta có 10 '< == lk Vì 10 == 1 (mod 9), = 1 (mod

9) với mọi số nguyên k âm.

Do đó,

n = 1 03 a + 102 b + lOe + d == l · a + 1 · b + 1 · e + d (mod 9).

Bạn không cần sử dụng ký hiệu đồng dư, nhưng nó là một cách viết tắt tiện lợi và
nó có thể giúp bạn hệ thống hóa suy nghĩ của mình. Điều quan trọng là nhận thức được
khả năng rằng một bất biến có thể là một môđun đại lượng m đối với m được chọn đúng .

Ví dụ 3.4.12 Một buồng bong bóng chứa ba loại hạt hạ nguyên tử: 10 hạt loại X, 11 hạt
loại Y, 111 hạt loại Z. Bất cứ khi nào một hạt X và Y va chạm, chúng đều trở thành hạt
Z. Tương tự như vậy, các hạt Y và Z va chạm và trở thành các hạt X và các hạt X và Z
trở thành các hạt Y khi va chạm. Các hạt trong buồng bong bóng có thể tiến hóa để chỉ
có một loại không?

Giải pháp: Hãy chỉ ra dân số bất kỳ lúc nào bằng một bộ ba có thứ tự (x, y, z).
Hãy thử nghiệm một chút. Chúng ta bắt đầu với popUlation ( 1 0, 1 , 111 ) . Nếu một X- và Y- par

1 ticle sau đó va chạm, popUlation mới sẽ là (9, 1 0,11 3) . Có điều gì bất biến không?
Nhận thấy rằng vẫn còn 1 số hạt Y nhiều hơn số hạt X như lần trước. Tuy nhiên, ban đầu
có số Z nhiều hơn số Y là 100 , bây giờ có thêm 103. Trông không ổn, nhưng hãy bình
tĩnh và thử nghiệm thêm. Một va chạm XY khác tạo ra (8, 9, 1 1 5). Khoảng cách dân số
giữa X và Y vẫn là 1, nhưng khoảng cách giữa Y và Z đã tăng lên 1 06. Bây giờ chúng ta
hãy có một vụ va chạm với XZ. Dân số mới của chúng tôi là (7, 1 1 1 1 4) . Khoảng cách
XY đã ,tăng từ 1 lên 4, trong khi khoảng cách YZ giảm trở lại 1 03.

Nếu bạn không hài lòng với khả năng của số học mô-đun, sự phát triển của khoảng
cách dân số từ 1 đến 4 hoặc từ 100 đến 103 đến 1 06 trở lại 1 03 có thể có vẻ hỗn loạn.
Nhưng bây giờ bạn đã đoán rằng

Khoảng cách dân số là môđun bất biến 3.

Để chứng minh điều này một cách chính thức, hãy đặt (x, y, z ) là tổng thể tại một thời
điểm nhất định. Nếu không mất tính tổng quát, hãy xem xét một vụ va chạm của XZ. Quần
thể mới trở thành (x - 1, y + 2, z - 1 ) và bạn có thể dễ dàng xác minh rằng sự khác biệt
giữa quần thể X và Z là không thay đổi, trong khi sự khác biệt giữa quần thể X và Y đã
thay đổi 3 (tương tự như vậy đối với sự khác biệt giữa dân số Z và Y).
Machine Translated by Google

3.4 LỜI MỜI 1 01

Chúng tôi kết luận giải pháp bằng cách lưu ý rằng dân số ban đầu là (1 0, II , Ốm ) .
Khoảng cách dân số XY là 1, và do đó nó phải luôn phù hợp với I modulo 3.
Vì vậy, không có cách nào mà các quần thể X và Y có thể giống nhau, vì vậy các quần thể này không
bao giờ có thể trở thành số không. Lập luận tương tự cũng áp dụng cho các cặp quần thể khác: không
thể có hai trong số các quần thể trở thành không.
_

Việc sử dụng màu có liên quan đến tính chẵn lẻ và số học mô-đun, ngoại trừ việc chúng ta không
bị ràng buộc bởi các tính chất đại số của số nguyên. Một ví dụ về tô màu là bài toán domino (Ví dụ
2.4.2 ở trang 54), có thể được đúc lại thành bài toán chẵn lẻ. Đây là một ví dụ khác, sử dụng 12
màu.

Ví dụ 3.4.13 Có thể gạch một hình chữ nhật 66 x 62 với các hình chữ nhật 12 x 1 được không?

Lời giải: Rõ ràng, bất kỳ hình chữ nhật lớn nào được lát bằng hình chữ nhật 12 x I phải có
diện tích chia hết cho 1 2. Thật vậy, 66 · 62 = 1 2 · 34 1. Vì vậy, không thể loại trừ được. Tuy
nhiên, thử nghiệm với các cấu hình nhỏ hơn có cùng thuộc tính (tức là m xn trong đó m và n không
phải là bội số của 1 2, nhưng mn là) khiến chúng ta phỏng đoán rằng hình chữ nhật 66 x 62 không
thể được lát bằng hình chữ nhật 12 x 1 .
Vì vậy, giả sử rằng có một lát gạch, và chúng tôi sẽ tìm kiếm sự mâu thuẫn. Tô màu các hình
vuông có hình chữ nhật 66 x 62 với 12 màu theo mô hình "đường chéo" tuần hoàn như sau (giả sử rằng
chiều cao là 66 và chiều rộng là 62):

12 11 ... 12

. . .
12 1

Tôi 2 3 Tôi 2 Tôi


... Tôi 2 3 2

. . .
5 4 5 4

. . .
6 5 3 4 6 5

Màu này có một đặc tính tốt là bất kỳ hình chữ nhật 12 x 1 nào trong tấm lát gạch bao gồm 12
hình vuông có màu khác nhau. Nếu hình chữ nhật lớn có thể được lát gạch, nó sẽ được lát bằng 66.
62/12 = 34 1 12 x I hình chữ nhật, và do đó hình chữ nhật lớn phải chứa 34 1 hình vuông trong mỗi
12 màu. Điều quan trọng không phải là con số 34 1, mà là thực tế là mỗi màu xuất hiện trong cùng
một số ô vuông. Chúng tôi sẽ gọi một màu sắc như vậy là "đồng nhất."

Chúng ta hãy quan sát kỹ hơn hình chữ nhật 66 x 62 được tô màu . Chúng ta có thể chia nó thành
bốn hình chữ nhật phụ:

60 x 60 60 x 2

6 x 60 6x2

Dễ dàng kiểm tra rằng các hình chữ nhật con 60 x 60, 60 x 2 và 6 x 60 đều giống nhau, vì mỗi
hình chữ nhật con này có kích thước là bội số của 12.
Nhưng hình chữ nhật phụ 6 x 2 được tô màu như sau:
Machine Translated by Google

1 02 CHƯƠNG 3 CHIẾN THUẬT GIẢI QUYẾT VẤN ĐỀ

12

1 2 1

3 2

4 3

5 4

6 5

_
Do đó, toàn bộ hình chữ nhật lớn không đồng nhất, trái ngược với điều mà một tấm lát
gạch tồn tại đã tồn tại. Vì vậy việc ốp lát là điều không thể.

Đơn biến

Đơn biến là đại lượng có thể thay đổi hoặc không thay đổi ở mỗi bước của bài toán, nhưng
khi thay đổi thì đơn biến (chỉ theo một hướng). Một thuật ngữ khác được sử dụng cho đơn
biến là bán bất biến. Các đơn biến thường được sử dụng trong phân tích các hệ thống đang
phát triển, để chỉ ra rằng các cấu hình cuối cùng nhất định phải xảy ra và / hoặc để xác
định thời gian tồn tại của hệ thống. Nhiều lập luận đơn biến cũng sử dụng nguyên tắc cực
trị (ít nhất là nguyên tắc sắp xếp tốt). Đây là một ví dụ rất đơn giản.

Ví dụ 3.4.14 Trong một giải đấu kiểu loại trực tiếp của trò chơi hai người (đối với môn
thi cầu, cờ vua hoặc judo), khi bạn thua, bạn bị loại và giải đấu tiếp tục cho đến khi chỉ
còn lại một người. Tìm công thức cho số trò chơi phải chơi trong một giải đấu loại trực
tiếp bắt đầu với n thí sinh.

Lời giải: Số người còn lại trong giải đấu rõ ràng là một biến thể đơn tính
theo thời gian. Con số này giảm đi mỗi khi một trò chơi kết thúc.
Do đó, nếu chúng ta bắt đầu với n người, giải đấu phải kết thúc sau đúng n - 1 ván đấu! _

Đây là một vấn đề tế nhị hơn, một vấn đề cho thấy tầm quan trọng của việc kết hợp
nguyên tắc cực trị với một đơn biến.
,
Ví dụ 3.4.15 . N thẻ của một bộ bài (với n là một số nguyên dương tùy ý) là n. Bắt đầu với
nếu thẻ trên cùng bộ
có bài
nhãntheo
là k,
thứhãy
tự đảo
bất ngược
kỳ, hãy
thứ lặp
tự của
lại k
thao
thẻ tác
đầu sau:
tiên.
cóChứng
nhãn minh
1,2 ,rằng
. . .
cuối cùng thẻ đầu tiên sẽ là 1 (vì vậy không có thay đổi nào xảy ra).

Điều tra: Ví dụ: nếu n = 6 và chuỗi bắt đầu là 362 1 54, các thẻ phát triển như sau:

--- + 5 13426 --- + 2431 56 --- + 4231 56 --- + 1 32456.


362 1 54 --- + 2631 54 --- + 6231 54 --- + 45 1326 --- + 3 1 5426

Sẽ thật tuyệt nếu số lượng thẻ ở vị trí thứ nhất giảm đi một cách ngẫu nhiên, nhưng
nó đã không xảy ra (chuỗi là 3, 2, 6,4, 3, 5, 2, 4, O. Tuy
về trình
nhiên,tựnónày.
là đáng
Chúngđểtasuy
sẽ nghĩ
sử dụng
một nguyên tắc chung rất đơn giản nhưng quan trọng:
Machine Translated by Google

3.4 LỜI MỜI 1 03

Nếu chỉ có rất nhiều trạng thái khi một cái gì đó tiến hóa, thì một trạng thái sẽ

lặp lại hoặc quá trình tiến hóa cuối cùng sẽ dừng lại.

Trong trường hợp của chúng ta, chuỗi các số hạng 1 lặp lại (vì chỉ có rất nhiều), hoặc cuối cùng

số hạng 1 sẽ là I (và sau đó quá trình tiến hóa tạm dừng). Chúng tôi xin chứng minh điều sau. Làm thế

nào để chúng tôi loại trừ khả năng lặp lại?

Rốt cuộc, trong ví dụ của chúng tôi, có rất nhiều lần lặp lại!

Một lần nữa, nguyên tắc cực đoan giúp tiết kiệm thời gian. Vì chỉ có rất nhiều khả năng xảy ra

khi một chuỗi phát triển, nên tồn tại một giá trị đứng thứ nhất lớn nhất từng xảy ra, mà chúng ta sẽ
gọi là LI (trong ví dụ trên, LI = 6). Vì vậy, tại một số thời điểm trong sự phát triển của dãy số, số

đứng thứ nhất là LI, và sau đó, không có số thứ nhất nào lớn hơn L1 • xuất
Điều hiện
gì xảy
ở vị
ra trí
ngaythứ
saunhất?
khi LI

Chúng tôi đảo ngược các thẻ LI đầu tiên, vì vậy LI xuất hiện ở vị trí thứ LI. Chúng tôi biết điều đó

nhưng liệu nó có thể bằng LI không? Thẻ vị trí thứ nhất không bao giờ được lớn hơn LI, câu trả lời là

hơn LI, thì việc đảo ngược sẽ không bao giờ chạm vào thẻ ở không;
vị trí miễn
thứ LI.
là giá
Chúng
trịtôi
ở vị
sẽ trí
không
thứbao
nhất
giờ
nhỏ

đảo ngược nhiều hơn các thẻ LI đầu tiên (theo mức tối đa của LI), vì vậy cách
có số
duy
LInhất
di chuyển
để làm được
cho thẻ

nếu chúng tôi đảo ngược chính xác các vị trí LI. Nhưng điều đó có nghĩa là cả thẻ xếp hạng 1 và thẻ

xếp hạng LI đều có giá trị LI, điều này là không thể.

Đó là bước đi mấu chốt. Bây giờ chúng ta xem xét tất cả các giá trị ở vị trí thứ nhất xảy ra sau

khi LI xuất hiện ở vị trí thứ nhất. Chúng phải nhỏ hơn LI . Gọi giá trị lớn nhất trong các giá trị này

là L2. Sau khi L2 xuất hiện ở vị trí thứ nhất, tất cả các giá trị vị trí thứ nhất tiếp theo sẽ nhỏ hơn

L2 bởi chính xác cùng một đối số như trước đó.

Do đó, chúng ta có thể xác định một chuỗi giảm dần các giá trị vị trí tôi lớn nhất.

Cuối cùng, chuỗi này phải đạt được I, và chúng tôi đã hoàn thành!

Ở trên là một chút mơ hồ. Sẽ là hướng dẫn để đưa ra một lập luận chính thức hơn, đặc biệt là để

minh họa việc sử dụng cẩn thận các chỉ số con và ký hiệu.

Lời giải chính thức: Gọi fi là giá trị của thẻ thứ nhất sau i bước, i = 1,2, .... Chúng tôi muốn

chứng tỏ rằng fm = I với m nào đó (và do đó, fn = I với mọi n 2: m). Kể từ khi tôi :::; f; :::; n, số

LI : = max {fi: i 2: I}

tồn tại. Cũng xác định

tl : = min {t: It = LI};

tức là, tl là bước đầu tiên tại đó thẻ I bằng LI. Trong ví dụ trên, LI = 6
và tl = 3.
Chúng tôi khẳng định rằng nếu t > tl, thì It < LI. Để thấy điều này, hãy lưu ý rằng ft không được

lớn hơn LI, theo định nghĩa của LI. Để thấy rằng Nó không thể bằng LI, chúng tôi
đốilập
lập.
luận
Gọibằng
t làcách
bước

đầu tiên sau tl sao cho It = LI. Tại bước tl, các thẻ LI đầu tiên

đã được đảo ngược, đặt giá trị LI ở vị trí thứ LI. Đối với tất cả các bước s sau tl và trước t, chúng
ta có fs < LI, có nghĩa không
là thẻthể
có có
giáIttrị LI ởvìvịđiều
= LI, trí đó
thứcóLInghĩa
đã không được
là thứ di Li
1 và chuyển. Do đó
là giống ở bước
nhau (trừ t,

khi LI = I, trong trường hợp đó chúng ta đã hoàn thành). Sự mâu thuẫn này thiết lập yêu sách.
Machine Translated by Google
Machine Translated by Google

3.4 LỜI MỜI 1 05

Có ba điều quan trọng về S '. Trước hết, nó đáp ứng

2
S' + S '- 1 = 0. (2)

Ngoài ra, nó là tích cực. Và cuối cùng, S ' < 1.


Đối với bất kỳ cấu hình (vô hạn) nào của bộ cờ, hãy cộng tất cả các giá trị được chỉ định cho

mỗi bộ cờ trên bàn cờ. Hãy để chúng tôi gọi đây là "Tổng Conway." Chúng tôi khẳng định điều này là

đơn phương của chúng tôi. Chúng ta cần kiểm tra một số thứ.

Trước hết, liệu tổng này có tồn tại không? Đúng; chúng ta có vô số chuỗi hình học vô hạn để xem

xét, 6 nhưng may mắn thay, chúng sẽ hội tụ. Hãy để chúng tôi tính tổng Conway cho cấu hình bắt đầu.

Các ô trên trục y (ngay bên dưới C) có các giá trị S '
S 6
, . ... Nói chung, các ô trên đường thẳng x = ± r có các giá trị S'S + r, S'6 +

r , . . . . Do đó, tổng Conway cho toàn bộ "nửa mặt phẳng" là


00 rS 00 rS + r
S 6 s + r - '=' -
( S ' + S ' + S'6 + r +. . . ) =
+. ..) + 2 � (S ' - '=' - +2 �
r = 1 1 - S ' r = 1 1 - S '

)
2S'6
-
S '+
1 -1 S(' s LÀ'

= _ 1_ (
1 - S '
S'S (1 - S ') + 2S'6

1-S '
)
S
_1 (
S' + S'6
=
) .
1 - S ' 1 - S '

2
Sử dụng (2), chúng tôi nhận thấy rằng 1 - S '= S' . Do đó, biểu thức trên đơn giản hóa thành

S
S'
�S'6 = S '+ S'2 = 1.

Do đó, cấu hình bắt đầu có tổng Conway là 1, và tất cả các cấu hình khác sẽ có tổng Conway có thể

tính toán được.

Tiếp theo, chúng ta phải chứng minh rằng tổng Conway là một đơn biến. Hãy xem xét một bước di
chuyển ngang di chuyển một quân cờ ra khỏi điểm đích C. Ví dụ: giả sử chúng ta có thể nhảy một bộ
ll
kiểm tra từ (9,3), có giá trị S ' ,thành (1 1,3). Điều gì xảy ra
với số tiền Conway? Chúng tôi loại bỏ các bộ kiểm tra tại (9,3) và (10,3) và tạo một bộ kiểm tra tại

(1 1,3). Thay đổi thực trong tổng Conway là (- 1 - S '+ S'2).

11 12 13 II
-S' - S ' + S ' = S '

Nhưng - 1 - S ' + S'2 = -2S', bởi (2). Do đó, sự thay đổi ròng là âm; tổng giảm dần. Rõ ràng đây là
một tình huống chung (hãy kiểm tra các trường hợp khác nếu bạn không chắc chắn): bất cứ khi nào một

người kiểm tra di chuyển khỏi C, tổng sẽ giảm.


Mặt khác, nếu quân cờ di chuyển về phía C, tình hình sẽ khác. Ví dụ, giả sử chúng ta đi từ (9,3)

đến (7,3). Sau đó, chúng tôi loại bỏ các bộ kiểm tra tại (9,3) và (8,3) và tạo một bộ kiểm tra tại

(7, 3), thay đổi tổng bằng (1 - S '- S'2) = o.

11 9
- S ' - S'1O + S'9 = S '

6Xem trang 1 60 để biết thông tin về chuỗi hình học vô hạn.


Machine Translated by Google
Machine Translated by Google
3.4 LỜI MỜI 1 07

ở trang 94, khám phá và chứng minh một phát biểu tương tự cho được phép), và đảo ngược màu sắc của chúng. Số lượng nước đi nhỏ

phép chia hết cho II. nhất của bà mẹ cần thiết để làm cho toàn bộ hàng có một màu là

bao nhiêu? Rõ ràng, n di chuyển sẽ hoạt động (đối với bài kiểm
3.4.23 Bắt đầu với một tập hợp các điểm mạng. Mỗi giây,
tra, đảo ngược hình vuông thứ 1, rồi đến hình vuông thứ 3, v.v.),
chúng ta có thể thực hiện một trong các thao tác sau;
nhưng bạn có thể làm tốt hơn thế không?
I. Điểm (x, y) “sinh” ra điểm (x + I, y + I).
3.4.29 Trả lời câu hỏi tương tự như trên, ngoại trừ bây giờ

hãy bắt đầu với "bàn cờ" 2n x 2n và nước đi hợp pháp bao gồm
2. Nếu x và y đều chẵn thì điểm (x, y) "sinh" ra điểm (x /
việc chọn bất kỳ hình chữ nhật con nào và tô màu của nó.
2, y / 2).

3. Cặp điểm (x, y) và (y, z) “sinh” ra (x, z).


3.4.30 Chứng tỏ rằng nếu mọi phòng trong nhà đều có số cửa
chẵn thì số cửa ra vào bên ngoài cũng phải là số chẵn.

Ví dụ: nếu chúng ta bắt đầu với điểm duy nhất (9, I), phép

toán số 1 cho ra điểm mới (10, 2), sau đó phép toán số 2 cho
3.4.31 Hai mươi ba người, mỗi người có trọng số tích phân,
kết quả (5, I) và sau đó là chín ứng dụng của phép toán số 1
quyết định chơi bóng đá, chia thành hai đội II người, cộng với
cho chúng tôi (14, 10), sau đó phép toán số 3 áp dụng cho (14,
một trọng tài. Để giữ cho mọi thứ công bằng, các đội được chọn
1 0) và (1 0, 2) cho chúng tôi (1 4, 2), v.v.
phải có tổng trọng lượng bằng nhau. Nó chỉ ra rằng bất kể ai
Nếu chúng ta bắt đầu với một điểm duy nhất (7, 29), thì
được chọn làm trọng tài, điều này luôn có thể được thực hiện.
liệu cuối cùng nó có thể lấy được điểm (3, 1 999) không?
Chứng minh rằng 23 người phải có cùng khối lượng.

3.4.24 Ban đầu, chúng ta được đưa ra dãy 1,2, ... là u và v và

thay chúng
, 1 bằng giá phút,
00. Mỗi trị uvchúng
+ u +tôi
v. xóa hai số bất kỳ
3.4.32 (Nga 1 995) Có n ghế trên một chiếc đu quay. Một

cậu bé đi n chuyến xe. Giữa mỗi lần cưỡi ngựa, anh ta di


Rõ ràng, chúng ta sẽ chỉ còn lại một con số sau 99 phút. Con
chuyển theo chiều kim đồng hồ một số địa điểm nhất định đến
số này có phụ thuộc vào những lựa chọn mà chúng ta đã thực
một con ngựa mới. Mỗi lần anh ta di chuyển một số lượng khác nhau
hiện không?
nơi. Tìm tất cả n mà cậu bé cưỡi mỗi con ngựa.

3.4.25 Chứng minh rằng không thể chọn ba số nguyên a, b và c

sao cho
3.4.33 Xét chín điểm mạng trong không gian ba chiều. Chứng

(ab) l (bc), (bc) l (ca), (ca) l (ab). tỏ rằng phải có một điểm mạng ở bên trong của một trong các

đoạn thẳng nối hai trong số các điểm này.


3.4.26 (Tom Rike) Bắt đầu với tập hợp {3,4, 1 2}. Sau đó bạn

được phép thay hai số a và b bất kỳ bằng cặp số mới 0.6a -


3.4.34 Sáu số hạng đầu tiên của dãy là o 1 2 3 4 5 Mỗi số hạng
0.8b và 0. 8a + 0.6b. Bạn có thể biến đổi tập hợp thành {4,
tiếp theo là chữ số cuối cùng
6, 1 2} không?
của ' s�� �f sáu số hạng trước đó. Nói cách khác,
3.4.27 Hai người thay phiên nhau cắt một thanh sô cô số hạng thứ bảy là 5 (vì 0 + I + 2 + 3 + 4 + 5 = 1 5), số hạng
la hình vuông có kích thước 6 x 8 hình vuông. Bạn chỉ
thứ tám là 0 (vì I + 2 + 3 + 4 + 5 + 5 = 20), v.v ... 1 3579
được phép cắt thanh dọc theo đường phân chia giữa các
xảy ra ở đâu trong dãy số này?
ô vuông và đường cắt của bạn chỉ có thể là một đường thẳng.
Ví dụ: bạn có thể biến thanh ban đầu thành mảnh 6 x 2 và mảnh
3.4.35 Giải pháp cho vấn đề người kiểm tra (Bài kiểm tra 3,4.
6 x 6 , và mảnh sau này có thể được biến thành mảnh I x 6 và
1 6) khá lắt léo, và gần như là gian lận. Tại sao bạn không
mảnh 5 x 6 . Người chơi cuối cùng có thể làm vỡ thanh sô cô
thể gán bất kỳ giá trị nào mà bạn muốn cho điểm C, ví dụ, I
la sẽ thắng (và được ăn thanh sô cô la). Có chiến lược chiến
OIOO? Sau đó, điều này sẽ đảm bảo rằng bạn không bao giờ có
thắng cho người chơi thứ nhất hoặc thứ hai không? Còn trường
thể đạt được điều đó. Ý kiến này có gì sai?
hợp tổng quát (thanh bắt đầu là mx n) thì sao?

3.4.36 Đảm bảo rằng bạn thực sự hiểu cách thức thực hiện đầy
3.4.28 Xét một hàng gồm 2n hình vuông được tô màu đen và
đủ của Ví dụ 3.4. 10 tác phẩm. Ví dụ, chính xác nguyên tắc
trắng tự nhiên. Một động thái hợp pháp bao gồm việc chọn
chuồng chim bồ câu được sử dụng bao nhiêu lần trong bài toán này?
bất kỳ tập hợp ô vuông liền kề nào (một hoặc nhiều ô vuông,
Sau khi đọc giải pháp, hãy xem bạn có thể giải quyết vấn đề
nhưng nếu bạn chọn nhiều hơn một ô vuông, các ô vuông của
này với người khác tốt như thế nào (một người
bạn phải nằm cạnh nhau; nghĩa là không có "khoảng trống"
Machine Translated by Google

1 08 CHƯƠNG 3 CHIẾN THUẬT GIẢI QUYẾT VẤN ĐỀ

đặt câu hỏi thông minh!). Nếu bạn không thể giải thích cho Bây giờ chúng ta muốn chứng minh rằng v -e + / = I cho đối

anh ấy hài lòng, hãy quay lại và đọc lại lời giải, ghi chú tượng mới. Để làm như vậy, hãy bắt đầu xóa từng cạnh và /

lại các điểm mấu chốt, các vấn đề phụ được đặt ra và giải hoặc đỉnh. Điều này làm gì với giá trị của v -e + /? Bạn

quyết, v.v. Hiểu biết vững chắc về ví dụ này sẽ giúp bạn phải kết thúc với điều gì?
giải quyết vấn đề tiếp theo. 3.4.37 (lMO 1 978) Một xã hội
3.4.41 Nếu bạn vẫn chưa thực hiện xong Vấn đề 2.4.24, bây

quốc tế có các thành viên đến từ sáu quốc gia khác nhau. giờ là lúc để xem xét lại vấn đề.

Danh sách thành viên có 1 978 tên, đánh số 1 , 2, ..., 1


3.4.42 (Olympic Toán học Vùng Vịnh 2 (00) Al ice chơi trò
978. Chứng minh rằng có ít nhất một mem ber có số bằng tổng
chơi solitaire sau đây trên bàn cờ 20 x 20. Cô ấy bắt đầu
số của hai
làthành
số một
viên
thành
đến viên
từ nước
đến mình,
từ đấthoặc
nướclớn
củagấp
mình.
đôi.
bằng cách đặt 1 00 xu, 1 00 niken, 100 dime và 1 00 phần
tư trên bàn cờ như vậy rằng mỗi ô trong số 400 ô vuông

chứa đúng một đồng xu.


Sau đó cô ấy chọn 59 trong số những đồng xu này và xóa

3.4.38 (lMO 1 997) Một ma trận n x n (mảng vuông) có các chúng khỏi bàn cờ. Sau đó, cô ấy loại bỏ từng đồng xu một,

phần tử đến từ tập S = {I, 2, ..., 2n -I} được gọi


trận lànếu,
bạc ma tuân theo các quy tắc sau:
với mỗi i = I , ... , n, hàng thứ i và cột thứ i tất
cùngcảchứa
các

phần tử của S. Chứng tỏ rằng không có ma trận bạc nào cho • Một xu chỉ có thể bị loại bỏ nếu có bốn hình vuông
n = 1 997.
của bàn cờ liền kề với hình vuông của nó (lên,

3.4.39 (Đài Loan 1 995) Xem xét phép toán biến dãy 8 số xuống, trái và phải) bị bỏ trống (không chứa đồng

xu). Các hình vuông "ngoài bàn cờ" không được tính
hạng XI, Xg thành dãy 8 số hạng mới X2 ,...
vào bốn hình vuông này: ví dụ: một hình vuông không

đồng bằng giáp với cạnh của bàn cờ có ba hình vuông

liền kề, vì vậy không thể xóa một xu trong một hình

vuông như vậy theo quy tắc này, ngay cả khi cả ba


Tìm tất cả các dãy số nguyên có 8 số hạng có tính chất mà
hình vuông liền kề hình vuông bị bỏ trống.
sau vô số lần áp dụng phép toán này, một dãy còn lại là
dãy, tất cả các số hạng của chúng đều bằng nhau.
• Một niken chỉ có thể bị xóa nếu có ít nhất ba ô

trống liền kề với ô vuông của nó. (Và một lần nữa,
3.4.40 Công thức của Euler. Xét một khối đa diện P.
các ô vuông "ngoài bảng" không được tính.)
Chúng ta muốn chứng minh rằng v - e + / = 2, trong đó v,

e, j lần lượt là số đỉnh, cạnh và mặt của P. Hãy tưởng


tượng P được làm bằng cao su trắng, với các cạnh được sơn
• Một đồng xu chỉ có thể bị loại bỏ nếu có ít nhất hai
đen và các đỉnh được sơn màu đỏ.
ô trống liền kề với ô vuông của nó (không tính ô
Cẩn thận cắt bỏ một mặt (nhưng không loại bỏ bất kỳ cạnh
vuông "tắt bảng").
hoặc đỉnh nào), và kéo căng đối tượng thu được để nó nằm

trên một mặt phẳng. Ví dụ: đây là hình khối trông như thế
• Một phần tư chỉ có thể bị loại bỏ nếu có ít nhất
nào sau khi "phẫu thuật" như vậy:
một hình vuông trống liền kề với hình vuông của nó

(các hình vuông "ngoài bảng" không được tính).

Alice thắng nếu cuối cùng cô ấy thành công trong


việc lấy tất cả các đồng xu. Chứng minh rằng cô ấy không thể
Thắng lợi.
Machine Translated by Google

Chương 4

Ba chiến thuật chuyển đổi quan trọng

Phép toán chéo (được đề cập lần đầu ở trang 54) là một ý tưởng kết nối hai hoặc nhiều nhánh toán
học khác nhau, thường theo một cách đáng ngạc nhiên. Trong chương này, chúng tôi sẽ giới thiệu có
lẽ ba chủ đề giao nhau hiệu quả nhất: lý thuyết đồ thị, số phức và hàm sinh.

Chúng tôi sẽ chỉ sơ lược bề mặt của ba chủ đề rất phong phú này. Bài thuyết trình của chúng

tôi sẽ là sự kết hợp của sự trình bày và các vấn đề để bạn có thể suy ngẫm ngay tại chỗ. Bạn có thể
thấy đáng giá khi đọc Chương 5-7 trước hoặc ít nhất là đồng thời, vì một số ví dụ dưới đây liên
quan đến toán học tương đối phức tạp.

4.1 Lý thuyết đồ thị

Khái niệm về đồ thị rất đơn giản: chỉ đơn thuần là một tập hợp hữu hạn các đỉnh và cạnh.
Các đỉnh thường được hình dung dưới dạng các chấm và các cạnh là các đường nối một số hoặc tất cả
các cặp đỉnh. Nếu hai đỉnh được nối với nhau bằng một cạnh, chúng được gọi là lân cận. Theo quy
ước, đồ thị không chứa nhiều cạnh (hai hoặc nhiều cạnh nối cùng một cặp đỉnh) hoặc vòng lặp (một
cạnh nối một đỉnh với chính nó).

Nếu có nhiều cạnh hoặc nhiều vòng, chúng ta sử dụng thuật ngữ đa đồ thị. Trong hình bên dưới, đối
tượng ở bên trái là một đồ thị, trong khi đối tượng bên phải của nó là một đồ thị đa đồ thị.

Bạn đã thấy nhiều ví dụ về đồ thị. Vấn đề về Hành động khẳng định


(Ví dụ 2. 1 .9 ở trang 21) có thể được trình bày lại thành một bài toán về đồ thị như sau.

Cho một đồ thị tùy ý, chứng tỏ rằng có thể tô màu các đỉnh là đen
và trắng sao cho mỗi đỉnh trắng có ít nhất bao nhiêu lân cận đen
như lân cận trắng và ngược lại.

Tương tự như vậy, bài toán Bắt tay (1.1.4) và giải pháp cho vấn đề Thư viện (2.4.3) đều có thể được
định dạng lại thành các câu hỏi về đồ thị, thay vì con người và cái bắt tay hoặc mạng lưới đường
ống.

Đây là điều làm cho lý thuyết đồ thị trở nên hữu ích một cách đáng ngạc nhiên. Chỉ về bất kỳ tình huống

nào trong các "mối quan hệ" giữa các "đối tượng" đều có thể được tái hiện dưới dạng đồ thị, trong đó các đỉnh

1 09
Machine Translated by Google

110 CHƯƠNG 4 BA CHIẾN THUẬT QUAY LỐI QUAN TRỌNG

là các "đối tượng" và chúng ta nối các đỉnh với các cạnh nếu các đối tượng tương ứng được "xếp lại".

Nếu bạn vẫn chưa bị thuyết phục, hãy xem vấn đề sau. Đừng đọc phân tích ngay lập tức.

Ví dụ 4. 1.1 (USAMO 1 986) Trong một bài giảng nào đó, mỗi nhà toán học trong số năm nhà toán học ngủ

quên đúng hai lần. Đối với mỗi cặp nhà toán học này, có một số động tĩnh khi cả hai đang ngủ đồng thời.

Chứng minh rằng, tại một thời điểm nào đó, ba người trong số họ đang ngủ đồng thời.

Giải pháp từng phần: Chúng ta hãy gọi các nhà toán học là A, B, C, D, E và biểu thị khoảng thời

gian mà mỗi người đã ngủ bằng Al , A2, B, B2, v.v. Bây giờ khoảng
hãy xácthời
địnhgian
một này
đồ thị
, với
có các
10 đỉnh
đỉnh là
được
những
nối

bằng một cạnh nếu khoảng thời gian trùng nhau. Có (�) = 1 0 cặp toán tử nên đồ thị này phải có ít nhất
Tôi

10 cạnh. Dưới đây là một ví dụ, mô tả tình huống trong đó giấc ngủ ngắn trùng
đầu tiên
với của
giấc
nhà
ngủ
toán
ngắn
học
đầu
A
tiên của C và giấc ngủ ngắn thứ hai của E, v.v. Lưu ý rằng Al và A2 không thể được nối với một đỉnh, B

và B2, v.v., vì mỗi nhà toán học có hai giấc ngủ ngắn riêng biệt.

Đồ thị cụ thể này chứa chu trình CIAI E2B2CI. Bởi điều này, chúng tôi muốn nói đến một con đường

khép kín "di chuyển" dọc theo các cạnh. Một chu kỳ rất hữu ích, bởi vì nó hạn chế thời gian chồng chéo.

Dễ dàng nhận thấy điều này nhất bằng cách "xếp chồng" các khoảng. Đây là một khả năng. Thời gian được

đo theo chiều ngang. Lưu ý rằng mỗi khoảng thời gian ngủ trưa phải trùng lặp với (các) hàng xóm dọc của

nó.

-------
cl

Chúng tôi sẽ chỉ ra rằng ba khoảng thời gian khác nhau phải trùng nhau. Vì C và B2 trùng nhau, và CI và

A l chồng lên nhau, chúng ta hoàn thành nếu B2 và A l chồng lên nhau trong CI. Nếu họ không (như trong

Đọc Phần 6. 1 nếu bạn không hiểu ký hiệu (;).


Tôi
Machine Translated by Google

4.1 G RAPH LÝ THUYẾT 111

ví dụ trên), sẽ có một khoảng cách giữa điểm cuối của B2 và điểm bắt đầu của AI.
Tuy nhiên, £ 2 phải nằm giữa khoảng cách này, vì nó chồng lên cả B2 và A I. Do đó C I £ 2 chồng ,B2, và
lên nhau (tương tự như vậy, C 1 , £ 2 và A I trùng nhau).

Lập luận này liên quan đến một chu kỳ 4 (chu kỳ có bốn đỉnh), nhưng một chút suy nghĩ
(lấy giấy ra và làm một số thí nghiệm!) Cho thấy rằng nó sẽ hoạt động với một chu kỳ có độ
dài hữu hạn bất kỳ. Do đó, chúng tôi đã giảm vấn đề thành một câu hỏi lý thuyết đồ thị
"thuần túy":

Nếu một đồ thị có 10 đỉnh và 10 cạnh thì nó phải chứa chu trình?

Khả năng kết nối và các chu

trình Bây giờ chúng ta thấy lý thuyết đồ thị có thể tái hiện hoàn toàn một vấn đề như
thế nào, chúng ta sẽ nghiên cứu cổng một số tính chất đơn giản của đồ thị, đặc biệt
là mối quan hệ giữa số lượng đỉnh và cạnh và sự tồn tại của các chu trình. Số lượng
các cạnh xuất phát từ một đỉnh cụ thể được gọi là bậc của đỉnh. Nếu đỉnh là x thì tung
độ thường được ký hiệu là d (x) . Bạn có thể dễ dàng xác minh sự thật quan trọng sau
đây, thường được gọi là bổ đề bắt tay (nếu bạn muốn có gợi ý, hãy đọc lại Ví dụ 3.4.7
trên trang 95).

Trong bất kỳ đồ thị nào, tổng tung độ của tất cả các đỉnh bằng hai lần số cạnh.

Một đồ thị được kết nối nếu mọi cặp đỉnh đều có đường đi giữa chúng. Nếu một biểu
đồ không được kết nối, người ta luôn có thể phân tách nó thành các thành phần được kết
nối. Ví dụ, đồ thị dưới đây có 10 đỉnh, 11 cạnh và hai thành phần liên thông.
Quan sát rằng bổ đề bắt tay không yêu cầu kết nối; trong biểu đồ này, các độ (quét từ
trái sang phải, từ trên xuống dưới) là 1, 2, 1, 1,3,3, 2, 4, 3, 2.
Tổng là 22 = 2. 11.

2
Một đồ thị liên thông không chứa chu trình được gọi là cây. Ví dụ,
đồ thị 8 đỉnh sau đây là một cây.

2
Một đồ thị không liên thông không chứa chu trình nào được gọi là một khu rừng; mỗi thành phần được kết nối của nó là một cây.
Machine Translated by Google

112 CHƯƠNG 4 BA CHIẾN THUẬT QUAY LỐI QUAN TRỌNG

3
Ở một cây, ta gọi các đỉnh có bậc 1 là lá.
Chắc chắn có vẻ hợp lý khi cây cối phải có lá. Trên thực tế,

Cây nào có từ hai đỉnh trở lên đều có ít nhất hai lá.

Không chính thức, điều này là khá rõ ràng. Vì cây không có chu trình nên nó phải có các đường
dẫn mà đỉnh bắt đầu và đỉnh kết thúc khác nhau. Hai đỉnh này sẽ có bậc 1.
Nhưng điều này hơi mơ hồ. Dưới đây là một bằng chứng chặt chẽ sử dụng nguyên tắc cực đoan và lập
luận mâu thuẫn:
Cho một cái cây, chọn một đỉnh bất kỳ. Bây giờ hãy xem xét tất cả các đường đi bao gồm đỉnh
này và chọn đường dài nhất , tức là đường dẫn chứa nhiều đỉnh nhất. Vì đồ thị là một cây, không
có chu trình, vì vậy không có sự mơ hồ nào ở đây - không có con đường nào có thể quay ngược trở
lại chính nó. Ngoài ra, vì các cây được kết nối với nhau, chúng tôi đảm bảo rằng có những con
đường để bắt đầu.
Gọi P : = XlX2 .. 'Xn là đường đi dài nhất, trong đó Xi là các đỉnh. Ta cho rằng Xl và Xn
phải có bậc 1. Giả sử rằng Xl có bậc lớn hơn 1. Khi đó Xl có các đỉnh X2 và y trong số các lân
cận của nó. Quan sát rằng y không thể là bất kỳ đỉnh nào trong số các đỉnh X3, X4, ..., Xn, vì
điều đó sẽ tạo
ta ra
có một
thể chu
tạo trình!
ra một Nhưng nếu ydài
con đường không
hơn phải
YXlX2là..một
'Xntrong những với
mâu thuẫn đỉnhtính
này,chất
thì tối
chúng
đa
của P. Do đó d (xJ) = 1, và một lập luận tương tự cho thấy rằng d (xn) = 1 .


Khi nào một đồ thị liên thông là một cây? Về mặt trực giác, có vẻ như cây cối khá nghèo nàn
về các cạnh so với các đỉnh, và thực tế, thử nghiệm (hãy làm điều đó!) Cho thấy rất rõ ràng rằng

Đối với cây, số cạnh nhỏ hơn số đỉnh đúng một.

Phỏng đoán này là một ứng cử viên tự nhiên cho quy nạp toán học. Chúng ta sẽ quy về số đỉnh
v, bắt đầu bằng v = 2. Cây duy nhất có hai đỉnh chứa một cạnh duy nhất nối hai đỉnh, vì vậy
trường hợp cơ sở là đúng. Bây giờ, giả sử rằng chúng ta biết rằng tất cả các cây có v đỉnh đều
chứa các cạnh v - I. Xét một cây T với v + 1 đỉnh. Ta sẽ chỉ ra rằng T có v cạnh. Hái một chiếc
lá (ta biết rằng T có những chiếc lá). Loại bỏ đỉnh này, cùng với cạnh sinh ra từ nó. Những gì
còn lại? Một đồ thị với v đỉnh vẫn được kết nối (kể từ khi T được kết nối, và rút ra một

3 Thuật ngữ này không hoàn toàn chuẩn. Theo thông lệ, người ta thường chỉ định một trong các đỉnh bậc là "gốc",
và thực sự là có cả một lý thuyết về cái gọi là "cây có gốc" khá quan trọng trong khoa học máy tính, nhưng sẽ không
được thảo luận ở đây. Xem [43] để biết thêm chi tiết.
Machine Translated by Google

4.1 G RAPH LÝ THUYẾT 113

lá không thể ngắt nó), và không có chu kỳ (vì T không có chu kỳ, và nhổ một chiếc lá không thể tạo ra một
_
chu kỳ). Do đó, biểu đồ mới là một cây. Theo giả thiết quy nạp, nó phải có v - I cạnh. Như vậy T có v -I +

1 = v cạnh.

4.1.2 Tổng quát hóa điều trên bằng cách chỉ ra rằng nếu một tập hợp các cây rời rạc (tất nhiên được gọi là

rừng) có k thành phần liên thông, e cạnh và v đỉnh, thì e = v - k.

4.1.3 Kết luận bằng cách chỉ ra

rằng Nếu một đồ thị có e cạnh và v đỉnh và e 2 v, thì đồ thị đó phải chứa một
chu trình.

Lưu ý rằng đồ thị có được kết nối hay không không quan trọng.

Bây giờ chúng ta có thể hoàn thành Ví dụ 4. 1.1, bài toán về các cians toán học chợp mắt. Đồ thị được

đề cập có 10 cạnh và 10 đỉnh. Bởi 4. 1 .3, nó phải chứa một chu trình.

Đường Eulerian và Hamilton

Bài toán 2. 1 .26 trang 24 có một công thức lý thuyết đồ thị đơn giản:

Tìm các điều kiện trên một đồ thị liên thông (hoặc nhiều đồ thị) để có thể
đi một con đường đi qua mọi cạnh đúng một lần.4

Những con đường như vậy được gọi là Eulerian, để vinh danh 1 nhà toán học Thụy Sĩ ở thế kỷ 8, người đầu tiên

nghiên cứu đồ thị một cách chính thức. Đây là hai ví dụ đã xuất hiện trong Bài toán 2. 1 .26.

Một B

Đồ thị A (các đỉnh không được đánh dấu bằng dấu chấm mà chỉ đơn giản là nơi các đoạn thẳng giao nhau) có

đường đi Eulerian, trong khi đồ thị B thì không. Nếu bạn vẽ đủ đồ thị (và nhiều đồ thị), bạn không thể tránh

khỏi việc tập trung vào các đỉnh có mức độ lẻ. Gọi v là đỉnh có bậc lẻ trong đồ thị có đường đi Eulerian.

Có ba trường hợp:

1. Đường dẫn có thể bắt đầu tại v.

2. Đường dẫn có thể kết thúc tại v.

3. Đường dẫn bắt đầu và kết thúc ở nơi khác, hoặc là một đường dẫn đóng (không có điểm bắt đầu hoặc

kết thúc). Điều này là không thể, bởi vì bất cứ khi nào đường dẫn đi vào v dọc theo một cạnh, nó

sẽ cần phải thoát ra v dọc theo một cạnh khác. Điều này có nghĩa là d (v) là chẵn.

4 Ngôn ngữ chính xác hơn giúp phân biệt giữa "lối đi" tránh các cạnh lặp lại, được gọi là "đường mòn" với lối đi
tránh các đỉnh lặp lại, được gọi là "đường đi". Ở đây không cần có những phân biệt này, nhưng để thảo luận rất rõ ràng,
hãy xem Chương 9 của [15].
Machine Translated by Google
114 CHƯƠNG 4 BA CHIẾN THUẬT ROSSOVER C QUAN TRỌNG

Do đó, nếu một đồ thị có đường đi Eulerian, nó phải có hoặc không hoặc đúng hai đỉnh bậc lẻ. Thực tế,

đây là điều kiện cần và đủ. Chính xác hơn,

Một đồ thị được kết nối (hoặc nhiều đồ thị 5) sở hữu một đường Eulerian nếu và
chỉ khi nó không có hoặc chính xác hai đỉnh bậc lẻ. Trong trường hợp trước
đây, đường dẫn là một đường dẫn đóng. Trong trường hợp thứ hai, đường đi phải
bắt đầu và kết thúc ở các đỉnh bậc lẻ.

Có thể chứng minh điều này bằng quy nạp, và thực sự, lập luận dưới đây có thể dễ dàng được viết lại

như một bằng chứng quy nạp. Nhưng sẽ hướng dẫn hơn nhiều nếu trình bày một kiểu lập luận mới, một bằng

chứng thuật toán , trong đó chúng tôi đưa ra một công thức chung để xây dựng một đường Eulerian.

Trước hết hãy xem xét một đồ thị có chính xác hai đỉnh bậc lẻ, chúng ta sẽ gọi là s và f. Chúng

ta hãy thử vẽ một con đường Eulerian một cách ngây thơ, bắt đầu từ s. Chúng ta sẽ đi du lịch một cách

ngẫu nhiên, nghĩ rằng chúng ta không thể thua: nếu chúng ta nhập một đỉnh bậc chẵn, thì chúng ta có

thể rời khỏi nó, và đỉnh này sẽ không còn lại các cạnh chưa được làm sáng hoặc một số chẵn trong số

chúng, trong trường hợp đó chúng ta có thể du lịch qua sau.

Nhưng điều này không hoàn toàn hiệu quả. Hãy xem xét đồ thị sau (thực ra, nó là một đồ thị đa,

nhưng chúng ta sẽ chỉ sử dụng thuật ngữ "đồ thị" lúc này; không nên có bất kỳ sự nhầm lẫn nào).

Các cạnh được gắn nhãn bằng chữ hoa và các đỉnh được gắn nhãn bằng chữ thường.

B
S ..... f
.... ---, -

G
L

Bắt đầu từ đỉnh s, điều gì sẽ xảy ra nếu chúng ta đi ngang qua các cạnh A, B, C, D, E, F, G, H theo

thứ tự? Chúng ta sẽ bị mắc kẹt ở đỉnh f, không có cách nào để "quay ngược lại" và đi qua các cạnh khác.

Trong trường hợp này, chúng ta hãy tạm thời loại bỏ các cạnh mà chúng ta đã đi. Chúng ta còn lại đồ

thị con chứa các cạnh I, J, K, L. Đồ thị con này có bốn đỉnh, mỗi đỉnh có tung độ chẵn. Vì đồ thị ban

đầu được kết nối nên đồ thị con "cắt" một số cạnh mà chúng ta đã loại bỏ, ví dụ, tại đỉnh có nhãn u.

Bây giờ chúng ta hãy áp dụng thuật toán "ngây thơ" cho đồ thị con, bắt đầu từ u. Chúng tôi đi qua, theo

thứ tự, J, K, L, I. Chúng tôi đã quay trở lại u, và đó không phải là ngẫu nhiên. Vì tất cả các đỉnh

của đồ thị con của chúng ta đều bằng nhau, chúng ta không thể bị mắc kẹt trừ khi chúng ta quay trở lại

điểm xuất phát của mình.

Vì vậy, bây giờ chúng ta có thể thực hiện "phẫu thuật tái tạo" trên đường ban đầu của mình và

nhận được đường Eulerian cho toàn bộ biểu đồ.

5
Như đã đề cập trước đó, chúng tôi giả định rằng đồ thị không phải là đồ thị đa đồ thị. Khi điều tra các đường đi của Eulerian,

nhiều biểu đồ có thể có liên quan, nhưng hầu như đây là vị trí duy nhất trong văn bản này có chúng.
Machine Translated by Google

4.1 G RAPH LÝ THUYẾT 115

1. Bắt đầu từ s, như trước, và di chuyển dọc theo các cạnh A, E, C cho đến khi chúng ta đạt đến đỉnh u.

2. Bây giờ đi dọc theo đồ thị con (các cạnh J, K, L, I), quay trở lại u.

3. Kết thúc chuyến đi dọc theo các cạnh D, E, F, G, H, đạt đến đỉnh f.

Phương pháp này sẽ hoạt động chung. Chúng ta có thể phải lặp lại bước "loại bỏ các cạnh đã
đi và di chuyển qua đồ thị con" nhiều lần (vì chúng ta có thể đã bị mắc kẹt trở lại điểm
bắt đầu mà không đi qua tất cả các cạnh của đồ thị con), nhưng vì đồ thị là hữu hạn, cuối
cùng chúng ta sẽ hoàn thành.
_

4. 1.4 Nếu bạn không bị thuyết phục về lập luận trên, hãy thử thuật toán trên đồ thị đa
phương tiện thấp. Sau đó, bạn sẽ hiểu các thuật toán.

4.1.5 Đồ thị có hướng (còn gọi là đồ thị ) là một đồ thị (hoặc nhiều đồ thị) trong đó mỗi
cạnh cho một hướng (thường được biểu thị bằng một mũi tên). Nói cách khác, đồ thị có hướng
giống như một mạng lưới các đường một chiều. Tìm điều kiện cần và đủ để đồ thị có hướng hoặc
đồ thị đa có đường đi Eulerian.

"Đối ngẫu" của một đường Eulerian là một đường Hamilton (được đặt tên theo 1 nhà toán
học người Ireland ở thế kỷ 9), một đường đi mà mỗi đỉnh chính xác một lần. Nếu đường đi bị
đóng, nó được gọi là chu trình Hamilton. Trong khi các đường đi Eulerian sở hữu một "lý
thuyết hoàn chỉnh", thì rất ít thông tin về các đường đi Hamilton. Hiện tại, điều kiện cần
và đủ của đường đi Hamilton vẫn chưa được biết rõ. Điều này là không may, bởi vì nhiều vấn
đề thực tế liên quan đến đường đi Hamilton. Ví dụ, giả sử chúng ta muốn mọi người ngồi quanh
bàn sao cho không ai ngồi cạnh người mà cô ấy không thích.
Sau đó, chúng ta có thể tạo một biểu đồ trong đó mọi người là đỉnh và chúng ta kết nối các
cạnh là bạn đôi. Một đường đi Hamilton, nếu có, sẽ cho chúng ta một sơ đồ chỗ ngồi. Nhiều
vấn đề liên quan đến lập lịch và tối ưu hóa đường dẫn mạng có thể được đúc kết lại khi tìm
kiếm đường đi Hamilton.
Sau đây là một phát biểu khá yếu đưa ra một điều kiện đủ cho các đường đi Hamilton.

4.1.6 Cho G là một đồ thị (không phải là một đa đồ thị) với v đỉnh. Nếu mỗi đỉnh có tung độ
nhỏ nhất là v / 2 thì G có chu trình Hamilton.
Machine Translated by Google

116 CHƯƠNG 4 BA CHIẾN THUẬT ROSSOVER C QUAN TRỌNG

Tuyên bố này là yếu, bởi vì giả thuyết quá mạnh. Ví dụ, giả sử rằng G có 50 đỉnh. Khi đó chúng ta
cần mỗi đỉnh có bậc ít nhất là 25 để kết luận rằng tồn tại một đường Hamilton.

Chúng tôi mong bạn chứng minh 4. 1 .6; lưu ý rằng một trong những điều đầu tiên bạn cần làm
là chỉ ra rằng giả thuyết buộc G phải được kết nối.

Hai người đàn ông của Tây Tạng

Mục tiêu của chúng tôi không phải là một nghiên cứu toàn diện về lý thuyết đồ thị, mà chỉ đơn
thuần là giới thiệu về chủ đề này, để cung cấp cho bạn một chiến thuật giải quyết vấn đề mới và
giúp bạn có thể suy nghĩ về việc lặp lại các vấn đề dưới dạng đồ thị bất cứ khi nào có thể. Nếu
bạn muốn tìm hiểu thêm về chủ đề này, có một tài liệu rất lớn, nhưng [18], [34], những
và [43]
nơiđều
tuyệt

vời để bắt đầu (đặc biệt là [18]).
Chúng ta sẽ kết thúc phần này với một bài toán kinh điển mà thoạt đầu dường như không liên
quan gì đến đồ thị.

Ví dụ 4.1.7 Hai người đàn ông của Tây Tạng. Hai người đàn ông nằm ở hai đầu đối diện của một dãy
núi, ở cùng độ cao. Nếu dãy núi không bao giờ giảm xuống dưới độ cao bắt đầu này, thì liệu hai
người có thể đi bộ dọc theo dãy núi và đến được nơi xuất phát của nhau, trong khi luôn ở cùng độ
cao không?
Đây là một ví dụ về "dãy núi". Không mất tính tổng quát, nó là "đoạn thẳng tuyến tính", tức
là, bao gồm các đoạn thẳng. Vị trí xuất phát của hai người đàn ông được biểu thị bằng hai dấu chấm.

Lúc đầu, nó có vẻ không quá khó. Miễn là hợp pháp để đi lùi, thì việc hai người đàn ông ở cùng
một độ cao là điều khá dễ dàng. Hãy để chúng tôi gắn nhãn các vị trí "thú vị" trên phạm vi (những
vị trí có độ cao bằng các đỉnh và đáy) bằng các chữ cái.
Machine Translated by Google

4.1 G RAPH LÝ THUYẾT 117

Khi đó chấm đen đi từ a đến c, còn chấm trắng đi từ s đến q. Tiếp theo, chấm đen đi lùi từ c
đến b trong khi chấm trắng đi từ q đến p, v.v. Khá dễ dàng để viết ra trình tự chính xác của
các bước tiến và lùi cần thực hiện.
Nhưng tại sao nó hoạt động? Và làm thế nào chúng ta có thể đảm bảo rằng nó sẽ luôn hoạt
động, ngay cả đối với những dãy núi thực sự phức tạp (miễn là dãy không có bất kỳ thung lũng
nào thấp hơn độ cao ban đầu)? Trước khi đọc tiếp, hãy dành một chút thời gian để cố gắng phát
triển một lập luận thuyết phục . Nó không dễ! Sau đó, bạn sẽ thích giải pháp lý thuyết đồ thị
của chúng tôi hơn thế nữa.

Giải pháp: Như trong sơ đồ trên, hãy dán nhãn tất cả các vị trí "thú vị". Chúng ta hãy
gọi tập hợp này là I, vì vậy trong ví dụ của chúng ta, I = {a, b, c, ..., s}. Khi các chấm di
chuyển, chúng ta có thể theo dõi các vị trí chung của chúng bằng một cặp có thứ tự có dạng (x,
y), trong đó x chỉ vị trí của chấm đen và y cho biết vị trí của chấm trắng. Sử dụng ký hiệu
này, tiến trình của hai dấu chấm có thể được viết tắt là

(a, s) --- + (c, q) --- + (b, p) --- + (e, m) --- + (I, l) --- +

••• - - + (s, a), trong đó cấu hình cuối cùng của (s, a) chỉ ra rằng hai dấu chấm đã đổi chỗ cho nhau.

Bây giờ hãy xác định một đồ thị r có các đỉnh là tất cả các cặp có thứ tự (x, y), trong
đó x, y E I và x và y ở cùng một đường cao. Nói cách khác, các đỉnh của r bao gồm tất cả các
cấu hình hợp pháp có thể có về vị trí của hai dấu chấm, mặc dù có thể một số cấu hình này
không thể đạt được từ các vị trí bắt đầu. Chúng ta sẽ nối hai đỉnh bằng một cạnh nếu có thể
di chuyển giữa hai khẩu phần cấu hình trong một "bước". Nói cách khác, đỉnh (a, s) không được
nối với (c, q), nhưng chúng ta nối (a, s) với (b, r) và (b, r) với (c, q). Đây là một bức
tranh không đầy đủ của r, sử dụng một hệ tọa độ [vì vậy cấu hình bắt đầu (a, s) nằm ở phần
trên bên trái]. Hình này thiếu nhiều đỉnh [ví dụ, (a, a), (b, b), (c, c), ...] và không phải
tất cả các cạnh đều được vẽ từ các đỉnh được hình.

Nếu chúng ta có thể chỉ ra rằng có một đường dẫn từ (a, s) đến (s, a), chúng ta đã hoàn thành.

[Trên thực tế, đường dẫn từ (a, s) đến (j, j) thực hiện một mẹo nhỏ, vì đồ thị là đối xứng.] Hãy xác

minh các dữ kiện sau.

1. Các đỉnh duy nhất của r có bậc 1 là (a, s) và (s, a).

2. Nếu một đỉnh có dạng (đỉnh, đỉnh), nó có bậc 4. Ví dụ, (e, m) có


độ 4.

3. Nếu một đỉnh có dạng (đỉnh, hệ số góc), nó có bậc 2. Một ví dụ là (e, i).

4. Nếu một đỉnh có dạng (hệ số góc, hệ số góc), nó có bậc 2. Một ví dụ là (d, n).
Machine Translated by Google

118 CHƯƠNG 4 CHIẾN THUẬT ROSSOVER C QUAN TRỌNG

5. Nếu một đỉnh có dạng (đỉnh, đáy) thì nó bị cô lập (có bậc bằng 0). Đỉnh (g, q) là một ví dụ

về điều này.

Bây giờ hãy xem xét thành phần được kết nối của r có chứa đỉnh (a, s). Đây là một đồ thị con của r

[nó không phải là tất cả của r, vì (g, q) và (q, g) là biệt lập]. Theo bổ đề bắt tay (trang 1 1 1),
tổng các bậc của các đỉnh của đồ thị con này phải là số chẵn. Vì hai đỉnh duy nhất có bậc lẻ là (a,

s) và (s, a) nên đồ thị con này cũng phải chứa (s, a) . Do đó, có một đường dẫn từ (a,Lập
s) luận
đến (s,
này a).

chắc chắn sẽ khái quát cho bất kỳ dãy núi nào, vì vậy chúng tôi đã hoàn thành.
_

- .. -
..., .. - 1 - .. r .. -, .. .. 1- .. T .. 1 - .. � ..., .. - 1 - .. r .. -Tôi ".., - .., .. - 1 - .. r "... , ,
S

r
.. .. ....
Tôi Tôi Tôi Tôi Tôi Tôi Tôi Tôi Tôi Tôi Tôi Tôi Tôi Tôi Tôi

- .. -

q
" ..

.. tôi" .. -. - .. 'i .. - ".. i' .. - ... " . - .. tôi ".. -. .." - .. "'tôi ..
. -.. . . .

P
o L .. J. .. _Tôi .... L .. J .... 1 _ .. L .. .... .. .. 1 .. _ 1 _ .. ... .. .J .. ..
Tôi Tôi

_TÔI

Tôi Tôi Tôi Tôi Tôi Tôi Tôi Tôi Tôi Tôi Tôi

N ..
.- .. "tôi .... , - .. r .. -, .." .- .. , .. -, ".. .- .. 1 .... Tôi

m
----------- - ----
Tôi Tôi Tôi Tôi Tôi Tôi

Tôi Tôi Tôi Tôi Tôi Tôi

k .. r .. -, ...., - .. 'T .. -, ".., ... ., .. .. Tôi

j
Tôi Tôi Tôi Tôi Tôi Tôi

.. j ".. -." .. .Tôi - .. tôi '.. - . " .. tôi" .. "tôi ...

h
g L .. J. .. _Tôi .... L .. .J .. _ 1 _ .. L .. _I .... 1_ .. 1 .. _ .... L .. J ...
Tôi Tôi

f
e

d � .. � ....: - .. T .. -: ....: - .. � .. -: .... � .. � ....: - .. T .. -: ....: - .. � .. -: .... � .. � ....

C r "" "" " , - .. r .. -, .... .- .., .. -, ".. r" .. , ...., - .. r .. -. ...., - .. 'T .. -, "" ... , ... Tôi

b
Tôi Tôi Tôi Tôi Tôi Tôi Tôi Tôi Tôi Tôi Tôi Tôi Tôi Tôi Tôi Tôi Tôi Tôi

.............................................. -_ .. ........................ - _ .. ..
một

abc defgh j k mn op qrs

Cuối cùng, chúng tôi đã giải quyết vấn đề khó khăn này với một phân tích chẵn lẻ rất đơn giản.

Tất nhiên, trước tiên chúng ta cần có cái nhìn sâu sắc về việc xây dựng một đồ thị và bước di chuyển

mấu chốt của việc xác định các đỉnh và cạnh một cách rất thông minh. Đạo đức của câu chuyện: bất cứ

điều gì cũng có thể được phân tích bằng biểu đồ!

Các vấn đề và bài tập Trong những

bài toán này, một đồ thị không phải là một đồ thị nhiều (không có vòng lặp hoặc nhiều cạnh) trừ khi được nêu cụ thể
như vậy.

4.1.8 Chứng tỏ rằng mọi đồ thị đều chứa hai đỉnh có tung khác. (Giả sử rằng "tình bạn" là lẫn nhau; tức là, nếu
độ bằng nhau. bạn là bạn của tôi thì tôi phải là bạn của bạn.)

4.1.9 Với sáu người, hãy chứng tỏ rằng ba người là bạn 4.1.10 Mười bảy người đang dự tiệc. Nó chỉ ra rằng đối
mu tual, hoặc ba người hoàn toàn xa lạ với một người với mỗi cặp người có mặt, chính xác một trong số
Machine Translated by Google
4.1 G RAPH LÝ THUYẾT 119

những câu sau luôn đúng: "Họ chưa gặp nhau", "Họ là bạn tốt", 4.1.17 Một quân cờ domino bao gồm hai hình vuông, mỗi hình vuông

hoặc "Họ ghét nhau." Chứng minh rằng phải có một bộ ba (3) người, được đánh dấu bằng 0, 1, 2, 3, 4, 5 hoặc 6 chấm. Đây là một ví dụ.

tất cả đều là những người xa lạ, bạn tốt của nhau hoặc là kẻ thù

của nhau.

4.1.11 Chứng tỏ rằng nếu một đồ thị có v đỉnh, mỗi đỉnh có ít

nhất (v - 1) / 2, thì đồ thị này được liên thông.

4.1.12 Đồ thị có n đỉnh phải có bao nhiêu cạnh để đảm bảo


Xác minh rằng có 28 dominos khác nhau. Có thể sắp xếp tất cả
rằng nó được liên thông?
chúng trong một vòng tròn sao cho các nửa liền kề của những con

4.1.13 Một ngôi nhà lớn chứa một chiếc tivi trong mỗi phòng domino lân cận hiển thị cùng một con số không?

có số cửa ra vào là số lẻ. Chỉ có một lối vào ngôi nhà này.

Chứng tỏ rằng bạn luôn có thể bước vào ngôi nhà này và đến
4.1.18 Liệu một kỵ sĩ có thể đi vòng quanh một bàn cờ tiêu chuẩn
một căn phòng có thiết lập tầm nhìn từ xa.
8 x 8 , bắt đầu và kết thúc tại cùng một ô vuông, trong khi

thực hiện mọi nước đi mà một kỵ sĩ có thể thực hiện trên bàn

4.1.14 Đồ thị hai phía là một trong đó các đỉnh có thể được phân cờ, chính xác một lần? Chúng tôi coi một bước di chuyển sẽ được

chia thành hai tập hợp U, v sao cho mỗi cạnh có một đầu là U và hoàn thành nếu nó di chuyển theo một trong hai hướng.

một đầu là V. Hình dưới đây cho thấy hai đồ thị hai phía. Hình

bên phải là một đồ thị lưỡng hợp hoàn chỉnh và được ký hiệu là
4.1.19 Các thành phố ,e2, ... ,eN được phục vụ bởi các hãng hàng không
K4,3.
el A "A2, ...,CóA".
dịch vụ bay thẳng trực tiếp tại hai thành phố

bất kỳ (của ít nhất một hãng hàng không) và tất cả các hãng hàng

không đều cung cấp dịch vụ theo cả hai chiều. Nếu N � 2 " + 1,

hãy chứng minh rằng ít nhất một trong các hãng hàng không có thể

thực hiện chuyến khứ hồi với số lần hạ cánh lẻ.

4.1.20 (IMO 1991) Gọi G là đồ thị liên thông có k cạnh. Chứng

Chứng tỏ rằng một đồ thị là lưỡng phân khi và chỉ khi nó không có minh rằng các cạnh có thể được gán nhãn I, ... k theo kiểu nào,

chu trình lẻ. đó, sao cho mọi đỉnh có bậc lớn hơn I, nhãn của các cạnh đó

trùng với đỉnh đó có ước số chung lớn nhất là 1.


4.1.15 Một nhóm người chơi một giải đấu cờ vua vòng tròn,

nghĩa là mọi người chơi một trò chơi với những người khác
4.1.21 (USAMO 1989) 20 thành viên của một câu lạc bộ quần vợt lo
đúng một lần (cờ vua là trò chơi một chọi một, không phải
là một môn thể thao đồng đội). Không có trận hòa nào. cal đã lên lịch chính xác 14 trận đấu dành cho hai người với nhau,

với mỗi thành viên chơi ít nhất một trận. Chứng minh rằng trong
(a) Chứng minh rằng luôn có thể sắp xếp các đấu thủ theo cách
lịch trình này phải có một bộ sáu trò chơi với 12 cách chơi riêng
mà người chơi thứ nhất đánh bại người thứ hai, người đánh
biệt
bại người thứ ba, v.v. xuống đến người chơi cuối cùng. Do
ers.
đó, luôn có thể xác định không chỉ là người chiến thắng

mà còn là thứ hạng có ý nghĩa của tất cả người chơi. 4.1.22 Một hình lập phương n được định nghĩa trực quan là đồ thị

bạn nhận được nếu bạn cố gắng tạo một hình lập phương n-chiều

không có dây. Nói một cách chặt chẽ hơn, đó là một đồ thị có 2 "
(b) Đưa ra một phát biểu định lý về đồ thị ở trên.
đỉnh được bao bởi các số nhị phân n chữ số, với hai đỉnh được nối

(c) Bảng xếp hạng này phải là duy nhất? bởi một cạnh nếu các chữ số nhị phân khác nhau đúng một chữ số.

Chứng tỏ rằng với mỗi n � 1, khối lập phương thứ n có chu trình
4.1.16 (USAMO 1981) Mỗi cặp cộng đồng trong một quận được liên
Hamilton.
kết trực tiếp với nhau bằng chính xác một phương tiện giao thông:

xe buýt, xe lửa hoặc ô tô. Tất cả ba phương thức vận tải đều được 4.1 .23 Xét một hình lập phương 3 x 3 x 3 được tạo ra từ 27 hol

sử dụng trong quận mà không có cộng đồng nào được phục vụ bởi cả ống con thấp. Các ống con được kết nối với nhau bằng các cửa trên

ba phương thức và không có ba cộng đồng nào được liên kết theo mặt của chúng (vì vậy mỗi khối con đều có sáu cửa, mặc dù tất

cặp theo cùng một phương thức. Xác định số lượng mối quan hệ cộng nhiên một số khối có cửa mở ra "bên ngoài"). Có thể bắt đầu ở
đồng tối đa trong quận. khối trung tâm và thăm tất cả các khối khác chính xác một lần

không?
Machine Translated by Google

1 20 CHƯƠNG 4 CÁC CHIẾN THUẬT QUAY LỐI QUAN TRỌNG CỦA TH REE

4.1.24 Nếu bạn đặt các chữ số 0, I, I, 0 theo chiều kim đồng hồ Bài toán hình chữ nhật của Bruijn (Ví dụ 3.4.1 1 trang 98).

trên một vòng tròn, bạn có thể đọc bất kỳ số nhị phân có hai chữ

số nào từ 00 đến II bằng cách bắt đầu từ một chữ số nhất định và
4.1.27 (Olympic Toán học Vùng Vịnh 2005)
sau đó đọc theo chiều kim đồng hồ. Nói chung là có thể làm được không?
Có 1000 thành phố ở đất nước Euleria, và một số cặp thành

4.1.25 Trong một nhóm có chín người, một người biết hai trong phố được nối với nhau bằng những con đường đất. Có thể dễ

số những người kia, hai người biết bốn người khác, bốn người dàng đi từ bất kỳ thành phố nào đến bất kỳ thành phố nào

biết năm người khác và hai người còn lại mỗi người biết sáu khác bằng cách đi dọc theo những con đường đất này. Chứng

người khác. Chứng tỏ rằng có ba người đều quen biết nhau. minh rằng chính quyền Euleria có thể lát một số con đường

đất này để thành phố ev ery sẽ có một số lẻ đường trải nhựa

dẫn ra khỏi thành phố.


4.1.26 Phát triển sơ đồ lý thuyết đồ thị của De

4.2 Số phức

Từ lâu, bạn đã học cách vận dụng các số phức C, tập hợp các số có dạng a + bi,
trong đó a, b là thực và i = R. Điều bạn có thể chưa học là số phức là ước mơ của
các nghệ sĩ chéo: giống như ánh sáng, tồn tại gần giống như sóng và hạt, các số
phức vừa là đại số vừa là hình học. Bạn sẽ không nhận ra toàn bộ sức mạnh của chúng
cho đến khi bạn cảm thấy thoải mái với bản chất hình học, vật lý của chúng. Điều
này trong tum sẽ giúp bạn trở nên thông thạo trong việc chuyển đổi giữa đại số và
hình học trong một loạt các bài toán.
Chúng tôi sẽ phát triển các tính chất cơ bản của số phức dưới đây chủ yếu là
một chuỗi các bài tập và bài toán. Phần này ngắn gọn, chỉ nhằm mục đích giúp bạn mở
rộng tầm mắt với một số khả năng thú vị.

Hoạt động cơ bản

4.2.1 Ký hiệu cơ bản và biểu diễn số phức. Một cách hữu ích để mô tả số phức là
thông qua mặt phẳng Gaussian hoặc Argand . Lấy mặt phẳng Descartes thông thường,
nhưng thay thế các trục x- và y bằng các trục thực và trục ảo, tương ứng. Ta có thể
xem mỗi số phức z = a + bi là một điểm trên mặt phẳng này có tọa độ (a, b). Ta gọi
a là phần thực của z và viết a = Re z. Tương tự, phần ảo Im z bằng b. Chúng ta cũng
có thể coi Re z và Imz là các thành phần thực và ảo của vectơ bắt đầu tại gốc và
kết thúc tại (a, b). Do đó số phức z = a + bi có nghĩa kép: nó vừa là điểm có tọa
độ (a, b) , vừa là vectơ bắt đầu tại gốc và kết thúc tại (a, b).
Tuy nhiên,
hãy nhớ rằng một vectơ có thể bắt đầu ở bất kỳ đâu, không chỉ tại điểm gốc và điều
xác định một vectơ duy nhất là magnitUde và hướng của nó. Độ lớn của số phức z = a
+ bi là

Để biết thêm thông tin, chúng tôi đặc biệt mong bạn đọc ít nhất vài chương đầu tiên của khẩu phần truyền cảm hứng chính

của chúng tôi cho phần này, Phân tích phức hợp thị giác của Tristan Needham [2 9]. Cuốn sách hấp dẫn này rất thú vị khi

đọc, được minh họa đẹp mắt và chứa hàng tá thông tin chi tiết về hình học mà bạn sẽ không tìm thấy ở đâu khác.
Machine Translated by Google

4.2 CÁC SỐ LINH KIỆN 1 21

tất nhiên là độ dài của vectơ từ điểm gốc đến (a, b). Các thuật ngữ khác cho độ lớn là môđun
và giá trị tuyệt đối, cũng như thuật ngữ không chính thức hơn "độ dài", mà chúng tôi thường sử
dụng. Hướng của vectơ này được quy ước là góc mà nó tạo với trục hoành (thực), được đo ngược
chiều kim đồng hồ. Đây được gọi là đối số của z, ký hiệu là arg z. Một cách không chính thức,
chúng tôi cũng gọi đây là "góc" của z. Nếu 0 = arg z và r = Izl, chúng ta có

z = r (cos 0 + i sin 0).

Đây được gọi là dạng cực của z. Viết tắt tiện dụng của cos 0 + i sin 0 là Cis 0; do đó chúng
tôi viết

z = r Cis 0.

Ví dụ (tất cả các góc đều tính bằng radian),

3
57 = 57 Cis O, - 1 2i = 1 2 Cis
;, 1 + i = VlCiS �.

a + bi
bi

một

4.2.2 Sự kết hợp. Nếu z = a + bi, chúng ta xác định liên hợp của z là

z = a - bi.

Về mặt hình học, z chỉ là phản xạ của z về trục thực.

4.2.3 Phép cộng và phép trừ. Các số phức thêm "theo thành phần", tức là,

(a + bi) + (c + di) = (a + c) + (b + d) i.

Về mặt hình học, phép cộng số phức tuân theo "quy tắc hình bình hành" của vector ad
dition: Nếu z và w là các số phức được xem như là vectơ thì tổng z + w của chúng là
đường chéo của hình bình hành với các cạnh z và w với một điểm cuối tại gốc tọa độ.
Giống như khôn ngoan, sự khác biệt z - w là một vectơ có cùng độ lớn và hướng với
Z,•.
vectơ có điểm bắt đầu tại w và điểm kết thúc tại z. Do đó, nếu các số phứctổng •Z2Zncó
l,bằng là

0, khi được vẽ dưới dạng vectơ và được đặt từ đầu đến cuối , chúng tạo thành một đa
giác đóng.
Machine Translated by Google

1 22 CHƯƠNG 4 BA CHIẾN THUẬT ROSSOVER C QUAN TRỌNG

4.2.4 Phép nhân. Tất cả các thao tác đại số của số phức đều tuân theo các quy tắc thông thường,
2
với điều kiện bổ sung là j = - 1. Do đó, ví dụ,

(2 + 3i) (4 + 5i) = 8+ l 2i + l Oi + l5P = -7 + 22i.

Việc sử dụng đơn giản các nhận dạng lượng giác xác minh rằng nếu z = rCis a và w = s
Cis f3, thì

zw = (rCis a) (sCis f3 ) = rs Cis (a + f3);

I E,

Độ dài của zw là tích của độ dài của z và w, và góc của zw là tổng của
các góc của z và w.

Đạo hàm lượng giác này là một bài tập hay, nhưng không thực sự sáng tỏ. Nó không thực
sự cho chúng ta biết lý do tại sao phép tính mUltiplication của các số phức lại có
tính chất hình học thỏa mãn này. Đây là một cách khác để xem nó. Chúng ta sẽ làm một
trường hợp cụ thể: hành động hình học của mUltiplies bất kỳ số phức z nào bằng 3 +
4i. Ở dạng phân cực, 3 + 4i = 5 C là e, trong đó e = arctan (4/3) , xấp xỉ 0,93 radian.

1. Vì i (a + bi) = -b + ai, nó theo sau (vẽ một bức tranh!)

"Phép nhân với i" có nghĩa là "xoay theo n / 2 ngược chiều kim đồng hồ: '

2. Tương tự, nếu a là thực,

"Phép nhân với a" có nghĩa là "mở rộng theo hệ số của a."

Ví dụ, phép nhân một vectơ phức z với 3 sẽ tạo ra một vectơ mới có cùng hướng
nhưng dài gấp ba lần. Phép nhân z với 1 15 tạo ra một vectơ có cùng phương, nhưng
chỉ dài bằng 1 15 .

3. Nhân z với 3 + 4i = 5 Cis e có nghĩa là z được biến thành (3 + 4i) z = 3z + 4iz.


Đây là tổng của hai vectơ, 3z và 4iz. Vectơ đầu tiên chỉ là z được mở rộng theo
hệ số 3. Vectơ thứ hai là z quay 90 độ ngược chiều kim đồng hồ, sau đó được mở
rộng theo hệ số 4. Vì vậy, kết quả ròng (vẽ hình!!) Sẽ là một vectơ có độ dài 5
1z1 và góc e + arg z.
Machine Translated by Google

4.2 CÁC SỐ LINH KIỆN 1 23

Rõ ràng lập luận này tổng quát thành phép nhân với bất kỳ số phức nào.

Phép nhân với số phức rCis e là một rota tion ngược chiều kim đồng
hồ với e sau đó kéo dài bởi thừa số r.

Vì vậy, chúng ta có cách thứ ba để nghĩ về số phức. Mọi số phức đồng thời là một điểm,
một vectơ và một phép biến đổi hình học, cụ thể là phép quay và kéo dài ở trên!

4.2.5 Phân chia. Bây giờ, thật dễ dàng để xác định ý nghĩa hình học của z / w, trong đó z =
rCis a và w = sCis f3 . Cho v = z / w = t Cis y. Khi đó vw = z. Sử dụng các quy tắc cho phép
nhân, chúng ta có

ts = r, y + f3 = a,

và do đó

t = r / s, y = a-f3.

Như vậy

Ý nghĩa hình học của phép chia cho rCis e là chuyển động quay theo chiều kim
đồng hồ bởi e (quay ngược chiều kim đồng hồ bởi - e) sau đó kéo dài theo hệ số 1 / r.

4.2.6 Định lý De Moivre. Một hệ quả dễ dàng của các quy tắc nhân và chia là nhận
dạng lượng giác đáng yêu này, đúng với mọi số nguyên n, dương hoặc âm và bất kỳ
số e thực nào:

(cos e + i sin et = cosne + i sinne.

4.2.7 Dạng lũy thừa. Hành vi đại số của đối số trong phép nhân và phép chia sẽ nhắc nhở bạn về phép

lũy thừa. Thật vậy, công thức của Euler nói rằng

Cis e = eiO ,

trong đó e = 2,7 1828 . . . là cơ số logarit tự nhiên quen thuộc mà bạn đã từng gặp trong giải
tích. Đây là một ký hiệu hữu ích, ít rườm rà hơn Cis e, và khá sâu sắc, bên cạnh đó. Hầu hết

các sách giáo khoa về giải tích và phân tích phức tạp đều chứng minh công thức của Euler bằng
cách sử dụng chuỗi lũy thừa cho e, sin x và cosx, nhưng điều này không thực sự mang lại nhiều
hiểu biết về lý do tại sao nó đúng. Đây là một vấn đề sâu sắc và thú vị nằm ngoài phạm vi của
cuốn sách này. Tham khảo ý kiến [29] để được điều trị triệt để và thử Bài toán 4.2.29 bên dưới.

4.2.8 Bài tập thực hành dễ dàng. Sử dụng phần trên để xác minh những điều sau đây.

(a) Izwl = Izl lwl và Iz / wl = Izl /

Iwl. (b) Re z = ! (z + z) và 1m z = d: z (z -
2
.
z). (c) zz = Iz1

(d) Trung điểm của đoạn thẳng nối các số phức z, w là (z + w) / 2.


Đảm bảo bạn có thể hình dung ra điều

này! (e) z + w = z + w và zw = zw và z / w = z / w.
lO
(f) (1 + i) = 32i và (1 - iv'3) 5 = 16 (1 + iV3).

2 = i.
(g) Cho thấy bằng cách vẽ hình mà z = {} (1 + i) thỏa mãn z
Machine Translated by Google
1 24 CHƯƠNG 4 BA CHIẾN THUẬT ROSSOVER C QUAN TRỌNG

(h) Quan sát rằng nếu a E C và r E � thì tập hợp các điểm a + rei t, trong đó 0 ::; t ::; 21t ',
mô tả một đường tròn có tâm tại a và bán kính r.

(i) Cho a, bE C. Chứng minh-bằng hình ảnh, nếu có thể-rằng diện tích tam giác có các đỉnh tại O,
a và b bằng một nửa giá trị tuyệt đối của Im (ab) .

4.2.9 Các vấn đề thực hành ít dễ dàng hơn. Những điều sau đây có phần khó khăn hơn.
Hãy vẽ những bức tranh cẩn thận, và đừng để bị dụ dỗ bởi đại số (ngoại trừ việc kiểm tra công việc
của bạn). (a) Thật dễ dàng để "đơn giản hóa"

a-bi
a + bi a2 + b2 bằng

cách nhân tử số và mẫu số với a - bi. Nhưng người ta cũng có thể xác minh điều này mà không
cần bất kỳ tính toán nào. Thế nào? (b) Iz + wi ::; Izl + Iwl, bằng nhau nếu và chỉ khi z và

w có cùng phương hoặc hướngnằm


ngược
trênnhau,
đườngtức
tròn
là đơn
nếu vị;
góc tức
giữalà,
chúng
Izl bằng
= 1. 0Chứng
hoặc tỏ
1t rằng
'. (c) Cho z

I l - zl = 2 sin ( ar; z)

mà không cần tính toán.

(d) Cho P (x) là một đa thức với hệ số thực. Chứng tỏ rằng nếu z là 0 của P (x) thì z cũng là 0;
tức là, các số không của đa thức với hệ số thực nằm trong các cặp liên hợp phức. (e) Không
cần tính toán nhiều, hãy xác định quỹ tích của z cho mỗi

' và
Re ( zz -+ Il -+ ii ) = 0 1m ( zz -+ 1l -+ ii ) = O.

1 0 lO = z
(f) Không giải phương trình, hãy chứng tỏ rằng tất cả chín nghiệm nguyên của (z nằm trên

- I) dòng Re (z) = 1. (g) Chứng tỏ rằng nếu Izl = I thì

(h) Gọi k là hằng số thực và a, b là các số phức cố định. Mô tả


quỹ tích của các điểm z thỏa mãn

tranh luận
= k.
( )z z- -a b

4.2.10 Lưới cho vòng tròn và ngược lại. Các bài toán dưới đây sẽ giúp bạn làm quen
với sự đan xen đáng yêu giữa hình học, đại số và hình học giải tích khi bạn suy ngẫm
về các phép biến đổi phức tạp. Phép biến đổi mà chúng tôi phân tích dưới đây là một
ví dụ về phép biến đổi Mobius. Xem [29] để biết thêm chi tiết. (a) Chứng minh mệnh đề

hình học đơn giản sau (sử dụng tam giác đồng dạng).
Machine Translated by Google
4,2 COMPLEX NU MBERS 1 25

Gọi AB là đường kính oj là đường tròn có đường kính k. Xét tam giác vuông
ABC có góc vuông tại B. Gọi D là giao điểm (D thuộc A) oj đường thẳng AC
với đường tròn. sau đó

2
QUẢNG CÁO. AC = k .

z
(b) Xét phép biến hình J (z) = _. Đây là một hàm có phức do z - 1 chính và phạm vi. Đây
là kết quả đầu ra của máy tính (sử dụng
Mathematica) về những gì J (z) thực hiện với miền (O: S Re (z) : s 2, -2: S 1m (z) :
s 2). Biểu đồ cho thấy J (z) trans tạo thành các đường lưới hình chữ nhật như thế
nào. Lưu ý rằng J dường như biến đổi lưới Descartes hình cầu trực tràng của mặt phẳng
Gaussian thành các đường tròn, tất cả đều tiếp tuyến tại 1 [mặc dù điểm 1 không nằm
trong phạm vi, cũng không phải là một vùng lân cận khoảng 1, vì z phải rất lớn để J
(z) gần bằng 1].

Xác minh hiện tượng này một cách rõ ràng, không tính toán, đối với trục tưởng tượng.
z
Chứng minh rằng hàm số J (z) = _ biến trục ảo thành đường tròn z - 1 có tâm
tại (�, 0) và bán kính �. Làm điều này theo

hai cách.
1. Về mặt đại số. Cho một điểm trên trục ảo là nó, với t là một số thực bất kỳ.
Tìm phần thực và phần ảo của JUt); tức là, đưa J (it) về dạng x + yi. Sau đó
2
chứng tỏ rằng (x - 1/2) 2 + y = 1/4.

2. Về mặt hình học. Đầu tiên, chỉ ra rằng J (z) là hợp phần của bốn ánh xạ, theo
thứ tự sau:

Z f ---> '- Z f ---> -Z, z f ---> z + l, Z f ---> - .


Z Z
Machine Translated by Google

1 26 CHƯƠNG 4 BA CHIẾN THUẬT QUAY LỐI QUAN TRỌNG

Nói cách khác, nếu bạn bắt đầu bằng z và sau đó đảo ngược, phủ định, dịch theo 1 và sau

đó đảo ngược lại, bạn sẽ nhận được J (z) .


Tiếp theo, sử dụng phép "phân tích" J (z) này cộng với bổ đề hình học mà bạn đã chứng

minh ở trên để chỉ ra rằng mọi điểm z trên trục ảo đều được ánh xạ thành một đường tròn

có đường kính 1 với tâm là 1/2. Vẽ một sơ đồ tốt! (c) "Ngược lại" của (b) là đúng: J

(z) không chỉ biến đổi lưới Descartes thành các đường tròn, nó còn biến đổi một số đường tròn

nhất định thành các đường lưới Descartes! Xác minh điều này một cách rõ ràng cho hình tròn
đơn vị (hình tròn có bán kính 1 tâm tại điểm gốc). Chứng tỏ rằng J (z) biến đường tròn đơn
vị thành đường thẳng đứng gồm tất cả các điểm có phần thực bằng 1/2. Trên thực tế, cho thấy

rằng

1 1 .
J e
( = 2 "-
i8) cũi
(J "2"
2" 1

Như trong (b), hãy làm điều này theo hai cách khác nhau - về mặt đại số và hình học. Đảm bảo

rằng bạn hiển thị chính xác cách vòng tròn đơn vị được ánh xạ vào dòng này. Ví dụ, rõ ràng

là - 1 được ánh xạ đến 1/2. Điều gì xảy ra khi bạn di chuyển ngược chiều kim đồng hồ, bắt đầu
từ -1 dọc theo đường tròn đơn vị?

Rễ của sự thống nhất

Các số không của phương trình y;! L = 1 được gọi là nghiệm nguyên thứ n của sự hợp nhất. Những con

số này có nhiều tính chất đẹp đẽ liên kết với nhau giữa đại số, hình học và lý thuyết số. Một lý do

giải thích cho sự phổ biến của nguồn gốc của sự thống nhất trong toán học là tính đối xứng: rễ của
sự thống nhất, theo một nghĩa nào đó, là biểu hiện của sự đối xứng, như bạn sẽ thấy dưới đây. (Chúng
tôi sẽ giả định một số kiến thức về đa thức và tính tổng. Nếu bạn không chắc chắn về tài liệu này,
hãy tham khảo Chương 5.)

. . - - - ---
, --
, � •
, = co s600 + isin 600
Machine Translated by Google

4.2 CÁC SỐ LINH KIỆN 1 27

4.2. 11 Với mỗi số nguyên dương n, có n nghiệm nguyên thứ n khác nhau, cụ thể là S'3,

2 l
1, S ', S'
... , S'n ,

ở đâu

2n
S '= Cis .
N

Về mặt hình học, căn bậc n của phép hợp nhất là các đỉnh của một n-gon đều nội tiếp trong vòng
. 2: : như trên. Sau đó,
tròn đơn vị (tập {z EC : Izl = I}) với một đỉnh tại 1 4.2.12 Cho S '= Cis

đối với mỗi số nguyên dương n, và đối với mỗi phức


số x,
2
(a) xn - 1 = (x - 1) (x - S ') (x - S' ) " . (x - S 'nl),
l
x n-1 + xn-2 + ... + x + 1 = (x - S') (x - S '2) ... (x - S'n (b ) ),
l
(c) I + S '+ S'2 + ". + S'n = o.

Bạn có thể thấy tại sao (c) đúng mà không sử dụng công thức tính tổng của chuỗi hình học không?

Một số ứng dụng

Chúng ta sẽ kết thúc phần này với một vài ví dụ về cách sử dụng thú vị của num bers phức tạp
trong một số ngành toán học, bao gồm lượng giác, hình học và lý thuyết num ber.

Ví dụ 4.2.13 Tìm công thức của tan (2a).

Giải pháp: Điều này tất nhiên có thể được thực hiện theo nhiều cách, nhưng phương pháp số
phức khá phức tạp và hoạt động dễ dàng với nhiều định danh trig khác. Ý tưởng chính là nếu z = x
+ iy, thì tan (arg z) = y / x. Cho t : = tan a và

z = 1 + nó.

Bây giờ z vuông, và chúng tôi nhận được

2 2
Z = (1 + nó) = 1 - t2 + 2it.

Nhưng

2
2t tan (arg z ) ='1
--2
- t

2
và tất nhiên đối số z = 2a, vì vậy chúng tôi kết luận rằng

2 tan a
tan 2a =
2.a
Tôi - tan •
Ví dụ 4.2.14 (Putnam 1996) Cho Cl và C2 là các đường tròn có tâm cách nhau 10 đơn vị và bán kính
là 1 và 3. Bằng chứng minh, tìm quỹ tích của tất cả các điểm M mà tại đó tồn tại điểm X trên C 1
và Y trên C2 sao cho M là trung điểm của đoạn thẳng XY.
Machine Translated by Google

1 28 CHƯƠNG 4 BA CHIẾN THUẬT ROSSOVER C QUAN TRỌNG

Giải pháp: Giải pháp của chúng tôi minh họa một ứng dụng hữu ích trong việc xem các bers num phức

tạp dưới dạng vectơ để tham số hóa các đường cong trong mặt phẳng. Hãy xem xét trường hợp tổng quát, được

mô tả trong hình bên dưới, của hai đường tròn nằm trong mặt phẳng phức, có tâm lần lượt là a, b và bán kính

u, v . Chú ý rằng a và b là các số phức, trong khi u và v là số thực. Chúng tôi đang giả định, như trong

bài toán ban đầu, v lớn hơn một chút so với

u.

Quỹ tích mà chúng ta tìm kiếm là tập hợp các trung điểm M của đoạn thẳng XY, trong đó X có thể là điểm bất

kỳ trên đường tròn bên trái và Y có thể là điểm bất kỳ trên đường tròn bên phải.

Do đó, X = a + ueit và Y = b + veis, trong đó t và s có thể là bất kỳ giá trị nào giữa 0
và 2n. Chúng ta có


X + Y a + b ueit + ve
M = = +
2 2 2
Hãy để chúng tôi diễn giải điều này một cách hình học, bằng cách cố gắng hiểu những gì

nó là + ve
ue

trông giống như 0 : S s, t : S 2n. Nếu ta sửa s thì P = veis là một điểm trên đường tròn có tâm v là tâm

tại gốc tọa độ. Bây giờ, khi chúng ta thêm ueit vào P, và để t thay đổi trong khoảng từ 0 đến 2n, chúng ta

sẽ nhận được một hình tròn có bán kính u, có tâm là P (được hiển thị dưới dạng đường chấm bên dưới).

o -----' --- ..--- 1

Bây giờ chúng ta hãy thay đổi. Vòng tròn chấm nhỏ sẽ di chuyển dọc theo chu vi của vòng tròn lớn, tạo ra

một hình khuyên hoặc "vòng" điền đầy. Nói cách khác, quỹ tích của các điểm
Machine Translated by Google
Machine Translated by Google

1 30 CHƯƠNG 4 BA CHIẾN THUẬT ROSSOVER C QUAN TRỌNG

Khi đó có một tam giác với độ dài các cạnh là a, b, c.

2. Cho 00 = e 27r: i / 3
. Không mất tính tổng quát, đặt 0 = 0 (tức là đặt tâm của đường tròn tại gốc

phức), và đặt A = 1, B = 00, C = 002 , và

p = z, trong đó z là một điểm tùy ý trong mặt phẳng phức thỏa mãn Izl < 1.

3. Sử dụng (1) để xác minh rằng có một tam giác với độ dài các cạnh được chỉ định. Bạn
sẽ cần phải chọn a, f3 một cách cẩn thận, nhưng bạn nên sử dụng câu nói nổi tiếng của
nhà viết kịch Anton Chekhov để lấy cảm hứng: "Nếu một khẩu súng nằm trên lớp lò sưởi
trong màn đầu tiên, nó phải nổ ở màn thứ ba."

4. Cuối cùng, tính diện tích tam giác, sử dụng công thức đơn giản mà bạn suy ra ở trang 124, và chỉ ra

rằng diện tích này chỉ phụ thuộc vào Izl , khoảng cách từ P đến O.

Ví dụ 4.2.16 Cho m và n là các số nguyên sao cho mỗi số có thể được biểu diễn dưới dạng tổng của hai bình

phương hoàn hảo. Chứng tỏ rằng mn cũng có tài sản này. Ví dụ: 17 = 44 + 1 2 và 13 = 2

+ 32 và chắc chắn là 17 ·

13 = 221 = 142 + 52. (1)

2
Lời giải: Cho m = a + b2 , 2 n = c + d2 , trong đó a, b, c, d là các số nguyên. Bây giờ con

sider sản phẩm

z: = (a + bi) (c + di).

Lưu ý rằng

Izl = l (a + bi) l lc + dil = V (a2 + b2) (c2 + d2).

2 2
Do đó mn = Iz1 . Nhưng Izl sẽ là tổng bình phương của các số nguyên, vì Re z và Imz là các số

nguyên. Điều này không chỉ chứng minh những gì chúng tôi đang tìm kiếm; nó cung cấp cho chúng ta một thuật

toán để tính toán các giá trị ở phía bên phải của các phương trình chẳng hạn như (1).

Ví dụ cuối cùng của chúng tôi là một vấn đề đáng ngạc nhiên cho thấy nguồn gốc của sự thống nhất có thể
được sử dụng để tạo ra một bất biến.

Ví dụ 4.2.17 (Jim Propp) Cho một vòng tròn gồm n bóng đèn, đúng một trong số đó sáng ban đầu, được phép thay

đổi trạng thái của một bóng đèn với điều kiện một bóng đèn cũng thay đổi trạng thái của mọi bóng đèn thứ d

sau nó (trong đó d là aa ước của n nhỏ hơn n), với điều kiện là tất cả n / d bóng đèn ban đầu ở cùng trạng

thái với nhau. Với giá trị nào của n thì có thể bật tất cả các bóng đèn bằng cách thực hiện một chuỗi chuyển

động như vậy?

Giải pháp: Cái nhìn sâu sắc ở đây là nhận ra rằng đây không phải là vấn đề về ánh sáng, mà là về

nguồn gốc của sự thống nhất

2 nl
1 , S, S , . .. , S ,

trong đó S = cos 2 :: + i sin 2 ::. Đặt mỗi đèn trên vòng tròn đơn vị nằm ở gốc của sự thống nhất và không
làm mất tính tổng quát, để đèn ở 1 sáng lúc đầu. Bây giờ, nếu d < n là ước của n và đèn ở

Sa, Sa + d, Sa + 2d ,. . . , Sa + (�-l) d
Machine Translated by Google
4.2 CÁC BỘ PHẬN HỢP LÍ 1 31

có cùng trạng thái thì ta có thể thay đổi trạng thái của n / d đèn này. Tổng của những thứ này là

d
sa + sa + d + sa + 2d + ... + sa + (�-l) d = Sa ( 1 + S + S2d + ... + S (il- 1) d).

Các thuật ngữ trong ngoặc đơn ở phía bên phải tạo thành một chuỗi hình học có thể tính tổng dễ
dàng và phía bên phải đơn giản hóa thành

Sự thật đáng ngạc nhiên này cho chúng ta biết rằng nếu chúng ta cộng tất cả các gốc rễ của sự
thống nhất "trên", thì tổng sẽ không bao giờ thay đổi, vì bất cứ khi nào chúng ta thay đổi trạng
thái của một chùm đèn, chúng sẽ cộng lại bằng không! Tổng ban đầu bằng 1 và mục tiêu là làm cho
tất cả các đèn được bật. Số tiền đó sẽ là

n- l 1 - Sn �
1 + S + S2 + ... + S = =
1 - S 1 - S = 0 = F 1.

Do đó, chúng tôi không bao giờ có thể nhào lộn trên tất cả các đèn! •

Các vấn đề và bài tập

4.2.18 Sử dụng số phức để tính đồng nhất với cos na và 4.2.26 Xét một n-gon đều nội tiếp trong một đường tròn
sin na, với n = 3,4, 5. bán kính I. Tích các độ dài của tất cả n (n - 1) / 2
đường chéo của đa giác là bao nhiêu (tính các cạnh của
4.2.19 Kiểm tra sự hiểu biết của bạn về Ví dụ 4.2. 16.
n- gon)?
Cho biết 17 = 42 + I và 101 = 102 + I, tính nhẩm các số nguyên
2
u, v sao cho 17. 10 I = u2 + v . 4.2.27 (USAMO 1976) Nếu P (x), Q (x), R (x), S (x) đều
là các đa thức sao cho
4.2.20 Chứng minh (không tính toán, nếu bạn có thể!)
5 5 4 3 2 + x
P (x ) + xQ (x ) + x2R (x 5 ) = (x + x + x + I) S (x),
e ' T .
2
e'l + e'S = 2 cos
.. (t - s ) . ( H / ) chứng minh rằng x - I là một thừa số của P (x).

� 4.2.28 Tập hợp các điểm (x, y) thỏa mãn x3 - 3xyZ 2: 3x2y -y3
4.2.21 Chứng tỏ rằng nếu x + = 2cosa, thì với mọi số nguyên
x và x + y = - I là một đoạn thẳng. Tìm chiều dài của nó.
ger n,

4.2.29 (T. Needham) Cố gắng suy ra công thức của Euler


Tôi

� + - = 2cosna. xn
tôi

là e
= cost + isin t theo cách sau:
Il
4.2.22 Tìm (khá) công thức đơn giản của sin a + sin 2a + sin có thể được
(a) Giả sử rằng hàm f (t) = e
3a +. . . + sin na và cosa + cos2a + cos 3a + ··· + cosna.
ferentiated đối với t "theo cách mà bạn (t) =
,
sẽ mong đợi "; nói cách khác, điều đó! ieil.
5 4.2.23 Yếu tố z + z + I. (Lưu ý rằng điều này không tự động, vì phạm vi
của hàm rất phức tạp; bạn cần phải xác định và
4.2.24 Giải ra z6 + z4 + z3 + z2 + I = o.
kiểm tra nhiều thứ, nhưng chúng tôi sẽ tránh điều
4.2.25 Cho n là số nguyên dương. Tìm một biểu thức dạng đó ngay bây giờ - đây là một đối số trực quan!)
đóng cho
(b) Nếu bạn xem biến t là thời gian và hàm f (t) như
. N . 2n . 3n là một đường cong trong mặt phẳng phức, thì phương
sm - sm-sm - ··· sm . (n - tôi) n .
N N N N trình f '(t) = ieil có tỷ lệ
Machine Translated by Google

1 32 CHƯƠNG 4 BA CHIẾN THUẬT ROSSOVER C QUAN TRỌNG

thay đổi cách diễn giải. Nhớ lại rằng mUltiplica tion các đỉnh của một tam giác là các số phức a, h, c,
theo i có nghĩa là "xoay 90 độ ngược chiều kim đồng trọng tâm là (a + h + c) / 3.
hồ." Chứng tỏ rằng điều này ngụ ý rằng f (t) là một
4.2.37 (Hungary 1941) Lục giác ABCDEF nằm trong một hình tròn.
đường tròn.
Các cạnh AB, CD và EF có độ dài bằng bán kính. Chứng minh rằng

(c) Hãy nghĩ về tốc độ tại đó đường cong tròn t > trung điểm của ba cạnh còn lại là đỉnh của một tam giác đều.

-> f (t) này đang được tìm ra, và xác định rằng
eit = cost + isint.

4.2.30 Gọi Ra (e) biểu thị phép biến hình của mặt phẳng quay 4.2.38 ( đề xuất lMO) Gọi n là số nguyên dương có ít nhất hai

mọi vật quanh tâm điểm a bằng e radian ngược chiều kim đồng hồ. thừa số nguyên tố phân biệt. Chứng tỏ rằng có một hoán vị (al,

Chứng minh điều thú vị rằng thành phần của Ra (e) và R b (l / a2,.., An) của (l, 2, ... , N ) sao cho

J) là một phép quay khác Rc ( a). Tìm c, a theo a, h, e, I / J.

2nak
Điều này có đồng ý với trực giác của bạn không?
± k cos = o.
N
k = 1

4.2.31 Chứng tỏ rằng không tồn tại bất kỳ góc tri


4.2.39 Đối với mỗi số nguyên dương n, xác định polyno mial
giác đều trong mặt phẳng có đỉnh là điểm mạng (tọa
độ nguyên).

4.2.32 Chứng tỏ rằng tam giác có các đỉnh a, b, c trong mặt

phẳng phức là cạnh đều khi và chỉ khi


Các số không của Pn nằm bên trong, bên ngoài hay trên vòng

một 2 + h2 + c2 = ah + hc + ca. tròn đơn vị Izl = I?

4.2.40 (Putnam 1998) Gọi s là cung bất kỳ của đơn vị


4.2.33 Tìm điều kiện cần và đủ để + az + h = 0, cộng với 0, tạo
nào đó nằm hoàn toàn trong góc phần tư thứ nhất. Gọi
2 hai gốc của z thành các đỉnh của một tam giác đều.
A là diện tích của vùng nằm bên dưới s và phía trên
trục x và gọi B là diện tích của vùng nằm bên phải
4.2.34 (T. Needham) Vẽ một tứ giác bất kỳ và trên trục y và bên trái s. Chứng minh rằng A + B chỉ phụ
mỗi cạnh vẽ một hình vuông nằm bên ngoài hình bên thuộc vào độ dài cung chứ không phụ thuộc vào vị trí của s.
của tứ giác đã cho. Vẽ các đoạn thẳng nối các tâm
4.2.41 Định lý Ptolemy. Cho a, h, c, d là bốn số phức arbi
của hình vuông op posite. Chứng tỏ rằng hai đoạn
trary. Xác minh danh tính
thẳng này vuông góc và có độ dài bằng nhau.

4.2.35 (T. Needham) Cho tam giác ABC với các điểm P, Q, R nằm (ac) (hd) = (ac) (cd) + (hc) (ad).

ngoài ABC sao cho các tam giác PAC, RCB, QBA đồng dạng với
Áp dụng bất đẳng thức tam giác để suy ra đẳng thức Ptolemy
nhau. Khi đó các trọng tâm (giao điểm của các đường trung trực)
Trong hình học phẳng (không liên quan đến số phức): Với bốn
của ABC và PQR là cùng một điểm.
điểm A, B, C, D bất kỳ trong mặt phẳng,

AC · BD "5. AB · CD + BC · DA,
4.2.36 (T. Needham) Vẽ một tam giác bất kỳ và trên mỗi cạnh vẽ

một tam giác đều nằm bên ngoài tam giác đã cho. Chứng tỏ rằng với đẳng thức nếu và chỉ khi tứ giác ABCD lồi và nội
các tâm của ba tam giác đều này là các đỉnh của một tam giác tiếp một đường tròn. Phần bình đẳng, thường được gọi
đều. (Trọng tâm của một tam giác là giao điểm của các trung là Ptolemy's The orem, là phần khó nhất của bài toán
tuyến của nó; nó cũng là trọng tâm.) Nếu này. Để biết các cách khác để chứng minh điều này,
hãy xem Vấn đề 8.4.30 và 8.5.49.

4.3 Tạo các chức năng

Chiến thuật kết hợp tạo ra các hàm có sức mạnh của nó nhờ hai sự thật đơn giản.

• Khi bạn nhân xn với Xl, bạn nhận được xn + n.


Machine Translated by Google
4.3 CÁC CHỨC NĂNG PHÁT ĐIỆN 1 33

• Kiến thức "cục bộ" về hệ số của đa thức hoặc chuỗi lũy thừa f (x) thường
cung cấp kiến thức "toàn cục" về hành vi của f (x) và ngược lại.

Thực tế đầu tiên là tầm thường, nhưng nó là "động cơ" kỹ thuật làm cho mọi
thứ xảy ra, vì nó liên quan đến phép cộng các số và phép nhân các đa thức. Thực
tế thứ hai sâu hơn và cung cấp động lực để làm những gì chúng ta sắp làm.

Ví dụ giới thiệu
Tuy nhiên, trước khi làm bất cứ điều gì, chúng ta cần xác định chủ đề của mình. Cho dãy
(có thể vô hạn) aO, a), a 2, ..., hàm sinh của nó là ao +

a) x + a2 x2 +. . . .

Dưới đây là một vài ví dụ đơn giản. Chúng tôi giả định rằng bạn có hiểu biết cơ
bản về dãy số, đa thức và các công thức tính tổng đơn giản (Chương 5), tổ hợp
và định lý nhị thức (Chương 6) và chuỗi vô hạn (Chương 5 và 9).
Nếu bạn cần tìm hiểu kỹ về những lĩnh vực này, bạn có thể chỉ cần đọc lướt phần này
ngay bây giờ và sau đó đọc lại nó sau. Chúng tôi không khuyên bạn nên tránh hoàn
toàn phần này, bởi vì ý tưởng tạo ra các hàm rất mạnh mẽ. Bạn càng sớm tiếp xúc với
nó thì càng tốt.

Ví dụ 4.3. 1 Cho I = ao = a) = a2 = ... . Sau đó, tion tạo ra func tương ứng
chỉ là

I + X + � + X3 +. . . .
1
Đây là một chuỗi hình học vô hạn hội tụ đến (xem _, với điều kiện Ixl < 1
Tôi - x

trang 60). Nói chung, chúng tôi không lo lắng quá nhiều về các vấn đề hội tụ với
các hàm tạo. Miễn là chuỗi hội tụ cho một số giá trị, chúng ta thường có thể đạt
được, như bạn sẽ thấy bên dưới.
Chuỗi hình học vô hạn được sử dụng ở trên là phổ biến trong thế giới của các hàm sinh.
Hãy ghi chú lại nó; chúng ta sẽ gọi nó là công cụ chuỗi hình học. Hãy nhớ rằng nó hoạt động
theo cả hai cách: bạn có thể đã thực hành tính tổng các chuỗi hình học vô hạn, nhưng đây là
một ví dụ về hướng ngược lại. Hãy nghiên cứu nó một cách cẩn thận.

_x_ x
= .
2 + x 2 ( 1 - (- 1x) ) = (!)2 (x _! � 2+ �x3 22
_ �x4 +23...)

Ví dụ 4.3.2 Với một số nguyên dương n cố định, xác định dãy ak = (:), k =
0, 1 , 2, ... ,N. Hàm tạo tương ứng là

(2)

bằng định lý nhị thức. Nếu chúng ta cắm x = 1 vào (2), chúng ta sẽ có được danh tính tốt
Machine Translated by Google

1 34 CHƯƠNG 4 BA CHIẾN THUẬT QUAY LỐI QUAN TRỌNG

Tất nhiên, danh tính này có thể được chứng minh theo những cách khác (và bạn nên biết một số cách trong số đó;

nếu không, hãy tham khảo Phần 6.2), nhưng lưu ý rằng phương pháp của chúng tôi vừa dễ dàng lại vừa dễ tổng

quát hóa. Nếu chúng ta cho x = - 1, chúng ta nhận được một danh tính hoàn toàn mới,

Hai ví dụ về việc cắm một giá trị để có danh tính là điển hình của quan điểm "cục bộ + - + toàn

cầu". Trên toàn cầu, hàm sinh chỉ là hàm đơn giản (1 + x) n. Chúng tôi có thể cài đặt bất kỳ giá trị nào

mà chúng tôi muốn. Nhưng mỗi khi chúng tôi cắm một giá trị, chúng tôi nhận được một câu lệnh mới liên

quan đến các hệ số (�) (thông tin cục bộ).


Điều quan trọng là di chuyển tiêu điểm qua lại giữa hàm và các hệ số của nó, để có được thông tin hữu ích.

Ví dụ 4.3.3 Việc cắm các giá trị chỉ là một trong những công việc chung mà chúng ta có thể làm. Hãy thử

phân biệt! Nếu chúng ta phân biệt cả hai mặt của (2), chúng ta nhận được

Bây giờ, nếu chúng ta cắm x = 1, chúng ta sẽ có được danh tính thú vị

Mối quan hệ lặp lại

Cho đến nay, chúng ta vẫn chưa sử dụng thực tế đơn giản được đề cập ở trang 1 32, rằng xn xn = xn + m.

Bây giờ chúng ta hãy làm như vậy, bằng cách sử dụng các hàm tạo để phân tích các quan hệ lặp lại (xem

Phần 6.4 để biết các ví dụ).

Ví dụ 4.3.4 Xác định dãy (an) theo ao = 1 và an = 3an l + 2 với n = 1,2, ....

Tìm một công thức đơn giản cho một.

Giải pháp: Có một số cách để giải quyết vấn đề này; thực sự, cách tiếp cận đơn giản nhất
mà bất kỳ người giải quyết vấn đề nào cũng nên thử trước tiên - là tìm ra một vài thuật ngữ
đầu tiên và đoán. Một số điều khoản đầu tiên là (xác minh!)

1,5, 17, 53, 161 ,485 , ... ,

điều này có thể khiến một người đoán có cảm hứng (người biết sức mạnh của cô ấy là 3) phỏng đoán rằng
N
một = 2 . 3 - 1, và điều này dễ dàng chứng minh bằng quy nạp.

Hãy xem xét một giải pháp thay thế, sử dụng các hàm tạo. Phương pháp "đoán và kiểm tra" ở trên kém

hiệu quả hơn nhiều so với phương pháp "đoán và kiểm tra" ở trên, nhưng nó có thể được áp dụng một cách

đáng tin cậy trong nhiều trường hợp mà việc đoán theo cảm hứng sẽ không hữu ích. Cho phép

2
f (x) : = ao + a lx + a2.x + ... = 1 + 5x + 1 7.x2 +. . .

là hàm sinh tương ứng với dãy (an). Bây giờ hãy nhìn vào

2 2 + ... + a2 .
xf (x) = aox + a1.x
Machine Translated by Google

4.3 CÁC CHỨC NĂNG PHÁT ĐIỆN 1 35

Đây là hàm tạo của dãy ban đầu, nhưng bị dịch chuyển. Nói cách khác, hệ số của XZ trong f (x) là an, trong khi

hệ số của XZ trong xf (x) là I. Bây giờ chúng ta sử dụng mối quan hệ giữa an và I. Vì an - 3an- I = 2 cho tất cả

n 2 tôi, chúng tôi có

f (x) - 3xf (x) = ao + 2 (x + � + x3 +...).

Trông có vẻ xấu xí, nhưng biểu thức trong ngoặc đơn ở phía bên tay phải chỉ là một chuỗi hình học vô hạn. Vì

vậy, chúng ta có (hãy nhớ, ao = I)

Gấp đôi

f (x) - 3xf (x) = 1 + .


Tôi - x

Vế trái là f (x) (1 - 3x) , vì vậy chúng ta có thể dễ dàng giải được f (x), nhận được

1 x + I
f (x) = 2x + ·
1 - 3x (I - x) (1 - 3x) (1 - x) (1 - 3x)

Mục tiêu của chúng ta bằng cách nào đó là khôi phục các hệ số khi f (x) được khai triển trong
một chuỗi lũy thừa. Nếu mẫu số chỉ là (1 - x) hoặc (1 - 3x) , chúng
cụ chuỗi
ta có
hình
thểhọc.
sử dụng
Một phần
công
phân số đến để giải cứu, mang 7lại

x + I 2
f (x) =
(l - x) (1 - 3x) 1 - 3x I - x

Sử dụng công cụ chuỗi hình học trên số hạng đầu tiên, chúng tôi nhận được

Và kể từ khi

--1 = 2
I + x + x + .r + ... ,
1

Tôi - x

chúng tôi nhận được

f (x) = 2 (1 + 3x + 32� + 33x 3


+ ...) - (1 + x + � + x3 + ...)
2 3
= 1 + (2 · 3 - I) x + (2. 3 - I) � + (2. 3 - 1) x3 + ... ,


N
từ đó nó ngay sau đó là an = 2 · 3 - 1.

Phương pháp này khá lộn xộn về mặt kỹ thuật, vì nó liên quan đến việc sử dụng công cụ chuỗi hình học lặp

đi lặp lại cũng như các phân số từng phần. Nhưng đừng để bị choáng ngợp bởi các chi tiết công nghệ - nó hoạt

động bởi vì mUltiplies một hàm tạo bởi x tạo ra hàm tạo cho chuỗi "đã dịch chuyển". Tương tự như vậy, chia cho

x sẽ chuyển dãy số theo hướng khác. Những kỹ thuật này chắc chắn có thể được sử dụng cho nhiều loại tái phát.

7Nếu bạn không nhớ kỹ thuật này, hãy tham khảo bất kỳ văn bản giải tích nào. Ý tưởng cơ bản là viết
(I �ti 3x) =
4 + I�X và giải các hằng số A và B chưa biết.
Machine Translated by Google

1 36 CHƯƠNG 4 BA CHIẾN THUẬT QUAY LỐI QUAN TRỌNG

Phân vùng

Xét tích đa thức sau: P (x) : = (2 + 3x + 1)

(2 + 2x + 1) = x4 + 5x3 + 82 + 5x + 1.

Làm cách nào để tính hệ số của: xk trong P (x)? Ví dụ, dấu X 2 số hạng là tổng

2. 1 + 3x. 2x + 1 · 2 = 8x2 ,

vậy hệ số là 8. Nói chung, để tìm số hạng: xk của P (x), chúng ta xem xét tất cả các cặp số
hạng, trong đó một số hạng đến từ thừa số thứ nhất và số hạng kia đến từ thừa số thứ hai và

số mũ cộng lại đến k. Chúng tôi nhân các cặp, và sau đó cộng chúng lại, và đó là câu trả lời
của chúng tôi.

Bây giờ chúng ta hãy viết lại P (x) dưới dạng

(x2 + x + x + x + 1) (2 + x + x + 1). (3)


2
Tất nhiên, sản phẩm sẽ không thay đổi, vì vậy, ví dụ: X hạn vẫn là 8X2. Cái này

tương ứng với tổng 2 · 1

+ x · x + x · x + x · x + x · x + x · x + x · x + 1 · 2.

Tất cả những gì thực sự quan trọng ở đây là số mũ; chúng ta có thể liệt kê các cặp số mũ theo
tổng

2 + 0, 1 + 1 , 1 + 1, 1 + 1, 1 + 1, 1 + 1, 1 + 1 , 0 + 2,

và tất cả những gì chúng ta cần làm là đếm số tổng (có tám). Đây là một cách khác
để nghĩ về nó. Hãy tưởng tượng rằng yếu tố đầu tiên của (3) có màu đỏ và yếu tố
thứ 2 có màu xanh
lam. Chúng ta có thể định dạng lại cách tính X hệ số theo cách

sau:

Giả sử rằng bạn có một màu đỏ 2, ba màu đỏ 1 và một màu đỏ 0; và một màu
xanh lam 2, hai màu xanh lam 1 s và một màu xanh lam O. Sau đó, có tám cách
khác nhau để bạn có thể cộng một số đỏ và một số xanh lam để có tổng là 2.

Tại thời điểm này, có thể bạn đang nói, "Vậy thì sao?", Vì vậy đã đến lúc làm ví dụ.

Ví dụ 4.3.5 Có bao nhiêu cặp có thứ tự phân biệt (a, b) của các số nguyên không âm thỏa mãn

2a + 5b = 100?

Giải pháp: Nhìn vào trường hợp tổng quát: Gọi Un là số thứ tự không âm,
các cặp giải được 2a + 5b = n. Do đó Uo = 1 Bây ,UI = 0, U2 = 1 , v.v. Chúng ta cần tìm UIOO.
giờ hãy xác định

A (x) = 1 + x2 + x4 + x6 + x8 + "' ,
'
B (x) = 1 + x5 + xlO + x15 +20 + " ,

và xét tích A (x) B (x) =

(1 +2 + x4 + .. ·) (1 + � + xlO + ...)

= 1 + x2 + x4 + x5 + x6 + x7 + x8 + x9 + 2x 1O + ... .
Machine Translated by Google
4.3 CÁC CHỨC NĂNG PHÁT ĐIỆN 1 37

. . . Cái này
Chúng tôi khẳng định rằng A (x) B (x) là hàm sinh cho chuỗi Uo, Ul không khó ,U2 , '
để nhận ra nếu bạn đã cân nhắc thảo luận "đỏ và xanh" ở trên: Rốt cuộc, mỗi số hạng trong A
(x) có dạng x2a trong đó a là một số nguyên không âm và tương tự như vậy, mỗi số hạng trong
B (x) có dạng �b. Do đó tất cả các số hạng trong tích A (x) B (x) sẽ là các số hạng có dạng
2a + 5b. Mỗi cặp (a, b) khác nhau thỏa mãn 2a + 5b = n sẽ tạo ra đơn thức XZ trong tích A (x)
B (x). Do đó hệ số của XZ sẽ là số nghiệm khác nhau của 2a + 5b = n.

Bây giờ chúng tôi sử dụng công cụ chuỗi hình học để đơn giản hóa

1
A (x) = và B (x) =
1 - x2 1Tôi- - 5x '

Như vậy

23
= UO + U1X + U 2 X + U 3 X + ... ). (4)
5 (1 -2 1x) (I - x

Theo nghĩa trừu tượng, chúng ta đã "xong", vì chúng ta có một hình thức đẹp cho hàm tạo.
Nhưng chúng tôi không hề biết UIOO là gì ! Đây không phải là quá khó để tìm thấy. Bằng cách
kiểm tra, chúng tôi có thể tính toán

(5)
Uo = U2 = U4 = Us = U6 = U7 = 1 và Ul = U3 = O.

Sau đó, chúng tôi biến đổi (4) thành

2
1 = (1 - x ) (1 - Y) ( Uo + U 1 x + U2 x2 + U3 x3 + ... )

7
= (1 - x2 - Y + x x2 + U3x3 + ... ) .
) ( Uo + U 1 x + U2

Hệ số l ' của tích của các số hạng bên tay phải phải bằng 0 (nếu k > 0). Nhân với hệ số
này là

Vì vậy, với mọi k > 7, chúng ta có quan hệ lặp lại

(6)

Đây là một bài tập khá đơn giản, mặc dù tẻ nhạt, để tính toán UIOO bằng cách sử dụng (5) và
(6). Ví dụ,

v.v ... Nếu bạn nghịch ngợm với điều này, bạn sẽ tìm thấy một số phím tắt (thử làm việc trở
lại phường và / hoặc lập bảng để giúp loại bỏ một số bước), và cuối cùng bạn sẽ nhận được
U100 = 1 1.

Ví dụ tiếp theo không liên quan đến việc tính toán hệ số của một hàm tạo gen, mà giải
quyết một vấn đề bằng cách đánh đồng hai hàm sinh một xấu, một đẹp.
Machine Translated by Google
1 38 CHƯƠNG 4 BA CHIẾN THUẬT QUAY LỐI QUAN TRỌNG

Ví dụ 4.3.6 Cho n là số nguyên dương bất kỳ. Chứng tỏ rằng tập hợp các trọng số

2 3 4
1, 3, 3 ,3 ,3 ,. ..

gam có thể được sử dụng để cân một trọng lượng n gam (sử dụng cả hai chảo của một chiếc cân) và điều

này có thể được thực hiện theo đúng một cách.

Giải: Ví dụ, nếu n = 1 0, thì khối lượng n-gam được cân bằng khối lượng I-gam và khối
lượng 9 gam. Thực tế số học tương ứng là
2
1 0 = 1 + 3 .

Nếu n = 73, khối lượng n-gam được ghép với khối lượng 9 gam trên một chảo, được đóng kiện
bởi khối lượng 8 I-gam và khối lượng I-gam. Điều này tương ứng với tuyên bố
2 4 73
+ 3 = 3 + 1,

tương đương với

Chúng ta sẽ hoàn thành nếu chúng ta có thể chỉ ra rằng với bất kỳ số nguyên dương n nào, có
thể viết n dưới dạng tổng và / hoặc hiệu của các lũy thừa riêng biệt của 3 và điều này có thể được
thực hiện theo đúng một cách. Nó giống như cơ số 3, nhưng không cho phép chữ số 2 và thay vào đó
thừa nhận "chữ số" - 1. Thật vậy, việc nghịch ngợm ý tưởng này có thể dẫn đến một thuật toán viết
n dưới dạng tổng / và hoặc hiệu số của lũy thừa trong tổng số 3, nhưng thật khó hiểu tại sao biểu
diễn sẽ là duy nhất.
Đây là một cách tiếp cận tạo kết quả học 8: Hãy xem xét chức năng

II (x) : = (1 + x + xI) (I + x3 + x-3).

Hai thừa số của II lần lượt chứa các số mũ 0, 1, - 1, và 0, 3, -3.


Khi nhân II ra, ta được II (x) = 1 + x +

xI + x3 + x4 + x2 + x-3 + x-2 + x-4. (7)

Mỗi số mũ nguyên từ -4 đến 4, được chứa trong tích này và mỗi số hạng có hệ số bằng 1.
Mỗi số hạng trong số chín số hạng này là kết quả của việc chọn đúng một số hạng từ thừa
số đầu tiên của II và một số hạng từ thừa số thứ hai, và sau đó nhân chúng (cộng số mũ
của chúng). Nói cách khác, các số mũ trong (7) chính xác là tất cả các cách kết hợp hai
số 1 và 3 với các phép cộng và / hoặc trừ và / hoặc bỏ sót. (Theo "bỏ sót", chúng tôi có
nghĩa là chúng tôi không cần phải bao gồm 1 hoặc 3 trong tổ hợp của mình. Ví dụ: tổ hợp
"+3" bỏ qua 1.)

Tiếp tục nào. Coi như

2
1
Jz (x) : = (1 + x + [ ) (1 + x3 + x-3) (I + x3 + x-3 \

8 "Công nghệ sinh học" tenn được đặt ra bởi Herbert Wilf, trong cuốn sách cùng tên của ông [47]. Chúng tôi kêu
gọi người đọc ít nhất hãy lướt qua cuốn sách giáo khoa được viết đẹp đẽ này, trong số nhiều cuốn sách khác của nó,
có câu mở đầu thơ mộng nhất mà chúng tôi từng thấy (trong một cuốn sách toán học). Ngẫu nhiên, [43] và [38] cũng
chứa tài liệu tuyệt vời và rất rõ ràng về các chức năng tạo ra.
Machine Translated by Google
4.3 CÁC CHỨC NĂNG PHÁT ĐIỆN 1 39

Khi nhân ra, mỗi kết quả là 3. 3. 3 = 27 số hạng sẽ có dạng x! 1 trong đó a là tổng và / hoặc hiệu ,

của lũy thừa là 3. Ví dụ, nếu chúng ta lấy số hạng thứ ba của thừa số thứ nhất bằng số hạng thứ nhất

của thừa số thứ hai bằng số hạng thứ hai thuật ngữ của yếu tố thứ ba, thuật ngữ tương ứng trong sản

phẩm là

(xI) (I) (x9) = x9-1.

Tất nhiên, những gì chúng tôi muốn hiển thị là các số hạng trong khai triển của h đều có hệ
số 1 (nghĩa là không trùng lặp) và phạm vi từ - (1 + 3 + 9) đến + (1 + 3 + 9) bao gồm ( nghĩa
là mọi số nguyên dương từ 1 đến 13 có thể được biểu diễn dưới dạng tổng / hiệu của các lũy
thừa của 3). Chúng tôi chắc chắn có thể xác minh điều này bằng cách mUltiplying ra, nhưng
chúng tôi tìm kiếm một lập luận chung hơn. Nhắc lại việc phân tích thừa số (xem trang 148) +

2
u3 -v3 = (u -v) (u uv + v2).

Áp dụng điều này cho các thừa số của h, chúng ta có

xZ + xI8 +

h (x) = ( ; + I) (x6 + ;: + I) ( x�9 +

= .
Tôi

x · x3. x9 x1)
- tôi
x3 - I) x3(x9
- tôi
- I) (
x9 - tôi
- I) (xZ7

Sau khi hủy, chúng tôi còn lại

xZ6 + xZ 5 + ... + x + I
_
'
_ -
h (x) -
13TÔIx (x-
xZ7 - tôi) _
Tôi 13 x

và như vậy

h (x) = xl 3 + x12 + ... + x + I + xI + x-2 + ... + x-13 ,

đó chỉ là những gì chúng tôi muốn. Chúng ta đã chỉ ra rằng các trọng số I, 3, 9 cho phép chúng ta cân

mọi số nguyên dương (hoặc âm!) N nhỏ hơn hoặc bằng 13, và theo đúng một cách (vì các hệ số đều bằng I).

Lập luận tổng quát, tất nhiên. Ví dụ, nếu

sau đó

=
x13
_TÔI x-
(xZ7tôi
- tôi) (xZ x27
· 27 + x27 + I)

=
Tôi

x13 · x27
( xZ7� x27(x81
- I) - tôi
- I

= .
) _1 (�) x40 x- I
Machine Translated by Google

1 40 CHƯƠNG 4 CÁC CHIẾN THUẬT ĐOẠN CHÉO QUAN TRỌNG CỦA TH REE

Bây giờ rõ ràng là các trọng số 1,3,32, ..., 3r có thể được sử dụng để cân bất kỳ giá trị
tích phân nào từ 1 đến (Y - 1) / 2, theo đúng một cách. Như r � 00, chúng ta nhận được trường
hợp giới hạn, một bản sắc đẹp của các hàm tạo:


Ví dụ cuối cùng của chúng tôi là từ lý thuyết về phân vùng của các số nguyên, một chủ đề
đầu tiên do Euler chủ trì. Cho một số nguyên dương n, một phân hoạch của n là một biểu diễn
của n dưới dạng tổng các số nguyên dương. Thứ tự của các summand không quan trọng, vì vậy chúng
được xếp theo thứ tự tăng dần. Ví dụ, 1 + 1 + 3 và 1 + 1 + 1 + 2 là hai phân hoạch khác nhau
của 5.

Ví dụ 4.3.7 Chứng tỏ rằng với mỗi số nguyên dương n, số phân hoạch của n thành các phần không
bằng nhau bằng số phân hoạch của n thành các phần lẻ. Ví dụ, nếu n = 6, có bốn phân vùng thành
các phần không bằng nhau, cụ thể là

1 +5, 1 + 2 + 3, 2 + 4, 6.

Và cũng có bốn phân vùng thành các phần lẻ,

1 + 1 + 1 + 1 + 1 + 1, 1 +1+ 1 + 3, 1 +5, 3 + 3.

Giải: Gọi Un và Vn lần lượt là số phân hoạch của n thành các phần không bằng nhau và các
phần lẻ. Cần phải thực hành suy nghĩ theo cách tạo ra chức năng, nhưng bây giờ bạn sẽ không
gặp khó khăn khi xác minh (ngay cả khi bạn gặp khó khăn khi tìm ra nó)

Ư (x): = (1 + x) (1 + �) (1 + x3) (1 + x4) (1 + x »...

là hàm tạo cho (un ) . Ví dụ, số hạng x6 trong U (x) bằng

xx 5 + x.� · x3 + � · x4 + x6 = 4x6.

Chú ý rằng không thể nhận được số hạng như � . �; tức là, chức năng tạo lỗ thông hơi trước
các phần lặp lại trong phân vùng.
Nếu bạn cảm thấy thoải mái với U (x) (vui lòng suy ngẫm về nó cho đến khi nó trở nên "hiển nhiên"

với bạn rằng đó là hàm tạo đúng), bạn nên thử tạo V (x), hàm tạo cho (vn ) . Các phần có thể
được trùng lặp, nhưng tất cả chúng phải là số lẻ. Ví dụ: nếu chúng ta muốn bao gồm khả năng
không, một hoặc hai số 3 trong một phân hoạch, thì thừa số (1 + � + x6) sẽ thực hiện thủ
thuật, vì x6 = x2-3 đóng vai trò của "hai số 3. "
Theo lý luận này, chúng tôi xác định hàm tạo

V (x): = (1 + x + x2 + x3 + .. ·) (1 + x3 + x6 + x9 + ...) (1 + x> + xlO + x15 + ...) ....

Bây giờ, tất cả những gì "còn lại" phải làm là chỉ ra rằng U (x) = V (x). Công cụ chuỗi
hình học cung cấp sự đơn giản hóa ngay lập tức, mang lại

V (x) = - - - - . . .5
I -x 1 -x3 l -x
( 1 ) ( 1 ) ( 1 ) ( 1 1 -x7 ) .
Machine Translated by Google
4.3 CÁC CHỨC NĂNG PHÁT ĐIỆN 1 41

Mặt khác, chúng ta có thể viết

Ư (x) = (1 + x) (1 + x2) (1 + x3) (1 + x4) (1 + x5) ...

= (\ � :) C = � :) C = � :) (� = �) Cl �: 50) ....

Lưu ý rằng trong biểu thức cuối cùng này, chúng ta có thể hủy bỏ tất cả các điều khoản của biểu mẫu (1 _� k

), chỉ để lại

.
--- x -) (- I-- -x3) - (1 -_- x 5) - (1 -_- x - = 7-) .-.
= V (x) . (l

Các vấn đề và bài tập

4.3.8 Chứng minh rằng với mọi số nguyên dương n, I. Hãy thử quy nạp bằng cách nhìn vào khai triển của 2n

số hạng đầu tiên của F (x). Nó sẽ không bằng I, nhưng nó


phải bằng một thứ không có bất kỳ số hạng xl < nào cho
"small" k. Ý tưởng là chỉ ra rằng khi n lớn lên, bạn có

thể đảm bảo rằng không có xl < số hạng cho k = 1, 2, ...,


4.3.9 Chứng minh rằng với mọi số nguyên dương k < m, n,
L, trong đó L là một số thứ lớn lên khi n lớn lên. Đó là

điều đó!
.
� (�)
J kJ ( m . ) = ( n k
+ m ) 2. Chứng tỏ rằng F (x) là bất biến dưới nhóm
f;: o

thế x > -> x2 • Sau đó, tiếp tục lặp lại


4.3.10 Sử dụng các hàm sinh để chứng minh công thức của
nhóm thế này (bạn cũng có thể muốn lưu ý
chuỗi Fibonacci được đưa ra trong Bài toán 1 .3. 18 trên
rằng sion expres chỉ hội tụ khi Ixl <I).
trang 10.
4.3.13 (Putnam 1 992) Đối với các số nguyên không âm n và k, xác
4.3.11 Đọc lại ví dụ 4.3.6. Chứng minh một cách chính thức bằng
định Q (n, k) là hệ số của xl < trong khai triển cũ của (I + x +
cách rút gọn rằng nếu
x2 + �) n. Chứng minh rằng

Jr (x) : = (I + x + [1) (1 + x3 + x-3)

x (I + �2 + [32) ... (I + � '+ x-3'),


Q (n, k) = }± =(�) J
o J(k�2 .) .
sau đó
4.3.14 Chứng tỏ rằng mọi số nguyên dương đều có một biểu diễn nhị

Jr (x) = x "+ x" - + ... + x2 + x + I


Tôi

phân (cơ số 2) duy nhất. Ví dụ, 6 được trả lại bằng 110 trong hệ
1
một
nhị phân, vì I · 22 + I. 2 +0. 20 =
+ x I + x-2 + ... + x ,
6. (Tính độc đáo này có thể được chứng minh theo nhiều
trong đó a = (3 '+ 1 - 1) / 2. cách; tất nhiên, bạn được khuyến khích thử tạo các hàm ở đây.)

4.3.12 Đây là một cách thay thế để kết thúc Bài kiểm tra 4.3.7: 4.3.15 Bộ lọc Unity gốc. Cho S = Cis 2: là căn bậc n của
Chứng tỏ rằng F (x) bằng I với mọi x, trong đó phép hợp nhất (xem trang 1 26).

(a) Chứng tỏ rằng tổng


F (x): = (l -x) (1 + x) x

(I-�) (I + �) x 1+ Sk + S2k + S3k +. .. + S (n I) k

(I _x5) (1 + �) x bằng n hoặc 0, tùy theo k có phải là đa bội của n hay

(I -x7) (1 + x4) x không.

()
Trong / 2J

.
Dưới đây là gợi ý cho hai đối số khác nhau: (b) Tìm một công thức đơn giản cho tổng } }
:; =
. o J
Machine Translated by Google

1 42 CHƯƠNG 4 BA CHIẾN THUẬT QUAY LỐI QUAN TRỌNG

Trong / 3J

4.3.19 Bạn có thể thực hiện đổi một đô la bằng bao nhiêu cách,

sử dụng đồng xu, niken, đồng xu, phần tư và nửa đô la? Ví dụ,
Tìm một fonnula đơn giản cho tổng � ;. . } = o (c)
() }
100 xu là một cách; 20 xu, 2 niken và 7 dimes là một loại khác.
(d) Tổng quát hóa!
Thứ tự không quan trọng.
4.3.16 (Olympic Toán học Leningrad 1991) Một dãy hữu hạn tại,

a2, tổng bất kỳ của fonn ak. + , an+ được


. . ak gọi
p + ak là +cân
+ 2p . ..bằng p tự
tương nếu
4.3.20 Hàm (l - x-x2 - x3 _ x4 - .r; _ x6) - t là hàm sinh cho
với bất kỳ k = 1,2, ..., p. Chứng minh rằng nếu một dòng có 50
dòng se dễ phát biểu nào?
phần tử là cân bằng p với mỗi p = 3,5,7, II

, 13, 17, thì tất cả các thành viên của nó đều bình đẳng
4.3.21 Một khuôn tiêu chuẩn được dán nhãn 1,2,3,4,5,6 (một

về không. trong teger trên mỗi mặt). Khi bạn tung hai viên xúc xắc tiêu

chuẩn, bạn có thể dễ dàng tính được xác suất của các tổng khác nhau.
4.3.17 Gọi p ( n) biểu thị số lượng mệnh không bị giới hạn của
Ví dụ, xác suất để tung hai con xúc xắc và nhận được tổng là 2
n. Đây là bảng một số giá trị đầu tiên của
chỉ là 1/36, trong khi xác suất nhận được 7 là 1/6.
p (n) .

Có thể tạo ra một cặp xúc xắc "không chuẩn" (có thể khác
N p (n) Các khoản tiền khác nhau
nhau) với các nhãn số nguyên posi tive tuy nhiên không thể phân
Tôi Tôi

biệt được với một cặp xúc xắc tiêu chuẩn, nếu tổng của xúc xắc
2 2 1 + 1,
là tất cả những gì quan trọng? Ví dụ, một trong những viên xúc
3 3 2 1 + 1 + 1, 1 +
xắc không tiêu chuẩn này có thể có nhãn 8 trên một trong các
4 5 2,3 I + I + I + I, I + I +2, I + 3,2
mặt của nó và hai mặt 3 . Nhưng xác suất để cuốn hai và nhận
5 7 + 2,4 I + I + I + I + I, I + I + I +
được tổng là 2 vẫn là 1/36 và xác suất để nhận được tổng là 7
2, I + I +3, I + 2 + 2,2 + 3, I +4,5
vẫn là 1/6.
Gọi f (x) là hàm sinh cho p (n) [nói cách khác, hệ số của
x * trong f (x) là p (k)]. Giải thích vì sao
4.3.22 Alberto xếp N con cờ vào một vòng tròn. Một số, có lẽ

tất cả, là màu đen; những cái khác màu trắng. (Sự phân biệt về
00 Tôi

· màu sắc là ngẫu nhiên.) Betiil đặt các séc mới giữa các cặp ô

f (x) = D I - xn cờ liền kề trong vòng Albertos: cô ấy đặt một ô cờ trắng giữa

hai ô cùng màu và một ô đen ở giữa


4.3.18 Chứng tỏ rằng số phân hoạch của một số nguyên posi tive

n thành các phần không phải là bội của ba bằng số phân hoạch
mỗi cặp màu đối lập. Sau đó, cô ấy loại bỏ các ô cờ gốc của
của n trong đó có nhiều nhất hai lần lặp lại. Ví dụ: nếu n = 6,
Alber để để lại một vòng mới gồm N ô cờ trong một vòng tròn.
thì có 7 phân vùng thuộc loại đầu tiên, cụ thể là

Sau đó, Alberto thực hiện thao tác tương tự trên vòng cờ

I + I + I + I + I + I + I + I + I + 2, của Betiils theo các quy tắc tương tự. Hai người chơi luân

I, 1 + 1 + 2 + 2, 1 + 1 +4, 1 +5, phiên thực hiện lại thao tác này.

2 + 2 + 2, 2 + 4;
Chứng tỏ rằng nếu N là một lũy thừa của hai, thì tất cả

và cũng có 7 phân vùng thuộc loại thứ hai, các ô cờ cuối cùng sẽ có màu trắng, bất kể dãy màu ar của

Alberto ban đầu đặt xuống. Có quan sát thú vị nào được thực
1 + 1 + 4, 1 + 1 + 2 + 2, 1 + 2 + hiện nếu N không phải là lũy thừa của hai không?

3, 1 +5, 2 + 4, 3 + 3, 6.

Bạn có thể khái quát vấn đề này?


Machine Translated by Google

Chương 5

Đại số học

Bạn có thể coi mình là một tay lão luyện về đại số. Tuy nhiên, bạn có thể đã mắc phải một số
thói quen xấu hoặc bỏ lỡ một vài thủ thuật trong giáo dục toán học của mình.
Mục đích của chương này là cải tiến: Chúng ta sẽ học lại đại số từ quan điểm của người giải
quyết vấn đề.

Đại số, tổ hợp và lý thuyết số có mối liên hệ mật thiết với nhau.
Vui lòng đọc phần đầu tiên / ew o / Chương 6 và 7 đồng thời với chương này.

5.1 Bộ, Số và Hàm

Phần đầu tiên này bao gồm phần đánh giá về ký hiệu bộ và hàm cơ bản, và có thể được đọc lướt
qua (nhưng hãy đảm bảo rằng bạn hiểu các ví dụ về hàm bắt đầu ở trang 145).

Bộ

Tập hợp là tập hợp các phần tử. Nếu một phần tử x thuộc (là phần tử của) tập A thì ta viết x
E A. Tập hợp có thể là tập hợp của bất kỳ thứ gì (kể cả các tập hợp khác). Một cách để xác
định một tập hợp là liệt kê các phần tử bên trong dấu ngoặc, ví dụ:

A = {2, 3, 8, Vi}.

Một tập hợp có thể không chứa phần tử nào cả; đây là tập trống 0 = {}.
Nhắc lại các phép toán tập hợp U (liên hiệp) và n (giao điểm). Chúng tôi định nghĩa A UB
là tập hợp mà mỗi phần tử của chúng được chứa trong A hoặc trong B (hoặc trong cả hai). Ví dụ,

{1,2, 5} Ư {I, 3, 8} = {1,2, 3, 5, 8}.

Tương tự, chúng tôi định nghĩa A n B là tập hợp có các phần tử được chứa trong cả A và
B, vì vậy, ví dụ:

{1,2, 5} n {1,3,8} = {I}.

Nếu tất cả các phần tử của tập A được chứa trong tập hợp B, ta nói rằng A là tập con của
B và viết A C B. Chú ý rằng A C A và 0 C A với mọi tập A.

1 43
Machine Translated by Google

1 44 CHƯƠNG 5 ALG EBRA

Chúng ta có thể xác định "phép trừ" cho các tập hợp theo cách tự nhiên sau:

A - B: = {aEA: a � B};

nói cách khác, A - B là tập hợp tất cả các phần tử của A mà không phải là phần tử của B.

Hai tập hợp cơ bản là các số tự nhiên N : = {I, 2, 3,4, ...} và các số nguyên Z : = {O, ± I,
Tôi

± 2, ± 3, ± 4, ...}.
Thông thường, có một tập hợp U "phổ quát" lớn hơn chứa tất cả các tập hợp mà chúng ta đang xem

xét. Điều này thường được hiểu theo ngữ cảnh. Ví dụ, nếu các tập mà chúng ta đang xem chứa các số,
thì U bằng Z, JR hoặc C. Khi đã biết tập phổ quát U, chúng ta có thể xác định phần bù A của tập A

là "mọi thứ" không thuộc A ; I E,

Đáp: = Ư - A.

Ví dụ: nếu U = Z và A bao gồm tất cả các số nguyên chẵn, thì A sẽ bao gồm tất cả các số nguyên lẻ.

(Nếu không có kiến thức về U, ý tưởng về phần bù của tập hợp là vô nghĩa; ví dụ: nếu U chưa biết và

A là tập hợp các số nguyên chẵn, thì các phần tử "không thuộc A" sẽ bao gồm các số nguyên lẻ, ảo
con số, cư dân của Paris, các vành đai của sao Thổ, v.v.)

Một cách phổ biến để xác định một tập hợp là ký hiệu "such that". Ví dụ, tập hợp các số hữu tỉ

Q là tập hợp tất cả các thương có dạng alb sao cho a, b EZ và b thuộc. O. Chúng tôi viết tắt "such

that" bởi "I" hoặc ":"; vì thế

Hỏi: = {� : a, b EZ, b của. O} .

Không phải tất cả các số đều hữu tỉ. Ví dụ, .j2 không phải là số hữu tỉ, mà chúng tôi đã chứng

minh ở trang 42. Chứng minh này có thể được mở rộng, với một số công việc, để tạo ra nhiều (thực tế
là vô số) số vô tỉ khác. Do đó, có một bộ giá trị "lớn hơn" trên "dãy số" bao gồm Q. Chúng tôi gọi
bộ này là các số thực JR. Người ta có thể hình dung JR một cách trực quan là toàn bộ tập hợp các

điểm "liên tục" trên trục số, trong khi Q và Z lần lượt là các tập con "sần sùi" và "rời rạc" của
JR.2

Chúng tôi thường đề cập đến khoảng các số thực. Ta dùng kí hiệu [a, b] để biểu thị khoảng đóng
{x E JR: a : s x : s b}. Tương tự, khoảng mở (a, b) được định nghĩa là {x E JR: a < x < b}. Các

phép lai [a, b) và (a, b] được định nghĩa tương tự.


Cuối cùng, chúng ta mở rộng các số thực bằng cách thêm phần tử mới i, được định nghĩa là căn

bậc hai của - 1; tức là, j2 = -1. Bao gồm i trong số các phần tử của JR tạo ra tập hợp các số phức
C, được định nghĩa chính thức bởi

C: = {a + bi: a, b E JR}.

Các số phức có thuộc tính quan trọng của phép đóng đại số. Điều này có nghĩa là bất kỳ sự kết hợp
hữu hạn nào của các phép cộng, trừ, nhân, chia (trừ 0) và rút gốc, khi được áp dụng cho một số phức,
sẽ bị dừng lại trong một số phức khác. Không có tập nào nhỏ hơn N, Z, Q hoặc JR có tính đại số

Chữ cái "Z" bắt nguồn từ tiếng Đức zahlen, có nghĩa là "số".
Tôi

2Có nhiều cách nghiêm ngặt để xác định các số thực một cách cẩn thận như là một "phần mở rộng" của số hữu
tỉ,. Ví dụ, xem Chương I trong [361.
Machine Translated by Google

5.1 BỘ, SỐ VÀ CHỨC NĂNG 1 45

Khép kín. Các số tự nhiên N không đóng dưới phép trừ, Z không đóng dưới phép chia, và cả Ql và

JR đều không đóng dưới căn bậc hai.


Cho hai tập hợp A và B (có thể bằng hoặc không bằng nhau), tích Descartes A x B được định
nghĩa là tập hợp tất cả các cặp có thứ tự có dạng (a, b ) trong đó EA và bE B. Về mặt hình
thức, chúng ta định nghĩa

A x B: = {(a, b): a EA, b EB}.

Ví dụ: nếu A = {I, 2, 3} và B = {Paris, London}, thì

A x B = {(I, Paris), (2, Paris), (3, Paris), (1, London), (2, London), (3, London)}.

Chức năng

Cho hai tập hợp A và B, ta có thể gán một phần tử cụ thể của B cho mỗi phần tử của A.
Ví dụ: với các tập hợp ở trên, chúng tôi có thể gán Paris cho cả 1 và 2 và London cho 3. Nói
cách khác, chúng tôi đang chỉ định tập hợp con

{(1 , Paris), (2, Paris), (3, London)}

của A x B.

Bất kỳ tập con nào của A x B với thuộc tính mà mỗi EA được ghép nối với đúng một bE B được
gọi là một hàm từ A đến B. Thông thường, chúng ta viết I: A --- + B để chỉ ra hàm có tên là 1
với miền tập hợp A và phạm vi trong tập hợp B. Chúng ta viết I (a) để chỉ phần tử trong B tương
ứng với một EA, và chúng ta thường gọi I {a) là ảnh của a. Một cách không chính thức, hàm 1 chỉ
là một "quy tắc" gán giá trị B I (a) cho mỗi giá trị A a. Dưới đây là một số ví dụ quan trọng
cũng phát triển thêm một số khái niệm và ký hiệu.

bình phương Xác định 1 : JR --- + JR bởi I (x) = 2x cho mỗi x E JR. Một thay thế không

2 tation là viết x ! --- * x hoặc x L x


2
. Chú ý rằng phạm vi 1 không phải là tất cả của
JR, mà chỉ là các số thực không âm. Cũng nhận thấy rằng I {x) = 9 có hai độ phân giải
như vậy x = ± 3. Tập hợp {3, -3} được gọi là ảnh nghịch đảo của 9 và ta viết (9) = {3,
1
1- bởi
-3}.
vì Lưu
nói ýchung,
rằng hình
như trong
ảnh nghịch
ví dụ đảo
này,không
hình phải
ảnh nghịch
là một đảo
phầncótử,
thểmàcólànhiều
một tập
hơn hợp,
một
phần tử.

3
Lập phương Định nghĩa g : JR --- + JR theo x ! --- * x cho mỗi x E JR. Trong trường hợp này, phạm
vi là tất cả của JR. Chúng tôi gọi các chức năng như vậy vào. Hơn nữa, mỗi hình ảnh
nghịch đảo chỉ chứa một phần tử (vì căn bậc hai của một số âm là âm và căn bậc hai của
một số dương là dương). Các hàm có thuộc tính này được gọi là I-to-l (hàm 1 ở trên là 2-
to- l, ngoại trừ ở 0, ở đó nó là I-to- 1). Một hàm như g, vừa là I -to- l vừa lên, còn
được gọi là hàm tương ứng 1-1 hoặc bijection.

Luỹ thừa và lôgarit Cố định một số thực dương bi = 1. Định nghĩa h: JR --- + JR bằng h
(x) = lr với mỗi x E JR. Phạm vi là tất cả các số thực dương (nhưng không phải là 0),
do đó h không nằm trên. Mặt khác, h là I -to- l, nếu y > 0 thì có đúng một nghiệm x của
phương trình lr = y. Chúng tôi gọi giải pháp này là
Machine Translated by Google

1 46 CHƯƠNG 5 ALG EBRA

logarit 10gb Y 'Ví dụ, nếu b = 3 thì log3 81 = 4 vì x = 4 là nghiệm duy nhất của
3 cho 3x = 81.
Bây giờ xét hàm x � logbx. Xác minh rằng miền là thực dương và phạm vi là
tất cả thực.

Sàn và Trần Đối với mỗi x E JR, xác định hàm sàn Lx J là số nguyên lớn nhất nhỏ
hơn hoặc bằng x (một ký hiệu khác cho Lx J là [xl, nhưng điều này hơi lỗi Ví
thời).
dụ,
L3,7J = 3, L2J = 2, L -2,4J = -3.
Tương tự như vậy, hàm trần IX 1 được định nghĩa là số nguyên nhỏ nhất lớn hơn
hoặc bằng x. Ví dụ, 13 . 1 1 = 4, 1- 1 . 21 = - 1. Đối với cả hai hàm, miền là
JR và phạm vi là Z. Cả hai hàm đều nằm trên và không phải là 1 -to-l. Trên thực
tế, các chức năng này là oo-to- l!

Các dãy Nếu miền của một hàm / là các số tự nhiên N, thì các giá trị của dãy sẽ
là / ( 1 ) , / (2) , / (3) , .... Đôi khi sử dụng ký hiệu sẽ thuận tiện hơn

/ 1 '/ 2' / 3, ... ,

trong trường hợp đó hàm được gọi là một chuỗi. Miền không cần chính xác là N ;
nó có thể bắt đầu bằng 0, và nó có thể là hữu hạn. Đôi khi một dãy vô hạn được
ký hiệu bằng (Ii) r, hoặc đôi khi chỉ bằng (Ii). Vì chỉ số phụ nhận các giá trị
nguyên nên các chữ cái quy ước được sử dụng là i, j, k, l, m, n.4 Các hàm chỉ
báo Gọi U là một tập hợp với tập con A. Hàm chỉ thị của A được ký hiệu là lA và

là một hàm với miền U và phạm vi {O, I} được xác định bởi

A x -
I if
1 () - { O nếu x EA,A,
x Ii

với mỗi x E U. Ví dụ, nếu U = N và A là tập hợp các số nguyên tố, thì lA (9) = °
và lA ( 1 7) = 1.

Các vấn đề và bài tập

5.1.1 Gọi A, B tương ứng là các số nguyên chẵn và lẻ hai bộ, và hàm chỉ báo của hàm hoàn chỉnh của một bộ
(nhớ rằng 0 là số chẵn). là sự khác biệt của I và hàm chỉ báo
của tập hợp đó.
(a) Có song ánh từ A đến B không?

(b) Có một sự phân ly từ Z đến A? 5.1.3 Đúng hay sai và tại sao: 0 = {0}

5.1.2 Chứng minh rằng với mọi tập A, 5.1.4 Chứng minh các phát biểu "đối ngẫu" sau đây, chứng tỏ
rằng, theo một nghĩa nào đó, cả số hữu tỉ và số vô tỉ đều
B, (a) 1A (X) lB (X) = 1AnB (X),
là "hạt".
(b) I - lA (x) = 1X (x).

Nói cách khác, tích của hai hàm chức năng chỉ (a) Giữa hai số hữu tỉ bất kỳ có một số vô tỉ.
báo là hàm chỉ báo của giao điểm của

3
Nếu chúng ta bao gồm các số phức, giải pháp không còn là duy nhất. Xem [29] để biết thêm thông tin.

4Trong ngôn ngữ lập trình máy tính tiên phong FORTRAN, các biến số nguyên phải bắt đầu bằng các chữ cái
I, J, K, L, M hoặc N (hỗ trợ ghi nhớ là "INteger.")
Machine Translated by Google

5.2 ALG EBRAIC MAN ĐÁNH GIÁ QUY ĐỊNH IPULATION 1 47

(b) Giữa hai số vô tỉ có một số hữu tỉ. số nguyên tive có thể được biểu diễn dưới dạng

l2xJ + l4xJ + l6xJ + l8xJ?


5.1.5 Số phần tử của một tập hợp được gọi là số lượng phần
5.1.9 Đúng hay sai và tại sao: l JfxJJ = l v'xJ với mọi x
tử của nó . Cardinality của A thường được chỉ ra bởi IAI
không âm.
hoặc #A. Nếu IAI = m và IBI = n thì chắc chắn IA x BI = mn.

Từ A đến B có bao nhiêu hàm khác nhau ? 5.1.10 Chứng minh rằng với mọi n EN,

5.1.6 (AIME 1984) Hàm f : Z - + Z thỏa mãn f (n)


= n - 3 nếu n � 1000 và f (n) = f (f (n + 5)) nếu
5.1.11 Thử Bài toán 2.4. 19 nơi trang 60.
n < 1000 . Tìm f (84).
5.1.12 Tìm công thức cho số hạng thứ n của dãy
5.1.7 (AIME 1984) Một hàm f được xác định cho mọi số thực
và thỏa mãn 1,2,2,3,3,3,4,4,4,4,5,5,5,5,5 ,. ..

f (2 + x) = f (2 -x) và f (7 + x) = f (7 -x) trong đó số nguyên m xảy ra đúng m lần.

5.1.13 Chứng minh rằng

:
với mọi x thực. Nếu x = 0 là một nghiệm nguyên của f (x)
0 2
N-'
= 0 thì số nghiệm nguyên f (x) = 0 phải có trong khoảng
J + l n; t J + l n; t J + ... + l n +2
-1000 : $ x : $ 1000 là bao nhiêu? l n ;, J = n

5.1.8 (AIME 1985) Có bao nhiêu trong số 1000 posi đầu tiên · với mọi số nguyên dương n.

5.2 Thao tác đại số được duyệt lại

Đại số thường được dạy như một loạt các kỹ thuật tính toán. Chúng tôi nói "compu tational" bởi vì thực sự

không có sự khác biệt về khái niệm giữa hai bài tập này:

l. Tính 42 x 57.

2. Viết (4x + 2) (5x + 7) dưới dạng một tam thức.

Cả hai đều là bài tập của các thuật toán thường ngày, nhàm chán. Thao tác đầu tiên sử dụng các con số thuần túy

trong khi thao tác thứ hai thao tác với cả số và ký hiệu. Chúng tôi gọi các thuật toán gây tê liệt (mặc dù hữu

ích) như vậy là "tính toán". Algebra chứa đầy những thuật toán này và chắc chắn bạn đã thực hành nhiều thuật

toán đó. Tuy nhiên, điều bạn có thể chưa học là đại số cũng là một môn học thẩm mỹ. Đôi khi người ta phải vượt

qua hàng đống hỗn độn các biểu thức đại số để giải quyết một vấn đề. Nhưng những trường hợp không may này là

khá hiếm. Một người giải quyết vấn đề tốt sẽ có cách tiếp cận tự tin hơn đối với các bài toán đại số. Chiến

lược tư duy mơ ước dạy cô ấy tìm kiếm một giải pháp thanh lịch .

Trau dồi bộ óc này: sử dụng một cái chạm nhẹ, gần như tinh tế, giữ cho các bộ điều chỉnh của đối thủ tránh

những thao tác xấu xí thay vào đó là các mẫu trang nhã, thường đối xứng .

Ví dụ đầu tiên của chúng tôi minh họa điều này.

3
Ví dụ 5.2.1 Nếu x + y = xy = 3, hãy tìm x + y3.

Giải pháp: Một cách để giải quyết vấn đề này - một cách không tốt - là giải hệ xy
= 3, x + y = 3 cho x và y (điều này sẽ sử dụng công thức bậc hai và các nghiệm 3 + y3.
Sẽ là
phức ) và sau đó thay thế các giá trị này vào biểu thức x số
Điều này sẽ hiệu quả, nhưng nó xấu xí và tẻ nhạt, lộn xộn và chắc chắn dễ xảy ra lỗi. + y3, Thay
3 2
liên lạc nhẹ. Mục tiêu của chúng tôi là x vào đó, vì
chúng tôita giữ
vậy chúng hãy cốmột
gắng cho x
+ tôi với tư cách là

2
y. bậc áp chót. Làm thế nào để có được x + tôi? Thử bằng cách bình phương x +
Machine Translated by Google

1 48 CHƯƠNG 5 ALG EBRA

32 = (X + y) 2 = � + 2xy + l,

và vì .xy = 3 nên ta có � + y2 = 3. Do đó, 3 · 3 = (x + y) (�

+ l) = .J + �y + .xy2 + l = x3 + l + .xy (x + y).

Từ điều này, chúng tôi kết luận rằng

đó là điều khá ngạc nhiên.

Thật tình cờ, điều gì sẽ xảy ra nếu chúng ta thực sự muốn tìm ra giá trị của x và y? Đây là một =
2 32, hoặc một cách thanh lịch
để làm điều đó. Phương trình x + y = 3 ngụ ý (x + y)

� + 2xy + l = 9.

Vì .xy = 3, chúng tôi trừ 4.xy = 12 từ phương trình cuối cùng này, được x2-2.xy

+ l = -3.

Đây là một hình vuông hoàn hảo, và lấy căn bậc hai cho chúng ta xy

= ± iV3.

Phương trình này đặc biệt hữu ích, vì nó được cho rằng x + y = 3. Cộng hai phương trình này ngay lập

tức cho chúng ta x = (3 ± iV3) / 2, và trừ chúng đi thì thu được y = (3 � iV3) / 2. Hai giải pháp của

chúng tôi cho (x, y) là

2
'
2
'
2
(3 + iV3 3-iV3) (3-iV3 3 + iV3) .
'
2 •
Chiến thuật nhân tố

Phép nhân hiếm khi đơn giản hóa mọi thứ. Thay vào đó, bạn nên

Yếu tố không ngừng.

Sau đây là các công thức cơ bản mà bạn đã học trong một lớp đại số. Đảm bảo rằng bạn
biết họ một cách chủ động, thay vì thụ động. Chú ý cách Công thức 5.2.4 giải ngay lập
tức Ví dụ 5.2. 1! 2 = x2 + 2.xy + y2. 5.2.2 (x + y)

5.2.3 (x-yf = x2 _2xy + y2. = X3


3
5.2.4 (x + y) + 3x2y + 3.xy2 + y3 = x3 + y3 + 3.xy (x + y). =
3
5.2.5 (x -y) � -3x2y + 3.xy2 -y3 = x3 -y3 -3.xy (x -y).

5.2.6 � -y2 = (x -y) (x + y).


N l
5.2.7 XZ -y = (x -y) (XZ-1 + XZ-2y + XZ-3y2 + ... + yn ) với mọi n.

5.2.8 XZ + yn = (x + y) (XZ-1 -XZ-2y + XZ-3y2 -... + yn-l) với mọi n lẻ (các số hạng của
thừa số thứ hai thay nhau trong dấu).
Machine Translated by Google

5.2 ĐÁNH GIÁ QUẢN LÝ ALG EBRAIC 1 49

Nhiều vấn đề liên quan đến sự kết hợp của các công thức này, cùng với các ý tưởng chiến lược
cơ bản (ví dụ, mơ tưởng), nhận thức về đối xứng và công cụ thêm 0 sáng tạo có giá trị.5 Đây là một
ví dụ.

4
Ví dụ 5.2.9 Hệ số x + 4 thành hai đa thức với hệ số thực.

Giải pháp: Nếu không yêu cầu các thừa số phải có hệ số thực, thì 4 chúng ta
có thể coi x
+ 4 là hiệu của hai hình vuông (Fonnula 5.2.6) và lấy

4 2
x +4 = x4 - (-4) = (�) - (2i) 2 = (� + 2i) (� -2i).

Mặc dù chúng ta không thể sử dụng trực tiếp phương pháp chênh lệch của hai bình phương, nhưng chúng
ta cũng không nên bỏ qua nó, vì biểu thức hiện có chứa hai bình phương hoàn hảo. Không may mắn
thay, nó không phải là sự khác biệt của hai hình vuông hoàn hảo. Nhưng có những hình vuông hoàn hảo
có thể có khác, và biểu thức của chúng ta gần như chứa chúng. Sử dụng mơ tưởng để làm cho các hình
vuông hoàn hảo hơn xuất hiện, bằng cách thêm các số 0 một cách sáng tạo.

4 x +4 = x4 + 4x2 + 4-4�.

Đây là động thái mấu chốt, hiện tại chúng tôi có

2 2
x 4 + 4� + 4-4� = (� + 2) - (2 lần) = (x2 + 2x + 2) (x2 -2x + 2).

Ví dụ hướng dẫn này cho thấy rằng bạn nên luôn luôn tìm kiếm những hình vuông hoàn hảo và cố gắng
tạo chúng nếu chúng chưa có.

Thao tác hình vuông


Cũng đáng nhớ là làm thế nào để bình phương một tam thức, chưa kể đến các đa thức chuyên dụng com.

Biết cách tạo và nhận biết các hình vuông hoàn hảo.

Để đạt được mục tiêu này, vui lòng chủ động áp dụng các công thức sau, không thụ động!

2
5.2.10 (x + y + z) = x2 + y2 + z2 + 2.xy + 2xz + 2yz.
2
5.2.11 (x + y + z + w) = x2 + y2 + z2 + w2 + 2xy + 2xz + 2xw + 2yz + 2yw + 2zw.

5.2.12 Hoàn thành hình vuông.

a2 a2 2 2
(a ) .
� + ax = � + ax + 4 -4 = x( +a 2) "- 2"

Suy ngẫm về hình vuông hoàn chỉnh ở trên. Một cách để "khám phá" nó là nhận dạng hình vuông
hoàn hảo bắt đầu bằng x2 + ax, sau đó thêm số 0 một cách sáng tạo. Một cách tiếp cận khác sử dụng
tính toán đơn giản, tiếp theo là một nỗ lực để sym metrize các tenns, cộng với việc thêm số 0 một
cách sáng tạo:

� + ax = x (x + a) = ( x + � - �) ( x + � + �) = ( x + �r - (�r ·

5Công cụ chị em với công cụ thêm 0 sáng tạo là phép nhân một cách thông minh với một công cụ.
Machine Translated by Google

1 50 CHƯƠNG 5 ALG EBRA

Chiến thuật giải nén hình vuông bao gồm nhiều công cụ ngoài việc hoàn
hình vuông. Đây là một vài ý tưởng quan trọng.

5.2.13 (xy) 2 + 4.xy = (x + y) 2.

5.2.14 Thay thế các biến trong phương trình trên bằng bình phương sẽ thu

được (�-i) 2 + 4�i = (x2 + i) 2,

mà tạo ra vô số bộ ba Pitago; tức là, các số nguyên (a, b, c) thỏa mãn a2 + b2 = c2. (Theo một
nghĩa nào đó, phương pháp này tạo ra tất cả các bộ ba số Pitago. Xem Ví dụ 7.4.3 trên trang 242.)

5.2.15 Phương trình sau đây cho thấy rằng nếu mỗi trong hai số nguyên có thể được viết dưới
dạng tổng của hai bình phương hoàn hảo, thì tích của chúng cũng có thể: (x2 + i) (a2 + b2) =

(xa-by) 2+ (ya + bx) 2. và 13 = 22 + 32 , và thật vậy,

2
Ví dụ: 29 = 22 + 5

29. 13 = 1 12 + 162.

Có thể dễ dàng thấy cách này hoạt động như thế nào, nhưng tại sao lại là một vấn đề khác. Hiện
tại, nhận thức muộn sẽ hoạt động: hãy nhớ rằng nhiều hình vuông hữu ích ẩn nấp và xuất hiện khi
bạn thao tác các "điều khoản chéo" một cách thích hợp (làm cho chúng bị loại bỏ hoặc làm cho chúng
tồn tại khi bạn thấy phù hợp). Để có giải thích "tự nhiên" về ví dụ này, hãy xem Ví dụ 4.2. 16 nơi
trang 1 30.

Thay thế và đơn giản hóa Từ "phân

số" đánh vào tâm hồn của nhiều học sinh toán giỏi. Điều này là do hầu hết
mọi người, bao gồm cả những người tiếp tục yêu thích và giỏi toán học, phải
chịu sự tra tấn về phân số ở trường lớp, nơi họ được yêu cầu hoàn thành
các phép tính dài và tẻ nhạt như
x2
Tôi 10 -
"S.yl X _"f" + Imp
I 17 -x I II +
1 -5x 3 ·

Bạn đã được dạy rằng "đơn giản hóa" là kết hợp mọi thứ trong "các thuật ngữ như".
Điều này đôi khi đơn giản hóa một cách diễn đạt, nhưng người giải quyết vấn đề giỏi có cách tiếp
cận tập trung hơn, theo định hướng nhiệm vụ, được thúc đẩy bởi chiến lược tư duy mơ mộng.

Tránh kết hợp thiếu suy nghĩ trừ khi điều này làm cho biểu hiện của bạn
trở nên tồi tệ hơn. Luôn luôn đi theo hướng đơn giản hơn và / hoặc symme
try và / hoặc beauty (cả ba thường đồng nghĩa với nhau).

(Tất nhiên, vẫn có những trường hợp ngoại lệ. Đôi khi bạn có thể muốn làm cho một biểu thức
xấu hơn vì khi đó nó mang lại nhiều thông tin hơn. Ví dụ 5.5.10 trên trang 1 75 là một minh họa
tốt cho điều này.)

Một ví dụ tuyệt vời về phép thay thế hữu ích (lấy cảm hứng từ tính đối xứng) là Exam ple 3.
1.1 0 trên trang 69, trong đó phép thay thế y = x + 1 / x làm giảm bậc 4
Machine Translated by Google

5.2 ALG EBRAIC MAN IPULATION R EVIS ITED 1 51

4
phương trình x + x3 + x2 + x + 1 = 0 về hai phương trình bậc hai. Dưới đây là một vài chi tiết
các ví dụ.

Ví dụ 5.2. 16 (AIME 1983) Tích của nghiệm nguyên của phương trình là gì

y. + 18x + 30 = 2Jx2 + 18x + 45?

Giải pháp: Đây không phải là một vấn đề quá khó. Trở ngại thực sự duy nhất cho
giải pháp tức thời là thực tế là có một căn bậc hai. Điều đầu tiên cần thử là loại bỏ
trở ngại này bằng cách mạnh dạn thay thế

y = Jx2 + 18x + 45.

Chú ý rằng nếu x là thực thì y phải không âm. Phương trình ngay lập tức đơn giản hóa thành

i- 15 = 2y,

thừa số nào thành (y -5) (y + 3) = O. Bác bỏ nghiệm nguyên y = -3 (vì y


phải không âm); thay thế gốc y = 5 trở lại gốc thay thế ban đầu sẽ thu được
x2 + 18x + 45 = 52

hoặc

y. + 18x + 20 = O.

Do đó, tích của các gốc là 20, sử dụng mối quan hệ giữa số không và công thức hệ số (xem trang 168).


Ví dụ 5.2. 17 (AIME 1986) Đơn giản hóa

(Vs + J6 + 0) (Vs + J6 -0) (Vs -J6 + 0) (-Vs + J6 + 0) .

Chúng tôi có thể giải thích tất cả các điều khoản, nhưng sẽ mất nhiều thời gian và có thể chúng tôi sẽ mắc sai

lầm. Chúng tôi cần một chiến lược. Nếu biểu thức này được đơn giản hóa, chúng ta có thể sẽ loại bỏ được tận gốc.

Nếu chúng ta nhân hai số hạng bất kỳ, chúng ta có thể sử dụng hiệu của hai công thức bình phương (5.2.6) và

nhận được các biểu thức chỉ chứa một căn. Ví dụ: tích của điều khoản thứ nhất và thứ hai là

2 2
(Vs + J6 + 0) (Vs + J6 - 0) = (Vs + J6) _ ( 0)

= 5 + 6 + 2v'30-7

= 4 + 2v'30.

Tương tự như vậy, tích của hai thuật ngữ cuối cùng là

(0+ (Vs-J6)) (0 - (Vs -J6)) = 7 - (5-2v'30 + 6) = -4 + 2v'30.

Sản phẩm cuối cùng, sau đó, là

(4 + 2v'30) ( -4 + 2v'30) = 4 · 30 - 16 = 104. •


Machine Translated by Google

1 52 CHƯƠNG 5 ALG EBRA

Ví dụ 5.2.18 (AIME 1986) Giải hệ phương trình

2x1 + X2 + X3 + X4 + x5 =

6 Xl + 2x2 + X3 + X4 + X5 =

12 Xl + X2 + 2x3 + X4 + X5 =

24 Xl + X2 + X3 + 2x4 + X5 =

48 Xl + X2 + X3 + X4 + 2x5 = 96.

Giải pháp: Quy trình tiêu chuẩn để giải hệ phương trình bằng tay là thay thế và / hoặc
loại bỏ các biến một cách có hệ thống (và tẻ nhạt). Nhưng lưu ý rằng mỗi phương trình gần như
là đối xứng, và hệ thống là đối xứng như một tổng thể.

Chỉ cần cộng tất cả năm phương trình lại với nhau; điều này sẽ dùng để đối xứng tất cả các hệ

số: 6 (Xl + X2 + X3 + X4 + X5 ) = 6 ( 1 + 2 + 4 + 8 + 16) ,


vì thế

Xl + X2 + X3 + X4 + X5 = 31.

Bây giờ chúng ta có thể trừ đại lượng này khỏi mỗi phương trình ban đầu để có ngay Xl = 6 -31, X
2 = 12 -31, v.v.
_

Bạn đã thấy công cụ xác định chức năng hoạt động trong Ví dụ 3.3.8 trên trang 89 và trong Ví
dụ 5.4.2 trên trang 167. Đây là một ví dụ khác, một ví dụ cũng sử dụng một lượng lớn đối xứng.

Ví dụ 5.2.19 Chỉ ra rằng --b - cc - aa -b (a - b) (b - c)

(ac) + - + - = abc abc , không nhân với số


.

Giải pháp: Mặc dù rất dễ hiểu, nhưng chúng ta hãy cố gắng tìm ra một cách tiếp cận thanh
lịch hơn. Chú ý các yếu tố bên tay phải như thế nào. Chúng ta có thể suy ra thừa số này bằng
cách xác định

b - cc - xx - b
f (x) :
= -x - -b - + c
'

+ Chú ý rằng f (b) = f (c) = O. Theo định lý nhân tử, nếu ta viết f (x) dưới dạng
thương của đa thức

P (x)
f (x) = ,
X bc

thì P (x) phải có X - b và X - c là thừa số. Ngoài ra, rõ ràng P (x) có bậc 3.
chúng tôi kết luận rằng
Cắm X = a vào f (x),
bc c - aa -b (a - b)
= (a - c) R (a)
- + - + - '
abc
trong đó R (x) là một đa thức tuyến tính. Bằng tính đối xứng, chúng ta cũng có thể xác định hàm

x - cc - aa - x
g (x) : = - + - + -,
một X c
Machine Translated by Google

5.2 ĐÁNH GIÁ QUẢN LÝ ALG EBRAIC ĐÃ ĐƯỢC LƯU Ý 1 53

và chúng ta có g (a) = g (e) = 0, suy ra thừa số là ea

ab (ba) (be) Q (a) + -b-


= + '
-một -e- abc

trong đó Q (x) là một đa thức tuyến tính khác. Chúng tôi kết luận rằng

- + - + - = K
b
làmộtea ab ((ab) (be)c (ea),) abc

đối với một số hằng số K. Việc cắm các giá trị (ví dụ: a = 1, b = 2, e = 3) sẽ thiết
lập rằng K = -1.

Ví dụ 5.2.20 (Putnam 1939) Cho x3 + b; (2 + ex + d = 0 có các hệ số và nghiệm nguyên
r, s, t. Tìm một phương trình đa thức với các hệ số nguyên được viết dưới dạng a, b,
e, d có gốc là r3, s3 ,t3.

Giải pháp: Một cách cực kỳ xấu để làm điều này là giải r, s, t, theo a, b, e và sau đó xây dựng đa

thức bậc ba (x -r3) (x -s3) (x -t3). Thay vào đó, chúng ta định nghĩa p (x): = x3 + b; (2 + ex + d và

lưu ý rằng

p (�) = 0

được thỏa mãn bởi x = r3, s3 , t3. Do đó chúng ta phải chuyển đổi

thành một phương trình đa thức tương đương. Lập phương đến trong tâm trí, nhưng chúng ta nên lập phương

những gì? Lập phương bất cứ thứ gì trừ một nhị thức là quá đau đớn. Nếu chúng ta đặt những người gốc ở
một bên và những người không gốc ở bên kia, chúng ta có

(1)

và bây giờ việc phân loại cả hai bên sẽ loại bỏ tất cả các gốc. Chúng tôi sẽ sử dụng biểu mẫu hữu ích hơn
của 5.2.4, trong đó nêu rõ rằng

(x + y) 3 = x3 + y3 + 3xy (x + y),

và tính toán cả hai bên của (1) sản lượng

3 3
_ (x + d) = (b�9) 3 + (e�) + 3b�e� (b� + e�)

= b3; (2 + e3x + 3bex (b� + e�).

Nhìn bề ngoài, điều này có vẻ không có nhiều cải tiến, vì phía bên tay phải vẫn chứa các gốc. Nhưng (1)

cho phép chúng ta thay thế - (x + d) cho những gốc khó chịu đó! Phương trình của chúng tôi trở thành

3
_ (x + d) = b3.x2 + e3x -3bex (x + d),

một khối với hệ số tích phân.



Machine Translated by Google

1 54 CHƯƠNG 5 ALG EBRA

Ví dụ 5.2.21 (AIME 1986) Đa thức 1 -x + x2 - .. J + ... + xI6 _xI7 có thể được


viết dưới dạng ao + aly + a2 y2 + a3 y3 + ... + aI6 yI6 + a17 yI7, trong đó y =
x + 1 và aiS là hằng số. Tìm giá trị của a 2.

Giải pháp: Sử dụng kiến thức tích cực của chúng tôi về các công thức tính thừa số, chúng
tôi nhận ra ngay rằng

x 18
- 1
1 _x + �_x3 + ... + xI6_x I7 = _ x - .
(- l)

(Ngoài ra, chúng ta có thể sử dụng công thức tính tổng của một chuỗi hình học.)

Thay y = x + 1, ta thấy đa thức trở thành

I8 - 1
_ = _� (y 18 _ (\ 8 ) y I7 + C;) y I6 _ ... + 1 -
(Y _ 1�

17 = _y 1) + (\ 8 ) y�)
I6 ·-... + C�) i -C!) Y + C

Như vậy


Vấn đề sau đây đã xuất hiện trong IMO 1972 . Giải pháp của nó phụ thuộc vào sym metry và
việc trích xuất cẩn thận các ô vuông, nhưng hơn bất cứ điều gì khác, dựa vào sự tin tưởng rằng
một giải pháp hợp lý thanh lịch tồn tại. Nó là khá giả, nhưng khá hướng dẫn.

Ví dụ 5.2.22 Tìm tất cả các nghiệm (XI, X2, X3, X4, X5) của hệ bất phương trình

(xi -X3 X5) (X� -X3 X5) �

0 (�-X4XI) (. xi -X4XI) � 0

(x� -X5X2) (� -X5X2) � 0

(x� -XI X3) (X � -XI X3) �

0 (x� -X2 X4) (xi -X2 X4) � 0

trong đó Xl, X2, X3, X4, X5 là các số thực.

Giải pháp: Vấn đề này khá đáng sợ, nhưng hãy lưu ý rằng nó diễn ra theo chu kỳ của sym
metric: mỗi bất đẳng thức có dạng (xT

-Xi + 2 Xi + 4) (xT + I -Xi + 2 Xi +

4), trong đó các chỉ số được đọc theo môđun 5. Ví dụ, nếu i = 3, thì bất đẳng thức trở thành

(x� -X5X 2) (X� -X5X2) � O.

Khi nhân các vế bên trái, chúng ta sẽ có tổng cộng 20 số hạng : tất cả m = 10 số hạng "bình

phương hoàn hảo" có dạng qX] (i - j) cũng như 10 "số hạng chéo", năm dạng -qxi + I Xi + 3 và

năm dạng -qxi + 2 Xi + 4.

6
Nếu bạn chưa quen với định lý nhị thức, hãy đọc Phần 6. 1.
Machine Translated by Google

5.2 ĐÁNH GIÁ QUẢN LÝ ALG EBRAIC ĐÃ ĐƯỢC LƯU Ý 1 55

Những tenns này trông đáng ngờ như chúng đến từ các bình phương của các nhị thức. Ví dụ,

2
(XI X2 -XI X4) = xTx� -2xTx 2 X4 + xT�

tạo ra hai trong số tenns hình vuông hoàn hảo và một tenn chéo. Chiến lược của chúng tôi: Viết tổng
các vế trái vào fonn

1 2
( 2 2 )
"2 YI + Y2 + ·· · + YIO,

trong đó mỗi Yk tạo ra một tenn chéo khác nhau và tất cả các số hạng bình phương hoàn hảo đều được

sao chép chính xác. Và thực sự, sau một số thử nghiệm, chúng tôi có

o � }:. (x7 -Xi + 2 Xi + 4) (x7 + 1 -Xi + 2 Xi + 4)


1 = 1

= 2
1 �(XiXi
"24. ( + 1 -XiXi + 3) + (Xi-I Xi + 1 -Xi-I Xi + 3) .
2 )
1 = 1

Vì chúng ta đã viết 0 lớn hơn hoặc bằng tổng các bình phương, nên giải pháp duy nhất là khi tất cả
các bình phương bằng 0, và điều này ngụ ý rằng

XI = X2 = X3 = X4 = Xs ·

Do đó, tập nghiệm của hệ bất phương trình là {(u, u, u, u, u): u E lR}.


Ví dụ cuối cùng xem xét một bất bình đẳng khó. Chúng tôi không giải quyết được nó, nhưng một
sự thận trọng tốt khiến nó ít nhất là dễ hiểu hơn một chút.

Ví dụ 5.2.23 (IMO 1995) Cho a, b, e là các số thực dương sao cho abc = 1 Chứng minh rằng .

Giải pháp một phần: Chúng tôi sẽ không giải quyết vấn đề này, nhưng chỉ ra một sự đơn giản hóa

alge braic cần phải được thực hiện. Điều tồi tệ nhất của vấn đề này là gì? Đó là một bất đẳng thức

liên quan đến các phân số khá xấu xí. Mơ ước cho chúng ta biết rằng sẽ đẹp hơn nếu các phân số ít
xấu hơn hoặc hoàn toàn không tồn tại. Làm thế nào điều này có thể đạt được? Có một sự thay thế khá

rõ ràng - nhưng chỉ rõ ràng nếu bạn có ý tưởng thay thế trong đầu trong ý thức của mình. Sự thay thế

X = 1la, y = 1 / b, z = lie,

biến bất đẳng thức ban đầu (sử dụng thực tế là xyz = 1) thành -x2 y2

3
z2 + - + -> - 2 y + -.
+ x x + y z z

Sự bất bình đẳng này vẫn không phải là dễ dàng để giải quyết, nhưng các mẫu số ít phức tạp hơn nhiều

và vấn đề đã được giảm bớt độ phức tạp. Xem Ví dụ 5.5.23 để tiếp tục.
Machine Translated by Google

1 56 CHƯƠNG 5 ALG EBRA

Các vấn đề và bài tập

5.2.24 (AIME 1 987) Tìm 3.?l nếu x, y là các số nguyên sao 5.2.31 (AIME 1989) Giả sử rằng XI, X2 ,..., X7 là

cho l + 3x2l = 30x2 + 517. số thực như vậy

5.2.25 Tìm tất cả các nghiệm nguyên dương (x, y) để (a) Xl + 4X2 + 9X 3 + 16x 4 + 25x5 + 36x6 + 49x7 = I

x2 - l = 20. (b) xy + 5x + 3y = 200. 4Xl + 9X2 + 16x 3 + 25x4 + 36x5 + 49x6 + 64X7 = 12 9Xl

+ 16x2 + 25x3 + 36x 4 + 49x5 + 64X6 + 81x7 = 123.

5.2.26 (Câu hỏi trắc nghiệm Mathpath 2006) Giả sử rằng a,


Tìm giá trị của
b, c, d là các số thực sao cho a 2 + b2 = I 2 + d2 = I

c 5.2.32 Chứng tỏ rằng mỗi số trong dãy

ac + bd = 0.
49.4489.444889.44448889, ...

Cho thấy
là một hình vuông hoàn hảo.

a 2 + c2 =
5.2.33 (Crux Mathematicorum, June / 7 , 1978) Chứng tỏ
I b 2 + d2 = I rằng n4 -20n2 + 4 là hợp khi n là bất kỳ số nguyên nào. +

ab + cd = 0. 2 5.2.34 Nếu x l + z2 = 49 và X + y + z = x3 + y3 +

z3 = 7, tìm xyz.
Vấn đề này có thể trở nên rất lộn xộn, nhưng không nhất thiết phải như vậy.

Cố gắng tìm ra một giải pháp thanh lịch và hoàn chỉnh. 5.2.35 Tìm tất cả các giá trị thực của X thỏa mãn (16x2 -
W + (9x2 - 16) 3 = (25x2 -25) 3.
5.2.27 (AIME 1 988) Tìm mầm nguyên dương nhỏ nhất có hình

lập phương kết thúc bằng 888 (tất nhiên, làm điều này mà không 5.2.36 Tìm tất cả các cặp số nguyên dương (x, y) có thứ tự

cần máy tính hoặc máy tính). thỏa mãn x3 -y3 = 721.

5.2.37 (Crux Mathematicorum, 4/1979 ) Xác định tôi


5.2.28 Tìm giá trị nhỏ nhất của xy + yz + xz, cho rằng x, y,

z là thực và x2 + l + z2 = 1. Vui lòng không tính tích!


khai thác bộ ba của số nguyên (x, y, z) thỏa mãn
hàm bằng

5.2.29 (AIME 1991) Tìm x2 + l nếu x, yEN và xy + x + y x3 + y3 + z3 = (x + y + z) 3.

= 71,.? Y + xi = 880. 5.2.38 (AIME 1987) Máy tính

5.2.30 Tìm tất cả các nghiệm nguyên (n, m) để (104 + 324) (224 + 324) ··· (584 + 324)

n4 + 2n3 + 2n2 + 2n + I = m2 . (44 + 324) (164 + 324) ... (524 + 324).

5.3 Tổng và Sản phẩm

Ký hiệu

Các chữ cái Hy Lạp viết hoa L (sigma) và II (pi) được sử dụng tương ứng cho các tổng
N N

và các sản phẩm. Ta viết tắt tổng XI + X2 +. . . + xn bằng , 6 Xi · Tương tự, [lXi

cho biết sản phẩm XIX2 " được · Xn • Biến i được gọi là chỉ số, và tất nhiên có thể

biểu thị bằng bất kỳ ký hiệu nào và giả định bất kỳ giới hạn trên và dưới nào, bao gồm cả vô hạn.

Nếu các chỉ số không phải là số nguyên liên tiếp, người ta có thể chỉ định chúng theo những cách khác.

Dưới đây là một vài ví dụ.


Machine Translated by Google

5.3 TỔNG KẾT VÀ SẢN PHẨM 1 57

2
• � d2 = 1 + 22 + 5 2 + 1 02 , vì dl lO bên dưới biểu tượng tổng kết có nghĩa là
lito

"d khoảng thông qua tất cả các ước


của 1 0." p225• 49
4 9 - = - . -. .. 7
một sản phẩm mfimte p2 - 1 '• }: / (i, j) = / .
f1.. (3,
-. -.
3 8 24 48
4) + / (3, 5) + / (4, 5).
p pnme

39 <j $ .5

Nếu các đặc tả chỉ mục được hiểu trong bối cảnh của một vấn đề, chúng chắc chắn có thể bị
bỏ qua. Trên thực tế, các chỉ số thường vướng vào một lập luận không chính thức, nhưng rõ ràng.
Ví dụ,

là một cách hợp lý, mặc dù không chính xác về mặt kỹ thuật để viết bình phương của một đa thức.
Ký hiệu chính xác là

Đảm bảo rằng bạn hiểu chỉ số phụ 1: S i < j: S n. Xác minh cẩn thận (xem
tại các ví dụ trong đó n = 2, 3, v.v.) mà

� XiX j
if.j = 2 ( 19
}:<jXiX
$ .n j) .
15, i, j $ .n

Đồng thời xác minh rằng một tổng kết với chỉ số con 1: S i < j: S n có (�) các số hạng
(bạn đang đọc Chương 6, phải không?).

Chuỗi số học

Một dãy số học là một dãy các số hạng liên tiếp với nhau có hiệu số không đổi;
tức là một
, chuỗi có dạng

a, a + d, a + 2d,….

Một chuỗi số học là một tổng của một chuỗi số học. Tổng của một dãy số học là
một ứng dụng đơn giản của công cụ ghép nối Gaussian (xem trang 67 trong Phần 3.
1). Xét một dãy số gồm n số hạng với số hạng đầu tiên là a và số hạng cuối cùng
là f. Chúng ta viết tổng hai lần (d là hiệu chung):

S = a + (a + d) + ... + (f - d) +

f, S = f + (f - d) + ... + (a + d) + a.

7 Có vô hạn số nguyên tố. Xem Vấn đề 2 .3 .2 1 trên trang 51 và Phần 7. 1. Ngẫu nhiên, giá trị của
tích vô hạn này là '/ t2 / 6. Xem Ví dụ 9.4.8 trên trang 349.
Machine Translated by Google

1 58 CHƯƠNG 5 ALG EBRA

Sau khi thêm, chúng tôi ngay lập tức suy ra rằng

thực tế hợp lý trực quan rằng tổng bằng giá trị trung bình của các số hạng nhân với số số hạng. Không phải

ngẫu nhiên mà một thuật ngữ khác cho "trung bình" là trung bình số học.

Chuỗi hình học và Công cụ kính thiên văn

Một dãy hình học hoàn toàn giống một dãy số học ngoại trừ việc bây giờ các số hạng liên tiếp có một tỷ lệ

chung; tức là, dãy có dạng

2 3
a, ar, ar ,ar , ....

Công cụ ghép nối Gauss không giúp ích gì cho việc tính tổng các chuỗi hình học, bởi vì các số hạng không

đối xứng cộng. Tuy nhiên, công cụ kính thiên văn tuyệt vời đã ra tay giải cứu. Xét một chuỗi hình học gồm n

số hạng với số hạng đầu tiên là a và tỉ số chung r (vì vậy số hạng cuối cùng là arn
l
). Thay vì viết tổng S hai lần, chúng ta nhìn vào S và rS:
l
S = a + ar + ar2 + ... + arn ,
2 3 n
rS = ar + ar + ar + ... + ar .

Quan sát rằng S và rS gần giống nhau, và do đó trừ hai đại lượng sẽ tạo ra một cái gì đó rất đơn giản. Thật

vậy, S - rS = a - ar + ar - ar2 + ar

2 nl
- ar3 + ... + ar - arn,

và tất cả các điều khoản đều hủy bỏ ngoại trừ điều khoản đầu tiên và điều khoản cuối cùng. (Đó là lý do

tại sao nó được gọi là "kính thiên văn", bởi vì cụm từ "hợp đồng" theo cách một số kính thiên văn làm.)

S - rS = a - arn
,

và giải quyết cho lợi suất S

a - arn
S - .
--l - r

Chuỗi hình học xuất hiện thường xuyên đến nỗi có lẽ đáng để ghi nhớ điều này đối với mula. Trong mọi trường

hợp, bước di chuyển mấu chốt-công cụ kính thiên văn-phải được thành thạo.

Có nhiều cách để kính thiên văn một loạt. Với chuỗi hình học ở trên, chúng tôi đã tạo ra hai chuỗi gần

như giống nhau. Loạt tiếp theo, mà bạn đã thấy lần đầu như Ví dụ 1.1.2 trên trang 1, yêu cầu cách xử lý khác.

Ví dụ 5.3.1 Viết
1 1 1 1
+ + + ... + 1 · 2 2 · 3 3 ·
4 99 · 100

dưới dạng một phân số trong điều kiện thấp nhất.

Giải pháp: Lưu ý rằng mỗi thuật ngữ có thể được viết dưới dạng

1 1 1

k (k + 1) k
Machine Translated by Google

5.3 TỔNG KẾT VÀ SẢN PHẨM 1 59

Toàn bộ số tiền là

(1 _ �) + (� _ �) + (� _ �) + ... + (� __ 1 ')
2 2 3 34 99 100

1 và tất cả các điều khoản đều hủy bỏ ngoại trừ điều khoản đầu tiên và điều khoản cuối cùng. Tổng của chúng tôi giảm xuống còn 1 -
100 ' •
Phần khó khăn ở trên là phát hiện ra rằng mỗi thuật ngữ có thể được viết theo cách mà kính
thiên văn. Điều này sẽ luôn hoạt động? Thật đáng buồn không. Điều quan trọng là nhận thức được
khả năng có thể sử dụng kính thiên văn, đây thực sự chỉ là một ứng dụng của công cụ thêm 0 một
cách sáng tạo. Và khá thường xuyên, một nỗ lực kính thiên văn sẽ không hoạt động hoàn hảo, nhưng
sẽ làm giảm độ phức tạp của một vấn đề.

Ví dụ 5.3.2 Tìm công thức tính tổng của n hình vuông đầu tiên.

Giải pháp: Nói cách khác, chúng tôi tìm kiếm một công thức cho

1 2 + 22 + 32 + .. · + n2 = � /.
J = l

Nếu chúng ta gặp may mắn như chúng ta đã làm với ví dụ trước, chúng ta sẽ phát hiện ra một chuỗi
ma thuật Ul, U2 ,... với tính chất uk + l

-Uk = k2.

Sau đó, chúng tôi sẽ được thực hiện; sản lượng kính viễn vọng

N N

� / = � (Uj + l -Uj) = (U2 -ud + (U3 -U2) +... + ( Un +! -Un) = Un + l -Ul ·


J =! J =!

Nhưng chúng ta không cần phải có được kính thiên văn hoàn hảo. Chúng ta chỉ cần tìm một chuỗi
Uk để các khác biệt liên tiếp trông giống như những gì chúng ta muốn. Điều này thực sự không
có gì khác hơn là theo đuổi một chiến lược tư duy mơ ước. Trong mạch này, hãy thử nghiệm với
một số trình tự đơn giản. Điều đầu tiên cần thử là phỏng đoán ngây thơ Uk : = k2. Ta nhận được

Uk + 1 -Uk = k2 + 2k + 1 -k2 = 2k + 1 .

Các số hạng bậc hai bị hủy bỏ, chỉ để lại một biểu thức tuyến tính. Tất nhiên, dự đoán tiếp
theo là thử Uk : = k3, kết quả là 3 -k3 = k3 + 3k2 + 3k + 1 - k

3 = 3k2 + 3k + 1 .
Uk + 1 -Uk = (k + 1)

Đây không phải là k2, nhưng nó sẽ làm khá tốt, hiện tại các phương pháp kính thiên văn mang lại
N N
3 3 3 - 1
= n + 3n2 + 3n.
� (3 / + 3j + 1) = � (Uj + l -Uj) = Un + l -Ul = (n + 1)
J = l J = l

Nói cách khác,


Machine Translated by Google

1 60 CHƯƠNG 5 ALG EBRA


N N

và chúng tôi có thể giải quyết cho }; P. Ta vẫn cần tính tổng của chuỗi số học }; (3 j + I), nhưng
J J =!

=! chúng tôi đã có một công thức cho điều này! Xác minh
N
điều đó . 2 n (n + l) (2n
};+ l)- } -
6
.
J =!

Đôi khi telescoping không phù hợp với những gì bạn bắt đầu, nhưng sự ra đời của một thuật
ngữ mới sẽ ngay lập tức biến đổi vấn đề. Chúng tôi gọi đây là công cụ xúc tác . Một khi bạn nhìn
thấy nó, bạn sẽ không bao giờ quên nó và sẽ dễ dàng áp dụng nó vào các bài toán khác.

Ví dụ 5.3.3 Đơn giản hóa sản phẩm

Giải: Gọi tích là P và xét hiện tượng xảy ra khi nhân P với I - l la. "Chất xúc tác" là sự

khác biệt đơn giản của hai công thức bình phương (xy) (x + y) =
2 x - tôi.

Vì thế
21


01 1 - l 1a
P = ---'--, -
1 - l la

Chuỗi vô tận

Chuỗi số có vô hạn số hạng đúng hơn là một chủ đề về giải tích và bạn sẽ tìm thấy thêm thông tin
trong Chương 9. Bây giờ, chúng ta hãy lưu ý một vài ý tưởng cơ bản. Một chuỗi vô hạn hội tụ hoặc
phân kỳ nếu tổng của nó tương ứng là hữu hạn hoặc vô hạn. Bạn có thể nhớ lại công thức cho chuỗi
hình học vô hạn hội tụ:

2
a a + ar + ar + ... =-, l
- r
Machine Translated by Google

5.3 SUMS VÀ PROD UCTS 1 61

hợp lệ nếu và chỉ khi Irl < I. Đây là hệ quả đơn giản của công thức cho một chuỗi hình học hữu
hạn.
Có nhiều cách để xác định xem một chuỗi đã cho hội tụ hay phân kỳ.
Tuy nhiên, các nguyên tắc đơn giản nhất là

• Nếu }: ak <00 (Le., Chuỗi hội tụ) và ak thống trị tất cả trừ một số hữu
hạn của hk (Le., Ak 2 hk với tất cả trừ một số hữu hạn giá trị của k),
thì }: hk• <00
• Tương tự như vậy, if }: ak = 00 (Le., Chuỗi phân kỳ) và hk chiếm ưu thế tất cả
trừ một số hữu hạn của ab thì}: hk = 00.

Nói cách khác, chiến lược đơn giản nhất khi xử lý một chuỗi vô hạn chưa biết là tìm một
chuỗi đã biết để so sánh với nó. Một chuỗi cơ bản mà bạn nên biết rõ là chuỗi điều hòa

1 I 1 + + + Tôi

2
+ ... .
3 4
Ví dụ 5.3.4 Chứng tỏ rằng chuỗi điều hòa phân kỳ.

Giải pháp: Chúng tôi sẽ tìm một số phép tính gần đúng thô cho các tổng một phần của loạt bài này.
Thông báo rằng

Tôi 1

+ Tôi 2
+ = =
1 4 4 4 2 4 Tôi 2 '

3 kể từ 1 và ! cả hai đều thống trị !. Tương tự như vậy,

1 1 + "6 1 4 1 2 8
+ + =
1 5 7 8 2

1 1 - 1 1
-
số 8

+ - + ... + - > - = 9 1 0 .
1 6 - 16 2
Nói chung, với mỗi n > 1,

1 Tôi
1 1
+ + ... + 2 2 ' 2n + 1 2n + 2
2n 2n +

N
số hạng lớn hơn hoặc bằng 2n�I ' Do đó, toàn bộ kể từ mỗi
chuỗi sóng hài lớn hơn hoặc bằng

1 1 1 1 1 + + + + 2
+ ""
2 2 2
phân kỳ rõ ràng.

Ý tưởng chính được sử dụng ở trên kết hợp sự thật hiển nhiên rằng

1 1
- <
ha-
với thủ thuật hay là thay mẫu số "phức tạp" bằng mẫu số "đơn giản" hơn.
Đây là một ví dụ về công cụ mát-xa nhiều mặt - kỹ thuật sờ soạng
Machine Translated by Google
1 62 CHƯƠNG 5 ĐẠI SỐ

một biểu thức, với bất kỳ phương pháp nào hoạt động (cộng không, nhân với một, cộng hoặc trừ một chút,

v.v.), để làm cho nó dễ quản lý hơn (nhưng một ý tưởng khác được kích thích bởi chiến lược tư duy mơ

ước). Đây là một ví dụ khác.

Ví dụ 5.3.5 Hàm zeta � (s) được xác định bởi chuỗi vô hạn
Tôi 1 1
� (các): = - + - + - + ....
IS 2S 3s

Khi s = I, chuỗi này trở thành chuỗi điều hòa và phân kỳ.8
Chứng tỏ rằng � (s) hội tụ với mọi s 2 2.

Giải pháp: Đây là một bài tập thông thường sử dụng phép thử tích phân từ phép tính tích phân, nhưng

sẽ hướng dẫn hơn nhiều nếu sử dụng các nguyên tắc đầu tiên. Trước hết, chúng ta lưu ý rằng mặc dù bài

toán yêu cầu chúng ta chứng minh một phát biểu về vô số giá trị của s, chúng ta chỉ cần chứng minh rằng

� (2) <00, vì nếu s > 2 chúng ta có

1 1
->
k2 - kS

với mọi số nguyên dương k và do đó sự hội tụ của � (2) sẽ có nghĩa là sự hội tụ của
� (s) với tất cả các giá trị lớn hơn của s.
Nhưng làm thế nào để chứng tỏ rằng � (2) hội tụ? Số hạng chung là l / k2; chúng ta phải tìm kiếm

một loạt phim tương tự mà chúng ta biết điều gì đó. Chúng tôi đã "sở hữu" một loạt bài lồng vào nhau độc

đáo có các số hạng là nghịch đảo của hệ tứ phân, Ví dụ 5.3. 1 trên trang 158, bộ truyện

N
Il = 1 -
.
� ! k (k + 1) n + 1

Chắc chắn chuỗi vô hạn hội tụ (thành 1), và bây giờ chúng ta sẽ hoàn thành nếu 1 / k2 nhỏ hơn 1 / (k (k

+ 1)). Nhưng sự bất bình đẳng đi sai đường!

Không phải lo lắng: Chúng tôi có thể viết shift chỉ số tổng kết và nhận được

N n +!
Il 1
=
2:
�! k (k + l) + � 2 (k- l) k '
Bây giờ chúng ta có thể sử dụng

k2
< '
1 k (k- l)

với mọi số nguyên dương k, để kết luận rằng

N
Il < 1 -
.
k�
k2 n + 1 2

Vì vậy, tổng (bắt đầu từ k = 2) hội tụ, và do đó toàn bộ tổng cũng hội tụ.
_

S
Nó chỉ ra rằng (các) có nhiều thuộc tính phong phú, với các kết nối tuyệt vời đến tổ hợp và số

học thuyết. Tham khảo Chương 2 của [47] như một điểm khởi đầu.
Machine Translated by Google

5.3 TỔNG KẾT VÀ SẢN PHẨM 1 63

Khi chúng ta quan tâm đến sự hội tụ, một vài thuật ngữ đầu tiên không quan trọng chút nào. Trên thực

tế, vài nghìn tỷ điều khoản đầu tiên không quan trọng! Đừng quên ý tưởng gọn gàng này về việc thay đổi

chỉ mục tổng hợp cho phù hợp với mục đích của chúng ta.

Các vấn đề và bài tập


N
5.3.6 Tìm công thức cho tích các số hạng của một dãy hình học. k (J ) •
5.3.15 Đánh giá sản phẩm J] cos ( 2

N
5.3.7 Tìm công thức tính tổng Tôi

5.3.16 Tìm tổng � -I . 2 gku 5.3.17


(AIME

1996) Đối với mỗi hoán vị al, a2, a3, ..., aw của các số nguyên
và khái quát hóa. 1,2, ... 10, tạo thành ,
Tổng
5.3.8 Tìm công thức cho

Tôi

- Tôi 1 · 2 · 3 .
+ - + . . . +
2 · 3 · 4 n (n + l) (n + 2)
Tìm giá trị trung bình của tất cả các tổng này.
Bạn có thể khái quát điều này?
5.3.18 Hãy thử Bài toán 1.3.8 ở trang 9, nếu bạn chưa làm được.
5.3.9 Tìm công thức cho

1 · 2 · 3 + 2 · 3 · 4 + ··· + n (n + 1) (n + 2). 5.3.19 (Canada 1989) Cho các số 1,2,22, •. • , 2n-1 , cho một hoán

vị cụ thể . .., Xn trong các


xl,sốS2này
(a)ta= xác
XI +định
X2 a = XI, X2, SI (a) =
Bạn có thể khái quát điều này?

5.3.10 Quan sát thấy l v'44 J = 6, l vi 4444 J = 66. , ... và Q (a) = SI (a)

S2 (a) ·· Sn (a). Đánh giá }; 1 / Q (a), trong đó tổng được lấy trên
Khái quát hóa và chứng minh.
tất cả các hoán vị có thể có.
5.3.11 (AIME 1983) Đối với {I, 2, 3, ..., n} và mỗi tập hợp con
5.3.20 Một sợi dây đàn hồi dài 2 inch được gắn chặt vào tường
khác của nó, một tổng thay thế duy nhất bị phạt như sau: Sắp xếp
ở một đầu, và có một lỗi ở đầu kia. Ở phút cuối (bắt đầu từ thời
các số trong tập con theo thứ tự giảm dần và sau đó, bắt đầu với
điểm 0), dải được kéo căng một cách tức thời và đồng đều theo
lớn nhất, lần lượt trừ và cộng các số liên tiếp. (Ví dụ: tổng
từng inch của tôi , và sau đó lỗi đi theo từng inch của tôi về
thay thế cho {I, 2, 4, 6, 9} là 9 -
phía cuối được buộc chặt. Liệu con bọ có bao giờ đến được bức

tường?
6 + 4 -2 + I = 6 và với {5} thì đơn giản là 5.) Với mỗi n, hãy tìm
5.3.21 Gọi S là tập hợp các số nguyên dương không có số 0 trong
công thức tính tổng của tất cả các tổng xen kẽ của tất cả các tập
con. biểu diễn cơ số 1O của chúng; I E,

5.3.12 Chứng minh rằng


S = {l, 2, ... , 9, Il, 12, ... , 19,21 ,. ..}.

Tổng các nghịch đảo của các phần tử của S là hội tụ hay phân kỳ?

Đây chỉ là một cách nói hoa mỹ rằng nếu bạn xem xét mỗi x trong U 5.3.22 Ví dụ 5.3.5 ở trang 162 cho thấy rằng , (2) <00 . Sử dụng
và viết ra I bất cứ khi nào x nằm trong A, thì tổng các I này tất
"massage" để chỉ ra rằng, trên thực tế, (2) <2. Sau đó, cải thiện
nhiên sẽ là số
ước tính của bạn hơn nữa để cho thấy rằng , (2) < 7/4. (Giá trị
của các phần tử trong A.
chính xác của , (2) là 1C 2/6. Xem Ví dụ 9.4.8 trên trang 349 để

5.3.13 Tìm tổng 1. l! + 2 · 2! +. .. + n. nL biết bản phác thảo của một bằng chứng.)

5.3.14 Tìm công thức tính tổng 5.3.23 (Putnam 1977) Đánh giá tích vô hạn

N
k
·

�I (k + I)!
Machine Translated by Google

1 64 CHƯƠNG 5 ALG EBRA


1 99 5
5.3.24 Bạn có thể khái quát ý tưởng được sử dụng trong Bài
1
thi 5.3.2 ở trang 159 không? VIri · Tìm �1 f (k) ·

5.3.25 (AIME 1995) Gọi f (n) là số nguyên gần nhất với

5.4 Đa thức
Còn nhiều thứ hơn đối với đa thức ngoài các phép toán thông thường như cộng, trừ ing,
mUltiplying và chia. Phần này chứa một vài tính chất quan trọng của đa thức để xem lại
và / hoặc học.
Đầu tiên, một số ký hiệu và định nghĩa. Gọi A là một tập hợp các số được đóng dưới
phép cộng và phép nhân. Định nghĩa
2
A [x] = {ao + alx + a2.x +. . . + an.tt: ai E A, n = 0, 1,2, 3 ,. . . }

là tập các đa thức có hệ số trong A. Các tập hệ số phổ biến nhất mà chúng ta sử dụng
là Z, Q, JR và C. Đôi khi chúng ta có thể sử dụng Zn, các số nguyên modulo n (xem
trang 230). Ta gọi mỗi biểu thức có dạng ajxi là một số hạng hay đơn thức.
Khi viết một đa thức tùy ý, hãy tuân theo quy ước gán nhãn ai là hệ số của x
. Ký hiệu nhất quán rõ ràng giúp tránh sai sót và nhầm lẫn với các
tôi

thao tác phức tạp. Chúng tôi xác định bậc của một đa thức là số mũ cao nhất với số mũ
khác 0. Hệ số này còn được gọi là hệ số hàng đầu. Nếu hệ số này là I thì đa thức được
gọi là monic. Hệ số ao được gọi là hằng số .

Phép toán đa thức


Phần lớn giáo dục đại số ban đầu của bạn được dành cho việc cộng, trừ, nhân và chia
các đa thức. Chúng tôi sẽ không xúc phạm trí thông minh của bạn khi xem xét hai phép
toán đầu tiên, nhưng bạn nên suy nghĩ về phép nhân và phép chia. Mẹo Mul khá dễ dàng,
nhưng điều quan trọng là phải sử dụng ký hiệu tốt. Đảm bảo rằng bạn hiểu ký hiệu sau
đây, bằng cách mô tả bằng tay một vài ví dụ.

= A (x) B (x), sau đó


tôi tôi tôi

Nếu A (x) = }: aix , B (x) = }: bix và C (x) = }: Cix

Cj = aobj + a \ bj - l +. .. + ajbo = }: asbt.


s + t = j;

s, t :::: O

Đa thức có thể được chia giống như số nguyên và kết quả sẽ là thương và phần dư.
Chính thức hơn, các đa thức với hệ số trong Z, Q, JR, C và Zn đều có một thuật toán
chia tương tự như phiên bản số nguyên (Bài toán 3.2. 1 7):

Gọi f (x) và g (x) là các đa thức trong K [x], trong đó K là một trong Z, Q, JR, C
hoặc Zn. sau đó

f (x) = Q (x) g (x) + R (x),

trong đó Q (x), R (x) E K [x] và bậc của R (x) nhỏ hơn bậc của g (x).
Ta gọi Q (x) là thương và R (x) là dư.
Machine Translated by Google

5.4 POLYNOM IALS 1 65

Ví dụ, cho f (x) = x3 + x2 + 7 và g (x) = x2 + 3. Cả hai đa thức đều trong

Z [x]. Bằng cách thực hiện "phân chia dài", chúng tôi nhận được

3 x + x2 + 7 = (x2 + 3) (x + 1) + (-3x + 4),

do đó Q (x) = x + I và R (x) = -3x + 4. Điều quan trọng là thương Q (x) cũng nằm trong Z [x], tức là

cũng có hệ số nguyên. Chúng ta có thể coi thuật toán chia là điều hiển nhiên, nhưng nó là một thuộc
tính rất quan trọng của đa thức, cũng như số nguyên.

Ví dụ 5.4. 1 (AIME 1986) Số nguyên n lớn nhất mà n3 + 100 hiển thị bởi n + 1O là bao
nhiêu?

Giải: Sử dụng thuật toán chia, n3 + 100 = (n + 1O) (n2 - IOn + 100) -
900, vì vậy

3 n + 100 900
= n2 _ IOn + 100 _ .
n + 1O n + 1O

3 Nếu n
+ 100 chia hết cho n + 10 thì n��O phải là số nguyên. Lớn nhất
dương n mà điều này đúng là n = 890, tất nhiên.
_

Zeros của một đa thức


Nó luôn luôn là tốt để giải quyết một phương trình đa thức; chắc chắn bạn biết công thức bậc hai, trong
đó nói rằng nếu

cd- + bx + c = 0,

sau đó

x =
2a

Mặc dù công thức này hữu ích, nhưng điều quan trọng hơn là phải nhớ nó được bắt nguồn như thế nào,

bằng cách sử dụng công cụ bình phương hoàn chỉnh . Chúng tôi sẽ xem xét điều này với một ví dụ đơn giản.

2
x 2 + 6x - 5 = 0 {= :::} x + 6x = 5 2 {= :::} x + 6x + 9 = 14.

Do đó (x + 3f = 14, do đó x + 3 = ± JI4, v.v.


9
Nhưng thường thì rất khó hoặc không thể xác định được các số 0 chính xác của một đa thức,

và trên thực tế, đôi khi những số không chính xác không phải là tất cả những gì quan trọng, nhưng thông

tin gián tiếp mới là thứ cần thiết. Vì vậy, điều quan trọng là phải hiểu càng nhiều càng tốt về mối

quan hệ giữa các số không của một đa thức và các thuộc tính khác. Dưới đây là một vài nguyên tắc hữu

ích.

9Các công thức cho số không của bất kỳ đa thức bậc ba hoặc bậc bốn nào được phát hiện vào thế kỷ 1 6 và Abel
đã chứng minh vào thế kỷ 1 9 rằng nói chung không thể tìm ra công thức "cơ bản" cho các số không của tất cả các
số 5- hoặc đa thức bậc cao hơn. Xem [41] để biết tài khoản rất dễ đọc về điều này.
Machine Translated by Google

1 66 CHƯƠNG 5 ALG EBRA

Định lý Phần dư

Nếu chia đa thức P (x) cho x - a thì phần dư sẽ là P (a).

Ví dụ, chia x3 - 2.x + 3 cho x + 2 và nhận (sau một số công việc) x3 -

2.x + 3
= 1 - 2.x + 2 ---;
2

x + 2
--...., .--- .r

x + 2

tức là, thương số là y. - 2x + 2 và phần còn lại là - 1. Và thực sự,

(_2) 3 - 2 (-2) +3 = - 1
.

Để thấy tại sao Định lý Phần dư nói chung đúng, hãy chia đa thức P (x) cho x - a, nhận
thương Q (x) với dư r. Sử dụng thuật toán chia, ta viết P (x) = Q (x) (x - a) + r.

Phương trình trên là một đồng nhất; tức là, nó đúng với mọi giá trị của x. Do đó, chúng ta có
thể tự do thay thế bằng giá trị thuận tiện nhất của x, cụ thể là x = a. Điều này tạo ra P (a) =
r, như mong muốn. Hãy ghi chú lại công cụ giá trị tiện lợi thay thế này. Nó có nhiều ứng dụng!

Định lý thừa số

Nếu a là số 0 của đa thức P (x) thì x - a phải là nhân tử; I E ,

P (x) là tích của x - a và một đa thức khác.

Điều này tiếp sau ngay từ Định lý Phần dư.

Định lý cơ bản của đại số

Định lý thừa số ở trên cho chúng ta biết rằng x - a là một nhân tử của đa thức P (x) nếu a là số
không. Nhưng làm thế nào chúng ta biết nếu một đa thức chẵn có số 0? Định lý Cơ bản của Đại số
đảm bảo điều này:

Mọi đa thức trong C [x] đều có ít nhất một số 0 phức.

Định lý này khá sâu và khó chứng minh một cách đáng ngạc nhiên. Bằng chứng của nó nằm ngoài phạm
1 0
vi của cuốn sách này.
Hệ quả của Định lý Cơ bản (sử dụng kết quả của Bài toán 5.4.6 bên dưới) là bất kỳ đa thức
bậc n nào đều có đúng n số không phức, mặc dù một số số không có thể không khác biệt. Do đó,
chúng ta có dạng thừa số sau cho bất kỳ polyno mial nào:

x "-1 +... + ao = an (x - rl) (x - r2) '" (x -


rn), anx "+ an_1 (2)

trong đó ri là số không, có thể không phải tất cả đều khác biệt.

Nếu các số không không khác biệt, chúng tôi nói rằng chúng có nhiều hơn 1. Đối với
ví dụ, đa thức bậc 8
2
(x - 1 ) (x - 2i) (x + 2i) (x - 7) 3 (x + 6)

IOFhoặc một bằng chứng cơ bản nhưng khó, xem [9]. Để có một lập luận đơn giản hơn nhưng ít cơ bản hơn, hãy xem [29].
Machine Translated by Google

5.4 POLYNOM IALS 1 67

có tám số không, nhưng chỉ có năm số không riêng biệt. Số 0 7 xuất hiện với bội số 3 và số 0 -6 có

bội số 2.

Dưới đây là một ví dụ sử dụng phân tích các số không kết hợp với thay thế thông minh

và xác định một công cụ chức năng mà bạn đã gặp trong Ví dụ 3.3.8 trên trang 89.

Ví dụ 5.4.2 (USAMO 1975) Nếu P (x) biểu thị một đa thức bậc n sao cho P (k) = k / (k + 1) với k
= 0, 1,2, ..., n, hãy xác định P (n + 1).

Giải pháp: Quay lại Định lý thừa số. Diễn giải lại định lý này từ quan điểm
của người giải quyết vấn đề là

Để biết các số không của một đa thức là biết đa thức.

Nói cách khác, nếu bạn không biết các số không của đa thức đang được xem xét, hoặc dành một số
nỗ lực để tìm chúng, hoặc chuyển trọng tâm của bạn sang một đa thức mới có số không rõ ràng.
Trong trường hợp của chúng ta, việc biết rằng P (k) = k / (k + 1) không cho chúng ta biết bất
cứ điều gì về các số không thức.
của P Chúng
(x), vì k loại
tôi / (k bỏ
+ 1) không
đồng phải
thời là 0khó
cả hai cũng không
khăn phải
bằng làmUltip
cách một đa
bằng (k + 1) và trừ đi:

(k + I) P (k) - k = 0.

Chúng tôi có thông tin về các số không của một đa thức khác, cụ thể là đa thức bậc (n + 1 )

Q (x) : = (x + I) P (x) - x.

Rõ ràng các số không của Q (x) chỉ là 0, 1, 2, ... ,n, vì vậy chúng ta có thể viết

(x + 1) P (x) -x = Cx (x - I) (x - 2) ··· (x - n),

trong đó C là một hằng số phải được xác định. Vì phương trình trên là một đồng nhất,
đúng với mọi giá trị x , nên chúng ta có thể thêm vào bất kỳ giá trị nào thuận tiện. N
Các giá trị x = 0, 1, ..., không hoạt động, vì chúng làm cho phía bên phải bằng 0. Vế
trái chứa số hạng rắc rối (x + 1) P (x), nên rõ ràng lựa chọn của chúng ta là x = - 1.
Cắm cái này vào năng suất

1 = C (-I) (- 2) (- 3) ··· (- (n + I)),

vì thế

!
(C = - 1)
'::n +
--'--: -: (n + I)!

Cuối cùng, chúng ta có thể cắm x = n + 1 và chúng ta có

: :
n +! !
1) 1) n + +!
((n + 2) P (n + 1) -n- I = ((n + I) n ··· 1 = (n + I)! = (- Tôi t ,
( + Tôi)! ( + Tôi)!

vì thế

!
P (n + 1) =
n + I + (-1) n +

n + 2
----'-


Machine Translated by Google

1 68 CHƯƠNG 5 ALG EBRA

Mối quan hệ giữa Zeros và Hệ số

Nếu chúng ta nhân vế phải của phương trình (2) ở trang 1 66, chúng ta có thể nhận được một loạt các biểu thức cho

các hệ số của đa thức theo các số không của nó. Đây có vẻ là một công việc khá phức tạp và tẻ nhạt, vì vậy chúng

ta hãy tiếp cận nó một cách thận trọng. Để xem điều gì đang xảy ra, chúng ta hãy xem xét một đa thức rất đơn

giản, một bậc hai monic (không mất tính tổng quát, tất cả các đa thức chúng ta sẽ xem xét sẽ là monic) với các số

không r và s.

Sau đó, theo phương trình (2), chúng ta có thể viết đa thức của chúng ta dưới dạng

� + a lx + ao = (x - r) (xs).

Vế bên phải bằng � - rx - sx + rs, và nếu chúng ta cân bằng các số hạng với các số hạng ở bên trái, chúng ta

nhận được

al = - (r + s), ao = rs.

Vì nói chung, chúng ta sẽ cần phải giải thích các biểu thức phức tạp hơn, chúng ta hãy suy nghĩ về cách chúng ta
II
vừa thực hiện một biểu thức dễ dàng này. Chúng tôi đã sử dụng "FOIL " , thực sự chỉ
thứccótrong
nghĩa(xlà- "nhân
r) vớimọi
mọiđơn
đơn

thức trong (x - s)." Nói cách khác, chúng ta đã tính (x - r) (xs) = (x + (-r)) (x + (-s)) = x · x + (-r) · x + x ·

(-s) + (-r) · (-s).

Quy trình tương tự cũng hoạt động khi chúng ta nhân ra các biểu thức phức tạp hơn.

Ví dụ, hãy xem xét

3 x + a2� + alx + ao = (x - q) (x - r) (x - s).

Sau khi bỏ qua vế phải, nhưng trước khi thu thập các số hạng tương tự, chúng ta sẽ có 2 · 2 · 2 = 8 số hạng, vì

chúng ta nhân mỗi đơn thức trong (x - q) với mỗi đơn thức trong (x - r) với mỗi đơn thức trong (x - s), và mỗi số

hạng chỉ có hai đơn thức trong đó. Nói cách khác, mỗi
cho
trong
sự lựa
số chọn
tám số
bahạng
phầntrong
tử, một
(x phần
- q) tử
(x được
- r) chọn
(x - từ
s) x
đại
hoặc
diện

-q, một phần tử được chọn từ x hoặc -r và một phần tử được chọn từ x hoặc -r và một được chọn từ x hoặc -so

Vì vậy, những loại điều khoản chúng tôi có thể nhận được? Nếu ba lựa chọn của chúng tôi đều là x, chúng tôi kết thúc với

số hạng x3
. Có ba cách để chúng ta có thể chọn hai x và một hằng số, tạo ra các số hạng -q�, -rx2
2
, -sx . Tương tự như vậy, có ba cách mà chúng ta có thể chọn
chỉ một x và hai hằng số, tạo ra qrx, qsx, rsx. Cuối cùng, chỉ có một cách để chọn không có xs, thuật ngữ -qrs.

Đây là tám điều khoản trong tất cả và thu thập những điều khoản giống như chúng tôi có

3 x + a2� + alx + aO = (x - q) (x - r) (x - s) =

x3 - (q + r + s) � + (qr + qs + rs) x - qrs.

Tương đương với các điều khoản, chúng ta thấy rằng

a2 = - (q + r + s), al = qr + qs + rs, ao = -qrs.

Hãy để chúng tôi làm một ví dụ nữa, lần này là một đa thức bậc bốn monic với các số không

p, q, r, s. Chúng ta viết đa thức dưới


4 3 + a3 x
x dạng + a2� + alx + ao = (x - p) (x - q) (x - r) (x - s).

II
Điều này là viết tắt của "đầu tiên, bên ngoài, bên trong, cuối cùng."
Machine Translated by Google

5,4 CHÍNH SÁCH 1 69

Sử dụng cùng một suy luận, vế phải sẽ có 16 số hạng đơn thức (trước khi thu thập các số hạng
giống nhau), mỗi số hạng được tạo thành bởi một lựa chọn của x hoặc -p, x hoặc -q, v.v. Đối
với bài kiểm tra, các số hạng sử dụng chính xác hai xs cũng sẽ có chính xác hai hằng số. 12
Số cách có thể chọn hai số hạng khác nhau từ tập {p, q, r, s}; tức là, (i) = 6 số hạng. Làm

việc với tất cả các điều khoản, chúng tôi


4
(x - p) (x - q) (x - r) (x - s) = x - (p + q + r + s) x3 +

(pq + pr + ps + qr + qs + rs) � - (pqr +

pqs + prs + qrs) x + pqrs.

Tương đương với các điều khoản, chúng tôi có

a3 = - (tổng các số không) a2 = +

(tổng của tất cả các tích của hai số không khác nhau) = - (tổng của

các số
tấtkhông),
cả các tích của ba số không khác nhau) a I ao = + (tích của

ở đây người ta hiểu “khác biệt” ở đây mang ý nghĩa tượng trưng thuần túy; tức là chúng ta chỉ
nhân các số không với các nhãn khác nhau, chẳng hạn như p và q, ngay cả khi các giá trị số
của chúng giống nhau.

Cuối cùng, chúng ta thấy mẫu và có thể viết các công thức nói chung:

Cho rl, r2 , ..., r n là số không của đa thức monic

Tôi

� + một I�- + ... + ao = O.

Khi đó đối với k = 1,2, ..., n,

ak k
= (- Nó (tổng của tất cả các sản phẩm của không - k số không khác nhau)
một
nk
= (_ Tôi) ril ri2 ... rin _k ·

LJ

1 5) 1 <h <·· - <iN -k5, n

Những công thức này rất quan trọng và cần được ghi nhớ vào bộ nhớ. Ngôn ngữ không chính
xác "tổng của tất cả các sản phẩm ..." dễ nhớ hơn,dụng
nhưng
cẩnhãy
thận
dành
cácthời
chỉ gian
số dưới
để hiểu
hình cách
thànhsửcác
tổng một cách chặt chẽ. k bằng nhau Cũng lưu ý vai trò của lũy thừa - 1. Chúng tôi sử dụng
thuận tiện rằng ( _ 1 ) thực tế
đến + 1 nếu k chẵn và - 1 nếu k lẻ.

Chúng ta hãy đi xuống trái đất từ cuộc thảo luận trừu tượng này bằng cách xem xét một ví
dụ cụ thể.

Ví dụ 5.4.3 (USAMO 1 984) Tích của hai trong bốn số không của phương trình bậc bốn

là -32. Tìm k.

12Bạn đã xem Phần 6. 1, phải không?


Machine Translated by Google

1 70 CHƯƠNG 5 ALG EBRA

Giải: Cho các số không là a, b, e, d. Khi đó, mối quan hệ giữa các số không và hệ số cho kết quả

a + b + e + d = 1

8, ab + ae + ad + be + bd + ed = k,

abc + Abd + aed + giường = -200,


abed = - 1984.

Không mất tính tổng quát, đặt ab = -32. Thay giá trị này thành abed = - 1984 thu được cd = 62 và thay thế điều

này bằng tum sẽ tạo ra hệ thống

a + b + e + d = 18 (3)

30 + ae + ad + be + bd = k (4)

-32e - 32d + 62a + 62b = -200 (5)

Chúng ta hãy suy nghĩ một cách chiến lược. Chúng ta cần tính k, không phải các giá trị a, b, e, d. Một

bước áp chót là đánh giá ae + ad + be + bd. Lưu ý rằng các yếu tố này:

ae + ad + be + bd = a (e + d) + b (e + d) = (a + b) (e + d).

Trong khi chúng ta đang ở đó, hãy cũng nhân tố (5):

-32 (e + d) + 62 (a + b) = -200.

Bây giờ nó sẽ rõ ràng làm thế nào để tiến hành. Ta chỉ cần tìm hai giá trị u: = a + b và v : = e + d. Phương

trình (3) và (5) trở thành hệ

u + v = 18,

62u - 32v = -200,

có thể dễ dàng giải được (u = 4, v = 14). Cuối cùng, chúng tôi có

k = 30 + 4. 14 = 86.

Định lý Rational Roots

Giả sử P (x) E Z [x] có hoành độ x = 2/3. Điều này có cung cấp cho bạn bất kỳ thông tin nào về P (x) không?

Theo Định lý Hệ số,

P (x) = (x - �) Q (x),

trong đó Q (x) là một đa thức. Nhưng Q (x) có những loại hệ số nào ? Tất cả những gì chúng ta biết chắc chắn

là các hệ số phải hợp lý. Tuy nhiên, nếu x - � là một thừa số thì 3 (x - �) = 3x - 2 cũng sẽ là một thừa số,

do đó ta có thể viết

P (x) = (3x - 2) S (x),

trong đó S (x) = Q (x) / 3. Ta biết rằng P (x) có hệ số nguyên ; chúng ta có thể nói điều tương tự về S (x)

không? Thật vậy, chúng tôi có thể; đây là Bổ đề Gauss:


Machine Translated by Google

5.4 CÁC CHÍNH SÁCH 1 71

Nếu một đa thức với hệ số nguyên có thể được tính thành nhân tử của đa
thức với hệ số hữu tỉ, thì nó cũng có thể được tính thành đa thức nguyên
thủy với hệ số nguyên.

(Một đa thức với hệ số nguyên được gọi là nguyên thủy nếu các hệ số của nó không có chung hệ số. Ví dụ:

3x2 + 9x + 7 là nguyên hàm trong khi 1 O.x2 - 5x + 15 thì không.) Xem Bài toán 7. 1 .30 để biết một số

gợi ý về chứng minh bổ đề Gauss.

Vì P (x) thừa số thành tích của (3x- 2) và một đa thức khác với hệ số nguyên nên hệ số của số hạng

đứng đầu của P (x) phải là bội của 3 và hệ số của số hạng cuối cùng phải là bội số của 2.

Nói chung, giả sử rằng một đa thức P (x) với hệ số tích phân có một số không hữu tỉ x = alb, trong

đó a và b là số hạng thấp nhất. Theo Định lý Nhân tố và Bổ đề Gauss,

P (x) = (bx - a) Q (x),

trong đó Q (x) là một đa thức với hệ số nguyên . Điều này ngay lập tức cho chúng ta Định lý Rational
Root:

Nếu một đa thức P (x) với hệ số tích phân có số hữu tỉ x = alb, trong đó
a và b là số hạng thấp nhất, thì hệ số đứng đầu của P (x) là bội số của b
và số hạng không đổi của P (x ) là bội số của a.

Trong thực tế, Định lý Rational Root không chỉ được sử dụng để tìm các số không mà còn để

chứng minh rằng các số không là không hợp lý.

Ví dụ 5.4.4 Nếu .x2 - 2 có bất kỳ số 0 hữu tỉ nào là alb (theo giá trị thấp nhất), chúng ta phải có bl

l và a1 2. Do đó, số 0 hữu tỉ duy nhất có thể là ± 2. Vì cả 2 và -2 đều không phải là số không, chúng ta

có thể kết luận rằng .x2 - 2 không có số 0 hữu tỉ. Đây là một cách khác để chứng minh rằng V2 là không

hợp lý!

Ta có thể khái quát suy luận trên khi áp dụng cho đa thức đơn thức. Nó là

một tiêu chí thú vị cho sự không hợp lý và cần được lưu ý như một công cụ:

Mọi số 0 hữu tỉ của một đa thức monic phải là một số nguyên. Ngược lại,
nếu một số không phải là số nguyên mà là số 0 của một monic poly 13
nomial, thì nó phải là số vô tỷ.

Chúng ta sẽ kết thúc phần này với một vấn đề khá khó sử dụng công cụ đa thức monic ở trên cùng với

một số ý tưởng khác.

Ví dụ 5.4.5 Chứng minh rằng tổng

\ hOO I2 + 1 + Vl O022 + 1 + ... + V20002 + 1

là phi lý.

Giải pháp: Chiến lược của chúng tôi có hai hướng: thứ nhất, để cho thấy rằng tổng được đề cập là

không phải là một số nguyên, và thứ hai, để chứng tỏ rằng nó là số 0 của một đa thức monic.

13 Phát biểu này cũng có thể được chứng minh trực tiếp mà không cần sử dụng định lý về căn số hữu tỉ (Bài toán 5 .4. 13). Nếu bạn

đang bối rối, hãy xem Ví dụ 7. 1.7 trên trang 226.


Machine Translated by Google

1 72 CHƯƠNG 5 ALG EBRA

Bước đầu tiên, hãy quan sát rằng nếu n > 1 thì n < J n2 + 1 < n + 1 / n. Bất đẳng thức đầu tiên

là hiển nhiên, và bất đẳng thức thứ hai theo sau n2 + 1 < n2 + 2 < (n + 1 / n) 2. Chúng ta hãy gọi
tổng trong câu hỏi S. Sau đó

S = 1001 + 81 + 1002 + lh + ... + 2000 + 81000,

trong đó mỗi 8i nằm trong khoảng từ 0 đến 1/1001. Do đó,

o < 81 + lh +. .. + 81000 < 1,

vì vậy S không phải là số nguyên.

Tiếp theo, chúng ta sẽ chỉ ra rằng S là số 0 của một đa thức monic. Nói chung, chúng tôi
sẽ chứng minh rằng với mọi số nguyên dương n, số lượng

Val + Ja2 + ... + JGn

là một số không của một đa thức monic nếu mỗi ai là một số nguyên không phải là một hình vuông hoàn

hảo. Chúng tôi tiến hành bằng cách cảm ứng. Nếu n = 1, khẳng định là đúng vì y'aI là một số không

của đa thức đơn thức x2 - al. Bây giờ giả sử rằng Y = Val + Ja2 +. .. + JGn là số 0 của đa thức đơn

thức P (x) = x + cr-1x '· -1 + ... + co. Chúng ta sẽ tạo ra một đa

thức monic có x = Y + v'an + 1 là số không. Ta có 0 = P (y) = P (x - v'an + d = (x - v'an + IY + Cr-

I (x - Jan + l y-1 +.. + Co.

Chú ý rằng khai triển của mỗi số hạng (x - v'an + I) k có thể được tách thành các số hạng có
hệ số nguyên và số hạng có hệ số bằng một số nguyên lần v'an + 1 .
Vì vậy, chúng tôi có

0 = P (x - Jan + d = x + Q (x) + Jan + IR (x), trong

đó Q (x) và R (x) là các đa thức với hệ số nguyên, mỗi bậc nhiều nhất là r - 1. Đặt các gốc ở một
phía của phương trình cho ra x + Q (x) = -Jan + IR (x),

và bình phương cả hai bên dẫn đến


2 2
x2r + lXQ (x) + (Q (x)) - an + 1 (R (x)) = Ô.

Thuật ngữ có mức độ cao nhất là x2r. Vì tất cả các hệ số bây giờ là số nguyên, chúng tôi đã
_
tạo ra một đa thức monic với x = Y + Jan + 1 là 0, như mong muốn.

Các vấn đề và bài tập

5.4.6 Chứng minh rằng đa thức bậc n có thể có nhiều nhất n giá trị x khác nhau thì hai đa thức bằng nhau.
số 0 khác nhau.
5.4.8 Chứng minh rằng nếu một đa thức có hệ số thực thì các

5.4.7 Sử dụng Bài toán 5.4.6 để chứng minh một ứng dụng hay số không của nó nằm trong các cặp liên hợp phức tạp; tức

được gọi là nguyên tắc đồng nhất, trong đó nói rằng nếu hai là, nếu a + hi là số 0, thì a - hi cũng là số 0.

đa thức bậc d f (x) , g (x) bằng nhau đối với d + I


Machine Translated by Google
5.5 CỔ PHIẾU 1 73

+ 5.4.9 Tìm phần dư khi bạn chia x 8 1 + x49 5.4.18 Cho p (x) là đa thức bậc n, không thuộc hệ số nguyên.
x 25 + x9 + x theo x3 - x. Đối với bao nhiêu giá trị nguyên bí mật của x thì ap (x) phải

5 + ... + 1. Tìm phần còn lại là một số nguyên để đảm bảo rằng p (x) là một số nguyên cho
5.4.10 Cho p (x) = x6 + x
tất cả các tegers x? 5.4.19 (lMO 1 993) Cho f (x) = x " + 5x" -
der khi p (x7) chia hết cho p (x).

5.4.11 (Gerald Heuer) Tìm một lời giải dễ dàng hơn cho Ví dụ
1 + 3 ở đâu
5.4.3 trang 1 69 bằng cách đầu tiên chỉ ra nhân tử của đa
n> I là một số nguyên. Chứng minh rằng f (x) không thể
thức quar thành hai tứ thức (x2 + ax - 32) (x2 + bx - 62).
được biểu diễn dưới dạng tích của hai đa thức, mỗi đa
thức đều có tất cả các hệ số nguyên và bậc ít nhất là I.
5.4.12 Tìm một đa thức với hệ số tích phân có các số không
5.4.20 (USAMO 1 977) Nếu a và b là hai nghiệm của + x4 +
bao gồm v'2 + v'5. 6
x 4 + x3 - I = 0, chứng minh rằng ab là một căn của x
5.4.13 Chứng minh rằng nếu một đa thức monic có một số 0 3 x - x2 -1 = 0.
hữu tỉ thì số 0 này thực tế phải là một số nguyên. 1

5.4.21 (Canada 1 970) Cho P (x) = Xl + al1_ 1 xn + ... + alx +


5.4.14 Cho p (x) là một đa thức với các hệ số nguyên thỏa ao là một đa thức với các hệ số tích phân. Giả sử tồn tại bốn
mãn p (O) = p (I) = 1 999. Chứng tỏ rằng p không có số số nguyên phân biệt a, b, c, d với Pta) = P (b) = P (c) = P
nguyên nào.
(d) = 5. Chứng minh rằng không tồn tại số nguyên k với P (k) =

5.4.15 Cho p (x) là một đa thức bậc I 999 với hệ số teger 8.

bằng ± 1 với 1 999 giá trị nguyên khác nhau của x. Chứng tỏ
5.4.22 (USAMO 84) P (x) là đa thức bậc 3n sao cho
rằng p (x) không thể được tính thành tích của hai đa thức với

hệ số coef nguyên.
P (O) = P (3) P (3n) 2,

P (I) = P (4) = P (3n - 2)


5.4.16 (Hungary 1 899) Gọi r và s là các gốc của
P (2) = P (5) và = P (3n - l) Tôi, 0,

2 x - (a + d) x + (ad - be) = O.
P (3n + I) = 730.
3 3 là
Chứng minh rằng r và s gốc rễ của Xác định n.

tôi - (a 3 + d3 + 3abc + 3bcd) y + (ad -bc) 3 = O. 5.4.23 (Hàng tháng Toán học Hoa Kỳ, ngày 1 tháng 10 năm

962) Cho P (x) là một đa thức với hệ số thực.


5.4.17 Cho a, b, c là các số nguyên phân biệt. Chứng tỏ rằng tồn tại một đa thức khác không Q (x) với các hệ

Liệu đa thức (x -a) (x -b) (x - c) - I có thể được số thực sao cho P (x) Q (x) có các số hạng có cùng bậc chia

tính thành tích của hai đa thức với hệ số nguyên không? hết cho 1 09 .

5.5 Tuổi bất bình đẳng

Các bất đẳng thức rất quan trọng vì nhiều cuộc điều tra toán học liên quan đến các ước lượng, tối ưu hóa, các trường

hợp tốt nhất và trường hợp xấu nhất, các giới hạn, v.v. Đẳng thức là tốt, nhưng thực sự khá hiếm trong "thế giới

thực" của toán học. Một ví dụ điển hình là việc sử dụng các bất đẳng thức khá thô sơ để thiết lập sự phân kỳ của

chuỗi điều hòa (Ví dụ 5 .3.4 trên trang 161). Một ví dụ khác là Ví dụ 2.3. 1 ở trang 41, 2 trong đó chúng tôi đã

chứng minh rằng phương trình b


2
+ b + 1 = a không có nghiệm nguyên dương nào bằng cách chứng

tỏ rằng đẳng thức bị cáo buộc, trên thực tế, là một bất đẳng thức.

Đây là một giải pháp khác cho vấn đề đó, một giải pháp sử dụng chiến thuật tìm kiếm 2 khẳng định rằng b
2 2 + b + 1 = a
bình phương hoàn hảo: phương trình b + b + 1 là một hình vuông hoàn hảo.
Nhưng
Machine Translated by Google
1 74 CHƯƠNG 5 ĐẠI SỐ

VẬY b2 + b + 1 nằm giữa hai hình vuông hoàn hảo liên tiếp . Một điều không thể!

Những ví dụ này đã sử dụng những bất đẳng thức rất đơn giản. Điều cần thiết là bạn phải nắm vững chúng,

cũng như một vài ý tưởng phức tạp hơn.

Ý tưởng cơ bản

Chúng ta hãy bắt đầu với việc xem xét ngắn gọn các ý tưởng cơ bản, nhiều ý tưởng trong số đó chúng tôi sẽ trình

bày dưới dạng một loạt các phát biểu là các vấn đề (bài tập?) Để bạn kiểm chứng trước khi chuyển sang phần tiếp theo.

Số học cơ bản

Những điều sau đây khá đơn giản, nhưng bạn nên suy ngẫm kỹ để chắc chắn rằng bạn thực sự hiểu tại sao chúng

lại đúng. Hãy chú ý đến các dấu hiệu của các biến số của bạn!

5.5. 1 Bổ sung. Nếu x : 2: y và a: 2: b thì x + a: 2: y + b.

5.5.2 Phép nhân. Nếu x: 2: y và a: 2: 0 thì ax : 2: ay. Ngược lại, nếu a < 0 thì ax: =:; ay.

5.5.3 Đối ứng. Nếu x : 2: y thì 1 / x: =:; l / y, với điều kiện cả x và y cùng dấu.

5.5.4 Giải thích khoảng cách của giá trị tuyệt đối. Bộ

{x : lx - al =

b} bao gồm tất cả các điểm x trên trục số thực14 nằm trong khoảng cách b của điểm
một.

G rowth Rates of Functions

Điều quan trọng là phải hiểu thứ bậc của tỷ lệ tăng trưởng cho các chức năng phổ biến nhất. Cách tốt nhất

để tìm hiểu về điều này là vẽ nhiều đồ thị.

5.5.5 Bất kỳ hàm bậc hai nào của x sẽ chiếm ưu thế của bất kỳ hàm tuyến tính nào của x, với điều kiện x là

"đủ lớn". Ví dụ: 0,001�> l00000x + 20000000

đúng với mọi x> 109.

5.5.6 Bằng cách lập luận tương tự, JCl cuối cùng sẽ "chiếm ưu thế" với điều kiện a> b> O.

5.5.7 Tương tự như vậy, nếu a là một số dương bất kỳ và b> 1, thì lr cuối cùng sẽ chiếm ưu thế trong JCl.

(Nói cách khác, hàm mũ phát triển nhanh hơn hàm đa thức.)

5.5.8 Ngược lại, nếu a là bất kỳ số dương nào và b> 1, thì JCl cuối cùng sẽ chiếm ưu thế đối với logb x.

Có thể bạn đã nhận thấy rằng chúng ta đang hạn chế sự chú ý của mình vào các con số thực. Đó là bởi vì
không thể xác định z> 0 một cách có nghĩa khi z phức tạp. Xem Vấn đề 2 .3. 16 nơi trang 50.
Machine Translated by Google

5.5 INEQUALITI ES 1 75

Tóm lại, thứ bậc của tốc độ tăng trưởng, từ chậm nhất đến cao nhất, là

logarit, lũy thừa, số mũ.

Chứng minh đơn giản

Trong số nhiều cách chứng minh các bất đẳng thức, cách đơn giản nhất là thực hiện các phép toán
tạo ra các bất đẳng thức tương đương nhưng đơn giản hơn về mặt logic. Các biến thể phức tạp hơn
cũng bao gồm một chút xoa bóp. Dưới đây là một số ví dụ.

Ví dụ 5.5.9 Cái nào lớn hơn, V19 + V99 hay JW + V98?

Giải pháp: Chúng ta sẽ sử dụng quy ước viết dấu chấm hỏi (?) Cho một bất đẳng thức được cho
là. Sau đó, chúng tôi có thể theo dõi. Nếu đại số bảo toàn hướng của bất đẳng thức được cho là,
chúng ta tiếp tục sử dụng dấu hỏi. Thay vào đó, nếu chúng ta làm điều gì đó đảo ngược hướng của
sự bất bình đẳng bị cáo buộc (ví dụ: lấy đi có lại của cả hai bên), chúng ta sẽ thay đổi dấu hỏi
thành dấu hỏi lộn ngược (,,). Vì vậy, chúng tôi bắt đầu với

V19 + V99? VW + V98.

Bình phương cả hai bên cho kết quả

19 + 2) 19 · 99 + 99? 20 + 2V20 · 98 + 98,

giảm xuống

Tất nhiên điều này tương đương với

19 · 99? 20 · 98.

Tại thời điểm này, chúng ta chỉ có thể thực hiện phép tính, nhưng hãy sử dụng kỹ năng tính toán của chúng

ta: Trừ 19. 98 từ cả hai bên để có được

19 · 99 - 19 · 98? 98,

giảm xuống

19? 98.

Cuối cùng, chúng ta có thể thay thế dấu "?" với "< " và chúng tôi kết luận rằng

V19 + V99 < VW + V98.



· · · 1998 1999 ?
E xample •. 10
5 5giờ h Igger,
W IC IS b hoặc .
1999 2000
Giải pháp: Điều này có thể được thực hiện bằng nhiều cách; đây là một đối số sử dụng công cụ xác
định một hàm. Cho phép

x
f (x): = -.
x + l

You might also like